Sunteți pe pagina 1din 369

DYP SURVIVAL GUIDE

FOR POST GRADUATE


DERMATOLOGY EXAMINATION

• Editors •

Nitin Nadkarni Sharmila Patil Kiran Godse


THE EDITORS
• Dr. Nitin Nadkarni (MD, DVD, DDV)
Professor, Department Of Dermatology,
Dr. D Y Patil Medical College and Research Center,
Nerul, Navi Mumbai. Notice

• Dr. Sharmila Patil (MD, DDV) Medical knowledge is a constantly


Professor and Head, Department Of Dermatology, changing science. Changes in treatment and
Dr. D Y Patil Medical College and Research Center, drug therapy are essential as new research
Nerul, Navi Mumbai. and clinical experiences enlighten our
knowledge. The authors and publisher of
• Dr. Kiran Godse (MD, DDV) this work have checked with sources which
Associate Professor, Department Of Dermatology, are believed to provide reliable and
Dr. D Y Patil Medical College and Research Center,
complete information in compliance with
Nerul, Navi Mumbai.
the standards, at the time of publication.
CONTRIBUTORS Readers are encouraged to confirm with
other sources the information that is
• Dr. Mrs. Manjyot Gautam (MD Dermatology),
provided in this work. As the possibility of
Associate Professor, Department Of Dermatology,
human error exists and due to the ever-
Dr. D Y Patil Medical College and Research Center,
Nerul, Navi Mumbai. changing advances in medical sciences,
neither the author nor the publisher nor any
• Dr. Nanma Nikalji (MD Dermatology), other party involved in preparation or
Assistant Professor, Department Of Dermatology, publication of this work assume any
Dr. D Y Patil Medical College and Research Center, liability for any injury and/or damage to
Nerul, Navi Mumbai. persons or property arising from this
publication.
• Dr. Shweta Agarwal (MD Dermatology),
Assistant Professor, Department Of Dermatology,
Dr. D Y Patil Medical College And Research Center,
Nerul, Navi Mumbai.

II III
Foreword
One of the most challenging tasks for a
student is to prepare for an examination. Laborious
years of study, hard work in the outpatients, wards,
regular attendance at lectures, symposia and
conferences finally culminating in the all important
test of knowledge assimilation – the examination.
However appearing for an examination requires a
skill which an astute mind can develop only by a
steadfast focus and intense preparation which will ultimately decide
success or failure. It is not unusual to find one amongst equally diligent
students to flounder while the others sail through an examination. It is
often the big match temperament that helps one score over the other, but
overall it is the preparedness that distinguishes the two.
This book is compiled by a group of dedicated teachers, ably led
by Dr. Nitin Nadkarni, who have kept their students on their toes by a
daily exercise of case presentation followed by a viva grilling. Their
experience has helped them understand common deficiencies in students
which they have attempted to rectify in this book. The book gives the
student a comprehensive approach to preparation for the practical
examination and discusses long, short and spot cases, as well as answers
a number of viva questions
It is my pleasure to write this brief foreword to this wonderful
book so thoughtfully prepared by Dr. Nadkarni and his colleagues. I
have no doubt that it will be of great help to both students and teachers
in Dermatology.

Dr. Rui Fernandes,


MD, DVD, DDV
Professor Emeritus
Department of Dermatology,
Seth GS Medical College and
KEM Hospital, Mumbai.
Former President,
IADVL.

IV V
Preface
Examinations are a test, not only of knowledge and competence,
but of technique and temperament. Having (collectively) appeared for
more than a dozen examinations, and having been examiners for a lot
more, we are only too aware of the foibles of examiners (including
ourselves) and the follies of examinees. This book is a humble attempt
to guide the unwary candidate through the pitfalls of dermatology post
graduate examinations at various levels (M.D., D.N.B., D.V.D., F.C.P.S.,
etc).
This book is intended for the examination going candidate who
is appearing (hopefully for the first and last time) for an important PG
examination. We aim to give reliable advice on both simple and tricky
questions asked by examiners in different clinical situations. We have
divided the book into various sections, including long cases, short cases
(and spots) as well as viva. We have collected common questions asked
in the examinations, through “chatting up” our examiner friends, as
well as quizzing candidates who have recently appeared for
examinations (when their traumatic experiences are still fresh!). We
have included lots of questions but have purposely omitted any
references, for the sake of brevity. Some of the questions (and the
answers) reflect our own personal preferences and idiosyncrasies, but
we think that is to be expected in a book of this nature.
We are extremely thankful to our teacher Dr. Rui Fernandez,
who went through the entire draft of this book and gave valuable
suggestions, besides consenting to write the foreword.We gratefully
acknowledge the help and cooperation given by the Director of the
D.Y.Patil Group of Hospitals, Mr. Vijay D. Patil as well as Dr. Shirish
Patil, Dean of our Hospital. We thank Torrent Pharmaceuticals for
printing and distributing this book and making it possible for us to
reach the target audience in all parts of India.
Lastly we would like to thank the residents of our department,
who helped with the nitty-gritty of preparing this book. Special mention
must be made of Dr. Aditya Mahajan and Dr. Zubin Mandlewala, whose
contribution to proof reading was invaluable. Many thanks to both of
them.
– Editors

VI VII
Contents 7. Actinic Keratosis 329
8. Alopecia Areata (AA) 333
1. Hints for asppearing in examination 1
9. Aphthous Stomatitis 341
2. Signs in Dermatology 5
10. Astratotic Eczema 346
11. Atopic Dermatitis 350
LONG CASES
12. Bacterial Infections Questions 357
1. Anogenital Warts 17 13. Becker’s Nevus 384
2. Chancroid 24 14. Café–Au-Lait Macules (CALM) 387
3 Dermatomyositis 30 15. Callus 389
4 Erythroderma 45 16. Carcinomas 391
5 Genital ulcer 65 17. Congenital Melanocytic Nevus 397
6 Gonorrhoea 112 18. Contactdermatitis 399
7 Hansens Disease 123 19. CORN (clavus) 404
8 Leg ulcers 152 20. Cutaneous Tuberculosis 407
9 Lymphogranuloma Venerum 172 21. Darier’s Disease 412
10 Pellagra 179 22. Dermatofibroma 415
11 Psoriasis 188
23. Dermatitis Artefacta 417
12 Systemic Lupuserythematosus 212
24. Dowling – Degos Disease 419
13 Scleroderma 229
25. Epidermal Nevi 421
14 Vesicobullous diseases 257
26. Erythema multiforme 425
27. Erythema Nodosum (EN) 430
SHORT CASES
28. Fixed Drug Eruptions 435
1. Acanthosis Nigricans (AN) 303 29. Freckles 441
2. Acrochordon 307 30. Fox Fordyce disease 443
3. Acna 309 31. Granuloma Annulare 444
4. Acne Excotiée 321 32. Haemangiona’s 447
5. Acquired Melanocytic Nevus 322 33. Hailey – Hailey disease 456
6. Aceokeratosis verruciformis of Hopf 326 34. Halo Nevi 459

VIII IX
35. Henoch–schönlein purpura (HSP) 461 64. Nevus spilus 575
36. Herpes simplex virus 468 65. Neurofibromatosis 577
37. Hidradenitis Suppurativa 479 66. Nipple eczema, hand eczema and pompholyx 587
38. Ichthyosis 482 67. Nummular Eczema 591
39. Idiopathic Guttate Hypomelanosis 485 68. Other Deep Fungal Infections 595
40. Incontinentia Pigmenti 487 69. Parapsoriasis 601
41. Keloid and Hypertrophic Scar 490 70. Palmoplantar Keratoderma 610
42. Keratolysis exfoliativa 493 71. Paronychia 615
43. Keratosis Pilaris 494 72. Parry Romberg Syndrome 618
44. Lentigo 495 73. Pearly Penile Papules - Hirsute Penis 619
45. Lichen Planus (LP) 501 74. Pediculosis 620
46. Lichen Planus Pigmentosus 512 75. Perioral Dermatitis 628
47. Lichen Simplex Chronicus (LSC) 514 76. Phrynoderma 630
48. Lichen Spinulosus 516 77. Piebaldism 633
49. Lichen Sclerosus Et Atrophicus 517 78. Pigmented Purpuric Dermatosis 635
50. Lichen striatus 522 79. Pityriasis Alba 638
51. Linear and Whorled Nevoid Hyoeroigmentation 525 80. Pityriasis Rosea 640
52. Lipoma 527 81. Pityriasis Rubra Pilaris (PRP) 644
53. Lipodermatosclerosis 530 82. Prurigo Nodularis 647
54. Macular and Lichen Amyloidosis 532 83. Pseudo Xanthoma Elasticum (PXE) 651
55. Miliaria 536 84. Pyoderma Faciale 655
56. Milium 541 85. Pyogenic Granuloma 656
57. Melasma 544 86. Reticulate Acropigmentation of Kitamura 658
58. Morphea 548 87. Rosaces 659
59. Mycetoma 553 88. Scabies 661
60. Nevus Comedonicus 565 89. Sebaceous Hyperplasia 670
61. Nevus Depigmentosus 568 90. Striae 672
62. Nevus of Ota and Nevus of Ito 570 91. Superficial Fungal Infection 674
63. Nevus sebaceous 573 92. Tattoo 690
X XI
93. Trichotillomania 693
1. HINTS FOR APPEARING
94. Tuberous sclerosis 697
95. Twenty Nail Dystrophy 704 IN THE EXAMINATION
96. Urticaria 706
97. Xanthomas 711 In our opinion, spots are the “make or break” section
of the practical’s. First impression is the last impression.
98. Xeroderma Pigmentosum 714 Since the examiner encounters the candidate for the first time
99. Yellow Nail Syndrome 715 during the spots, what impression he gets will have a
100. M olluscum Contagiosum 717 bearing on the final result. Hence it is imperative that a
101. Chickenpox 719 candidate does reasonably well in this section. If you get
through the spots comfortably, for all practical purposes,
you have passed!
Important Tips:
1. The candidate should be well equipped. He should
be wearing a clean (not tattered) apron with multiple
pockets without any name on it. The pockets should
contain: Pen, Magnifying lens (illuminated), Small Pen
torch, Measuring tape, Glass slide, Pin, Cotton piece
and a blotting paper (to demonstrate Auspitz sign).
However, there is absolutely NO need to carry a
mobile (actually, it is banned!).
2. Since most of the candidates are females, it is advisable
(please accept our apologies if we sound sexist) that
they should avoid the following items: Heavy
makeup, streaked hair, painted and untrimmed nails,
mehendi, jewellery, dangling ear rings and prominent
nose rings, pencil heels (they make a distracting noise).
Both males and females should wear modest clothes.
You should fit in the crowd and not stand out (for the
wrong reasons!).
3. Greet the examiners respectfully during the first
interaction. The following instructions are targeted
towards the spots.
XII 1
2 DYP SURVIVAL GUIDE FOR POST GRADUATES DYP SURVIVAL GUIDE FOR POST GRADUATES 3

4. Make the patient comfortable and take his permission case of palmo-plantar warts. Remember, sounds of
before examining hoofs imply approaching horses and not zebras.
5. Keep both your hands free. All the accessories should 12. Usually the question asked is “how will you treat”?
be kept in the pocket s of your apron. The examiner wants ONE treatment for the
“SPECIFIC” patient sitting before you. Do not impress
6. Look at the lesion through a magnifying glass (which
the examiner with a long list of possible treatments.
you should carry and not borrow from the examiner!).
First say the most common treatment you would give,
7. Touch the lesion. You cannot make out whether the with the dose (or concentration and frequency, in case
lesion is flat or raised by just looking. If you suspect of a topical medication) and duration. Also mention
erythema or purpura, do a diascopy. If you suspect the general measures you would advice the patient
psoriasis, or pityriasis versicolor, do grattage. Remove (after talking about the specific treatment). If there is
the scale to see the “carpet tacking” if you suspect no satisfactory or specific treatment for a condition,
lupus erythematosus. Spots do not mean you take a say it, BUT immediately come out with SOME
cursory glance at the patient and give your diagnosis. treatment or at least some general measures you
Please do not put your hands in your pockets or fold would advice in case of an absolutely untreatable
them. This implies you have already become a condition. Don’t say, there is no satisfactory treatment
consultant (which you have not become yet). and stop. In a series of cases of viral infections (warts,
molluscum, etc.), a candidate said, there is no specific
8. If you want to look at other sites besides the one you treatment at least 3 consecutive times, without saying
are asked to see, ask the permission of the examiner anything more. The examiner asked him why he was
before doing so. trying to become a dermatologist when he didn’t want
9. If you want to ask questions of the patient, seek the to treat patients. Such embarrassing encounters should
permission of the examiner (some examiners do not be avoided.
allow it, but no harm in asking!). We will try to discuss the common “spots” which are
10. If you suspect any infectious or neoplastic process, kept in the examination. The list is NOT inclusive,
palpate the regional nodes (after taking permission). since potentially any lesion on the skin (including a
tattoo) is worthy of being kept as a spot!
11. As far as possible, give a single diagnosis, which is
SIMPLE and COMMON. DON’T use short forms like In case of short and long cases, following hints will be
LP, LE, PLE, (though they are used in this book for helpful:
sake of brevity). Spots test your ability to come out 1. Write down all the history and examination findings
with a single diagnosis. Try not to impress the as you would write in the ward notes. Many examiners
examiner with “rare” diagnosis. We know of a look at them and criticize them.
candidate who came into trouble by giving a diagnosis 2. Take a targeted history in short cases but a detailed
of “porokeratosis palmaris et plantaris” when it was a history in long cases.
4 DYP SURVIVAL GUIDE FOR POST GRADUATES

3. When time is limited, try to do the history taking and


examination at the same time. This is especially true
2. SIGNS IN DERMATOLOGY
of leprosy cases, where you will be really pressed for
time. 1. AUSPITZ SIGN
4. Give a specific yet detailed diagnosis. For example, The scales are completely scraped off, seen as a moist
in psoriasis, tell about the type of psoriasis, extent of red surface (membrane of Bulkeley) through which dilated
involvement, stable or unstable, etc. capillaries at the tip of elongated dermal papillae are torn
5. When examiner asks “how will you manage”, start leading to multiple bleeding points.
with investigations; when he asks how you will treat, Parakeratosis, suprapapillary thinning of the stratum
start with the treatment. malpighii, elongation of dermal papillae and dilation and
6. Always talk about counseling and advice to the patient tortuosity of the papillary capillaries, psoriasis, darier’s
in the beginning of your treatment. disease and actinic keratosis
7. DON’T use trade names of medications. Always 2. CARPET TACK SIGN
mention the dose or concentration of the medications Also called cat’s tongue sign and tin tack sign in DLE,
even without being asked. partially adherent scales entering a patulous follicle. When
8. Do not forget to look at special sites: genitals, palms, the scale is removed, its undersurface shows horny plugs
soles, mouth, peri-anal areas, scalp, nails. that had occupied follicles.
9. In case a patient refuses to allow you to examine a Seborrheic dermatitis and pemphigus foliaceous
particular site, do not forcibly make him/her strip, but
just inform the supervisor about it. 3. SCRATCH SIGN
10. Ask for a tongue retractor, vaginal speculum but do Also called coup d’ongle sign, besnier’s sign, stroke
not insist that the patient allows you to use it. In case of the nail sign. Seen in Pityriasis versicolor and is an
of any controversy, inform the supervisor. important diagnostic tool. Occurs may be due to the loosing
In general, please remember, DO NOT ARGUE WITH of barely perceptible scale with a fingernail.
THE EXAMINERS. They are INFALLIBLE during the course Scale is usually white or light tan and flakes off rather
of the examinations. As one of my respected teachers said, easily.
if the examiner says that the sun rises in the west, nod 4. DARIER’S SIGN
respectfully and say that it has been reported. Do NOT
however, quote textbooks at the examiners to quote your Refers to the urtication and erythematous halo that are
point. Remember, there is such a thing as winning the battle produced in response to rubbing or scratching of lesions of
and losing the war! cutaneous mastocytosis.
Named after the French dermatologist Ferdinand-Jean
h Darier.
5
6 DYP SURVIVAL GUIDE FOR POST GRADUATES DYP SURVIVAL GUIDE FOR POST GRADUATES 7

Conditions associated with Darier’s sign: Cutaneous give rise to gaping ‘fish mouth wounds’ over bony
mastocytosis, leukemia cutis, juvenile xanthogranuloma, prominences like shins, knees and elbows.
histocytosis and lymphoma. Wide, thin, papyraceous scars over the knees and
A transient piloerection and elevation or increased elbows are called ‘cigarette paper scars’.
induration of a lesion induced by rubbing and is observed 12. GORLIN’S SIGN
in congenital smooth muscle harmartomas. Is the ability to touch the tip of the nose by extension
6. DERMATOGRAPHISM of the tongue.
A form of physical urticaria that consists of local Seen in pseudoxanthomaelasticum.
erythema due to capillary vasodilation, followed by edema 13. ANTENNA SIGN
and a surrounding flare due to axon reflex induced dilation
It is seen in keratosis pilaris in which individual
of arterioles, which is observed after the firm stroking of
follicles show a long strand of keratin glinting when
skin.
examined in tangentially incident light.
7. MARFAN SYNDROME:
14. ASBOE-HANSEN SIGN
Unusually tall habitus, long, thin extremities,
(BLISTER SPREAD SIGN)
arachnodactyly, joint hypermobility, excessive length of
the lower extremities, armspan exceeding the individual’s Gustav Asboe Hansen first described it in 1960.
length. He demonstrated enlargement of bulla by applying
8. WRIST SIGN: finger pressure to small, intact, and tense bulla in patients
with pemphigus and bullous pemphigoid.
Wrist sign is the overlapping of the thumb and fifth
fingers when these encircle the opposite wrist. In the traditional bulla spread sign, pressure is applied
to the blister from one side, whereas in Asboe-Hansen sign
9. THUMB SIGN
pressure is applied at the center of the blister and
Also called Steinberg sign perpendicular to the surface due to smaller size of the lesion.
Is the extension of the thumb past the ulnar border of 15. BARNETT’S SIGN
the hand when apposed to the palm (SCLERODERMA NECK SIGN)
10. EHLERS-DANLOS SYNDROME It is ridging and tightening of the skin of the neck on
Joint hypermobility limited to the digits and extending the head with a visible and palpable tight band
hyperextensible skin over platysma in the hyperextended neck.
11. BEIGHTON SIGN 16. BRANHAM’S SIGN (Nicoladoni sign)
Is the passive apposition of the thumb to the flexor It is to be elicited in cases of arterio-venous fistula
aspect of the forearm. where there is slowing of the heart rate in response to
India-rubber man: Hyperextensibility of the skin and (manual) compression.
8 DYP SURVIVAL GUIDE FOR POST GRADUATES DYP SURVIVAL GUIDE FOR POST GRADUATES 9

17. “BREAKFAST, LUNCH, AND DINNER” SIGN 23. COUDABILITY SIGN


The bites of bed bugs (Cimex lectularius) usually It was first described by Shuster in cases of alopecia
follow a linear pathway in a group of three to five blood areata in 1984. Coudability sign is normal-looking hairs
meals and are often referred to as “Breakfast, lunch, and tapered at the proximal end in the perilesional hair-bearing
dinner” or “Breakfast, lunch, and supper” sign. scalp and can easily be made to kink when bent or pushed
18. BUSCHKE-OLLENDORFF SIGN inward.
This is a sign to be elicited in case of secondary 24. CULLEN’S SIGN
syphilis and cutaneous vasculitis, where there is deep Periumbilical ecchymosis in cases of acute
dermal tenderness on pressing the lesion (e.g., papular hemorrhagic pancreatitis and ruptured ectopic pregnancy
lesions of syphilis) with a pinhead. is termed Cullen’s sign. Similar changes in the flank is called
19. BUTTERFLY SIGN as Grey-Turner sign.
This refers to sparing of the mid scapular region in 25. DECK-CHAIR SIGN
patients having prurigo nodularis with neurodermatitis as It was classically described in Papulo-erythroderma
they are unable to reach the region for scratching. of Ofuji, wherein there are flat-topped red papules that
20. BUTTONHOLE SIGN become generalized erythrodermic plaques without the
involvement of abdominal skin folds.
In type 1 neurofibromatosis (Von-Recklinghausen’s
disease), neurofibromas can be invaginated with the tip of 26. DIMPLE SIGN (FITZPATRICK SIGN)
index finger back into the subcutis and again reappear after Squeezing the skin adjacent to a dermatofibroma
release of pressure. Other condition where one can find causes a dimpled appearance on its surface, also termed a
positive buttonhole sign are anetoderma and positive “pinch sign” or “dimple sign”.
dermatofibroma. 27. DORY-FLOP SIGN
21. CORAL BEAD SIGN It is described in relation to syphilitic chancre on the
Papules seen around the nail fold in multicentric coronal border of the prepucial skin in an uncircumscribed
reticulohistiocytosis are called as coral bead sign. male, whereupon on retracting the foreskin the entire ulcer
22. CHAGAS-MAZZA-ROMAÑA’S SIGN flips out all at once because it is too hard to bend due to
underlying button like induration.
In about eighty percent of cases of Chagas’ disease
(American trypanosomiasis), conjunctiva is the portal of 28. FORSCHEIMER’S SIGN
entry for TrypanosomaCruzi Seen in 20% of rubella patients, where there is an
Unilateral swelling of eyelids and orbit after enanthem of dull-red macules or petechiae confined to the
conjunctival inoculation is called as eye-sign or Chagas- soft palate during the prodromal period or on the first day
Mazza-Romaña’s sign or Romaña’s sign. of the rash. Can also be seen in infectious mononucleosis.
10 DYP SURVIVAL GUIDE FOR POST GRADUATES DYP SURVIVAL GUIDE FOR POST GRADUATES 11

29. FRANK’S SIGN 34. HAMBURGER SIGN


Diagonal crease in the earlobes of adults has been This sign has been described in relation to
associated with an increased risk for atherosclerotic heart trichotillomania, wherein there is vertically oriented split
disease. of hair shafts and proteinaceous material and erythrocytes
30. FRIAR TUCK SIGN are present in the split resembling a hamburger within a
bun.
Friar tuck was a companion of Robin Hood in the
legendary stories who had alopecia of the vertex with 35. HERTOGHE’S SIGN
sparing of occipital region. This is described in relation to (QUEEN ANNE’S SIGN)
trichotillomania, where patient plucks his own hair either It is defined as loss of lateral one third of eye-brows
in a wave like pattern across the scalp or centrifugally from (superciliary madarosis). It is seen in leprosy, myxedema,
a single starting point. Hairs over the occipital area are follicular mucinosis, atopic dermatitis, trichotillomania,
mostly spared in trichotillomania and is referred as Friar ectodermal dysplasia, discoid lupus erythematosus,
Tuck sign. alopecia areata, syphilis, ulerythema ophryogenes, systemic
31. GOTTRON’S SIGN sclerosis, HIV infection, and hypothyroidism.
It is a characteristic finding in dermatomyositis 36. HANGING CURTAIN SIGN
typified by scaly erythematous eruption seen on the dorsa It is seen in patients with pityriasis rosea. When the
of hands, metacarpophalangeal joints, and proximal skin is stretched across the long axis of the herald patch, the
interphalangeal joints. scale is noted to be finer, lighter, and attached at one end,
32. HAIR COLLAR SIGN which tends to fold across the line of stretch.
It is a marker of cranial dysraphism, including 37. HOLSTER SIGN OF DERMATOMYOSITIS
encephalocele, meningocele, and heterotropic brain tissue. Literally, holster means an extra pocket made from
Ectopic neural tissue in the occipital and parietal areas takes various materials to hold something where it can quickly
the form of smooth dome-shaped hairless nodules and be taken out for use (e.g., Gun holster). Confluent macular
sometimes a collar of hypertrichosis surrounds them, this violaceous erythema present on the lateral side of hip and
is called as hair-collar sign. thighs is called as “Holster sign” corresponding to the site
33. HAIRY PALM SIGN of hanging a holster.
It is a characteristic histopathological finding seen in 38. HYPOPYON SIGN
prurigo nodularis, where there is a presence of thick Hypopyon sign describes the presence of small,
compact orthohyperkeratosis; the hairy palm sign discrete, vesicles either flaccid or tense that become
(folliculosebaceous units seen with a thick and compact secondarily infected and pus accumulates in the lower half
cornified layer, seems like that biopsy has been taken from of the pustule. It is a clinical sign seen in pyodermas and
palm, i.e., volar skin but contains pilosebaceous unit). secondarily infected vesicobullous disorders (e. g.,
12 DYP SURVIVAL GUIDE FOR POST GRADUATES DYP SURVIVAL GUIDE FOR POST GRADUATES 13

pemphigus, bullous pemphigoid, and linear IgA involvement can result in a lipstick like mark left on the
dermatosis), where there is a transverse fluid level rim of a glass mug after consuming a hot beverage (Meffert’s
comprising of purulent material at the bottom when the sign).
patient is in a standing position and is called hypopyon 45. MILIAN’S EAR SIGN
sign.
Erysipelas and cellulitis have traditionally been
39. INGRAM’S SIGN defined as acute inflammatory processes of infectious origin
Inability to retract the lower eye-lid in patients of that primarily affect the dermis (in the case of erysipelas) or
progressive systemic sclerosis due to underlying sclerosis deeper dermis and subcutaneous tissue in cellulitis.
is called Ingram’s sign. It is a sign used to distinguish between erysipelas and
40. KAPOSI-STEMMER SIGN cellulitis of the facial region, where there is involvement of
Inability to pinch or pick up a fold of skin at the base ear in erysipelas and sparing in cellulitis, as there is no
of the second toe because of its thickness. It is seen in chronic deeper dermal tissue and subcutaneous fat.
lymphedema. 46. MIZUTANI’S SIGN
41. LESER-TRELAT SIGN (ROUND FINGER PAD SIGN)
First described by Edmund Leser and Ulysse Trelat, It is seen in Raynaud’s phenomenon associated with
characterized by sudden eruption of numerous seborrhoeic systemic sclerosis. This sign refers to disappearance of the
keratosis, usually associated with pruritus and is considered peaked contour on fingerpads and replacement with a
as a marker of internal malignancy. hemisphere-like fingertip contour especially on ring fingers.
42. LOVE’S SIGN 47. NAZZARO’S SIGN
Exact localization of tenderness with the help of pin Follicular hairy hyperkeratosis (horny follicular
head in glomus tumor is called as Love’s sign. spicules) commonly located on the face which shows
43. MATCHBOX SIGN compact follicle bound hyperkeratosis is a rare but typical
clinical finding in multiple myeloma and is termed as
Patient having delusions of parasitosis (acarophobia,
Nazzaro’s sign.
entomophobia) collects skin debris with mistaken belief that
such collected material contains alleged parasite in a 48. PASTIA’S SIGN
matchbox, tissue paper, or small container. This whole Linear petechial eruption in the skin folds especially
exercise executed by the patient is referred to as “matchbox on the ante-cubital fossa and axillary fold seen in
sign.” streptococcal scarlet fever is called Pastia’s sign.
44. MEFFERT’S SIGN 49. PATRICK YESUDIAN SIGN
It is described in Fordyce’s disease, characterized by Palmar melanotic macules (palmar freckling) seen in
presence of ectopically located sebaceous glands on the lips, type 1 neurofibromatosis was first reported by Patrick
oral mucosa and less commonly on gums. Prominent lip Yesudian and hence the name.
14 DYP SURVIVAL GUIDE FOR POST GRADUATES DYP SURVIVAL GUIDE FOR POST GRADUATES 15

50. PRAYER SIGN 55. RUSSELL’S SIGN


It is described in relation to diabetic cheiroarthropathy, Crusted callosity on the knuckles of dominant hand
wherein the patient is requested to bring both the palmar due to repeated self-induced vomiting in patients of
surface of the hands together as at prayer. Prayer sign is bulimia.
said to be positive when patient is unable to bring both the 56. SANDWICH SIGN
palmar surface together completely and it indicates limited
In dermatophytosis, fungi are present in the horny
joint mobility. Limited joint mobility is secondary to
layer between two zones of cornified cells, the upper being
nonenzymatic glycosylation of collagen and its deposition
orthokeratotic and lower consisting partially parakeratotic
in the small joints of the hand.
cells.
51. PUNSHI’S SIGN
57. SHUSTER’S SIGN
In young women and girls having from vitiligo the
Scarring of the concha due to lesions of discoid lupus
original white color of vitiligo macules turns to red-pink
erythematosus is called as Shuster’s sign and it can be
during menstruation and after the menstruation, it turns to
present in 30% of the cases.
the original colour
58. SHAWL SIGN
52. PUP-TENT SIGN
Confluent macular violaceous erythema on the
It is seen in nail lichen planus, in which the nail splits
posterior neck and shoulders in patients of dermatomyositis
and elevates longitudinally with downward angle of lateral
is called Shawl sign.
nail edge.
59. UGLY DUCKLING SIGN
53. REVERSE NAMASKAR SIGN
In 1998, Grob and Bonerandi introduced the “ugly
Namaskar’ is the typical Indian way of greeting
duckling” concept to demonstrate that nevi in the same
people, where the forearms are folded in front of the chest
individual tend to resemble one another and that atypical
and the palms are closely approximated together. In patients
mole often deviates from the individual’s nevus pattern. In
with hyperextensible joints as seen in Ehler Danlos
other words, nevus that does not resemble other nevi is
syndrome, they are able to fold their forearms at the back
more likely to be suspicious of melanoma.
and oppose their palms to say “Namaskar,” demonstrating
the hyper extensible joints. 60. VOLCANO SIGN
54. ROPE SIGN It is descriptive term for the morphologic feature of
Old World cutaneous leishmaniasis. The lesion starts as a
It refers to the thick indurated inflammatory cord like
small nontender papule, which enlarges in size and ulcerates
structure that extends from the lateral trunk to the axillae
in the centre. The border of the crusted ulcer often has an
and said to be a classical finding of interstitial
erythematous rim and is called as “Volcano sign.”.
granulomatous dermatitis (Ackerman’s syndrome) with
arthritis. h
1. ANOGENITAL WARTS
Q. What are Anogenital Warts?
A. Anogenital Warts are the most common
manifestations of sexually transmissible infection
caused by Human Papilloma Virus(HPV 6, HPV 11).
Q. What are the genital lesion associated with HPV
infections?
A. 1] Condyloma acuminata- HPV-6, 11
2] Non-condylomatous lesions and/or intraepithelial
neoplasia- HPV 6, 11, 16, 18, 30, 31, 33, 34, 35, 39,
40, 42, 43, 45, 51, 52, 56, 57, 59, 61, 62,64.

LONG CASES 3] Carcinoma- HPV 16, 18, 31, 33, 35, 39, 45, 51, 52,
66, 68
4] Vulvar Papilloma - HPV 70
5] Busckhe Lowenstein tumor - HPV 6,11
Q. How are anogenital HPV subdivided?
A. 1] Low or no oncogenic risk - HPV types 6, 11, 42,
43 and 44.
2] Intermediate risk - HPV types- 31, 33, 35, 51 and 52.
3] High risk - HPV types-16, 18, 45 and 46.
Q. What are the types clinical types of genital warts?
A. 1] Condyloma acuminata.
2] Papular warts.
3] Macular warts.
4] Verruca vulgaris or Keratotic Warts.
5] Sessile Warts.
6] Flat Warts.

16 17
18 DYP SURVIVAL GUIDE FOR POST GRADUATES DYP SURVIVAL GUIDE FOR POST GRADUATES 19

7] Intraepithelial Neoplasia (Bowenoid Papulosis growth that is verrucous carcinoma. This is caused
and Bowen's disease) by HPV-6 and 11.
8] Giant Condyloma (Buschke Lowenstein tumor). Site-Glans and prepuce of an uncircumscribed male;
Q. How anogenital warts are transmitted? less often in perianal skin and vulva.
A. ADULTS - Mostly sexual transmission. It may invade deeply into the underlying dermal
Perianal warts may accompany genital warts, either structures.
due to local spread of infection or to direct contact Q. Describe the clinical appearance of flat warts?
during anal coitus. A. Flat warts are slightly raised, may exhibit an
CHILDREN- Anogenital warts are uncommon in undulating wavy surface. Located anywhere on
children, but their occurrence frequently stimulates genital epithelium.
the possibility of sexual transmission and sexual
On vulval vestibule - velvety, granular, or
abuse.
cobblestone-like surface known as ''vulval
Infection from the mother's genital tract at delivery papillomatosis''. On colposcopic examination, these
is regarded as a frequent source of childhood lesions are described as ''Arizona cactus-like
anogenital warts, probably including those projections'', ''Camel hump like projections'' and
presenting up to 2 years of age. ''Stony colonial pavement-like projections''.
Postnatally, transmission from adults with genital Q. Describe about papular warts?
warts may occur nonsexually such as by sharing a
bath with an infected adult. A. Protruberant, non-pedunculated, dome-shaped, or
hemispherical masses, about 1-4mm in diameter and
Q. Describe the clinical appearance of Condyloma are located in fully keratinized epithelium.
acuminata(acuminate warts)?
Q. Describe about Macular variants?
A. (condyloma = knuckle, acuminatum= pointed; plural
A. Macular lesions are usually found on mucosal
condylomata acuminata). Soft, Pink, Pedunculated
papilliferous masses (cauliflower like) with finger- surfaces, characterized by subtle changes in the colour
of mucosa, a greyish-white colour being most
like peduncles and an irregular surface. Does not
common manifestation. Occassionally, there may be
bleed on touch.
slight capillary punctuation and slight elevation at
Site-Labia minora, introitus, vagina, cervix, anus, the edges of the lesion may be present.
prepuce, frenulum, glans penis, urinary meatus.
To visualize them, 3-5% acetic acid is applied and
Q. What is Giant Condyloma Acuminata? the affected area is covered with a gauze piece soaked
A. Giant Condyloma acuminata, also known as Buschke in acetic acid for 5 mins. Macular lesion are stained
Lowenstein tumor is a rare, aggressive wart-like grayish white. This whitish appearance is attributed
20 DYP SURVIVAL GUIDE FOR POST GRADUATES DYP SURVIVAL GUIDE FOR POST GRADUATES 21

to an overexpression of cytokeratin 10 in suprabasal special situation, includes interferons, 5-flurouracil,


cells. Bleomycin, cidofovir, retinoids, gluteraldehyde,
Q. Describe the histopathology of warts? formaldehyde, cantharidin etc.
Q. Are any vaccine available for HPV infections?
A. Hyperkeratosis of stratum corneum, papillomatosis
and acanthosis of stratum malpighii and thickening A. Yes.
and elongation of rete ridges. Most characteristic Two HPV vaccines are available, both of which offer
feature is presence of epithelial cells with perinuclear protection against the HPV types that cause 70% of
vacoulization known as Koilocytes. There is cervical cancers (i.e., types 16 and 18);
peripheral rim of condensed cytoplasm with 1] The quadrivalent vaccine(Gardasil) protects
hyperchromatic and large nucleus. against the types 6, 11, 16 and 18 of HPV.
Q. What is the treatment of external genital warts? Contains recombinant virus-like particles from
the L1 proteins of HPV-6, 11, 16, and 18.
A. Patient-Applied: Recommended to be given to females between
Podofilox 0.5% solution or gel 9-26years. Given in arm muscle in three doses.
OR Dose 1, Dose 2- 1 month after dose 1, Dose-3-
6months after dose 1.
Imiquimod 5% cream
2] The bivalent vaccine(Cervarix) protects against
OR
HPV 16, 18 types. Contains L-1 protein of viral
Sinecatechins 15% ointment capsid. Vaccine has no live virus so do not infect
Provider-Administered: the patient. Given to females between 10-45 years
Cryotherapy with liquid nitrogen or cryoprobe. of age.
Repeat applications every 1-2 weeks. Cervarix is more effective against HPV than Gardasil.
OR Q. How podophyllotoxin is applied?
Podophyllin resin 10%-25% in a compound tincture A. Podophyllotoxin is a purified extract of
of benzoin Podophyllum plant which binds to cellular
OR microtubules, inhibits mitotic division at metaphase,
and induces necrosis of warts that is 3-5 days after
Trichloroacetic acid (TCA) or Bichloroacetic acid
administration.
(BCA) 80%-90%
OR Podofilox solution(0.5%) or 0.15% cream should be
applied with a cotton swab, or podofilox gel with a
Surgical removal either by tangential scissor excision, finger, to visible genital warts twice a day for 3 days,
tangential shave excision, curettage, or electrosurgery. followed by 4 days of no therapy. This cycle can be
Therapies not generally recommended: used only in repeated, as necessary, for up to four cycles. The total
22 DYP SURVIVAL GUIDE FOR POST GRADUATES DYP SURVIVAL GUIDE FOR POST GRADUATES 23

wart area treated should not exceed 10 cm2, and the persons, immunocompromised persons, or persons
total volume of podofilox should be limited to 0.5 with clinical genital herpes.
mL per day. Mild to moderate pain or local irritation Q. How Podophyllin resin is applied?
might develop after treatment. The safety of podofilox
A. Podophyllin resin 10%-25% should be applied to
during pregnancy has not been established.
each wart and allowed to air-dry before the treated
Q. How imiquimod is applied? area comes into contact with clothing; overapplication
A. Imiquimod is a topically active immune enhancer or failure to air dry causes local irritation caused by
that stimulates production of interferon and other spread of the compound to adjacent areas. The
cytokines. treatment can be repeated weekly, if necessary.
Imiquimod cream should be applied once daily at To avoid the possibility of complications associated
bedtime, three times a week for up to 16 weeks. The with systemic absorption and toxicity, two guidelines
treatment area should be washed with soap and water should be followed:
6-10 hours after the application. Local inflammatory 1) Application should be limited to <0.5 mL of
reactions, including redness, irritation, induration, podophyllin or an area of <10 cm2 of warts per
ulceration/erosions, and vesicles, are common with session, and
the use of imiquimod, and hypopigmentation has 2) The area to which treatment is administered
also been described. should not contain any open lesions or wounds.
Q. How is Sinecatechin ointment applied? The preparation should be thoroughly washed
A. Sinecatechin ointment, a green-tea extract with an off 1-4 hours after application to reduce local
active product (catechins), should be applied three irritation.
times daily (0.5cm strand of ointment to each wart)
h
using a finger to ensure coverage with a thin layer of
ointment until complete clearance of warts. This
product should not be continued for longer than 16
weeks. The medication should not be washed off after
use. Sexual (i.e., genital, anal, or oral) contact should
be avoided while the ointment is on the skin.
The most common side effects of sinecatechins 15%
are erythema, pruritus/burning, pain, ulceration,
edema, induration, and vesicular rash. This
medication may weaken condoms and diaphragms.
The medication is not recommended for HIV-infected
DYP SURVIVAL GUIDE FOR POST GRADUATES 25

Infiltration of granulocytes, Macrophages and lymphocytes.


2. CHANCROID
Q. What is chancroid also known as? INFLAMMATION
A. Soft Chancre, Soft Sore, Ulcus molle Spread of infection causes regional lymphadenitis.
Q. What is causative organism for chancroid? Q. In which conditions do you get kissing ulcers ?
A. Haemophilus ducreyi [Ducrey bacillus].
A. 1] Syphilis.
Q. Describe the biology of organism?
2] Chancroid due to auto-inoculation.
A. Gram-negative, facultative anaerobic coccobacillus
Q. Can extragenital regions also involve in disease?
with tendency to occur in 'short chains' or 'end-to-end
pairs'. May be arranged in small groups or whorls or A. Many cases of extragenital infection on hands, eyelids,
in parallel chains giving a 'School of fish', 'rail-road lips and breasts have been reported.
track' and 'fingerprint' appearance. Q. Describe the classical lesion of chancroid?
Q. What is the incidence of acquiring infection after a A. Intercourse after 1-5 days or rarely as long as 2
single sexual exposure? weeks Papule forms pustules which soon
A. The probability of transmitting chancroid after a ruptures to form a tender non indurated, bleeding,
single sexual exposure is 35% and if left untreated ragged ulcer, with undermined edges.
infected women are estimated to transmit bacilli for Q. What is Chancrous bubo?
about 45 days while the clinical lesions are present.
A. If chancroid is untreated, the site of perforation of
Q. What is the incubation period of chancroid?
broken-down bubo may assume the features of soft
A. 3-10 days. chancre [chancrous bubo].
Q. Does carrier state exists for the disease? Q. What are the types of chancroid?
A. Asymptomatic carrier seems to be rare. A. - Dwarf chancroid(herpetiform ulcers)
Q. Describe the pathogenesis of chancroid? - Giant chancroid
A. Microscopic barrier defect in the epidermis causes
- Transient chancroid
penetration of organism
- Follicular chancroid
- Phagedenic chancroid
Th1-mediated immune response and pyogenic - Pseudogranuloma inguinale
inflammation.
- Serpinginous chancroid
- Mixed chancroid(chancroid with syphilis)
24
26 DYP SURVIVAL GUIDE FOR POST GRADUATES DYP SURVIVAL GUIDE FOR POST GRADUATES 27

Q. What is chancroidal ulcer? Q. What features are seen in biopsies of chancroid?


A. Tender, nonindurated single or multiple large A. Shows three zones of inflammation.
ulcers with no lymphadenopathy, caused by First zone- Necrotic Debris, Fibrin and Neutrophils.
organisms other than H.Ducreyi most commonly
anerobic organisms like Vincent's bacilli. Middle Zone- Area of granulation tissue.
Q. Why is chancroid more common in males? Deepest Zone- Lymphocytes and Plasma cells.
A. Occlusive environment provided by the prepuce Gram negative bacilli are rarely found in tissue gram
makes men more more susceptible to the infection. stain or giemsa stain and are readily found in smears.
Also there is underreporting in women because of Q. What diagnostic modalities are used in diagnosis of
asymptomatic and hidden nature of lesions in
chancroid?
women.
Q. What are the risk factors for chancroid? A. 1] Smears.[Diagnostic in only 50% of cases in best
hands]
A. - Frequent change of sexual partners
2] Culture.[Provides definitive
- Migratory population
- Intensive commercial sexual activity diagnosis] Not used
3] Antigen detection methods. routinely for
- Poor hygiene
diagnosis.
- Alcoholism 4] Molecular diagnostic
- Cocaine abuse techniques.
- Immunosuppression Q. Which culture media are used for diagnosis?
Q. Which coinfections can occur in patients with A. Chocolate Agar, enriched with ISOVITALEX and fetal
chancroid? calf serum, and containing Vancomycin as a selective
A. Co-infection with Herpes Simplex virus, Syphilis, agent with 5-10% CO2 , at a temperature of 33-35
HIV can occur. degree celsius for 24-72 hours.
Q. Does Chancroid increases the chances of acquiring Q. Which is more sensitive than culture?
HIV infection? A. Multiple PCR
A. Yes. The presence of increased number of T Q. What is the time within which the specimens should
lymphocytes [CD4+] and macrophages in ulcers due be sent to microbiology laboratory?
to H.ducreyi provides an ideal opportunity for A. Within 4 hours or directly placed on media since
H.ducreyi is fastidious.
latent HIV infections to become active, with excretion
of virus into ulcer secretions. Genital lesions thus Q. What are the complications of chancroid?
become a portal of entry for non-infected individuals A. 1] Mixed infections causing Phagedenic (spreading
and exit for already infected HIV individuals. or sloughing ulceration) and Gangrenous
features.
28 DYP SURVIVAL GUIDE FOR POST GRADUATES DYP SURVIVAL GUIDE FOR POST GRADUATES 29

2] Secondary bacterial infections Q. Is history of circumcision important in this case?


3) Phimosis/fistula A. Yes. Healing is slower for some uncircumcised men
4) Balanitis, Balanoposthitis who have ulcers under the foreskin.
3] Penile amputation from deep transverse ulceration. Q. Even after 7 days of treatment, if patient doesn't
Q. What are the differential diagnosis of chancroid? improve. What can be the reasons?
A. Genital Herpes, Primary Syphilis, Lymphogranuloma A. 1. Incorrect diagnosis
venerum, Donovanosis, Fixed drug eruption, trauma 2. Coinfection with another STD
and occasionally carcinoma.
3. Patient infected with HIV
Q. What is the treatment of Chancroid?
4. The treatment was not used as instructed
A. Recommended Regimens
5. H. ducreyi strain is resistant to the prescribed
Azithromycin 1 g orally in a single dose
antimicrobial.
OR
Q. How does chancroid in HIV differ?
Ceftriaxone 250 mg intramuscularly (IM) in a single
dose A. - Large size and numbers
OR - Extragenital localization
Ciprofloxacin 500 mg orally twice a day for 3 days* - Longer time to heal
OR h
Erythromycin base 500 mg orally three times a day
for 7 days
* Ciprofloxacin is contraindicated for pregnant and
lactating women.
Q. When will you follow up this patient?
A. On the seventh day. Ulcers start showing
symptomatic improvement in 3 days and substantial
reepithelialisation after 7 days. Larger lesions require
more than 2 weeks.
Q. What is the outcome of lymphadenopathy after
treatment?
A. Resolution is slower compared to ulcers and might
require incision and drainage. (Repeated aspiration
must be done)
DYP SURVIVAL GUIDE FOR POST GRADUATES 31

• h/o dry/ fissured hands


3. DERMATOMYOSITIS
• h/o hard swellings on the body with/without
Name/Age/Sex/Occupation/Marital status/ ulceration/whitish discharge
Residing at/Hailing from….. Systemic symptoms
Chief complaints h/o difficulty in breathing/cough/chest pain
Muscle weakness since……. h/o palpitations/acute chest pain/
Swelling around the eyes since….. h/o regurgitation/bloating/incontinence
Itchy purplish discolouration of skin since…. h/o difficulty in vision
Pink purplish asymptomatic /itchy lesions on
h/o sudden unexplained weight loss/lump in the
elbows/knuckles since…..
breast or abdomen(for malignancy)
Photosensitivity since….
h/o joint pain/swelling
Muscle weakness
h/o any respiratory/gastrointestinal infections
• h/o difficulty in standing from squatting
(childhood DMS)
position/combing hair/getting up from chair/
climbing up stairs history of similar complaints in family
• h/o recurrent episodes of pain in shoulder and For overlap/MCTD
hips(for bursitis which often preceeds myositis) h/o dry mouth/dry eyes
• h/o difficulty in swallowing /pain/regurgitation h/o tightness of skin/raynauds/difficulty in opening
• h/o difficulty in speech mouth
• h/o fever/malaise/tiredness h/o malar rash/oral ulcers/discoid rash
• c/o swelling around eyes,whether present h/o psychosis/seizures
throughout the day/any aggravating/relieving
factors /pigmentation of skin(violaceous/ History of treatment taken for the same
purplish/reddish) Any improvement with treatment
• h/o increased itching/burning/stinging/ h/o any complications with treatment
redness on sun exposure.(photosensitivity)
Past history
• h/o darkening or pigmentation around the neck/
h/o TB/DM/HT/IHD/Bronchial asthma/drug
upper back/chest
allergies
• h/o pink/purple asymptomatic/itchy lesions on
back of fingers/elbow/knee/ankle h/o medical/surgical illnesses
30
32 DYP SURVIVAL GUIDE FOR POST GRADUATES DYP SURVIVAL GUIDE FOR POST GRADUATES 33

QOL : Gottrons sign: Violaceous erythema in the


h/o interfering with work/social/interpersonal/ interdigital spaces of dorsa of hands.
sexual life Calcinosis cutis: Firm yellow/flesh coloured
Personal history : nodules-plaques on bony prominences
Sleep/Bowel/Bladder/Appetite Hypertrichosis/Icthyosis/Lichen planus/Vesicles
Menstrual/Obstetric history Hair - alopecia
Contraception Mucosa-oral
General examination : -genital
General condition Musculoskeletal system
Patient is conscious,cooperative well oriented in time Upper and lower extremities
space and position Power: (grade 0-5)
Pulse/B.P/R.R. Systemic examination :
pallor/cyanosis/icterus/lymphadeopathy/edema RS-crepts/rhonchi
feet CVS- S1S2 normal
B.M.I. CNS
Cutaneous features
P/A
Face: periorbital edema with violaceous hue /
Diagnosis: Dermatomyositis
lilac(heliotrope rash)
Q. Why is it dermatomyositis?
Telangiectasia
A. This is dermatomyositis because of proximal muscle
Neck:poikiloderma
weakness,heliotrope rash,gottrons papules,gottrons
Shawl sign-poikiloderma over the upper back and sign,shawl sign and holsters sign.
shoulders
Q. Any radiological finding specific for
Holsters sign-poikilodermatous changes over waist area. dermatomyositis?
Hands:- A. Calcinosis in muscles and no terminal resorption of
Nails- Ragged cuticles tips of digits(contrast with systemic sclerosis)
- Periungual telangiectasia Q. How do you differentiate it from other myopathies?
Fissuring/scaly/hyperkeratotic/hyperpigmentation A. DMS is more rapid in onset and remissions are
on fingers suggestive of mechanics hands. frequent.
Gottrons papules: Violaceous papules and plaques ESR- Moderately increased
on knuckles/elbow/knee/ankle Serum globulin may be elevated
34 DYP SURVIVAL GUIDE FOR POST GRADUATES DYP SURVIVAL GUIDE FOR POST GRADUATES 35

Q. What are the potential markers of malignancy in Q. What are Gottron’s papules and Gottron’s sign?
DMS? A. Gottrons papules: small, erythematous or violaceous,
A. Elevated ESR and cutaneous necrosis. flat topped papules over knuckles,on the dorsa of
Q. What are markers for overlape syndrome ? finger joints, around nail folds, dorsa of toes, front of
A. ANA, Anti-Rnp, Anti ds DNA, ds dna- for overlap knees,back of elbows.
syndromes Gottrons sign: Rash of dermatomyositis on the dorsa
Q. Heliotrope sign is seen in how many patients? of hands occurs as a linear streaking of violaceous
A. 30-60% erythema (macules) over extensor tendon sheaths or
plaques. May be hyperpigmented in Afro-Carribeans.
Q. Which sign parallels clinical course?
Q. What are the nail changes in DMS?
A. Heliotrope rash : lilac or violaceous, purplish red
erythema over eyelids, upper cheeks, forehead and A. Ragged cuticles (thickening, hyperkeratosis,
temples roughness and irregularity of cuticles)
Q. Why is there a violaceous hue? Diffuse redness and shininess of nail folds
A. Because of mucin deposition and edema,the dermal Dilated,tortuous,irregular capillary loops of nail
vasculature goes deep. Deoxygenation of blood folds
through abnormal dermal vasculature causes tyndall Q. What do capillary changes correlate to?
effect resulting in a violaceous hue.
A. More in Raynauds phenomenon,arthritis and
Q. How will you differentiate rash of SLE from that of
pulmonary involvement.
DMS?
Q. DD of ragged cuticles?
A. Presence of pruritus and involvement of nasolabial
fold is suggestive of DMS. A. - Systemic sclerosis
Q. What is shawl sign and Holster sign? - Lupus erythematosus
A. Shawl sign- Erythematous,poikilodermatous macule - Dermatomyositis
on the v area of the neck.(V sign) Q. What are the less common physical signs associated
Holster sign- Reticulated erythema and edema in with dermatomyositis?
lateral thighs,hips A. 1. Spreading erythema and fleeting or persisting
Q. What is amyopathic dermatomyositis? oedema of the face and neck or of the limbs
A. Only skin rash without muscle involvement (may 2. Urticarial lesions with dermographism
have good prognosis but severe calcinosis can occur)
3. Bullous lesions
Q. In how many patients does the rash precede muscle
weakness? 4. Photosensitivity
A. 50% 5. Erythema nodosum, erythema multiforme
36 DYP SURVIVAL GUIDE FOR POST GRADUATES DYP SURVIVAL GUIDE FOR POST GRADUATES 37

6. Follicular keratoses, PRP like lesions (WONG type) Q. What is sclerodermatomyositis?


7. Hypertrichosis, scalp erythema with hair loss A. This is an overlap syndrome in which cutaneous
8. Hyperhidrosis changes of systemic sclerosis and DMS are associated
with myositis and homogenous nucleolar pattern of
9. Psorasiform eruptions and pitting of the nails ANA but there is no visceral involvement and changes
10. Gingival telangiectasia (childhood disease) of DMS are transient.
11. Linear violaceous itchy and edematous streaks Q. What is the effect of dermatomyositis on pregnancy
on the trunk (flagellate erythema) and vice versa?
12. Exfoliative dermatitis A. Half of pregnancies end prematurely and overall fetal
13. hyperpigmentation, icthyosis, LP, LABD loss is over 50%.Fertility is decreased.
14. livedo reticularis with ulceration Pregnancy should be planned at time of remission
and patient monitored atleast every month. If possible
15. malakoplakia treatment to be confined to oral steroids. Pregnancy
16. anasarca(ominous) exacerbates disease in 50% and remission in 20%.
17. panniculitis Q. What are Mechanics hands?
Q. How is childhood DMS different from adult DMS? A. Hyperkeratotic, scaly, fissured hands, hyperpigmen-
A. Calcification(subcutaneous,intermuscularfascial tation of radial and palmar aspect of fingers.
planes,in panniculitis), vasculitis, hypertrichosis, Correlates with antisynthetase antibodies.
partial lipodystrophy is more common in juvenile Q. How many percentage of patients with dermatomyositis
DMS while malignancy is more common in adult form have Raynaund's phenomenon positive?
Q. What are the variants of juvenile DMS? A. 10%
A. 1. Banker type (widespread vasculitis of skin, Q. Which are the infections associated with DMS?
muscle, subcutaneous tissue, git) A. - Toxoplasmosis
2. Bronstig type (slow course, progressive - Staphylococcal osteomyelitis and arthritis
weakness, calcinosis and steroid responsiveness) - Streptococcal infection (antigenic homology)
Q. What is the differential diagnosis of calcinosis cutis? - Parvovirus B19
A. - Dermatomyositis - Coxsackie B virus
- Systemic sclerosis Q. Which are the autoantibodies found in DMS?
- MCTD A. Jo-1 and PL-12 antibodies are directed against tRNA
- SLE synthetase enzymes and are associated with
38 DYP SURVIVAL GUIDE FOR POST GRADUATES DYP SURVIVAL GUIDE FOR POST GRADUATES 39

interstitial pulmonary fibrosis. This may be detected - USG/CT scan abdomen/pelvis/thorax


before the appearance of the lung disease and Jo-1 - Tumour specific markers including PSA,CEA
may be linked to HLA-DR3.
- Mammography
Q. Which muscles are most affected?
- Pelvic examination
A. Proximal striated muscles of lower limb and upper
limb(less common) specially extensors, limb girdle, Q. Any radiological finding specific for dermatomyositis?
pharynx, tongue. A. Calcinosis in muscles and no terminal resorption of
Q. How will you investigate? tips of digits (contrast with systemic sclerosis)
A. - CBC, leucocytosis/anemia with low serum iron Q. What are the noninvasive tests for DMS?
- ESR may be increased A. - MRI scan
- CPK, CPKMB, Aldolase, lactic dehydrogenase - EMG
- Positive ANA test (60 -80 %) Q. How will you assess disease activity?
- Anti Jo, anti MI2, anti t-RNA synthetase, A. - Serial estimation of the 24hr urinary creatinine.
Anti SRP(signal recognition particle) ACLA(juvenile - MRI of the muscles
dms)ANA, anti DsDNA
Q. D/D of puffy heliotrope eyelids?
- SGOT, SGPT
A. 1. Dermatomyositis
- RFT
2. Systemic sclerosis
- EMG
3. Aldosteronism with adenoma of adrenal glands
- Muscle biopsy and hypokalemia
- MRI scan 4. SLE
- Barium swallow-weak pharyngeal muscles and a
5. Angioedema
collection of barium in the pyriform sinuses
6. EM
Q. How will you investigate a patient suspected to have
malignancy? 7. Erysipelas
A. - History and physical examination (wt, lymph Q. What are D/Ds of myositis?
nodes/breast/pelvis/rectum) A. 1. DMS
- Blood count on biochemistry 2. Systemic sclerosis
- Stool for occult blood Q. What are the lung manifestations of DMS?
- Chest x ray A. Interstitial lung disease, pulmonary hypertension,
40 DYP SURVIVAL GUIDE FOR POST GRADUATES DYP SURVIVAL GUIDE FOR POST GRADUATES 41

pneumonia, interstitial pneumonitis, pneumothorax, dermal oedema maybe more extensive involving all
infections, malignancy, Bronchiolitis obliterans. layers of the dermis, there is usually some
Q. What are the cardiac manifestations? lymphocytic infiltrate, mucin deposits in the dermis
is suggestive of dermatomyositis.
A. Conduction abnormalities,Pericarditis,Pericardial
effusion,arrhythmias,CCF. Late: thickening homogenisation and sclerosis of the
collagen and, thickening of the walls of cutaneous
Q. How is the eye involved in DMS? blood vessels. Atrophic epidermis and increased
A. Iritis, retinitis, cotton wool spots, conjunctival edema, basal layer pigmentation.
nystagmus, optic atrophy, extraocular muscle Q. How will you differentiate skin biopsy of LE and
imbalance. DMS?
Q. What is the significance of arthritis in dermatomyositis? A. DMS shows less eccrine coil involvement and fewer
A. Patients with arthritis frequently have pulmonary vertical coloums of lymphocytes in fibrous tract
involvement remnants.
Q. Which are the malignancies associated with Q. Why is MRI of muscle preferred to a muscle biopsy?
dermatomyositis? A. The muscle involvement in dermatomyositis is not
A. Lung, breast, female genital tract, stomach, rectum, uniform so histology may often be negative so MRI
kidney or testes. is preferred. Also MRI is a non invasive procedure.
Q. What are the factors associated with malignancy? Q. From which muscle should a biopsy be taken?
A. 1. Older age A. 1. From a muscle that is tender and clinically weak.
2. constitutional symptoms 2. Muscle identified as abnormal on EMG/MRI scan
3. Rapid onset of dermatomyositis 3. non atrophic
4. lack of raynauds phenomenon Usually deltoid, trapezius and quadriceps
5. grossly elevated ESR Q. What are the h/p findings of a muscle biopsy in
6. presence of leucocytoclastic vasculitis dermatomyositis?
7. relapse A. Loss of transverse striations, hyalinization of the
sarcoplasm, increase in sarcolemmal nuclei, later the
Q. How many percentage of DMS have malignancy? fragmentation and vacuolar degeneration of fibres
A. 26% and in 40% it precedes neoplasm.Myositis - takes place. Eosinophilic intimal thickening of blood
related antigens are expressed by tumors triggering vessels may be seen.
an autoimmune response directed against muscle. Q. Why is ulceration and hemorrhage seen in skin/fat/
Q. What are the findings in skin biopsy? g i tract?
A. Early: changes resemble subacute LE although the A. Intimal proliferation and thrombosis of arteries and
arterioles of skin, fat and alimentary tract causes
42 DYP SURVIVAL GUIDE FOR POST GRADUATES DYP SURVIVAL GUIDE FOR POST GRADUATES 43

ulceration. - Cyclophosphamide 100mg/day or as pulsed i.v.


Q. Which are the systems involved in dermatomyositis? - IVIG 1 mg/kg for 2 days every month for 4-6 months
A. GIT, lungs, kidney, heart/vasculitis of cerebral vessels - Infliximab (directed against tumour necrosis factor)
Q. Which are the drugs which can cause dermatomyositis?
- Rituximab (directed against cd20 molecule on B
A. Penicillamine,tamoxifen(for ca breast), oral lymphocyte precursors)
progesterone.
- Plasmapheresis
Q. What are the trigger factors of dermatomyositis?
A. Heavy muscular exercise and stress - Dapsone, pentoxifylline and aspirin(for digital
ischemia) have also been tried with some success
Q. What are the sites of oedema?
A. Eyelids, periorbital tissues, hands and arms, - Octreotide (for constipation)
generalized anasarca. - Bisphosphonates and calcium supplements (for
Q. What are the diseases associated with DMS? Osteoporosis)
A. - Thymoma - Aluminium oxide and dilitiazem (for calcinosis)
- Hyperthyroidism - Physiotherapy
- Aplastic anaemia Q. How will you treat the rash of DMS?
- Hemolytic anemia A. 1. Antimalarials (hydroxychloroquine 200mg twice
- Genaralised amyloidosis a day)
- Multiple myeloma 2. Topical calcineurin inhibitors(tacrolimus 0.03-
Q. How do you treat DMS? 0.1%)
A. - Advice rest 3. Topical steroids
- Rule out underlying carcinoma in adults
Q. What are the causes of weakness in DMS?
- 60mg/day prednisolone gradually reduced as a
patient improves clinically and biochemically, A. 1. Myopathy because of DMS
maintenance dose 5-50 mg/day for months/years 2. Steroid induced myopathy
- 1g methylprednisolone i.v. for 3 days (if patient 3. Hypokalemia(serum electrolytes should be
doesn't respond to the above therapy) monitored,potassium supplements to be given)
- Azathioprine 1.5-3mg/kg/day in divided doses Q. How will you differentiate myopathy of DMS from
- Oral methotrexate 7.5-15mg weekly steroid induced myopathy?
Q. What if patient does not respond to above line of A. Serum enzymes will be normal in dms while 24 hr
treatment? urinary creatinine excretion is raised in steroid
- Cyclosporine 5mg/kg/day myopathy.
44 DYP SURVIVAL GUIDE FOR POST GRADUATES

Q. How much time does it take for steroid myopathy


to recover?
4. ERYTHRODERMA
A. Recovery on reduction of dosage takes upto four
Name/Age/ Sex/Occupation/Residing at... /
months.
Hailing from.../Marital status
Q. What are the causes of GI haemorrhage in DMS?
Chief complaints:
A. 1. Disease itself
Redness and scaling all over body since……
2. Steroid therapy
Itching since……
Q. Why can a patient of DMS get osteoporosis?
Fever since….
A. 1. immobility caused by DMS
Edema feet since….
2. steroid therapy
Patient was apparently alright …… back when he
Q. What are the adverse prognostic factors ? developed generalized redness and scaling
A. - Acute pulmonary infiltration on……scalp/limbs/trunk
- Dysphagia Started from…..
- Cutaneous necrosis Spread to …. involve….(was it cephalo caudal)
- Increase in age Gradual/sudden onset, rapidly increased /slow
Q. What are the good prognostic factors? spread
A. - Calcinosis Starting from…cephalocaudal spread? Any areas
- Amyopathic DMS which were spared.(On enquiry)
Q. What are the causes of death in dermatomyositis? How are the scales?
A. 1. respiratory tract infection Type/Colour/Amount(whether handful scales or
2. heart failure bed is full of scales)
3. malnutrition Any associated pruritus/burning/dryness/irritation
4. carcinoma Any preexisting red raised scaly lesions on the body/scalp?
5. weakness or debility because of difficulty in (psoriasis /PRP/Atopic eczema)
swallowing h/o joint pains(psoriasis)
6. side effect of steroid therapy personal/familial h/o bronchial asthma/allergic
rhinitis/allergic conjunctivitis
h
h/o preceding itchy raised red lesions in the flexural
areas(atopic dermatitis)
45
46 DYP SURVIVAL GUIDE FOR POST GRADUATES DYP SURVIVAL GUIDE FOR POST GRADUATES 47

h/o any areas that are spared(Pityriasis rubra pilaris) h/o oliguria, edema feet (renal)
h/o photosensitivity(photodermatitis) h/o diarrhoea (dermatopathic enteropathy)
h/o any exposure to airborne allergens(congress h/o cramps in hands and feet(electrolyte imbalance-
grass) hypokalemia)
h/o weight loss(malignancy) h/o pigmentation,itching/watering(ectropion) if
h/o sexual exposure(HIV erythroderma/Secondary longstanding
syphilis/Reiters disease) Past history
h/o discharge per urethra/diarrhoea (Reiter's Past history similar episode
syndrome)
Past history of illness in detail
h/o any preceding drug taken for any other condition
h/o TB/DM/HT/IHD/Drug allergy/medical/
h/o fluidfilled lesions in the past(Pemphigus
surgical illness
foliaceus)
h/o proximal muscle weakness/rash around the eyes Family history of similar complaints
or photoexposed areas/malar rash with Personal history
photosensitivity(Dermatomyositis and SLE) h/o addictions/Sleep/Bowel/Bladder/Appetite
h/o varicose veins/oozing itchy lesions on the legs Menstrual history
h/o edema feet(stasis dermatitis) Sexual exposure/HIV
h/o fever/illness before episode Quality of life(work/personal)
h/o acute stress General examination
Winter exacerbation/Summer remission
Patients general condition fair
h/o overtreatment with any topical medication
Conscious,cooperative,welloriented in time place
application/withdrawal of any medicines or
drugs(steroids) and person
h/o any systemic medications prior to scaling/ Pallor/Clubbing/ /Icterus/Edema (feet/face)/
phototherapy Lymphadenopathy
h/o response to previous treatment TPR/BP(tachycardia/Poikilothermic/hypotension/
tachypnea)
h/o shivering/feeling of extremes of temperature
(cold)/malaise and bodyache Respiratory rate
h/o shortness of breath or easy fatigability (cardiac Local examination:
failure) Distribution- Face/Trunk/Neck/Upper limb/Lower
h/o increased thirst (dehydration) limb
48 DYP SURVIVAL GUIDE FOR POST GRADUATES DYP SURVIVAL GUIDE FOR POST GRADUATES 49

Arrangement CVS- S1,S2 normal/Increased JVP/tachycardia/High


Morphology output cardiac failure
Face/Trunk/Neck/Upper limb/Lower limb CNS-
Diffuse Erythema and scaling- P/A- soft/No T/G/R, Hepatosplenomegaly
Amount-minimal/copious P/R
Colour(White/Brown) P/V
Types of scale(Adherent) Impression:
Shedding/Adherent Erythroderma secondary to ……with…..metabolic
complications.
More in flexures?
Q. What is the definition of erythroderma?
Any oozing?
A. It is defined as generalized scaling and redness of
Any raw areas?
skin that involves more than 90% of the body surface.
Look for tenderness(TEN,Secondary syphilis) and It was described by Wilson Brocq
Nikolskys sign for Pemphigus foliaceous.
Q. Is erythroderma is the same as exfoliative dermatitis?
Sparing of nose(Pavithrans sign)
A. Yes
Eyes-normal/ectropion/entropion
Q. What are the causes of erythroderma ?
Oral mucosa
A. Preexisting skin diseases:
Hair loss(axillary/inguinal)/Palms and soles
- Dermatitis- Atopic dermatitis, Seborrheic
involvement
dermatitis, Stasis dermatitis
Nails
- Papulosquamous- Psoriasis,PRP/Reiters
Genital mucosa
- Vesiculobullous diseases-Pemphigus foliaceous,
Lymph nodes(to rule out secondary syphilis/ Pemphigus erythematosus/PNP (melanoerythroderma),
malignancy) Hailey Hailey disease,Transient acantholytic
Bones and joints(psoriatic arthritis) dermatosis
Testicular sensation - Collagen vascular disease- Dermatomyositis/SLE
Gynaecomastia-liver failure - Infections-Norwegian scabies, SSSS, Dermatophytosis
Systemic examination /Viral exanthems/Candidiasis/HIV
RS-AEBE,No added sounds - Disorders of keratinization-Dariers/Icthyosis/CBIE
50 DYP SURVIVAL GUIDE FOR POST GRADUATES DYP SURVIVAL GUIDE FOR POST GRADUATES 51

- Drug induced erythroderma Q. What suggests diagnosis of phytophotodermatitis?


- Airborne contact dermatitis A. involvement of retroauricular area, upper eyelid,
- Malignancies (GI, lung, leukemia, lymphoma, submental area suggests phytophotodermatitis
Mycosis Fungoides) (contrast with photodermatitis).
- Mastocytosis Q. What is Alibert -Bazin disease?
- Histiocytosis A. Classical MF characterized by typical evolution of
- Sarcoidosis patches,plaques and tumors or a clinicopathologic
variants showing a similar clinical course.
- Graft Versus Host disease
Q. What is pagetoid reticulosis?
- Papuloerythroderma of ofuji
A. Woringer -Kolopp disease
- Nutritional(Zinc/Biotin)
Q. What is Erythrodermic CTCL (E-CTCL):
- Immunodeficiency disorders(Job’s syndrome)
A. A variant of CTCL characterized by chronic
- Metabolic(lipid storage diseases)
erythroderma and three types
- Systemic contact dermatitis(neomycin cream or
silversulphadiazine cream) 1. SS: A leukemic expression of E-CTCL
characterized by numerous Sézary cells in
Q. Why is it not icchthyosiform erythroderma? the skin, blood, and other tissues, typically
A. Absence of history of blisters, nail dystrophy, with evidence of T-cell clonality.
ectropion.
May include lymphadenopathy,
Q. What is Leiner’s disease (erythroderma desquativum/ hepatosplenomegaly, palmoplantar
red scaly syndrome)? keratoderma, ectropion, and alopecia.may
A. - Failiure to thrive develop de novo (classic SS), but may be
- Low birth weight preceded by a prodrome of pruritus alone
or nonspecific dermatitis.
- Diarrhoea
- Erythroderma-lesions similar to seborrheic 2. E-MF: E-CTCL lacking the blood findings of SS
dermatitis that develops during the course of disease
in patients with otherwise clinically typical
- C5 deficiency: MF.
1. In a case of erythroderma if bouts of fever with
3. E-CTCL, not otherwise specified (NOS):
chills with itchy lesions over the face (Kaposis
varicelliform eruption [Kve]in atopic dermatitis) Cases of E-CTCL that fail to fulfill the
diagnostic criteria for either E-MF or SS as
2. h/o painful lesions
defined above.
3. h/o joint pains
52 DYP SURVIVAL GUIDE FOR POST GRADUATES DYP SURVIVAL GUIDE FOR POST GRADUATES 53

Q. What is Pseudo-CTCL erythroderma? Q. Where do you get Lutzner cells?


A. Benign non-malignant inflammatory conditions A. - actinic reticuloid,
characterized by erythroderma and other findings that - certain drug eruptions
suggest the diagnosis of CTCL. Typically this refers
- severe eczemas
to compatible skin histology, but occasionally cases
of actinic reticuloid and certain drug eruptions may Q. Types of sézary cell?
have increased numbers of Sézary cells in the blood A. A. Small Sézary cell: A Sézary cell with cell
that are suspected to be SS. diameter < 12 microns, that is, the size of a normal
Q. What are the types of SS? resting lymphocyte
A. Small cell variant of SS when more than 80% of the B. Large Sézary cell: A Sézary cell with cell
Sézary cells are small (<12 micron in diameter) and diameter 12 microns, that is, larger than the size
large-cell variant of SS when 20% or more of the Sézary of a normal resting lymphocyte
cells are very large (>14 micron in diameter). mixed C. A very large Sézary cell with cell diameter > 14
cell variant is the one with an intermediate picture. microns is clearly neoplastic
Q. What is Russell-Jones and Whittaker criteria for SS? Q. What are the drugs commonly causing
A. erythroderma, compatible skin histology, more than erythroderma?
5% circulating mononuclear cells (Sézary cells), and A. Anticonvulsants/Antituberculosi sdrugs/
evidence of a peripheral blood T-cell clone by means sulphonamides/ cephalosporins/vancomycin/
of at least one of the following specific tests: fluoroquinolones/NSAIDS/allopurinol/Muscle
- Very large Sézary cells (>14 micron diameter) relaxants
- Cytogenetic evidence of an abnormal clone Q. How do you differentiate acute from chronic
- Loss of pan-T cell antigen by immunophenotyping erythroderma?
- T-cell clone shown by Southern blot or PCR analysis A. In acute stages scales are large and crusted, while in
Q. What is a Sézary cell? chronic stage the scales are small and dry.

A. An atypical lymphocyte with a moderately to highly Q. An erythrodermic patient with lymphadenopathy


infolded or grooved nucleus (eg, a cerebriform and hepatosplenomegaly, What are the possibilities?
lymphocyte with nuclear contour index of 6.5 or A. Particularly in association with liver dysfunction and
more). Sézary cells, mycosis cells, Lutzner cells, and fever, may suggest a drug hypersensitivity syndrome
cerebriform lymphocytes are synonymous terms. or malignancy.
54 DYP SURVIVAL GUIDE FOR POST GRADUATES DYP SURVIVAL GUIDE FOR POST GRADUATES 55

Q. Enumerate different types of scales seen in various intracellular compartment to the extracellular
disorders compartment
A. 1.) Micaceous scales - Psoriasis 2.) Proteins are lost in scales resulting in
2.) Branny (furfuraceous) scales - Pityriasis hypoalbuminemia.
Versicolor 3.) High-output cardiac failure
3.) Collarette of scales - Pityriasis Rosea 4.) Inflammation resulting from primary skin
4.) Wafer like scale - Pityriasis lichenoides chronica disease
5.) Fish like scales - Ichthyosis 5.) Renal involvement
6.) Double edged scale - Nethertons syndrome Q. What is the normal requirement of proteins?
7.) Carpet tack like scales - Discoid lupus A. 1gm/kg/day
erythematosus Q. How many scales and proteins are lost in
Q. Why does one lose hair in erythroderma? erythroderma?
A. In erythroderma, A. 20-30 gms of scales(9gms/metersquare/day) and 200
1. scales are lost which results in sulphur loss. gms proteins per day.
(Sulphur is needed for hail and nail growth), Q. Why hypoalbuminemia?
hence it is attributed in 50% of cases.
A. - Decreased synthesis of albumin?
2. Hypoproteinemia
- Increased metabolism of albumin
3. Hyperestrogenemia
- Proteins lost in scales
Q. What are Shoreline nails?
Q. Why does a patient get gynaecomastia?
A. Transverse line of discontinuity preceeded by
A. Decreased perfusion in liver results in increased
transverse leuconychia seen in drug-induced
estrogens which causes gynaecomastia.
erythroderma.
Q. What is ebonisation? Q. What is the rule of 9?
A. Shiny nails because of constant rubbing A. Face-9%
Q. What are the other nail changes? Upper limbs-9+9%
A. Nails become thickened ,ridged and may be shed. Chest-18%
Beaus lines may be seen. Back - 18%
Q. Why does one get oedema in this condition? Lower limbs- 18+18%
A. 1.) Vasodilatation resulting in fluid seeping from Genitals-1%
56 DYP SURVIVAL GUIDE FOR POST GRADUATES DYP SURVIVAL GUIDE FOR POST GRADUATES 57

Q. What % of body area is involved in erythroderma? Q. What are the characteristics of pemphigus foliaceous?
A. >90% BSA involved A. Cornflake like scale crusts in seborrheic distribution
Q. Which are the common types of eczema notorious usually without mucosal involvement and mousy
to cause erythroderma? odour
A. - Atopic dermatitis Q. What is Norwegian scabies?
- Seborrheic dermatitis A. Hyperkeratotic crusted lesions all over the body in
immunocompromised and debilitated people.
- Airborne contact dermatitis,phytophotodermatitis
Q. What are the causes of erythroderma in neonates?
- Stasis dermatitis
A. 1. Congenital bullous and nonbullous icthyosiform
- Irritant and allergic contact dermatitis
erythroderma
Q. Most common cause in india?
2. Congenital syphilis
A. Psoriasis and phytophotodermatitis
3. Leiners disease
Q. Which drugs can precipitate erythroderma in
4. Infantile seborrheic dermatitis
psoriasis?
Q. What are the causes of collodion baby?
A. - Overtreatment with tar,salicylic acid and dithranol
A. 1. Non bullous icthyosiform erythroderma
- Phototherapy in unstable psoriasis
2. Lamellar icthyosis
- Gold, lithium, Cimetidine, Carbamazepine,
Omeprazole, Hydantoin 3. Sjogren- Larsson syndrome
- Beta blockers 4. Conradi-Hunnermann syndrome
Q. Most common causes of erythroderma in children? 5. Netherton syndrome
A. - Atopic dermatitis Q. What are the uncommon causes of erythroderma?
- Congenital bullous icthyosiform erythroderma A. - Sarcoidosis
- Collodion baby - Sezary syndrome
- Candidiasis - Actinic reticuloid (chronic actinic dermatitis)
- Congenital syphilis - HIV
- Leiners disease(C5 deficiency) - Mycosis fungoides
- Mastocytosis, Histiocytosis - Graft versus host disease
Q. In AIDS when does a patient get erythroderma? Q. What is postoperative erythroderma?
A. Seroconversion phase and late stage A. A type of graft-versus-host disease following surgery
along with blood transfusion, is marked by
58 DYP SURVIVAL GUIDE FOR POST GRADUATES DYP SURVIVAL GUIDE FOR POST GRADUATES 59

erythroderma, fever, pancytopenia, hepatic 2.) Blood flow through skin is increased resulting
insufficiency and diarrhea and may be fatal. in high output cardiac failure.
Q. What is Pavithran sign? 3.) Increased skin perfusion leading to hypothermia
A. Sparing of nose and paranasal areas in erythroderma 4.) Excessive loss of heat leads to compensatory
Q. What is dermatopathic lymphadenopathy ? hypermetabolism and increased BMR.
(lipomelanotic reticulosis) 5.) Loss of proteins in scales-hypoproteinemia
A. Reactive lymphadenopathy because of macrophages 6.) Barrier function is lost leading to cutaneous,
which collect melanin from dermis and take it to subcutaneous and respiratory infections
lymphnode. 7.) Death in 7% most commonly due to pneumonia
Q. What is Parklands formula? Q. What is the effect of erythroderma on quality of life?
A. Input=4ml/kg × BSA involved for burns A. Since it's a chronic disease,patient can get depression.
In TEN ¾ of this is given(Ringer lactate) Also because of decreased circulation to the brain,
One half in the first 8 hours patients having borderline psychosis can manifest
overt signs. If steroids are given, they can also cause
Other half over next 16 hrs
psychosis.
Q. What are the complications of erythroderma?
Q. What should be the humidity of the room?
A.
A. 70%
Cutaneous Systemic
Q. What is the normal core body temperature?
Hair changes Hypothermia
A. The normal core body temperature of a healthy,
Nail changes Hypoproteinemia resting adult human being is stated to be at 98.6
Epiphora Anaemia degrees fahrenheit or 37.0 degrees celsius.
Ectropion Renal failure Q. What is the axillary,oral and rectal temperature?
Pruritus Dermatopathic enteropathy A. Oral temperatures, which are the most convenient
TEWL High Output cardiac failure type of temperature measurement, is at 37.0 °C.
Axillary temperatures is the longest and most
Dry and fissured palms Acute skin failure
inaccurate way of measuring body temperature, the
and soles normal temperature falls at 97.6 °F or 36.4 °C. Rectal
Q. What are the reasons for the above systemic temperatures fall at 99.6 °F or 37.6 °C. It is the least
complications? time consuming and most accurate type of body
A. 1.) Increased TEWL causes fluid and electrolyte temperature measurement, being an internal
imbalance which results in pre renal failure. measurement but not comfortable.
60 DYP SURVIVAL GUIDE FOR POST GRADUATES DYP SURVIVAL GUIDE FOR POST GRADUATES 61

Q. Is erythroderma infectious? There is non-specific infiltrate, vasodilatation and


A. Scales harbor bacteria which thus serve as a source perivascular infiltrates.
of infection to others. It is more prone for infections In chronic cases there is acanthosis, elongation of rete
because barrier function of skin is lost and immune ridges. Histopathology can help identify the cause
responses are altered.(CD4 T cells lymphocytopenia in 50% of cases.
and increased gammaglobulin) Q. What is the significance of eosinophilia?
Q. What is Concealed pyrexia? A. It is generally a nonspecific finding, although a highly
elevated count raises the possibility of a lymphoma
A. In erythroderma, body feels warm because of
or drug hypersensitivity syndrome.
increased peripheral skin perfusion and so even if
patient gets fever,its not noticed. This is concealed Q. What are Sezary cells?
pyrexia.Therfore rectal temperature must be taken. A. Circulating Sezary cells may be present; but whereas
Q. Why is barrier nursing important in erythroderma? less than 10% is considered nonspecific in the setting
of erythroderma, the presence of 20% or more raises
A. Scales harbor bacteria which thus serve as a source the suspicion for Sezary syndrome.
of infection to others.
Q. What is sezary syndrome?
Q. What is the pathogenesis of erythroderma? A. It is a lymphoma of skin homing T-cells characterised
A. The number of germinative cells as well as their by erythroderma, lymphadenopathy and Sezary cells
mitotic rates is increased, meanwhile their transit time in blood.
through the epidermis is decreased. Q. In which circumstances DIF should be done?
Q. What are the investigations in erythroderma? A. Direct immunofluorescence studies may be of
A. 1.) CBC - Anemia due to chronic disease diagnostic utility in cases of erythroderma secondary
leukocytosis with eosinophilia, to pemphigus foliaceous, bullous pemphigoid, graft-
versus-host disease and connective tissue disorders.
2.) ESR - increased
Q. What is red man syndrome?
3.) Serum Albumin - hypoalbuminemia
A. It is characterized by dermatopathic
4.) Serum electrolytes - electrolyte imbalance lymphadenopathy, palmoplantar keratoderma and
5.) Serum IgE - increased in atopic erythroderma, relapsing erythroderma.
6) increased uric acid, abnormal serum protein Q. What is Omenn syndrome?
electrophoresis with elevation in the gamma A. It is characterized by immunodeficiency, failure to
globulin thrive, hypogammaglobinemia, increased IgE,
6.) Histopathology - In acute stages there is erythroderma and diffuse alopecia.Treatment is by
spongiosis and parakeratosis. bone marrow transplantation It is fatal if untreated.
62 DYP SURVIVAL GUIDE FOR POST GRADUATES DYP SURVIVAL GUIDE FOR POST GRADUATES 63

Q. What are the types of scales seen in erythroderma? The diagnosis of underlying cause maybe very
A. Fine in atopic dermatistis,bran like in seborreic difficult.The history and multiple biopsies is often
dermatitis,crusted in pemphigus foliaceous and helpful.
exfoliative in drug reaction Q. How will u manage a case of psoriatic erythroderma?
Q. What is pavitran sign? A. In psoriatic erythroderma low dose methotrexate ,
A. There is sparing of nose in erythroderma. acitretin or ciclosporin can be given.avoid topical tar
and uv therapy.
Q. How will u manage a case of erythroderma?
Q. What is deckchair sign?
A. Hospitalize the patient.Initial management consists
A. Papuloerythroderma of Ofuji presents in elderly men
of correction of fluid and electrolyte imbalance and
as flat-topped red pruritic papules that become
treatment of secondary infections.The environmental
generalized erythrodermic plaques. There is sparing
temperature must be regulated.Cooling and
of abdominal skin folds ("deck chair" sign)
overheating must be avoided by use of extra blankets
and fans respectively.The possibility that the Q. What is emulsifying ointment BP?
erythroderma is due to drug reaction must be A. Emulsifying wax - 20gms
considered in every case and all non essential drugs Liquid paraffin - 30gms
must be withdrawn.Sedating oral anti histaminics will
White soft paraffin - 50 gms
help to ease the pruritus.The cutaneous inflammation
should be treated with blande emollient creams or It is a good moisturizing agent. It can be used by
making a soapy solution in warm water which can
mild topical steroids.Systemic steroids maybe
be used in place of soap for bathing.
necessary in idiopathic erythroderma and drug
reactions. Prednisone can be given in a dose of 1 - 2 Q. How will you treat sezary syndrome ?
mg/kg/d and a maintainence dose of 0.5mg/kg/d. A. - Retinoids
Due to increased transcutaneous absorption topical - Bexarotene
salicylic acid or lactic acid should be avoided. In
- Interferon
refractory cases cyclosporine( 5mg/kg/d), retinoids,
vitamin-A, methotrexate and azathioprine can be - Extrocorporeal photopheresis
given. - GM-CSF
Many dermatologists prefer to avoid systemic Q. What suggests diagnosis of phytophotodermatitis?
steroids due to chances of fluid retention secondary A. Involvement of retroauricular area, upper eyelid,
infection diabetes but in persistent cases it may submental area suggests phytophotodermatitis
become necessary. (contrast with photodermatitis).
64 DYP SURVIVAL GUIDE FOR POST GRADUATES

Q. How will you investigate?


5. GENITAL ULCER
A. - Peripheral smear for abnormal lymphocytes (sezary
cells)
Name/Age/ Sex/Occupation/Residing at... /
- Skin biopsy Hailing from.../Marital status
- Chest xray(malignancy/kochs) Chief complaints
- Tb quantiferon gold test HOPI:
- Mantoux test Urethral / vaginal discharge
- Serum IgE levels (atopic dermatitis) Urinary complaints
Q. Why should you examine the eyes in erythroderma? Genital raw areas/sores (GUD)
A. To rule out cataract in atopic dermatitis as a cause of Groin swelling (lymphadenopathy)
erythroderma
H/o constitutional symptoms
Q. What is Jobs syndrome?
joint pains (Reiter's, disseminated gonococcal
A. recurrent staphylococcal infections of skin and lungs, infections), fever
(beginning in infancy ) Recurrent pyogenic associated complaints: h/o skin rash for S2
pneumonias, mucocutaneous candidiasis, facial
asymmetry, prominent forehead, deep-set eyes, a conjunctivitis (red eye)/uveitis
broad nasal bridge, mild prognathism, and rough urethral discharge since….
appearance of the facial skin with prominent pores Amount/colour/smell/site/irritation/relation with
arterial aneurysms and associated with Hodgkin and intercourse/bloodstained/threads
non-Hodgkin lymphomas, leukemia, and cancers of Vaginal discharge since…….
the vulva, liver and lung Amount/Colour/Odour/Itching/irri tati on/
Q. What is acute skin failure? bloodstained/relationship with menses and
A. Skin failure has been defined as loss of normal intercourse.
temperature control with inability to maintain the Genital sore
core body temperature, and failure to prevent Since……
percutaneous loss of fluid, electrolytes and protein, Number/Site/Size/Pain/Discharge/Colour
with resulting imbalance, and failure of the
mechanical barrier to prevent penetration of foreign Started initially as what?
materials. h/o preceding fluid filled lesions
Progression in size/morphology
h
H/o preceding drug intake
55
66 DYP SURVIVAL GUIDE FOR POST GRADUATES DYP SURVIVAL GUIDE FOR POST GRADUATES 67

h/o mucosal membrane involvement: oral/rectal/ Swelling of the penis/scrotum


conjunctival. Foul smelling discharge with black colour
H/o application of medications H/O etiology
GROIN SWELLING Syphilis-Pri mary (painless ulce r) / shotty
Bilateral/Unilateral lymphadenopathy
Number / Site / Size / Pain / Overlying skin / Secondary-Skin lesions/Oral lesions/Painless
Associated ulcer lymphadenopathy
Associated complaints in systemic Tertiary-Tabes dorsalis/neurological CVS
Fever/arthralgia/myalgia/eye complaints/ complications
abdominal pain/headache Chancroid- Painful/ multiple/Extragenital/
H/O OF EXPOSURE: Lymphadenopathy
1st exposure: protected/unprotected with sex worker LGV- Swelling of scrotum/leg/vulva/groin
or known person - Ribbon like stools/fissures/ulcers
Last exposure: same as above - Ophthalmic complaints
h/o of peno-vaginal/peno-oral/peno-anal exposure Donovanosis
h/o condom used-regular/occasional Beefy red/chronic
h/o breakage of condom Scabies-h/o itching in the groins with crawling
h/o multiple sex partner sensation.
h/o trauma PAST HISTORY
h/o drug intake Past h/o of STD or similar complaints
h/o fever ,loss of wt/loss of appetite. What type?
h/o of joint pain If treated or not
h/o of itching all over the body HIV- Known serostatus
H/O OF COMPLICATION: - Weight loss/Diarrhoea/Recurrent URTI
If discharge-abdominal pain/obstruction/stools - Herpes zoster/Tuberculosis/Bedridden/Cachexia
If ulcer-extent/bleeding/nonhealing lesion - H/O IV drug abuse
If discharge-burning/fever/chills Inability to retract - h/o blood transfusion/needle prick
the prepucial skin Any h/o jaundice(HBV)
68 DYP SURVIVAL GUIDE FOR POST GRADUATES DYP SURVIVAL GUIDE FOR POST GRADUATES 69

TB/DM/HT/IHD/Bronchial Asthma/drug allergy - wt/ht/tpr/bp


H/o medical and surgical illnesses - pallor, cyanosis ,clubbing icterus, edema feet,
SEXUAL HISTORY: clubbing
marital status/extramarital/ number of partners - lymphadenopathy(mental/submental/occiput/
axilla/epitrochlear/inguinal
Mode of intercourse(penovaginal/peno-anal/peno-
oral/oral sex) - Unilateral (lgv/chancroid)
Contraception-always/frequent/rare - Any rash/colour of the rash
Last exposure was how many days back - Ichthyosis
In females: - Wasting
Menstrual history - Pigmentation
irregularity/flow/painful/intermenstrual bleeding/ - Herpes zoster scar
dyspareunia - Postinflammatory hyperpigmentation
Obstetric history: - Any infection in the skin
Gravida/Para/Livebirths/Abortion - diffuse hyperpigmentation/hair loss
Infertility(PID) - Oral cavity
Abortion - Hair Palms and soles
stillbirth - Eyes(conjuctival redness, watering of eyes)
PERSONAL HISTORY: - Per vaginum
Sleep/ bowel-bladder/ addiction - Per rectum-Local examination
Family history: - Pubic hair/Nits(Pthirus pubis)/Folliculitis
H/o any complaints in the family - Groin-Lymphadenopathy
H/o any similar complaints in the past Bilateral/Unilateral,discrete/matted
H/o blood transfusion in wife Which group of lymph nodes are involved?
H/o itching(scabies/pediculosis) Shape/Size/Tenderness/Fluctuation/
Obstetric history in wife Warmth/Consistency(firm/hard/shotty)
General Examination Impulse on coughing
General condition fair Skin overlying the lymph node: normal/
sinuses/bluish/erythematous/Ruptured
Patient is conscious/cooperative,well oriented in
buboes
time space and person
70 DYP SURVIVAL GUIDE FOR POST GRADUATES DYP SURVIVAL GUIDE FOR POST GRADUATES 71

Ulcer: multiple, firm, discrete, mobile, non tender, move


Number/Site/Shape/Edge/Floor/Base/Margin// freely under finger (shotty, rubbery)
Painful/tender/smell Chancroid: Soft tender papule breaks down to form
Discharge- Purulent/Serous/Serosanguinous/Blood an ulcer,multiple 1-2 mm ulcers with sharp ragged,
undermined edge, nonindurated ,painful base with
Staining of undergarments
yellow gray necrotic exudate covering granulation
Discharge-Site(Urethral/Vaginal) tissue which bleeds easily.
- Retract prepuce Edema of prepuce present or not
- Colour/Amount/smell/adherent/meatal Unilateral tender fluctuant warm mass in inguinal
redness. region of ……size with normal overlying skin.
- Milk urethra Syphilis
Scrotum/testes/epididymis/spermatic cord/
cervix/vagina/bartholin/fourchette Q. what was syphilis earlier called ?
Perianal area-patulous A. The Great pox , because of its rapid spread and
considerable effects throughout Europe.
- Warts/skin tags/Sphincter elasticity/fissures
Q. How is the name of syphilis derived?
- Scars
A. It is derived from the poem by Fracastorius about
- BMPE(Bimanual pelvic examination) afflicted shepherd named syphilis who was
P/A- soft immortalized in a poem named "syphilis got the
Liver/Spleen French disease"
Tenderness q. What is the other name for syphilis?
RS Lues (means pest in french)
Cvs Q. When is a syphilis patient infectious?
CNS A. During the first year of untreated infection.
Imp- Sexually transmitted disease with complications Q. What is the rate of acquisition of syphilis from an
with HIV status,VDRL status/HBV in a high risk infected sexual partner after a single sexual contact
patient A. 30%.
Syphilis: Single painless rounded well defined Q. What is the risk of mother infecting foetus with
indurated button like induration on the ulcer with subsequent pregnancies?
dull red clean granulation tissue with serous A. Patient is infectious for many years , although risk of
discharge on ….site Lymph nodes : bilateral,
72 DYP SURVIVAL GUIDE FOR POST GRADUATES DYP SURVIVAL GUIDE FOR POST GRADUATES 73

infecting foetus declines gradually during untreated kissing ulcers.


illness after 8 years there is little risk even in Q. What is condom chancre?
untreated mother . A. An ulcer occurring at the hilt of shaft of penis is called
Q. Why is the incubation period of syphilis so long? ( condom chancre.
9- 90 days) Q. What are the usual sites of chancre?
A. As the doubling time of Treponema Pallidum for each A. Males- Coronal sulcus, Glans, Shaft, Prepuce,
cell division is 33 hours. frenulum, urinary meatus
Q. How will we differentiate between syphilitic chancre Females-vulva, vagina or cervix
and chancroid. Extra genital sites (12-14%) - anorectal (receptive anal
A. sex),oral cavity, hands and arms, rarely as paronychia.
Syphilitic chancre Chancroid Q. What is chancre redux (monorecidive)?
(Hunterian chancre) A. Recurrence of a primary chancre at the site of original
Painless Painful lesion.
Usually single Multiple Q. What is pseudochancre redux?
Clear ham coloured base Necrotic base A. Also occurs at the site of the original lesion but unlike
true chancre redux, it is non-infectious and is a
No exudate Yellow exudate granulomatous, non-infectious lesion of late syphilis.
Button hole induration - Q. What is Syphilitic balanitis of Follmann?
Rolled edges Jagged edges. A. Inflammatory reaction on the glans penis in primary
Bilateral inguinal Unilateral inguinal syphilis to treponema pallidum. A superficial
lymphadenopathy lymphadenopathy balanitis in which treponemes are seen only in
epidermal cell layers.
Overlying skin normal Overlying skin thin and shiny
Q. What is syphilis d'emblee?
Q. What is Dory flop sign? A. Syphillis without chancre as a result of direct
A. The induration of the ulcer on the inner side of the inoculation into the blood as a result of transfusion
prepuce often gives rise to flip on retraction of the into the infected blood or blood component or
prepucial skin. This is Dory flop sign. (dory-a small punctured with an infective needle.
flat bottomed boat) Q. What are the complications of primary chancre?
Q. What are kissing ulcers? A. Oedema, phimosis, erosive balanitis, lymphangitis,
A. Ulcers on the coronal sulcus and prepuce are called phagedenic chancre.
74 DYP SURVIVAL GUIDE FOR POST GRADUATES DYP SURVIVAL GUIDE FOR POST GRADUATES 75

Q. What is phagedenic chancre? Q. What is necklace/collar of venus?


A. Phagedenic chancre is due to coinfection with A. Resolving rash of secondary syphilis which leaves
fusospirochaetes charecterised by necrotizing depigmented patches on the back and sides of the
perforation of prepuce or in some patients gangrene neck referred as leucoderma syphiliticum.
Q. What are the non STD causes of genital ulcers ? Q. Which variants of syphillis are associated with
A. Fixed drug eruption, squamous cell carcinoma, itching ?
Behcets disease, Crohn's disease. A. Follicular syphillis
Q. Which is an important diagnostic clue in patients Lichenoid syphillis
presenting with rash of secondary syphilis?
Q. Enumerate the morphological variant of syphillis?
A. A thin depressed scar seen in 10% patients on the
A. Macular or roseolar/ maculopapular rash
site of pre-existing ulcer.
- Papular(Nickel and Dime)
Q. In how many patients the chancre persists even after
the appearance of secondary rash? - Papulosquamous
A. 10-40% - Psorasiform
Q. Which are the most common manifestations of - Annular
secondary syphillis? - Pustular
A. Rash(raw-ham or copper coloured) - Follicular
Lymphadenopathy - Lenticular
Q. What is roseolla syphilitica? - Corymbose
A. Macular eruptions consisting of 0.5 to 2 cms pink, - Lichenoid
discrete, round or oval over trunk, flexor aspect of
- Rupioid
upper extremities and palms and soles which are
transient. - Nodular(seen in late syphilis more than
Q. What is Corona veneris? secondary syphilis)
A. Papules along the hairline on forehead arranged in a Ulcerative or necrotic(lues maligna)
crown like pattern Q. Describe rash of secondary syphilis?
Q. What is Buschke Ollendorf sign? A. Generalised rash of skin and mucous membranes,
A. Deep dermal tenderness elicited by wincing on bilateral and symmetrical, more prominent on upper
applying pressure with a blunt end of a pin over a limbs than on abdomen and lower extremities with
papule. Also seen with furuncle, vasculitis, erythema special predilection for palms and soles,more
nodosum etc. discrete than confluent, sharply demarcated and
76 DYP SURVIVAL GUIDE FOR POST GRADUATES DYP SURVIVAL GUIDE FOR POST GRADUATES 77

having a coppery hue with deep dermal tenderness Q. How do we differentiate condyloma lata with
positivity. condyloma acuminata?
Q. What is lichenoid syphilis? Condyloma lata Condyloma acuminata
A. Characterised by maculopapular lesions with Pale Pinkish
lichenoid(violaceous)hue with increased incidence Flat topped Thrown into folds
now being attributed to HIV. Moist Dry
Q. What is lenticular syphilid? Broad at the base so a paper Narrower at the base so a
A. Pinhead to lentil bean sized papules on face and cannot be passed under it paper can be passed under
genitalia. it (paper test)
Bilateral, firm, rubbery, No lymphadenopathy
Q. Which are the most common sites of annular syphilis?
shotty, painless discrete
A. Face,anogenital area, axillae, palms and soles.
and non tender
Q. What are bombshell like eruptions?
Dark ground illumination Negative DGI
A. They are corymbose lesions characterised by a central
positive
plaque and surrounding smaller satellite papules
Caused by treponema Caused by human
which occurs months after infection.
palladum papilloma virus
Q. What is syphilis cornee?
A. Corn like lesions over soles. Q. What are split papules?
Q. What is Clous syphilitiques? A. Split papules are lesions similar to condyloma lata
which become fissured at the labial commissures,
A. Diffuse hyperkeratosis of soles seen in syphilis. nasolabial folds,behind ears,axilla, inframammary
Q. What is Biette's collarette area.
A. A zone of scaling surrounding papules of syphilis Q. Why are the mucosal lesions morphologically
on palms and soles. different from the rash on the body?
Q. What is the initial finding in secondary syphilis? A. Local irritation, moisture and trauma modify the
morphology of rash at mucosal sites.
A. A coppery evanescent macular rash often overlooked.
Q. What are snail track ulcers?
Q. Describe mucous patches?
A. They are serpignious ulcers in the mucosa.
A. large greyish plaques or small superficial ulcerated Q. Why are mucosal lesions more common in women?
areas with greyish borders.
A. They are due to moisture friction and some irritation
78 DYP SURVIVAL GUIDE FOR POST GRADUATES DYP SURVIVAL GUIDE FOR POST GRADUATES 79

Q. What are the types of hair loss seen in secondary Q. Differential diagnosis of fever with sore throat and
syphillis? adenopathy?
A. Irregular patchy non-scarring alopecia (moth eaten) A. Infectous mononucleosis , hiv , secondary syphilis
Telogen effluvium (diffuse pharyngitis in secondary syphilis)
Q. Where is moth eaten alopecia observed? Q. What do you mean by latent syphilis ?
A. Margins of the scalp, beard, eyebrows, legs A. Serological or historical evidence syphilis but who
Q. Nail changes in secondary syphillis? has never received treatment for this disease and who
A. Pitting, onycholysis, onychodystrophy, beaus lines, have no clinical manifestations.
paronychia,onychia Q. What are the types of latent syphilis?
Q. What are the CSF findings in secondary syphillis?
A. Early-Potentially infectious and defined by one of the
A. 1/4th of the patients with early syphillis regardless
following
of HIV status show elevated wbc count and proteins
in CSF with a reactive VDRL 1. Documented seroconversion
Q. Why does a patient get syphilitic periosteitis? 2. Unequivocal symptoms of primary or secondary
A. It is due to vasculitis syphilis
Q. Which lymph nodes are commonly involved ? 3. Sexual partner documented to have primary or
A. inguinal nodes - 75% secondary or early latent syphilis.
axillary - 38% Late- More than 1 year
posterior cervical - 28% On the basis of time when untreated individuals
femoral - 18% likely to have spontaneous mucocutaneous relapses
epitrochlear - 17% Q. What is interval from acute infection to clinical onset
Q. In which bones will you see peri-osteal inflammation? of late or tertiary stage of disease?
A. Skull , tibia , sternum , ribs . A. 1- 20 years
Q. Why is liver function test to be done in secondary Q. What is Merritts classification of neurosyphilis?
syphilis ?
A. 1. Asymptomatic
A. Sub - clinical hepatitis may occur and ocassionaly
symptomatic hepatitis. (increased alkaline phosphatase) Early
Q. Why does a patient get systemic symptoms like Late
glomerulonephritis/ nephrotic syndrome / iritis/ 2. Meningeal
anterior uveitis ?
Acute syphilitic meningitis
A. Circulating immune complexes get deposited on the
respective sites. Meningovascular
80 DYP SURVIVAL GUIDE FOR POST GRADUATES DYP SURVIVAL GUIDE FOR POST GRADUATES 81

Cerebral Q. After how many days of a primary chancre will a


Spinal form patient get secondary syphilis ?
3. Parenchymatous A. Few weeks or months .( both may co- exist )
General paresis Q. What is lues maligna?
Tabes dorsalis A. Malignant secondary syphilis is widespread form
Taboparesis (mixed) produced by prodrome of fever,myalgia and
headache followed by papulopustular eruption that
Optic atrophy
rapidly transforms into necrotic, sharply marginated
4. Gummatous ulcers with erythematous halo, clean looking floor
Cerebral hemorrhagic brown crusts that are organised in
Spinal rupioid layers.(layers of crusts resembling oyster
shell)more common in HIV positive patients.
Q. When can a patient get meningovascular syphilis
and tabes dorsalis? Q. What are the four clinical characteristics of lues
A. 5-12 years after primary infection (meningovascular maligna?
syphilis) A. 1. Short incubation period
8-25 years after primary infection(tabes/paresis) 2. Prodrome
Q. Why is syphilis not diagnosed in women until the 3. Pleomorphic lesions
secondary stage . 4. Noduloulcerative lesions of skin and mucosa
A. Because the chancre is not visible in women and it is Q. What is Fishers criteria for lues maligna?
painless .
A. 1. Compatible gross and microscopic morphology
Q. How much time does it take for a primary chancre
to heal if untreated ? 2. A high titre of positivity for serologic tests for
syphilis
A. 3- 6 weeks.
3. Jarisch Herxheimer reaction following treatment
Q. Can a single ulcer on the genitals be due to herpes
simplex virus ? 4. Dramatic response to antibiotic treatment
A. Yes, recurrent herpes simplex virus often present with Q. What are the systemic manifestations of secondary
a single ulcer . syphilis?
Q. How many patients of syphilis have abnormal csf A. 1. Malaise, low grade fever, headache mild
findings in primary and secondary stage ? hepatitis, hepatosplenomegaly, jaundice
A. primary syphilis - 13 % 2. Pulmonary changes
secondary syphilis - 25% 3. Glomerulonephritis
82 DYP SURVIVAL GUIDE FOR POST GRADUATES DYP SURVIVAL GUIDE FOR POST GRADUATES 83

4. Asymptomatic neurosyphilis Q. From which sites should a DGI be done?


5. Gastrointestinal involvement (anorexia, nausea, A. Moist lesions on mucous membranes which are full
vomiting, antral ulcers) of treponemes are the best sites.
6. myocarditis rarely, periosteitis Q. Are lesions on the mouth or genital area more better
7. iritis,uveitis,parotitis for DGI?
Q. What is the natural course of secondary syphilis? A. Nonpathogenic treponemes are part of their normal
flora . And so lesions from mouth should not be
A. It can last for several weeks till 1 year but relapses
selected as a site for DGI.
can occur in 20-25% of patients.
Q. Which diseases mimick the pustular rash of
Q. How do you differentiate relapse from reinfection?
secondary syphillis?
A. Occurrence of a chancre at a different location from
A. Vons zumbusch type of pustular psoriasis.
the previous one or detection of early syphilis in a
current sexual partner indicates reinfection. Q. Does the duration of infection hold any significance
in the treatment of syphilis?
Q. What are the types of relapse?
A. Yes, as the duration of infection increases T.pallidum
A. Clinical relapse
divides more slowly requiring a more prolonged
Serological relapse-reagin tests becomes positive duration of therapy.
after having been negative,or shows a progressive
Q. What is syphilis incognito?
rise in titres after a decline.(often precedes a clinical
relapse- CSF examination is recommended) A. It is a subtype of latent syphilis(early/late)which runs
a subclinical course from the time of infection until
Q. What do you mean by transplacental relapse?
its diagnosis by routine serological screening.
A. The birth of a syphilitic child to an apparently cured Improved personal hygiene-soaps or antiseptics may
mother. cause the chancre to stop developing or to heal
Q. How do you diagnose a case of secondary syphilis? rapidly. Coincidental use of antibioitcs for some other
A. Positive serology for syphilis disease may minimize or abolish symptoms of early
disease but the dose may be insufficient to cure
Demonstration of Treponema Pallidum
syphillis.
Q. How many treponemes are required for syphilis?
Q. What is the risk of getting late syphilis in the above
A. 1000 treponemes. case?
Q. Difference between initial episode and relapse? A. 1/3 of these patients develop late syphilis.
A. Lesions are fewer in number and smaller in size in Q. How do you visualise treponemes in histopathology?
relapses and asymmetrical. It is less severe.
A. Levaditi or Warthin Starry stains.
84 DYP SURVIVAL GUIDE FOR POST GRADUATES DYP SURVIVAL GUIDE FOR POST GRADUATES 85

Q. What is the histopathology in secondary syphilis? Q. Why don't you use a light microscope?
A. Lymphomononuclear infiltrate most intense in A. Treponemes cant be detected with light microscope
papillary dermis. Plasma cells are seen as focal because of its narrow width.
corrections around the skin appendages and on the Q. What is the principle of dark field microscopy?
periphery of epitheloid cell granulomas.
A. In dark -field microscopy, dark field condenser
Q. How does Syphilis in HIV differ from the usual form? allows the light rays to strike the object in the field at
A. Larger primary chancre, multiple ulcers, more an oblique angle so that no direct light but only rays
frequent systemic symptoms in the secondary reflected from from the object enter the microscope
stage,simultaneous multiorgan involvement, objective. This gives the object a luminous
accelerated course and development of uveitis and appearance against a black background.
neurosyphilis(within 2 years) in the early stages of Q. How do you collect specimen for the above test?
infection.
A. The lesion is cleaned gently with a saline soaked
Q. What is the rate of transmission to the foetus in gauze and then squeezed with the index finger and
untreated men in various stages of syphilis? thumb to produce a serous exudate. It is then
A. 70-100% in primary and secondary syphilis transferred directly on a glass slide by pressing it on
40% in early latent syphilis the lesion. A drop of saline is added to make it
homogenous.
10% in late latent syphilis
Q. Why should the specimen be immediately be
Q. Which trimester infection is associated with high
examined?
foetal mortality?
A. The specimen should be immediately examined
A. First and Second trimester infection.
because any delay reduces the motility of treponemes.
Q. What is Kassowitz's law?
Q. From where all can treponemes be isolated?
A. Longer the interval between infection and pregnancy,
1. Mucocutaneous lesions
the more benign is the outcome in the infant.
2. Lymph node puncture
Q. Infection of the fetus does not occur before 18 weeks
-TRUE/FALSE? 3. Amniotic fluid
A. False. It has been shown that T.pallidum gains access Q. How is the lymphnode puncture done?
to the foetal compartment as early as 9-10 weeks. A. The skin over the enlarged lymphnode is infiltrated
Q. Which is the most specific test in early congenital with 1% lignocaine the overlying skin is stretched and
syphilis and early acquired syphilis? the lymphnode is held firmly. 0.2 ml of sterile normal
saline is injected, the lymphnode is massaged gently,
A. Darkfield miocroscopy
fluid is aspirated and expressed on a glass slide.
86 DYP SURVIVAL GUIDE FOR POST GRADUATES DYP SURVIVAL GUIDE FOR POST GRADUATES 87

Q. How is T.Pallidum differentiated from non Q. How much is the sensitivity of the above test?
pathogenic treponims? A. The sensitivity of VDRL test is 70-90% in primary
A. It is distinguished from other treponemes by the syphilis, while it is 100% in secondary syphilis.
tightness of spirals, regular coils and characteristic Q. What is the advantage of RPR test over VDRL?
cork screw movement. It is shorter and thinner than
non pathogenic treponemes. A. It can be read macroscopically because of addition
of charcoal particles, the antigen used is stabilised
Q. What are the advantages of direct flourocent
and cards are used instead of slides.
antibody- T.Pallidum (DFA-TP) over dark field
microscopy? Q. What are the causes of false positive VDRL tests?
A. 1. It is more sensitive and specific. A. As the antigen used in non treponimal test is a
2. Samples from oral mucosa can be examined by component of all mammalian cell membranes, the
this method. damage to the host tissue by infection (leprosy, HIV
tuberculosis, malaria), immunisation, pregnancy, age
3. Slides need not be examined immediately and
releated changes, or autoimmune diseases(
may be sent to a laboratory.
connective tissue diseases) can result into false
Q. Which is the screening and confirmatory test for positive VDRL.
syphilis?
Q. What are the types of false positive reactions?
A. Non treponimal serological test like VDRL are
inexpensive, high in sensitivity and have a high A. 1. Acute ( less than 6 months duration)
negative predictive value, therefore are suitable for 2. Chronic ( more than 6 months duration). It is
screening. Treponimal test(FTA-Abs) are more recommended that serology should be repeated
specific and have a high positive predictive value, at 10 weeks as by that time most cases will return
therefore are used for confirmation. to VDRL non reactivity.
Q. What is the antigen in VDRL? Q. What is prozone phenomenon?
A. The antigen comprises of lecithin, cholesterol and A. This phenomenon occurs when there is excess of
purified cardiolipin ( a component of mammalian cell antibodies in the patient's serum that interfers with
membrane) the binding to antigen in the floculation test, so if
Q. How is the VDRL test done? syphilis is strongly suspected the test should be
A. Serum is heated at 56 degree celsius for 30 mins. repeated in serial dilutions.
Serum and antigen are mixed within a ring on a glass Q. What is the zone of equivalence?
slide by rotating it mechanically and results are read
A. The optimal ratio of antigen to antibody yields an
in a microscope at 100x magnification. If
insoluble precipitate that is visible in a positive test.
anticardiolipin antibodies are present, the antigen
The zone of equivalence defines this optimal ratio.
rods aggregate to form clumps.
88 DYP SURVIVAL GUIDE FOR POST GRADUATES DYP SURVIVAL GUIDE FOR POST GRADUATES 89

Q. What is the appropriate temperature for storage of Q. Which is the only test for sexually transmitted
blood samples? diseases which can be used with CSF?
A. 27 degree Celsius since false negative treponemal VDRL
tests can occur in patients with titres of less than 1: 4 Q. When does the VDRL test becomes reactive?
dilutions.If the blood samples are stored at cold A. 2-3weeks after infection which reverts to negative in
temperature(4degree Celsius) before testing. 1/4rth cases during late latency.
Q. Can trponemal tests be used for monitoring Q. How are the reports read?
purposes?
A. Reactive, Non reactive, weakly reactive or in titres
A. No, because these tests remain reactive for years 1:2, 1:4, 1:8.
inspite of adequate therapy.
Q. What is the treatment of syphilis?
Q. what are the causes of positive and negative
A. Primary,secondary or early latent syphilis
treponemal tests?
RECOMMENDED: Benzathine penicillin 2.4 million
A. False positive - Leprosy, malaria, infectious units in a single dose IM.
mononucleosis, SLE
Penicillin allergy- Doxycycline 100 mg oral twice
False negative - advanced immunodeficiency daily for 14 days
associated with HIV and early congenital syphilis.
Late latent syphilis, syphilis of unknown duration,
Q. which is the test to come positive first in early tertiary syphilis
syphillis?
RECOMMENDED: Benzathine penicillin 2.4 million
A. The FTA-Abs IgM is the first to become positive in units weekly for three weeks IM
early syphyllis and has a higher sensitivity than the Penicillin allergy- Doxycycine 100mg twice daily for
VDRL test in late syphyllis. 28 days
Q. How is FTA-Abs done? Neurosyphilis, Syphilitic eye disease, Syphilitic
A. In this test, T. pallidum subsp. Pallidum (Nichol's auditory disease
strain are fixed on glass slides). The patient's serum Recommended: Aqueous Crystalline penicillin G,18-
is first diluted 1:5 in sorbent (an extract from cultures 24 million units/day administered as 3-4 million
of the non-pathogenic reiter treponeme) to remove units IV every 4 hours/continuos infusion for 10-14
non-specific treponeme antibodies. The serum is then days.
added to the glass slide. If it contains antibodies, it Alternate: Procaine penilcillin 2.4 million units IM
coats the treponemes. Then FITC - Labelled anti- once daily with probenecid 500 mg orally 4 times a
human immunoglobulin is added to the slide that day,both for 10-14 days.
combines with the patient's antibody attached to the
Q. How is penicillin administered?
T. Pallidum. The slide is examined under fluorescent
microscope. A. Benzathine penicillin 2.4megaunits,1.2 in each
90 DYP SURVIVAL GUIDE FOR POST GRADUATES DYP SURVIVAL GUIDE FOR POST GRADUATES 91

buttocks deep IM with a wide bore 18 no needle after Primary syphilis


test dose. - Edema and phimosis
Q. How will you treat a pregnant woman with syphilis - Chancre becomes more prominent
who also has a penicillin allergy?
Secondary syphilis
A. Desensitisation and treatment with penicillin. Oral - Lesions become more obvious
step-wise penicillin dose challenge or skin testing
might be helpful in identifying women at risk. - Papule at the site of primary chancre
Erythromycin and azithromycin should not be used, Signs of meningeal irritation
because neither reliably cures maternal infection or - arthralgia/myalgia
treats an infected fetus. Data are insufficient to Late syphilis - CNS and CVS affected
recommend ceftriaxone for treatment of maternal Occlusion of coronary arteries
infection and prevention of congenital syphilis
headache, nausea/vomiting, tender adenopathy,
Q. A woman in her second trimester diagnosed as pharyngitis or fever.The reaction is believed to be a
syphilis.How will you manage? result of the rapid killing of spirochetes and is seen
A. When syphilis is diagnosed during the second half most commonly among patients treated for primary
of pregnancy, management should include a or secondary syphilis (1/3 to 2/3 of cases). Treatment
sonographic fetal evaluation for congenital is supportive utilizing antipyretics and analgesics
syphilisand usual treatment.Sonographic signs of and symptoms usually resolve within 24 hours.
fetal or placental syphilis (i.e., hepatomegaly, ascites, During pregnancy, the Jarisch-Herxheimer reaction
hydrops, fetal anemia, or a thickened placenta) has been associated with fetal distress and preterm
indicate a greater risk for fetal treatment failure. labor (the greatest risk occurring during the first 48
hours after treatment).
Q. What are the reactions which a patient can get with
injection penicillin? Q. How do you prevent Jarisch Herxheimer reaction?
A. 1. Anaphylactic reaction A. Oral steroids
2. Vasovagal reaction Q. What is Mazzotti’s reaction?
3. Hoigne phenomena A. Sudden increase in fever after starting within a few
hours after antifilarial medication.
4. Jarisch Herxheimer reaction
Q. Why does a patient get CNS symptoms after
Q. What is Jarisch Herxheimer reaction?(Thereupeutic treatment?
paradox)
A. After treatment,organisms is killed resulting in
A. It is an all or none phenomenon which may occur liberation of toxins from organism- act as
within the first 24 hours (usually 2-8 hours) after superantigens-meningism like symptoms but CSF is
initiation of penicillin. normal.(Shwartzman phenomenon).
92 DYP SURVIVAL GUIDE FOR POST GRADUATES DYP SURVIVAL GUIDE FOR POST GRADUATES 93

Q. What is Hoigne’s phenomenon? Q. When should screening for syphilis be done in a


A. Microcrystals of procaine peniciilin enter the cerebral pregnant woman?
circulation resulting in abnormal behavior/ anxiety/ A. Most states mandate screening at the first prenatal
angor animi(feeling of impending death). visit for all women. For communities and populations
Q. How do you desensitise? in which the prevalence of syphilis is high and for
A. Intravenous protocol for penicillin patients at high risk, serologic testing should be
desensitization performed twice during the third trimester (ideally
at 28-32 weeks' gestation) and at delivery. Any woman
STEP* SOLUTION DOSE (mL) DOSE (U) who delivers a stillborn infant after 20 weeks'
(U/mL)† gestation should be tested for syphilis.
1 100 0.1 10 Q. When will you repeat the titres?
2 100 0.2 20 A. Serologic titers should be repeated at 28-32 weeks'
3 100 0.4 40 gestation and at delivery as recommended for the
4 100 0.8 80 disease stage. Inadequate maternal treatment is likely
5 1,000 0.15 150 if delivery occurs within 30 days of therapy, if clinical
6 1,000 0.30 300 signs of infection are present at delivery, or if the
7 1,000 0.60 600 Serologic titers should be repeated at 28-32 weeks'
8 1,000 1.00 1,000 gestation and at delivery as recommended for the
9 10,000 0.2 2,000 disease stage. Inadequate maternal treatment is likely
10 10,000 0.4 4,000 if delivery occurs within 30 days of therapy, if clinical
11 10,000 0.8 8,000 signs of infection are present at delivery, or if the
12 100,000 0.15 15,000 maternal antibody titre at delivery is fourfold higher
13 100,000 0.30 30,000 than the pretreatment titre. Serologic titres can be
14 100,000 0.60 60,000 checked monthly in women at high risk for
15 100,000 1.00 100,000 reinfection or in geographic areas in which the
16 200,000 25 200,000 prevalence of syphilis is high.
17 400,000 25 400,000 Q. In which stage of syphilis does one get bullous
18 800,000 25 800,000 lesions?
19 1,600,000 25 1,600,000
A. Congenital syphilis.
20 3,200,000 25 3,200,000
21 5,000,000 25 5,000,000 Q. What is Hutchinsons triad?
*Each step is administered at 15-minute intervals
A. Sir Jonathan Hutchinson (1828-1913) from England
†Use penicillin G for dilutions in 0.9% sodium chloride
described a triad in late congenital syphilis consisting
94 DYP SURVIVAL GUIDE FOR POST GRADUATES DYP SURVIVAL GUIDE FOR POST GRADUATES 95

of notched incisors, interstitial keratitis, and eighth Q. What are the differentiating features of various STD
cranial nerve deafness. ulcers?
Q. What is Dattner Thomas concept? FEATURES SYPHILIS HERPES CHANCROI LGV DONOVANOSI
D S
A. If cellular count in CSF is high(> 5 cells/cubic mm) Incubation 9-90 days 2-7 days 1-14 days 3 days-6 1-4 weeks
then it is suggestive of active neurosyphilis in which Period weeks (up to 6 months)

case retreatment is essential even if VDRL is negative. Primary Papule Vesicle


(erosion)
Papule/Pust
ule
Papule,
pustule,or
Papule
lesion
vesicle
While if the protein count is high with the cell count Number of Usually one Multiple , may Multiple , Usually one Variable
being low then it is not significant. lesions coalesce may
coalesce
Edges Sharp, Well Erythematous, Ragged, Elevated, Elevated,
Q. Stigmata of syphilis? demarcated, Cratered Undermined round, or irregular
Round or , oval
Syphilitic facies Frontal bossing, saddle nose, Oval Irregular
short maxilla, protuberant Depth Deep or Superficial Excavated Superficial Superficial or
Superficial or deep deep
mandible Base Smooth,
Nonpurulent
Serous,
Erythematous,
Purulent,
Vascular,
Variable,
nonvascular
Red and velvety,
bleeds
, Nonvascular Friable Readily
Ophthalmologic Interstitial keratitis, Relatively
chorioretinitis, secondary nonvascular
Induration Indurated None Soft Occasionall Firm
glaucoma, corneal scarring, y firm
Pain Rare Often Tender Very Variable Uncommon
optic atrophy tender
Ears Sensorineural hearing loss Lymph
nodes
Oropharynx Hutchinson teeth, mulberry Fluctuance Firm None May suppurate May None;
suppurate pseudobuboes
molars, perforation of hard Tenderness None Tender Tender Tender None;
pseudobuboes
palate Distributio Bilateral Bilateral with Usually Usually None;
n primary Unilateral Unilateral pseudobuboes
Cutaneous Rhagades, gummas infection

Central nervous system Intellectual disability, arrested


hydrocephalus, seizures, optic HERPES GENITALIS
atrophy, juvenile general
paresis Q. What does the word HERPES means?
Skeletal Saber shins (anterior bowing of A. The word Herpes is derived from Greek word which
the tibia), Higoumenakis sign means to creep.
(enlargement of the Q. What is the causative organism for Herpes genitalis?
sternoclavicular portion of the
A. Human herpes virus-2 is the most common organism
clavicle), Clutton joints
(painless arthritis), scaphoid implicated in Herpes Genitalis. However HHV 1 can
scapula cause the same in case of oral intercourse.
96 DYP SURVIVAL GUIDE FOR POST GRADUATES DYP SURVIVAL GUIDE FOR POST GRADUATES 97

Q. What is Primary Herpes infection, First Episode Virus will replicate at the site of infection (i.e. the
Infection and what is recurrent herpes infection? mucocutaneous surface),
A. Primary Herpes Infection- 'Primary infection' denotes PRIMARY HERPES INFECTION
initial HSV infection in individuals without pre-
existing antibodies to HSV-1 or HSV-2.
First episode infection denotes initial clinical attack Virus then travel by retrograde axonal flow to the
of HSV in individuals with pre existing antibodies dorsal root ganglia and establish latency. Latency
to HSV 1 or HSV 2 due to previous subclinical enables the virus to exist in a relatively non-infectious
infection with the virus state for varying periods of time in its host.
Recurrent Herpes Infection- The reactivation of HSV LATENCY
after the establishment of latency. Stress, UV light, Fever, Tissue damage,
Non Primary genital infection- refers to an infection Immunosuppression.
with one HSV type in an individual who already has
pre-existing antibodies to the other HSV type.
Q. What are the risk factor for transmission of genital REACTIVATION OF VIRUS
herpes? RECURRENT HERPES INFECTION.
A. Risk factors associated with the transmission of
genital herpes include Q. How do you differentiate primary and recurrent
1] an age of 15-30 years (period of greatest sexual herpes genitalis?
activity), A.
2] an increased number of sexual partners,
Primary Recurrent
3] black or Hispanic race,
Prodromal symptoms More severe Less severe
4] lower income levels and education,
Number of lesions more Limited
5] female gender,
Severity(Pain and extent) more Less
6] homosexuality and
Time taken for resolution 2-6 weeks 1 week
7] HIV positivity.
Complications More common Uncommon
Q. Describe the pathogenesis of Herpes infection?
Regional lymph nodes Enlarged and Not
A. The causative organism of Herpes genitalis, HSV-2
spreads primarily by sexual contact. tender involved
Scarring Sometimes Never
98 DYP SURVIVAL GUIDE FOR POST GRADUATES DYP SURVIVAL GUIDE FOR POST GRADUATES 99

Q. Why does Non-Primary Genital Herpes lesion heal collected by vigorously rubbing with a cotton-
faster? tipped swab on a wire shaft and sent for culture.
A. Patients with non-primary first episode have To collect cervical specimens, swabs should be
neutralising antibodies to HSV-1 which interrupts the taken from ectocervix and the entry of the
spread of HSV infection. Thus they have lower endocervical canal, as the HSV involves
frequencies of systemic symptoms, shorter duration squamous rather than columnar epithelium.
of pain, fewer lesions and shorter healing time The specimen should be placed immediately
compared to true primary infections. into vials containing 1ml of viral transport
Q. What are the atypical presentations of genital medium and should be kept at 4 degree until
herpes? cultured.
A. 1] vaginal discharge(unrelated to candidiasis) 2] Culture- Gold Standard for diagnosis of acute
2] genitourinary pain HSV infections.
3] nonspecific vulvar erythema Human diploid fibroblast cell lines is used- HSV
takes 12-18 hrs for replication.
4] prostatitis and lower back pain
3] Confirmatory tests- Neutralization with type
5] itching, burning, soreness, pain over genital area(
specific antisera, Immunological assays, Nucleic
without obvious lesions)
acid hybridization.
6] unexplained systemic symptoms- fever, malaise,
Q. Describe the bedside test for Herpes infection?
myalgia
A. Tzanck test is the bedside investigation for herpes
7] vulvar, penile or perianal fissures
Infection.
8] folliculitis.
Specimen collected.
Q. How does Herpes in HIV present?
Slide is air-dried.
A. 1] hyperkeratotic, verrucous lesions
2] vegetating plaques
3] chronic, persistent ulceration Slide is covered with Giemsa stain (which is diluted 1:10
with distilled water) for 15 mins.
4] generalised papular eruptions
Q. How will you investigate the case of genital HSV ?
A. 1] Collection of specimen - A large vesicle should Wash with water.
be chosen and fluid should be aspirated with a
tuberculin syringe. If large vesicle is not present,
Air dry the slide and examine under oil immersion.
then exudates of a small vesicle or open lesion is
100 DYP SURVIVAL GUIDE FOR POST GRADUATES DYP SURVIVAL GUIDE FOR POST GRADUATES 101

Q. What will you see in Tzanck smear? 2]


Limiting number of sexual partners.
A. Acantholytic cells(secondary acantholysis) with 3]
Avoiding partners who have a history of genital
multinucleated gaint cell. herpes.
Q. What is the significance of HSV antibody titres? 4] Consistent and correct condom use.
A. HSV IgG- Past infection. 5] Vaginal and rectal microbicides.
HSV IgM- Recent infection. 6] Antiviral therapy.
Q. What are the sequelae of genital herpes? 7] Vaccines.
A. 1] Psychological morbidity associated with Q. How will you manage genital herpes infections?
recurrences. A. General Measures-
2] Psychological morbidity related to fear of 1] Counselling
transmission to partner or fetus or newborn.
2] Maintainence of local hygiene.
3] Systemic complication.
3] Sietz bath.
4] Increased risk of acquiring or transmitting HIV
Topical Treatment-
infections.
Topical antibiotics to avoid secondary infection.
Q. What are the systemic complications associated with
Herpes Infections? Systemic Treatment-
A. 1] Aseptic meningitis. Paracetamol- 500mg, 4 hourly may relief pain.
2] Radiculomyelopathy- urinary retention, Anti-Virals (Acyclovir, Valacyclovir, and famciclovir)
numbness, paresthesias, neuralgic pain of reduce viral shedding, itching, average healing time,
buttocks, constipation, impotency. and frequency of new lesion formation.
3] HSV encephalitis. CDC GUIDELINES-2010
4] Disseminated infections FIRST EPISODE EPISODIC SUPPRESSIVE GENITAL HERPES IN
THERAPY FOR THERAPY FOR HIV INFECTED
5] Transmission to newborn. RECURRENT RECURRENT INDIVIDUALS AND
GENITAL GENITAL HERPES OTHER
6] Secondary microbial infection. HERPES IMMUNOCOMPROMISED
Acyclovir 400 Acyclovir 400 mg Acyclovir 400 mg EPISODIC
7] Pelvic inflammatory disease mg orally three orally three orally twice a day TREATMENT-Acyclovir
times a day for times a day for 5 OR 400 mg orally three
8] extragenital lesions 7–10 days days Famiciclovir 250 times a day for 5–10
Q. What measures can be helpful for prevention of HSV OR OR mg orally twice a days
Acyclovir 200 Acyclovir 800 mg day OR
infections? mg orally five orally twice a day OR Famciclovir 500 mg
times a day for for 5 days Valacy clovir 1 g orally twice a day for 5 –
A. 1] Abstinence. 7–10 days OR orally once a day 10 days
102 DYP SURVIVAL GUIDE FOR POST GRADUATES DYP SURVIVAL GUIDE FOR POST GRADUATES 103

OR Famciclovir 125 OR Q. How is Acyclovir injection available?


Famciclovir 250 mg orally twice Valacyclovir 1 g orally
mg orally three daily for 5 days. twice a day for 5–10
A. It is available as sterile solution containing acyclovir
times a day for OR days containing 25mg/ml (as 20ml and 40ml vials).
7–10 days Valacyclovir 1 g
OR orally once a day SUPPRESSIVE Q. How will you manage asymptomatic sex partners of
Valacyclovir 1 g for 5 days TREATMENT- patients with genital herpes?
orally twice a Acyclovir 400–800 mg
day for 7–10 orally twice to three A. Asymptomatic sex partners of patients who have
days times a day
*Treatment can OR
genital herpes should be questioned concerning
be extended if Famciclovir 500 mg histories of genital lesions and offered type-specific
healing is orally twice a day serologic testing for HSV infection.
incomplete after OR
10 days of Valacyclovir 500 mg Q. What is the risk of transmission of genital herpes
therapy. orally twice a day.
from mother to neonate?
Q. What are the indications of IV acyclovir? A. The risk for transmission to the neonate from an
infected mother is high (30%-50%) among women
A. Immunocompromised patient with disseminated who acquire genital herpes near the time of delivery
herpes. and low (<1%) among women with histories of
CNS complications (Herpetic meningoencephalitis) recurrent herpes at term or who acquire genital HSV
pneumonitis, or hepatitis during the first half of pregnancy.
Severe initial clinical episodes in immunocompetent Q. What is the treatment of Genital herpes in
pregnancy?
patient.
A. Oral or IV Acyclovir in standard doses.
Neonatal Herpes simplex virus infections.
Caesarean section should be considered for women
Varicella zoster infection in immunocompromised
developing genital herpes within 6 weeks of delivery.
infections.
Q. What is the treatment of Neonatal Herpes Infection?
Patient is unable to swallow or tolerate oral acyclovir
A. The recommended regimen for infants treated for
because of vomiting.
known or suspected neonatal herpes is acyclovir 20
Q. How is IV Acyclovir administered? mg/kg IV every 8 hours for 21 days for disseminated
A. 5-10 mg/kg, 8 hourly infused at a constant rate over and CNS disease or for 14 days for disease limited to
1 hour for 3-7 days or until clinical improvement the skin and mucous membranes.
observed, followed by oral antiviral therapy to Q. Name some vaccines available for HSV?
complete at least 10 days of total therapy. Acyclovir A. 1] Inactivated virion derived vaccines.
dose adjustment is required in renal failure. 2] Adjuvated subunit vaccines.
104 DYP SURVIVAL GUIDE FOR POST GRADUATES DYP SURVIVAL GUIDE FOR POST GRADUATES 105

DONOVANOSIS [GRANULOMA INGUINALE] 3] Poor personal hygiene - who do not retract the
prepuce and clean the area regularly.
Q. What is Donovanosis? 4] Low socio-economic status.
A. Donovanosis is chronic, progressive ulcerative 5] HLA-B57
bacterial infection with Calymmatobacterium
(Klebsiella) granulomatis, a Gram-negative bacillus, 6] Climate- High humidity and heavy rainfall.
as the causative microorganism. Q. What is the primary lesion of Donovanosis?
Q. What is Donovanosis also known as? A. The primary lesion is a small cutaneous papule or
A. Granuloma venereum, Granuloma inguinale nodule that contains mononuclear cells with
tropicum, Serpiginous ulcer. cytoplasmic vacuoles filled with microorganisms.
Q. Is granuloma inguinale a misnomer? The cytoplasmic vacuoles can rupture and release
bipolar Donovan bodies of coccoid, coccobacillary
A. Yes, this is because inguinal lesions occur only in up
to 20% of patients. and bacillary morphology.
Q. Describe the bacillus Calymmatobacterium Q. What is the incubation period of Donovanosis?
granulomatis? A. The exact incubation period is unclear: although the
A. Calymmatobacterium granulomatis is a small, average period is about 17 days, may range from 1
pleomorphic, nonmotile, Gram negative, fastidious day to 1 year.
bacillus showing bipolar staining(Safety Pin Q. Describe the clinical features of Donovanosis?
appearence). A.
Q. In which conditions Safety pin appearance seen?
Small papule or nodule
A. Burkholderia pseudomallei (pseudomonas pseudomallei)
Francisella pestis(Yersinia pestis)
Francisella tularensis. Breaks to form a non-tender beefy red ulcer
Calymmatobacterium granulomatis Which bleeds on touch with sharply defined
Q. Who identified the causative organism? overhanging edge.
A. The microorganism was first identified in 1905 by and no adenitis. Inguinal lesions occur only in up
Donovan, who noted the characteristic Donovan to 20% of patients and are often combined
bodies in macrophages and epithelial cells of the with genital involvement.
stratum malphigii.
Q. What are the extragenital sites of Donovanosis?
Q. What are the predisposing factors for Donovanosis?
A. Often due to autoinoculation or secondary to
A. 1] Sex- Male preponderance
dissemination, occurs in skin, bones(most common),
2] Uncircumscised state. intra-abdominal cavity and oral cavity.
106 DYP SURVIVAL GUIDE FOR POST GRADUATES DYP SURVIVAL GUIDE FOR POST GRADUATES 107

Q. Describe the morphological variants of donovanosis?


A. 1] Ulcerative/Ulcerogranulomatous form (most
common) Slides are air-dried.
2] Hypertrophic /Verrucous form. The smears are then stained with Giemsa, Leishmann
or Wright's stain. In tissue scrapings stained by
3] Necrotic type(Phagedena)
Wright's method. Within large mononuclear cells, C.
4] Sclerotic / Cicatricial type. granulomatis organisms are found with bipolar
Q. What is the culture media for donovanosis? staining at its two ends(Donovan bodies), giving them
A. Human peripheral blood mononuclear cells and a 'safety pin' appearance.
Hep2 cells. Q. Describe the procedure of Leishmann staining?
Q. Which serological test available for diagnosis of A.
Donovanosis? Specimen collected.
A. No serological test available for diagnosis of Slide is air-dried
donovanosis.
Q. What is Gold Standard in diagnosis of Donovanosis?
A. Demonstration of organism and identification of Slide is covered with diluted Leishmann stain for 10mins.
Donovan bodies in lesions is Gold Standard for
diagnosis of donovanosis.
Q. Where and how crush smears are done? Rinse in steam of buffered water[ph-7.0-7.2]
A. For confirming the diagnosis of donovanosis, first the Water is left on smear for 2-3 minutes.
lesion is cleaned with gauze soaked in saline and then
a dry gauze.
The infected tissue is usually friable and can be easily Air dry the slide and examine under oil immersion.
detached with a toothed forceps, a curette, scalpel Q. What are the other methods used to stain
blade, or a biopsy punch. The active edge of ulcer C.granulomatis?
gives best yield. A. Leishman Stain
Alternatively, lesion can be scraped with a scalpel or Wright stain
a cotton swab can be rolled over the lesion.
Giemsa stain
Q. What are the complications of donovanosis?
The pinched tissue is crushed in between two slides. A. 1] Lymphedema [common in women] leading to
pseudoelephantiasis.
108 DYP SURVIVAL GUIDE FOR POST GRADUATES DYP SURVIVAL GUIDE FOR POST GRADUATES 109

2] Auto-amputation of penis in males, mutilation subcutaneous tissue involvement in the groins. The
and destruction of labia in females in long lymph nodes are NOT involved.
standing cases. Q. Can Donovanosis be eradicated?
3] Phimosis, Paraphimosis and stenosis of A. For eradication of any disease the following criteria
uretheral, vaginal, oral, or anal orifices in sclerotic should be fulfilled-
variant of disease.
1) Humans should be essential for the life cycle of
4] Local spread- involvement of fallopian tubes and the agent, which should not be able to amplify
ovaries leading to Pelvic Inflammatory disease. in the environment.
Q. How is pregnancy and donovanosis related? 2) Diagnostic tests with sufficient sensitivity and
A. Lesions of Donovanosis tend to proliferate or reoccur specificity should be available to detect levels
during pregnancy. Cervical lesions may extend to of infection that can lead to transmission.
pelvic structures and disseminate resulting in fatal 3) Effective intervention should be available to
haemorrhage at the time of delivery. These events interrupt the transmission of agent.
are due to increased vascularity of tissues and
immunosuppressive effects of pregnancy. DONOVANOSIS FULFILLS ALL THE ABOVE
Q. What are the differential diagnosis of Donovanosis? CRITERIA FOR ERADICABILITY.
A. 1] Squamous cell carcinoma, Q. How will you treat donovanosis?
2] Amoebiasis, A. Recommended Regimen
3] Tuberculosis, Doxycycline 100 mg orally twice a day for at least 3
4] Dimorphic fungal infections, weeks and until all lesions have completely healed.
5] Pyoderma vegetans, Alternative Regimens
6] Crohn's disease and Azithromycin 1 g orally once per week for at least 3
7] Pyoderma gangrenosum. weeks and until all lesions have completely healed.
8] Atypical variant of secondary syphilis. OR
Q. What is pseudogranuloma inguinale? Ciprofloxacin 750 mg orally twice a day for at least 3
weeks and until all lesions have completely healed.
A. The lesions of chancroid may closely resemble
granuloma inguinale and is then known as OR
pseudogranuloma inguinale. Erythromycin base 500 mg orally four times a day
Q. What is Pseudobubo? for at least 3 weeks and until all lesions have
completely healed.
A. The bubo like swellings in the groins due to
110 DYP SURVIVAL GUIDE FOR POST GRADUATES DYP SURVIVAL GUIDE FOR POST GRADUATES 111

OR 2. Doxycycline 100 mg 2 times a day in 3 weeks


Trimethoprim-sulfamethoxazole one double-strength (or until lesions heal)
(160 mg/800 mg) tablet orally twice a day for at least Alternative regimen
3 weeks and until all lesions have completely healed.
1. Ciprofloxacin 750 mg 2 times a day for 3 weeks
Q. What is the treatment of donovanosis in pregnancy? (or until lesions heal)
A. Erythromycin regimen and consideration should be 2. Erythromycin 500 mg every 6 hours for 3 weeks
given to the addition of a parenteral aminoglycoside (or until lesions heal)
(e.g., gentamicin) according to CDC guidelines 2010.
3. Azithromycin 1g orally once per week for
Q. How will you treat sex partners of donovanosis? atleast 3 weeks.
A. Persons who have had sexual contact with a patient 4. Add Gentamicin 1 mg/kg body weight by slow
who has granuloma inguinale within the 60 days I/v infusion every 8 hours for 3 weeks (or until
before onset of the patient's symptoms should be lesions heal) if slow healing
examined and offered therapy.
Q. How does donovanosis in HIV differ from the usual Donovanosis like any other STD can increase the
pattern? chances of acquiring HIV infection.
A. h
Donovanosis and HIV infection-
Clinical features
Morphology : Longer ulcer duration, greater tissue
destruction Atypical sites: extra genital sites (chin)
Course
Increased severity, slow healing and treatment
failure
Investigation
Tissue smear and biopsy
Therapy
Recommended
1. Trimethoprim sulfamethoxazole double
strength tablets 2 times a day or 3 weeks or until
lesions heal
DYP SURVIVAL GUIDE FOR POST GRADUATES 113

Asymptomatic infections, local symptomatic mucosal


6. GONORRHOEA infections (with or without local complications), and
systemic dissemination.
Q. What is Gonorrhoea?
A. Gonorrhoea is a sexually transmitted bacterial MEN WOMEN
disease, caused by gram-negative, pear-shaped or 10% asymptomatic infections. 50% asymptomatic infections.
reniform, fastidious, diplococcus, Neisseria Most common manifestation- Endocervicitis.
gonorrhoeae. Derived from greek word 'gono' means
Acute anterior urethritis.
seed; 'rrhoea'' means flow (the discharge was
confused with semen!) Uretheral discharge- purulent Increased vaginal discharge
Q. which structures are more prone to get infected? and profuse. (pre pubertal females).
A. N. gonorrhoeae principally infects host mucous Dysuria Dysuria
membranes lined by columnar, cuboidal epithelium Without treatment, the clinical Intermenstrual uterine bleeding.
and non cornified cells. symptoms disappear in most Menorrhagia.
Q. What is the mode of transmission of gonorrhoea? patients after about 6 months. Deep dyspareunia.
A. 1] Sexual contact- most common mode (vaginal, Swabbing of the endocervical
anal or oral intercourse). canal results in a characteristic
2] Vertical transmission- infected mother to the yellow swab, indicating
newborn during parturition, may lead to gonococcal cervicitis.
gonococcal conjunctivitis, pneumonia or even
vulvovaginal infections. Occasionally, inflammation
of Bartholin's glands , with
3] Fomites or nonsexual transmission- extremely
acute swelling of the labial
rare (many patients claim to have been infected
folds and a discrete purulent
through toilet seats!)
discharge that appears
Q. What is the risk of acquiring gonorrhoea for a man
when pressure is applied to
following single vaginal intercourse with an infected
the gland.
female?
A. 20%, rising to an estimated 60-80% following 4 Q. Why is gonorrhoea not a cause of vaginal discharge
exposures. in adult females?
Q. What are the clinical features of gonorrhoea? A. This is because gonorrhoea does not involves
A. Incubation period- 1-14 days. stratified squamous epithelium.

112
114 DYP SURVIVAL GUIDE FOR POST GRADUATES DYP SURVIVAL GUIDE FOR POST GRADUATES 115

Q. What are the features of local complications of Gonococcal ophthalmia:


gonorrhoea? Major cause of blindness in the developing world,
A. primarily due to self-inoculation and unusual sexual
MEN WOMEN practices. Characterised by purulent conjunctivitis
progresing to a devastating keratitis followed by
Inflammation of the Cowper's Inflammation of Bartholin's corneal opacification.
and Tyson's glands. gland. Q. What are the syndromes associated with
Epidydimitis. Salpingitis. disseminated gonococcal infections?
Acute and chronic prostatitis. Oophoritis A. 1] Arthritis-dermatosis syndrome (gonococcemia)
Seminal vesiculitis. Pelvic inflammatory disease. 2] Acute perihepatitis (Fitz-Hugh Curtis syndrome)
Proctitis. Proctitis. Q. Describe Arthritis-Dermatosis Syndrome?
Phimosis, Paraphimosis, Parametritis. A. Occurs in 0.5-1% of patients with mucosal gonorrhoea
Trigonitis, Sterility. characterised by fever, gonococcal tenosynovitis
Watering can perineum affecting primarily larger joints, sometimes visible
(urine coming out through as erythema overlying the tendons and necrotic
multiple openings on the pustules due to embolic septic vasculitis seen on the
distal portions of the extremities.
perineum as a consequence
Q. What is Fitz-Hugh-Curtis Syndrome?
of urethral stricture)
A. Occur with either gonorrhoea, Chlamydia or mixed
Q. What are the complications of genital gonorrhoea infections. It presents as fever, and a complaint of right
in pregnancy? upper quadrant pain and tenderness that mimics
A. Spontaneous abortion. acute cholecystitis.
Premature ruptures of membranes. Fitz-Hugh-Curtis syndrome is inflammation of the
liver capsule associated with genital tract infection.
Premature delivery.
Q. What are the risk factors of disseminated gonoccoal
Acute Chorioamnionitis.
infection?
Ophthalmia neonatorum.
A. Risk factors for disseminated gonococcal infection
Pharyngeal infections . In newborn. include-
Q. Ennumerate extragenital gonorrhoea? 1] Menstruation (with the majority of cases in
A. Pharyngeal gonorrhoea women developing during or immediately
Rectal gonorrhoea (Asymptomatic in 50%) following menses)
116 DYP SURVIVAL GUIDE FOR POST GRADUATES DYP SURVIVAL GUIDE FOR POST GRADUATES 117

2] Deficiencies in the late complement components Swabs of the posterior pharynx, including the
(C5-C9). tonsillar area, can be plated directly onto
3] Systemic lupus erythematosus. supplemented selective media.
Q. What is NGU? Staining- using grams stain or methylene blue stain.
A. NGU- Non specific Urethritis that is not caused by Culture- gold standard for the diagnosis of gonococcal
either N.gonorrhoea or C.trachomatis. infection.
NGU- Non gonococcal urethritis that is not caused by Culture media available- Thayer Martin Media,
N.gonorrheae but might be caused by C.trachomatis. NewYork media.
Q. What are the causes of non-gonococcal urethritis? Nucleic acid amplification tests (NAATs)-
A. - Chlamydia trachomatis. Polymerase chain reaction.
- Ureaplasma urealyticum. Q. What is the treatment of gonorrhoea?
- Mycoplasma genitalium/hominis. A. • Uncomplicated gonococcal infections of the
- Trichomonas vaginalis. urethra, cervix, rectum:
- Gardenella vaginalis. Recommended Regimens:
- HSV - Ceftriaxone 250 mg IM in a single dose
- Adenovirus. OR, IF NOT AN OPTION
- Enteric bacteria. - Cefixime 400 mg orally in a single dose
Q. What is the most common cause of gonococcal OR
infections in children? - Single-dose injectable cephalosporin regimens
A. Sexual abuse is the most frequent cause of gonococcal PLUS
infection in preadolescent children.
Azithromycin 1g orally in a single dose
Q. How will you diagnose gonococcal infections?
OR
A. Collection of specimen- In women, samples are
obtained from the endocervical canal (after cleansing Doxycycline 100 mg orally twice a day for 7 days.
of any external exudate of vaginal secretions) and • Uncomplicated Gonococcal Infections of the
from the urethra. Pharynx-
In men, specimen collection from the urethra is - Ceftriaxone 250 mg IM in a single dose
performed by using a small swab or bacteriologic PLUS
loop.
Azithromycin 1g orally in a single dose
Anorectal specimens should be obtained under direct
vision using anoscopy. OR
118 DYP SURVIVAL GUIDE FOR POST GRADUATES DYP SURVIVAL GUIDE FOR POST GRADUATES 119

Doxycycline 100 mg orally twice a day for 7 days Ceftriaxone 25-50 mg/kg IV or IM, not to exceed 125
• Gonococcal Conjunctivitis: mg, in a single dose.
• Gonococcal infections in pregnancy:
Ceftriaxone 1 g IM in a single dose
Pregnancy/breastfeeding:
• Gonococcal Meningitis and Endocarditis:
- Ceftriaxone, 250 mg im as single dose or
Ceftriaxone 1-2 g IV every 12 hours
- Other single-dose cephalosporin regimens (see
Therapy for meningitis should be continued for 10- above) or
14 days; therapy for endocarditis should be continued
- Spectinomycin, 2 g im as single dose
for at least 4 weeks.
• Disseminated gonococcal infection and gonococcal
• Disseminated gonococcal infections: scalp abscesses in newborns:
Recommended Regimen - Ceftriaxone, 25-50 mg/kg iv or im qd for 7 days
Ceftriaxone 1 g IM or IV every 24 hours (10-14 if meningitis)
Alternative Regimens Q. How will you prevent Ophthalamia Neonatorum?
Cefotaxime 1 g IV every 8 hours A. Erythromycin is the only antibiotic ointment
recommended for use in neonates. Silver nitrate
OR ointment (1%) is also effective (Crede's method).
Ceftizoxime 1 g IV every 8 hours Q. Would you prefer Low dose ceftriaxone or high dose
All of the preceding regimens should be continued ceftriaxone and Why?
for 24-48 hours after improvement begins, at which A. A 250-mg dose of ceftriaxone is now recommended
time therapy can be switched to cefixime 400 mg over a 125-mg dose given as
orally twice daily to complete at least 1 week of 1) increasingly wide geographic distribution of
antimicrobial therapy. isolates demonstrating decreased susceptibility
• Ophthalmia neonatorum: to cephalosporins in vitro,

Ceftriaxone 25-50 mg/kg IV or IM in a single dose, 2) reports of ceftriaxone treatment failures,


not to exceed 125 mg. 3) improved efficacy of ceftriaxone 250 mg in
pharyngeal infection (which is often unrecognized),
• Prophylactic Treatment for Infants Whose
and
Mothers Have Gonococcal Infection:
4) the utility of having a simple and consistent
Recommended Regimen in the Absence of Signs of recommendation for treatment regardless of the
Gonococcal Infection anatomic site involved.
120 DYP SURVIVAL GUIDE FOR POST GRADUATES DYP SURVIVAL GUIDE FOR POST GRADUATES 121

Q. How will you investigate a case of urethral discharge? Coinfection with Chlamydia Diagnosis:NGU?
A. GC and Chlamydia infection
Urethral discharge, dysuria
Appropriate Rx Test for other Positive result
Physical examination Partner notification causes of NGU. Appropriate Rx
and Rx Repeat culture, Confirm
Gram stain Confirm response Chlamydia test response to Rx
to Rx Partner testing.

PMNs with diplococci PMNs without


diplococci

Diagnosis: Gonorrhea Diagnosis:


Gonorrhea? NGU? (Non gonococcal
urethritis) Q. What is the difference between Gonococcal
Urethritis and Non-gonococcal Urethritis?
A.
Gonococcal culture Gonococcal Urethritis Non-Gonococcal Urethritis
Chlamydia test Thick purulent discharge. Thin mucoid discharge.
Incubation period within a week. More than 2-4 weeks
More symptomatic Less symptomatic
GC culture GC culture GC Culture Associated with Strangury Pruritus
positive negative negative
Q. What are the two varieties of resistant organisms
Chlamydia Chlamydia Chlamydia encountered in gonorrhoea?
positive positive negative A. PPNG (Penicillinase producing NG)
QRNG (Quinolone resistant NG)
P.T.O. P.T.O. P.T.O. Due to the latter, nowadays fluoroquinolones are not
used in gonorrhoea)
122 DYP SURVIVAL GUIDE FOR POST GRADUATES

Q. What are the other differential diagnosis of urethral


discharge?
7. HANSENS DISEASE
A. - Trauma/Foreign body/stricture
Case proforma
- Intraurethral ulcers(Chancre/Chancroid/Herpes
HISTORY TAKING IN LEPROSY
simplex)
- Benign or malignant neoplasms Name/Age/ Sex/Occupation/Residing at... /
Hailing from.../Marital status
- Rarely tuberculosis
chief complaints :
h 1. Light coloured patches / noticed since___days /
months (I/ T/BT)
2. Stuffiness of nose / Sensation of ants crawling under
the skin/ glove and stocking type of anaesthesia/
swelling of the feet since___ months.
3. Redness and pain over pre existing light coloured
lesions associated with fever since___ days
4. Fever, constitutional symptoms associated with red
raised painful lesions on bilateral shins since____
days
5. Deformity like claw hand /wrist drop / foot drop
since ___ days.
6. Non healing ulcer since ___ days/ weeks
ODP:
Patient was apparently alright ____days/ months
ago.
1) TT / BT
• Patient then noticed light coloured patch over the
___area of the body since ___ days/ months.
• The lesion was approximately pea sized and then
gradually increased in size over ___time period
to the present size.
123
124 DYP SURVIVAL GUIDE FOR POST GRADUATES DYP SURVIVAL GUIDE FOR POST GRADUATES 125

• The patient also noticed ___ (number) such 3. H/O swelling of the breast tissue in males
similar lesions. 4. H/O infertility
• On enquiry patient reveals that there is decreased 5. H/O bone pains in the small bones of the
hair, decreased sweating and decreased hand and feet
sensations over that area 6. H/S/O facial palsy
2. BL/LL 7. H/O hoarseness of voice
• Patient was apparently alright ___months ago. 8. H/O alopecia
• Patient then developed bleeding from the nose/ 9. H/O inability to perceive smell ( anosmia)
epistaxis
10. H/O dyspnoea on exertion
• occasionally associated with stuffiness of the 11. H/O swelling of feet.
nose.
3. Type 1 reaction
• Patient also C/O swelling over B/L ankle and
• C/o redness ,swelling and pain over the pre
feet which is more in the evening
existing lesions since ___ days.
• On enquiry patient also C/O dryness on both
• H/O similar complaints in past.If yes-how many
L/L
episodes
• Patient C/O asymptomatic skin coloured / • H/o associated fever
reddish/ light coloured lesions over the trunk
since ___months • H/o associated weakness of hands and feet
• H/S/O claw hand or foot drop or other
• The lesions gradually increased in number and
deformities.
few lesions have increased in size.
4. Type 2 reaction:
• Patient C/O of glove and stocking of anaesthesia
since__ weeks / months. • H/o fever - intermittent
• There is also decreased sensation over the palms -- low grade
and soles which has lead to unnoticed injury in -- more in the evening
the form leading to non healing wounds. • H/O crops of red raised painful nodules over
• To rule out systemic involvement: the B/L shins occurring since___days
1. H/O eye involvement in the form of redness • H/O similar complaints in past.If yes-how many
/irritation and increased lacrimation episodes
2. H/O reduced libido • H/O joint pain/ bone pain / muscle pain
126 DYP SURVIVAL GUIDE FOR POST GRADUATES DYP SURVIVAL GUIDE FOR POST GRADUATES 127

" H/O epistaxis • H/O fluid filled blister at the pressure points
" H/O blurring of vision with deep pain in the eye • Followed by development of an ulcer at the same
with redness of eyes site after ___days.
" H/O cloudy urine (proteinuria), flank pain • No H/O pain over site of trauma.
7. Ask for
Rule out exacerbating factors that precipitate the reaction:
• H/O loss of lateral 1/3 rd of the eyebrows
• Spontaneous
• H/O drooping of eye lids
• H/O intercurrent infections
• H/O hanging of ear lobes
• H/o physical or mental stress • H/O sagging of face
• H/o onset of puberty 8. H/O treatment taken
• H/o pregnancy / parturition/ lactation. • Rx received in the form of blister combi packs?
• H/O recent surgery • Was the treatment taken in the form of monthy
• H/O drugs like OCP, Potassium iodide. supervised dose as well as daily unsupervised
doses?
• H/o Vaccinations.
• Was the treatment taken regularly or not?
5. Deformity • Rx taken for how long?
• H/o numbness over affected area since ___days. • Any untoward reaction during the course of
• H/O difficulty in doing fine movements like treatment in the form of drug reaction or in the
buttoning of shirt form of type 1 or type 2 reaction.
• H/O feeling like walking on cotton wool Past history :
• H/O TB /DM/ HT/ Ischemic heart disease/
• H/O Slipping of chappals while walking
Bronchial Asthama
• H/O development of frank deformity like claw Personal history :
hand, wrist drop, claw toes, foot drop
• Sleep
6. Ulcer • Appetite
• C/O non healing ulcer over the ball of the great • Bowel/ Bladder
toe since ___days • H/O addictions --- Alcohol
• H/O decreased sesations over the glove and --- beedi/cigarette
stocking area since ___weeks --- chewing beetle nut
• H/O multiple unnoticed minor trauma • Hobbies
128 DYP SURVIVAL GUIDE FOR POST GRADUATES DYP SURVIVAL GUIDE FOR POST GRADUATES 129

• Menstrual history 2. Temperature


LMP: Pulse - increased in C/O reaction
4 to 5 days Blood pressure - increased in C/O renal
involvement in LL or type 2 lepra reaction
Regular cycle of 28 days
Pedal edema - in C/O LL and type 2 reaction
Normal bleeding
Lymphadenopathy - seen in type 2 reaction,
• Obstetric history advanced LL
G_P_A_L_ Respiratory rate
Contraceptive used BMI
Tubal ligaton done or not CUTANEOUS EXAMINATION
FAMILY HISTORY TT/BT:
1. Distribution of lesions: few/ ___ number of lesions
• H/O similar complaints in the family
over the ___ areas of the body
• H/O TB in the family contacts 2. Arrangement: assymetrical
QUALITY OF LIFE 3. Morphology:
• Personal: effect of disease on the interpersonal • well defined skin coloured /coppery /
relationship with spouse and other family erythematous round to oval plaque measuring
members approximately ___ x ___cms with raised edge and
• Professional: effect of disease in terms of numbers flattened centre( TT )
of working days lost. • well defined to ill defined skin coloured /
coppery/ erythematous round to oval plaque
SOCIOECONOMIC STATUS
measuring approximately ___ x ___ cms with
• Total monthly income of the family sloping edges. Few peripheral satellite lesions
• Total number of dependant family members seen.( BT )
• Type of house: pucca house or hutment 4. The surface of the lesions show
• Hair: decreased
• Sanitation facility
• Sweating: decreased
ON EXAMINATION
• Surface : dry
GENERAL PHYSICAL EXAMINATION
• Sensation to cotton wool/ hot and cold/ pin prick
1. Pt is well oriented in time, place and person reduced
130 DYP SURVIVAL GUIDE FOR POST GRADUATES DYP SURVIVAL GUIDE FOR POST GRADUATES 131

5. Feeding nerve palpable or not Look for the following points on palpation :
6. Tinel's sign: tingling pain on tapping of the feeding Thickened +/-
nerve. Tender +/-
BL/LL nodules +/-
1. Distribution: multiple lesions scattered over almost Nerve Right Left
the Entire integument sparing the scalp and axillae Supra orbital
Infra orbital
2. Arrngement: Lesions are tending towards symmetry
Temporal
/ or are symmetrical Greater auricular
3. Morphology: Supra clavicular
Infraclavicular
• Multiple ,smooth ,shiny and skin coloured / Radial
erythematous nodules seen. Ulnar
• The hair /sweating and sensations over skin Median
Radial cutaneus
lesions are preserved.
Common peroneal
NERVE EXAMINATION: Anterior tibial
Posterior tibial
Sural

SENSORY EXAMINATION:
1. Check for the loss of sensations over the lesions ( The
permissible limit for mis reference is 1 cm on hand,
2 cm on face and 7 cm on back)
2. Check for glove and stocking anaesthesia
132 DYP SURVIVAL GUIDE FOR POST GRADUATES DYP SURVIVAL GUIDE FOR POST GRADUATES 133

• Froment's book test: if book is held between


thumb and index finger, the tip of the thumb
flexes because the short flexor and adductors of
thumb flexes because the short flexor and
adductors of the thumb are paralysed and only
way of tightening pinch is using FPL
• Muscle of forearm like FDP and FCU are
paralysed- e/o flattening of these muscles.
Median:
• Look for thenar wasting
• Pen test to check abductor pollicis brevis ( ape
thumb)
• Ask patient to stabilize hand and with the thumb
parallel to the palm touch the tips of the other
3. Protective sensation over palms and soles ( ideally fingers-to check for opponens pollicis brevis
with SW filaments )
• Ask patient to flex at metacarpo phalangeal joint
4. Sensations over the dorsomedial forearm and with inter phalangeal joint extended - to check
anterolateral legs for the 1st and 2nd lumbricals
MOTOR EXAMINATION: • Ochsner's clasping finger test i.e pointing index
Ulnar: finger --- to check for flexor digitorum
• e/o wasting of thenar /hypothenar muscles superficialis and profundus.
• e/o guttering over the palmar and dorsal Radial:
interosseii • Ask patient to extend the wrist against resistance.
• e/o failure of adducting fingers ( palmar Superficial peroneal nerve:
interosseii)-card test / wartenberg's sign • Look for e/o fanning of toes
• e/o failure of abduction of fingers (dorsal • Ask patient to evert the foot against resistance
interoseii)- unable to separate fingers against
Anterior tibial nerve or deep peroneal nerve:
resistance
• Ask patient to dorsiflex i.e extend the foot against
• e/o partial claw ( lumbricals of little and ring
resistance
finger)- ask patient to flex MP joint while
extending IP joint against resistance • Ask patient to invert the foot against resistance
134 DYP SURVIVAL GUIDE FOR POST GRADUATES DYP SURVIVAL GUIDE FOR POST GRADUATES 135

Posterior tibial nerve: EXAMINATION OF SPECIFIC DEFORMITIES


• Ask patient to plantar flex i.e flex the foot against • Madarosis
resistance. • Buddha ear
• Ask patient to invert the foot against resistance. • Nose - epistaxis / nasal collapse
EXAMINE FOR TROPHIC CHANGES: • Sagging of face
• Asymptomatic, deep , non healing fissures on • Leonine facies
heels and toes • Gynecomastia
• Callosities on palms and soles EXAMINATION OF SPECIAL AREAS:
• Multiple spontaneous blisters on trauma prone Oral : look for
areas • papules over lips /macrochelia
• Frank trophic ulcer • Nodules over tongue/ palate/ uvula
EXAMINATION OF ULCER ( IF PRESENT) • Perforation of the hard palate
• Site • Loss of upper incisor teeth
• Size ( Muller Christensen facies leprosa on radiology)
• Margin : elevated Nasal mucosa : look for
• Edges : hypertrophic • Nasal crusts that may lead to mouth breathing
• Floor :covered with slough • Insensitive, pale , thickened nasal mucosa
• Base : hard • Destruction of inferior turbinates
• Local lymphadenopathy Eyes:
• Look for any skin lesions around peri ocular
EXAMINATION OF RELEVANT CRANIAL NERVES:
region
5TH NERVE:
• Look for frequency of blinking, blink interval and
• Corneal reflex loost blinking completeness. ( normal blinking is about
7th NERVE: 6/minute)
• Look for e/o ptosis • Look for photophobia, redness of eyes, watering
of eyes
• e/o deviation of angle of mouth
• Corneal reflex
• e/o dribbling of saliva
• Light reflex
• e/o flattening of nasolabial groove.
• Intra ocular pressue by manual palpation
136 DYP SURVIVAL GUIDE FOR POST GRADUATES DYP SURVIVAL GUIDE FOR POST GRADUATES 137

Digits : Q. What are the diagnostic criteria?


• Tapering A. There are 3 relative and 1 absolute criterias:
• deformity due to bone atrophy A) Relative criteria:
Nails: # skin lesions compatible with leprosy i.e.
hypopigmented patches, infiltrated plaques,
• dry nodules
• lustureless # nerve lesions compatible with leprosy i.e.
• shrunken thickened and tender nerves
• narrow # sensory deficit either in skin or in areas
supplied by thickened nerves
• longitudinal ridges.
B) Presence of AFB in the slit skin smear
• Inverse pterygium For diagnosis atleast 2 relative or 1 absolute
External genitals: criteria should be present.
• Size and consistency of the testicles If only one relative criterion then keep under
• Testicular sensation observation.
Positive AFB in nasal secretions is not diagnostic
• Tenderness
because of presence of saprophytic organisms
SYSTEMIC EXAMINATION Q. What are the investigations to be done?
Respiratory system: A. Routinely done:
• Hoarseness of voice a) Slit Skin Smear
• Dryness of throat b) Skin biopsy
• R/O bronchopneumonia Done only in doubtful cases:
• R/O tuberculosis a) Histamine test
DIAGNOSIS: b) Pilocarpine sweat test
c) Nerve biopsy and USG nerves to study
My patient ____, aged __,unmarried / married with enlargement, structural abnormalities & vascularity.
__ kids is a case of
Done rarely:
• TT/BT/ BL/ LL hansens disease
a) Animal inoculation model
• Active / Inactive b) Polymerase chain reaction
• In type 1 or 2 reaction c) Phenolic glycolipid-1 test
• With _____ deformity/disability (Grade) Q. Describe slit skin smear?
• With /without trophic ulcer A. Take verbal consent. Area cleaned with spirit and
• Who has been treated/not treated for the same. allowed to dry. Firm pressure is applied with thumb
138 DYP SURVIVAL GUIDE FOR POST GRADUATES DYP SURVIVAL GUIDE FOR POST GRADUATES 139

and index finger so that skin blanches, for 30 seconds e) In upgrading type-1 reaction: granulomatous
after this using a no. 15 blade make a 2-3mm long 0.5 erosion of the epidermis, Langhans giant cells
mm deep slit. wipe if any blood is present The blade are larger, sometimes fibrinoid necrosis within
is then turned 90 degree. Scraping is done at base of the granulomas
but once or twice only. The pulpis collected and
f) In downgrading type-1 reaction: necrosis less
smeared on a slide (circular fashion)-Smear is then
heat dried or natural dried. The slit is sealed with common, bacilli density increases over time
Tincture benzoin and stained with Z-N stain. g) In type-2 reaction: foci of acute inflammation
The sites used are 6 for MB- Both ear, elbow, knuckles, superimposed on chronic multibacillary leprosy,
skin lesion, forehead and 2 for PB- either ear and skin polymorph nuclear lymphocytic infiltrate,foamy
lesion. macrophages
Q. What histopathological features are seen on skin Q. What is histamine test?
biopsy?
A. Two symmetrical sites selected (test & control).
A. Biopsy should be taken from active border of the skin,
but normal skin should not be included. One drop of histamine acid diphosphate on each site.
A deep biopsy should be taken. Superficial prick made through each drop.
Slides should be stained with H&E and Fite-faraco Delayed and feeble flare in borderline and
stain (modified acid fast stain). indeterminate leprosy, absent in tuberculoid.
Following features are observed: Flare indicates intact dermal nerves.
a) In lepromatous leprosy: Flattened epidermis, Q. What is sweat test?
grenz zone present, foamy/Virchow/lepra cells
seen (with globi in their cytoplasm), destruction A. 0.2 ml of 1 in 1000 sol of pilocarpine nitrate is injected
of cutaneous appendages, excessive intradermally.
inflammatory infiltrate. Injection site is painted with iodine.
b) In tuberculoid leprosy: dense lymphocytic Starch powder dusted over it.
infiltrate, absent grenz zone, large epitheloid cell
Sweating causes blue discolouration.
granulomas, Langhan's giant cells, infiltration of
dermal nerves Sweating indicates intact dermal nerves.
c) In borderline lepromatous: lymphocytes more Q. Comment on PCR & PGL-1 tests?
prominent, perineural fibroblast proliferation A. PCR is polymerase chain reaction for amplification
(onion skin appearance), foamy cells are present of specific sequences of bacterial DNA. Sub-clinical
but not prominent cases can also be detected.
d) In borderline tuberculoid: ill defined PGL-1 stands for phenolic glycolipid-1 antigen
periappendageal granulomas, foreign body present in M.leprae. This test lacks sensitivity and
giant cells present specificity.
140 DYP SURVIVAL GUIDE FOR POST GRADUATES DYP SURVIVAL GUIDE FOR POST GRADUATES 141

Q. Comment on animal inoculation models? Q. Differential diagnosis of raised infiltrated lesion?


A. Since M.leprae cannot be cultured in artificial A. Differentiate Hansens from the following:
medium, animal models have been used. a) Tuberculosis verrucosa cutis: initially papule
Foot pad of mouse or nine banded armadillo used later thickened crusted sharply demarcated
because of lower temperature. (other animals used warty mostly at site of injury, mostly farmers
are Slender Indian Loris, Korean Chipmunk, butchers
Hedgehog).
b) Lupus vulgaris: lesions mimic lesions of
Differential Diagnosis in Leprosy tuberculoid form, erythematous indolent
infiltrated asymptomatic, tendency to ulcerate,
Q. Differential diagnosis of a flat hypopigmented lesions heal at one edge v/s leprosy lesions
lesion? which heal centrally, apple-jelly nodules on
A. Differentiate hansens from the following: diascopy
a) Pityriasis versicolor : fine branny scaling, c) Lupus erythematosus: erythema, skin atrophy,
presence of fungal hyphae follicular plugging, adherent scales
b) Vitiligo: depigmentation, leucotrichia, no d) Granuloma annulare: indolent asymptomatic
thichened nerves, negative histamine test papules or nodules in an annular pattern
c) Pityriasis alba: ill-defined hypopigmented e) Granuloma multiforme: early papulo-nodular
macules with scaling, associated features of lesions later plaques, sensations preserved
atopy may be present f) Pellagra: asymptomatic symmetrical patches
d) Post-kala-azar dermal leishmaniasis: with burnt-out appearance, associated dementia
hypopigmented macules over trunk and limbs, diarrhoea may be present
erythematous papules and nodules on face,
g) Annular syphilides: positive serological tests
facial erythema may be present, leishman-
donovan bodies h) Seborrhoeic dermatitis: itchy scaly lesions, scalp
e) Oncocerciasis: depigmentation confined to may be involved, cardinal signs of leprosy absent
pretibial regions, gastrointestinal symptoms i) Post-kala-azar dermal leishmaniasis:
present, stool examination positive erythematous infiltrated raised lesions, LD
f) Yaws: depigmentation over hairless areas of bodies can be demonstrated
palms, hyperkeratosis of palms and soles, sabre j) Oriental sore: discoid infiltrated indurated
tibiae, juxta-articular nodes, positive serological plaques with central ulceration
tests h) Tinea circinata: lesions are pruritic, fungal
g) Morphea: may be slightly raised with purple elements can be detected in the scrapings, raised
edges, hair growth and sweating are lost erythematous edge with vesicles
142 DYP SURVIVAL GUIDE FOR POST GRADUATES DYP SURVIVAL GUIDE FOR POST GRADUATES 143

i) Sarcoidosis: sensations preserved, Kveim test Diabetes


positive, relatively uncommon in tropics Alcoholic
j) Pityriasis rosea: erythematous plaques with Nutritional
collarette of fine scales, herald patch may be Guillan Barre syndrome
confused with tuberculoid hansens
Toxic and drug induced
k) Psoriasis: erythematous scaly infiltrated plaques,
Hereditary
auspitz sign positive
Refsum's disease
Q. Differential diagnosis of nodular lesions?
Amyloidosis
A. Differentiate hansens from the following:
Periarteritis nodosa
a) Post-kala-azar dermal leishmaniasis: in advanced
cases nodules on face neck extremities, absence Infections (including leprosy and HIV)
of AFB, LD bodies diagnostic Systemic diseases (autoimmune)
b) Cutaneous leishmaniasis: LD bodies diagnostic Tumors (carcinomatous neuropathy)
c) Syphilis: nodular and nodulo-ulcerative lesions Of the above the following is discussed in some
in late syphilis, abscence of AFB , serological detail
tests for syphilis positive Diabetes: very symmetrical glove and stocking
d) Sarcoidosis anaesthesia, accompanied by dysesthesia. Associated
with cold, shiny and thinned out skin with loss of
e) Onchocerciasis hair & anhidrosis. Loss of deep reflexes (usually never
f) Leukaemia cutis: nodules with diffuse lost in leprosy). Nerve size normal.
infiltration, leukaemic nodules are bluish red, Alcoholic and Nutritional: Usually history, presence
rubbery in consistency of calf tenderness, absence of thickened nerves.
g) Mycosis fungoides: intensely pruritic plaques Hereditary sensory neuropathy: Loss of sensation and
and red shiny tumors on face and trunk, presence sweating more in lower limbs. Motor function normal.
of mycosis cells are diagnostic Deep painless plantar ulcers, tendon reflexes lost.
Q. Differential diagnosis of sensory impairment with Nerves normal. Positive family history.
or without muscle wasting. Bernhardt-Roth syndrome: Neuropathy of lateral
A. Following conditions should be ruled out: femoral cutaneous nerve, due to pressure or edema
of the femoral canal. Burning sensation and numbness
a) Neuropathy: Think of the mnemonic DANG THE
over the lateral part of thigh. There may be slight
RAPIST (Dr. Dang- the villain in a famous Hindi
reduction in hair. Nerves normal
movie!)
144 DYP SURVIVAL GUIDE FOR POST GRADUATES DYP SURVIVAL GUIDE FOR POST GRADUATES 145

Q. Causes of thickened nerves without neurological Q. Differential diagnosis of deformities?


deficit? A. Following conditions should be kept in mind:
A. Following conditions should be ruled out: a) Dupuytren's contracture: due to involvement of
a) Excessive muscular development: in body palmar aponeurosis. Involves middle aged,
builders or people with excessive muscle usually diabetic males. Ring finger involved first.
development nerves are thickened secondary to There is flexion of MCP joints but extension of
muscle hypertrophy IP joints (reverse of the claw hand found in
b) Thin persons can have easily palpable nerves leprosy)
which may be mistaken for thickened nerve
b) Syringomyelia: Lesions only on upper limb.
c) Traumatic neuropathy due to recurrent minor Touch sensation normal while there is inability
traumas, for e.g. ulnar nerve of the right hand to feel temperature. Associated with wasting of
which is often subjected to minor injuries during hand muscles. Nerves normal.
the stress and strains of writing.
c) Sclerodacytly: Skin is edematous in early phases,
d) Pachydermoperiostosis: Clubbing of the nails,
thickened skin of the forehead accompany later hide bound. Flexion deformities are found.
thickened nerves Painful ulcers at tips of fingers. H/o Raynaud's
phenomenon. Females more. Similar appearance
e) Neurofibromatosis: Neurofibromas, cafe au lait
is found unilaterally due to cervical rib (thoracic
macules.
outlet syndrome).
Q. Causes of palpable nerves with some neurological
deficit? d) Ainhum (saw): Seen in Africa. Painless
constricting bands develp at base of little toes,
A. Following conditions should be ruled out:
leading to auto amputation. No sensory loss, no
a) Primary amyloidosis: Lower limbs, impaired thickened nerves.
sensation, muscle wasting, foot drop
e) Facial Palsy: Isolated facial palsy almost never
b) Familial hypertrophic interstitial neuropathy due to leprosy. Bell's palsy is the common cause.
(Dejerine- Sottas) associated with muscular
Of the lower motor neuron type. Bell's
atrophy, claw hand, foot drop and loss of deep
reflexes phenomenon (upward rolling of the eyeball when
attempting to close the eye) seen. Preceded by
c) Peroneal muscular atrophy (Charcot Marie pain in the ear.
Tooth). Muscle weakness, hammer toes,
callosities and reduced deep reflexes Q. Differential diagnosis of neuropathic ulcers?
d) Endotoxic polyneuritis: cause unknown, adult A. Following conditions should be ruled out:
onset, tendon reflexes absent a) Diabetes: Same site as leprosy (under great toe).
e) Sarcoidosis: rare cause of thickened nerves. No thickened nerves.
146 DYP SURVIVAL GUIDE FOR POST GRADUATES DYP SURVIVAL GUIDE FOR POST GRADUATES 147

b) Tabes dorsalis & taboparesis : Lightning pains, Treatment of leprosy and reactions
broad based gait, positive Romberg's sign, loss
of knee reflexes, pupillary changes Q. What is MDT?
A mnemonic may help to remember signs of A. It stands for multi drug therapy. In this multiple drugs
neurosyphilis. P: ersonality changes A: are given at the same time in order to avoid resistance
Depressed affect (emotional state) R: Reflexes to any single drug.
change (increased in paresis, decreased in tabes)
E:ye changes S:ensorium altered I:mpaired Q. How is leprosy MDT given?
intelligence S:peech defects A. In an adult following regimen is followed:
c) Syringomyelia, spina bifida, hereditary sensory Type of Monthly Daily self- Duration of
neuropathy can lead to trophic ulcer Leprosy supervised administered therapy
d) Plantar corns: After excessively paring a corn, Multibacillary a) Rifampcin: a) Dapsone: 1 year
deep cavities can be left behind which can be 600mg 100mg
mistaken for a plantar ulcers. b) Dapsone100mg b)Clofazimine:
e) Yaws: now rare. Painful hyperkeratotic ulcers Clofazimine: 50mg
(crab yaws). 300mg
Paucibacillary a)Rifampcin: a)Dapsone: 6 months
Q. Causes of Leonine Facies?
600mg 100mg
A. Leonine facies can be seen in following conditions: Dapsone 100mg
a) Lepromatous leprosy
b) Diffuse cutaneous Leishmaniasis Q. What are the paediatric doses?
c) Post Kala Azar Dermal Leishmaniasis A. In children following doses are given in a similar
d) Sarcoidosis fashion:
e) Primary systemic amyloidosis Age group Drug doses
f) Scleremyxedema <6 years a) Dapsone: 25mg/day
g) Pachydermoperiostosis b) Rifampcin: 150mg/month
h) Acromegaly c) Clofazimime : 100mg/month &
i) Myxedema 50mg 2 times/week
j) Actinic reticuloid 6-12 years a) Dapsone: 50mg/day
k) Parthenium dermatitis b) Clofazimine: 150mg/month &
l) Mycosis fungoides
50mg on alternate days
m) Leukemia cutis
c) Rifampcin: 300mg/month
n) Sezary's syndrome
148 DYP SURVIVAL GUIDE FOR POST GRADUATES DYP SURVIVAL GUIDE FOR POST GRADUATES 149

12-15 years a) Rifampcin: 450mg/month b) Steroids: In a dose of 0.5-1 mg/kg/day


b) Clofazimine: 200mg/month & c) Clofazimine: In a dose of 300 mg/day
50mg/day d) Thalidomide: In a dose of 100 mg 3-4 times/day
c) Dapsone: 50mg/day e) Colchicine: In a dose of 0.6 mg/day
15-20 years Same as adults Q. Treatment of type-1 lepra reaction?
A. Following drugs are used along with MDT:
Q. Common side effects of MDT drugs? a) Steroids in a dose of 0.5-1 mg/kg/day
A. Side effects are: b) NSAIDs can be used in mild cases
Drug Side effects c) Chloroquine: 250mg twice/day
Rifampcin RIPAS : Respiratory symptoms, Q. Treatment of neuritis?
Infuenza-like symptoms, Pupura,
A. Steroids, perineural injections (lignocaine +
Abdominal pain, Shock (hypotension)
hyaluronidase + hydrocortisone), surgical
hepatotoxicity, red discolouration of
decompression
body fluids
Q. Side effects of drug used in treatment of reactions?
Clofazimine Pigmentation of initially infiltrated
skin, GI symptoms, nephrotoxicity, A. Side effects are:
ichthyosis, phototoxicity Drugs Side effects
Dapsone Hemolytic anaemia in G6PD deficient Steroids Hypertension, diabetes, sodium
patients, methemoglobinemia (treated retention, peptic ulcers
with i/v methylene blue), peripheral
Thalidomide Teratogenecity, infertility, constipation,
neuropathy, nephrotoxicity, hepatotoxi-
city, light headedness, dapsone syndrome drowsiness, peripheral neuritis
Colchicine Diarrhoea, abdominal pain, CNS
Q. What is ROM therapy? depression, apastic anaemia
A. Single dose of the following drug combination: Clofazimine As mentioned earlier
a) Rifampcin: 600mg (300mg for children > 7years) Aspirin Internal bleeding, peptic ulceration,
b) Ofloxacin: 400mg (200mg for children > 7 years) gastritis
c) Minocycline: 100mg (50mg for children > 7yrs)
Q. Management of other associated conditions?
Q. Treatment of type-2 lepra reaction?
A. The following conditions should be treated:
A. Following drugs are used along with MDT:
a) Acute iridocyclitis: 1% hydrocortisone eye drops
a) Aspirin: In a dose of 600 mg 3-4 times/day
150 DYP SURVIVAL GUIDE FOR POST GRADUATES DYP SURVIVAL GUIDE FOR POST GRADUATES 151

b) Acute epididymo-orchitis: treated with & scrape hyperkeratotic areas, use micro cellular
prednisone and scrotal support rubber chappals (MCR chappals).
c) Care of hands and feet: treatment of ulcers and Most of them heal within 6 wks, if not healing then
deformities suspect non-compliance, deep secondary infections,
osteomyelitis, bony spurs, malignancy
d) Management of rhinitis
Prevention of trophic ulcers (tips of patients): 4 S’s
e) Care of the eyes: In case of lagophthalmos,
artificial tear drops See- inspect feet at end of the day
Q. Should MDT be given in pregnancy? Soak- in luke warm water for 15days
Scrape- callosities with scraper, pumice stone
A. Under any circumstances MDT shoult not be stopped
Smear- apply oil on extremity to soften and lubricate.
Q. Leprosy & TB - How to treat together?
Q. What are the recent trends in treatment of leprosy?
A. Start AKT in the proper doses. Start MDT for leprosy
(without monthly rifampcin, since daily rifampcin is A. In cases of rifampicin resistance:
being given in daily regimen. After AKT is over start Regimen I: Ofloxacin 400mg, minocycline 100mg and
giving monthly rifampcin) clofazamine 50 mg for 6 months. Followed by
Ofloxacine 400mg or Minocycline 100mg and
Q. Treatment of trophic ulcers?
clofazamin 50mg for 18 months.
A. Following points should be considered:
Regimen II: Minocycline 100mg, moxifloxacin
a) Rest 400mg, clarithromycin 500mg, clofazamine 50mg for
b) Leg elevation 6 months. Follwed by a continuation phase with Once
a month dosing of Minocycline 200mg, Moxifloxacin
c) Eusol soaks
400m, clarithromycin 1000mg for 18months.
d) Systemic antibiotics in case of secondary
In rifampicin sensitive cases:
infections
1. Rifampicin 600mg or Rifapentin 900mg
e) After infection subsides, below knee walking
plasters 2. Moxifloxacin 400mg
f) Graded physical rehabilitation 3. Clarithromycin 1000mg
4. Minocycline 200mg
g) Preventive treatment through instituting
gradually increasing walking, frequent stopping all this daily for 12 months.
and resting, visual inspection & palpation of
h
soles at night for wounds or warm areas (hot
spots, indicative of necrosis under the skin), soak
DYP SURVIVAL GUIDE FOR POST GRADUATES 153

Any similar family history


8. LEG ULCERS
Treatment history- topical/systemic
Leg ulcer Whether aggravated/relieved symptoms
History Personal-Smoking/Alcohol intake
Name/Age/ Sex/Occupation/Residing at... / Obstetric-h/o recurrent/spontaneous abortions
Hailing from.../Marital status Physical examination:
Chief complaints: General examination: General constitution fair
h/o red raw lesions on …..sites, since…… Patient is conscious/cooperative/well oriented in
where did it begin….did it start like a red raised solid time/place/position
lesion/pus filled lesion/fluid filled lesion/ Pallor/clubbing/cyanosis/icterus/lymphadenopathy/
swelling(gradual/rapid enlarging?)/ on preexisting edema feet
scar? Lymphadenopathy (tuberculosis)
spontaneous/post trauma Look for nutrition/syphilitic stigma
is the ulcer painful? Check both limbs (Buergers disease)
Is there any discharge For atherosclerosis
If yes..Serum/pus/blood Palpate the arteries for calcification
History of limb heaviness/aching/swelling. Auscultate for carotid bruit
Is it worse at the end of the day/swelling relieved by Capillary refilling time should be within 2-3 seconds
leg elevation?
All peripheral pulsations to be felt
Is there a history of prolonged standing?
Local examination:
Does the shoes become more tightly fitting in the
Inspection
evening?
Size and shape:
h/o fever/joint pains/ recurrent oral ulcers/
photosensitivity/malar rash Oval/circular/semilunar/serpinginous/irregular
h/o History of sexual exposure Measure the size-Press a sterile gauze on the ulcer
to get its measurement
Any associated diseases(tabes dorsalis/syringomyelia/
transverse myelitis/peripheral neuritis/generalised Number of ulcers
TB/nephritis/DM/tertiary syphilis/autoimmune Position of the ulcer (medial malleolus/lateral
CTD/Atherosclerotic cardiovascular disease malleolus/tibia/heel and ball of the foot)
152
154 DYP SURVIVAL GUIDE FOR POST GRADUATES DYP SURVIVAL GUIDE FOR POST GRADUATES 155

Edge(inflammed/edematous/blue zone?/white Neurological insufficiency(trophic)


zone?) Presence of any hypopigmented anaesthetic patches
Undermined/Sloping/Punched out/Rolled out Palpation
(everted)/Raised and pearly white beaded Tenderness(tender/slightly tender/never tender)
Floor - covered with red/pale/yellow granulation Edge and margin- Corraborate inspectory findings
tissue
Palpate edge and surrounding tissue- for marked
Wash leather slough (like wet chamois leather) on induration(squamous cell carcinoma or adenocarcinoma)
the floor(gummatous syphilis) Slight induration in any other chronic ulcer except
Is the bone forming the floor of the ulcer?(trophic malignancy(gummatous/syphilitic/trophic)
ulcer) Base
Is there any black mass in the floor(malignant If an attempt is made to pick up the ulcer between
melanoma) the thumb and the index finger,base will be felt.
Discharge - Character/Amount/Smell should be Slight/ Marked Induration
noted
Depth
Scanty serous/Purulent/Serosanguineous(malignant Record in millimetres
or tuberculosis)
Check whether it reaches bone
Greenish discharge?(B. pyocyanea)
Bleeding
Surrounding area:
Bleeds to touch?(malignant)
Glossy, red, edematous(inflammed ulcer)
Relation with deeper structures
Pigmentation/verrucous surface(stasis dermatitis-
The ulcer is made to move over the deeper structures
venous ulcer)
to know whether its fixed to these structures.(fixed-
Telangiectasia(atrophie blanche) gummatous and malignant ulcers)
Scar or wrinkling surrounding the ulcer(old case of Surrounding skin
tuberculosis) Skin around the ulcer must be palpated and examined
Fibrosis/necrosis/cellulitis Check temperature/ tenderness
Thin, atrophic, hairless dry skin Mobility of surrounding skin
Whole limb should be examined Loss of sensation or motor deficit
Deep vein thrombosis/varicose veins Feel the peripheral nerves thickening for abnormal
156 DYP SURVIVAL GUIDE FOR POST GRADUATES DYP SURVIVAL GUIDE FOR POST GRADUATES 157

thickening/tenderness Q. What is Marjolins ulcer?


Feel the main artery supplying the part A. Squamous cell carcinoma developing on pre-existing
State of vein around the ulcer(varicose veins) scar of a burn. It is more aggressive and metastasizes
Lymph node examination: early.
Enlarged/tender/abscess? Q. Differential diagnosis of tender ulcers
Matted/nonmatted? A. - Inflammatory ulcers (Pyoderma gangrenosum,
Stony hard and fixed to the neighbouring structures Vasculitis, Polyarteritis nodosa, wegener’s
Examination for vascular insufficiency: granulomatosis), Panniculitis/Erythema induratum
/ Hypertensive ulcers.
Varicose veins
- Tuberculosis ulcers.
Check for deep vein thrombosis
- Malignant ulcers infiltrating structures supplied
Arteries proximal to ulcer
by pain nerve endings.
Examination for nerve lesion:
Q. Painless ulcers-DD?
Sensory/motor examination/palpation and testing of
sensation of any erythematous plaques for Hansens. A. - Syphilitic ulcers
(peripheral neuropathy) - Trophic ulcers(leprosy/tabes dorsalis/Peripheral
Deep tendon reflexes neuritis/Transverse myelitis)
Questions - Deep fungal infections

Q. What is an ulcer? - Cutaneous Tuberculosis


A. Ulcer is a break in the skin or mucous membranes - Ulcerative DLE
with associated necrosis (cell death). An ulcer heals - Squamous cell carcinoma, Basal cell carcinoma
with scarring. - Venous ulcers
Q. What is an erosion?
Q. Why is the shape of ulcers important?
A. Erosion is a break in skin or mucous membranes
without associated necrosis. It heals without scarring. Shape Condition
Q. What are the common causes of leg ulcers as far as a Oval/irregular crescentic border Tuberculosis
dermatolotist is concerned? Semilunar/circular/serpinginous Syphilis
A. Venous, Neuropathic (including due to diabetes or Vertically oval Varicose ulcers
leprosy), arterial and vasculopathic (most important
Irregular Carcinomatous
being pyoderma gangrenosum).
158 DYP SURVIVAL GUIDE FOR POST GRADUATES DYP SURVIVAL GUIDE FOR POST GRADUATES 159

Q. What information do you get by knowing the Q. Is base and floor the same?
location of a particular ulcer? A. Floor is the exposed surface of the ulcer which can
Site Condition be both seen and felt on palpation while base is the
Medial malleolus Varicose ulcer area on which the ulcer rests. It cannot be seen but
only felt. Floor is exposed surface within the ulcer
Ball of foot/heel Trophic
Q. What is importance of floor of ulcer?
Tibia Gummatous ulcer
Anywhere Malignant ulcer Floor Condition
Pretibial area,breast,hand,neck, Pyoderma Red granulation tissue Healthy and healing
peristomal skin,head gangrenosum Pale and smooth granulation tissue Slowly healing ulcer
Wash leather slough Gummatous ulcer
Q. What is the difference between edge and margin?
Bone Trophic ulcer
A. Margin is the junction between normal epithelium
and the ulcer. So it is the boundary of the ulcer. Black mass Malignant melanoma
Edge is the area between the margin and the floor of Q. What is importance of discharge of ulcer?
the ulcer.
Edge Condition Discharge Condition
Undermined Tuberculosis Scanty serous discharge Healing ulcer
Punched out Gummatous ulcer/Deep Purulent discharge Spreading and inflamed ulcer
trophic ulcer Serosanguineous discharge Tuberculous ulcer,
Sloping Healing/Traumatic/ Malignant ulcer
Venous ulcer Greenish discharge B-pyocyanea
Raised/pearly white/ Rodent ulcer
beaded Q. Why do you get undermined edge in tuberculous ulcer?
Rolled out/everted Squamous cell carcinoma/ A. Because tuberculosis spreads in and destroys the
ulcerated adenocarcinoma subcutaneous tissue faster than it destroys the skin.
Inflammed and edematous Spreading ulcer The overhanging skin is thin, friable, reddish blue
red granulation tissue in Healing ulcer and unhealthy. The other ulcer which is undermined
centre, towards periphery is pyoderma gangrenosum.
blue(growing epithelium) Q. What do you mean by punched out edge?
and a white zone(fibrosis) A. The edge drops down at right angles to the skin
surface as if it has been cut out with a punch.
160 DYP SURVIVAL GUIDE FOR POST GRADUATES DYP SURVIVAL GUIDE FOR POST GRADUATES 161

Q. Why do you get everted edges in Squamous cell - FBS/PLBS


carcinoma? - VDRL
A. Fast growing cellular disease, growing portion at the - Urine for sugar
edge of the ulcer heaps up and spills over the normal
- Pus/discharge for culture and sensitivity
skin resulting in everted edge.
(inflammed and spreading)
Q. Why should you look at the surrounding skin?
- Mantoux test
Surrounding skin Condition - Chest Xray(tuberculous ulcers, metastatic deposits)
Glossy red and edematous Acutely inflamed - Biopsy (malignancy from edge and surrounding
healthy tissue)-type/invasiveness/differentiation)
Eczematous and pigmented Venous ulcer(varicose)
- X-ray of bone and joints
scarred or wrinkling skin Tuberculosis
- New bone formation(gummatous ulcers and
Q. Which conditions cause arterial/ischemic ulcer? sabre tibia)
A. Atherosclerosis - Contrast radiography-Arteriography
Raynauds disease - Imaging techniques- Doppler(venous and
arterial)
Buergers disease
Q. Classification
Q. What type of lymphadenopathy does one get in
various conditions? A. • Clinical - Spreading ulcer
- Healing ulcer
Lymph nodes Conditions - Callus or chronic ulcer
Regional tender lymphadenitis Acutely inflamed • Pathological
Matted,enlarged,slightly tender Tuberculous • Nonspecific
Shotty,discrete,firm Hunterian chancre • Specific
Not usually involved Gummatous ulcer • Malignant
Stony hard/fixed to neighbouring Malignant ulcer Q. What are the nonspecific ulcers?
stages(late stages) A. - Traumatic
- Arterial
Q. What are the investigations done in case of ulcer?
- Venous
A. - Total count/DLC(Lymphocyte count)/ESR
- Neurogenic
- Anemia
- Associated with other diseases
- Increased lymphocyte count and raised ESR for
tuberculosis - Certain other types
162 DYP SURVIVAL GUIDE FOR POST GRADUATES DYP SURVIVAL GUIDE FOR POST GRADUATES 163

Q. What are specific ulcers? Surrounding skin – loss of Eczema and pigmentation Thick rim of callus.
hair, cold to Dryness,fissuring
A. Tuberculosis touch,shiny,atrophic, dry because of autonomic
skin, pallor, involvement
Syphilitic fissuring,thckening of nails
Soft sores - - Lipodermatosclerosis -
Peripheral pulsations not Peripheral pulsations felt Felt
Actinomycosis felt
Prolonged capillary refill Varicosities/ankle Peripheral
Q. Which are the malignancies causing ulcers? time>3-4 secs. edema/lymphedema neuropathy/decreased
A. Epithelioma Pallor on leg elevation sensations/foot
Dependent rubor appears deformities
Marjolins ulcer after a delay of normal 10-
15 secs in dependent
Rodent ulcer position
Malignant melanoma Audible bruit-femoral - -
artery
Q. What is Bazins ulcer? Most common cause Most common cause is Leprosy/transvers
atherosclerosis varicose vein/deep vein myelitis/tabes dorsalis
A. Found in obese adolescent girls particularly on the thrombosis
calves as purplish nodules followed by indolent
ulcers.
Q. What is Martorell’s ulcer? Q. Which artery should always be felt when an arterial
A. Hypertensive patient with ulcer which affects legs ulcer is suspected?
where patches of skin necrosis are seen. A. Dorsalis Pedis artery and posterior tibial artery
q. Difference between arterial and venous ulcer and Q. What is the role of leg elevation in arterial ulcer?
neuropathic ulcer. A. If the leg is kept elevated above the heart level, there
VENOUS NEUROPATHIC will be more pain and the ulcer will not heal.
ARTERIAL
Rare Common Common Q. What is the gaiter area of the leg?
Painful Painless Ulcer painless but
paraesthesia of distal A. Extends from lower medial calf to just below the
extremities medial malleolus
Small Large Large or small
On anterior and outer Medial aspect of the lower Pressure sites(plantar Q. What is a callus ulcer ( trophic/ neurogenic)?
aspect of the leg/ dorsum third of the lower limb surface overlying 1st
of the foot /toes,ankle or (Below medial malleolus) and 5th metatarsal A. It is an ulcer which starts with callosity under which
the heel (Above medial heads,plantar surface suppuration takes place, the pus comes out and the
malleolus/ bony of great toe and heel) central hole forms the ulcer with punched out corny
prominences)
Intermittent No rest pain/ No pain edge.
claudication/ rest pain intermittent claudication Q. What is a tropical ulcer?
Punched out edges Flat / Sloping edges Punched out
Base is dry,covered with Base covered by healthy red Bone A. Tropical ulcers occur on the leg and feet of people in
necrotic debris.tendons granulation tissue
may be exposed tropical countries, associated with malnutrition
164 DYP SURVIVAL GUIDE FOR POST GRADUATES DYP SURVIVAL GUIDE FOR POST GRADUATES 165

caused by vincent's organism (bacteroides fusiformis) Q. What is lymphedema praecox and tarda?
refractory to heal. A. Lymphedema praecox is at puberty while
Q. Why does a diabetic patient get an ulcer? lymphedema tarda occurs after 35 years.
A. 1. Diabetic neuropathy Q. What is Milroys disease?
2. Diabetic atherosclerosis causing ischaemia A. Congenital lymphedema
causing ulcer Q. How do you differentiate lymphedema from
3. Glucose laden tissue is quiet vulnerable to lipedema?
infection A. Bilateral swelling of lower extremities but sparing of
feet.
Q. What is the most characteristic feature of tuberculous
ulcer? Q. What is Kaposis stemmer sign?
A. It is its edge which is thin reddish blue and A. Inability to pinch the area between the 4th and the
undermined. 5th toe characteristic of lymphedema.
Q. Which other skin disease is common in patients with Q. What is elephantiasis nostras verrucosa?(mossy leg)
chronic venous ulcers? A. Nostra- of our region
A. Allergic contact dermatitis. It is a complication of chronic lymphedema
characterised by hyperkeratosis, verrucous changes
Q. What is lipodermatosclerosis? and fibrosis as well as massive enlargement of body
A. It represents fibrosed subcutaneous tissues. part, usually scrotum or lower leg.
(firm,indurated,woody) Q. What are the causes of secondary lymphedema?
Early(acute) -diffuse indurated erythema, warm, A. Recurrent lymphangitis and cellulitis
tender. Parasitic infections(filariasis)
Advanced-inverted champagne bottle leg appearance. Lymhnode dissection(for melanoma/breast cancer)
Q. What is inverted champagne bottle leg appearance? Rosaceous lymphedema
A. Proximal leg swells as a result of chronic venous Granulomatous disease
obstruction and lower leg constricts because of Amyloidosis
fibrosis and loss of subcutaneous fat.
Malignant obstruction(lymphoma/kaposis sarcoma)
Q. What is atrophie blanche? (livedoid vasculitis)
Radiation injury
A. Smooth ivory white atrophic plaques of sclerosis, Obesity
multiple telangiectasias and surrounding brown
Q. What is yellow nail syndrome?
discolouration.(40% of cases with venous
insufficiency/APLS like hypercoaugulability states) A. Yellow thickened nails with primary lymphedema of
ankles and recurrent pleural effusion/bronchiectasis.
166 DYP SURVIVAL GUIDE FOR POST GRADUATES DYP SURVIVAL GUIDE FOR POST GRADUATES 167

Q. What is intermittent claudication? muscles leads to cocked up toes and prominent


A. Pain in one or both legs(calfs),relieved by rest and is metatarsal heads.
characteristic of arterial insufficiency. Q. What is acroangiodermatitis of mali?
Rest pain in advanced disease and tend to keep limb A. Pseudokaposis sarcoma in which violaceous patches
in dependent position. and plaques on extensor surface of distal lower
Q. How is capillary refill time measured? extremities and dorsal aspect of feet and is associated
with venous hypertension/AV malformation or fistula.
A. It is obtained by compressing tip of the great toe until
it blanches and then releasing the pressure. Q. What is cryofibrinogenemia?
Q. What is Ankle brachial pressure index? (ABI) A. In cryofibrinogenemia there is a circulating protein
A. Divide the ankle systolic pressure by the higher of that precipitates when the plasma is cooled. It may
the two systolic pressures obtained from brachial be idiopathic or secondary to malignancy/connective
artery in both arms. tissue disease/infection.
Normal > 1 It presents with purpura, livedo reticularis,
ecchymoses or leg ulcers that are refractory to
0.5-0.8-= occlusive arterial disease standard therapy.
<0.5- severe occlusive disease(rest pain) Q. What is Buergers disease?
Q. False high index seen in….. A. Inflammatory and occlusive condition of Medium
A. Diabetes sized arteries and veins caused by autoimmune
Calcified vessels process trigerred by tobacco in young adults.
Q. How will you check the sensory perception in a case Asymmetric coldness of extremities and decreased
of neuropathic ulcer? peripheral pulses.
A. Apart from the pin prick method, Semmes Weinstein Q. What is pyoderma gangrenosum? .
monofilament is utilized to test sensory neuropathy. A. Pyoderma gangrenosum (PG) is a primarily sterile
A 4.17 monofilament is normally felt(1gm of linear inflammatory neutrophilic dermatosis characterized
pressure) by recurrent very tender cutaneous ulcerations with
If a 5.07 monofilament(10gm linear pressure)cannot mucopurulent or hemorrhagic exudate,undermined
be detected, patient is said to have lost protective bluish borders and surrounding erythema.
sensation. Q. What is the initial lesion of pyoderma gangrenosum?
Q. Why are the metatarsal heads more prominent in (PG)
neuropathic ulcer? A. Pustule or papulopustule on an erythematous or
A. Because motor neuropathy affects the small intrinsic violaceous base.
muscles of the foot, the predominance of extrinsic Erythematous nodule/Bulla
168 DYP SURVIVAL GUIDE FOR POST GRADUATES DYP SURVIVAL GUIDE FOR POST GRADUATES 169

Q. Variants of PG? syndrome is caused by mutations in the PSTPIP1 gene


A. - Ulcerative on chromosome 15 involved in regulation of
inflammatory response .
- Bullous
Q. Why is h/o jointpains important?
- Pustular
A. The PG-ulcers associated with arthritis( poorer
- Superficial granulomatous
prognosis)
Q. Describe the ulcer of PG?
Sickle cell anemia with crisis
A. Ulcers are painful necrotic,undermined border with
purulent or vegetative base on lower extremities Connective tissue disorder
(pretibial area) and rarely mucous membranes and Q. Why is location of PG important to know?
peristomal sites. A. PG involvement of the hands shows a higher
Q. Histopath of PG? percentage of lymphoproliferative than chronic
A. A sterile dermal or appendageal abscess inflammatory bowel disease.
Q. Which are the illnesses associated with PG? Q. What are the extracutaneous manifestations of PG?
A. - Inflammatory bowel disease (Ulcerative colitis, A. Involvement of upper airway mucosa, eye, genital
Crohns disease) mucosa, sterile pulmonary neutrophilic infiltrates or
spleen infiltrates, and neutrophilic myositis . Sterile
- Arthritis
cortical osteolysis has been observed adjacent to PG
- Hepatitis C, ulcers as another extracutaneous manifestation of the
- Seronegative rheumatoid arthritis, spondylitis disease.
- Lymphoproliferative disorders ( monoclonal Q. What is the differential diagnosis?
gammopathies, leukaemia, lymphoma, and A. (a) Vascular occulusive or venous disease including
myelodysplastic syndrome ) calciphylaxis which is particularly painful and
Q. Which drugs cause pyoderma gangrenosum? rapidly evolving.
A. - Propylthiouracil, pegfilgastrim - a granulocyte- (b) Vasculitis. That is a particular challenge when
stimulating factor PG develops in a patient with vasculitic
- gefitinib - an inhibitor of epidermal growth factor rheumatoid arthritis. Differential diagnoses
receptor. include Wegener's disease,anti phospholipid
syndrome, or in Adamantiades-Behcet's disease
Q. What is PAPA syndrome?
etc.
A. The autosomal dominant inherited disease is
(c) lymphomas or leukaemia.
characterized by familial recurrent nonaxial
destructive arthritis, severe cystic acne and PC. PAPA (d) Infectious disease. Eccthymata and deep
170 DYP SURVIVAL GUIDE FOR POST GRADUATES DYP SURVIVAL GUIDE FOR POST GRADUATES 171

[tropical] mycoses like sporotrichosis may Q. Can PG lesions be painless?


resemble PG. The rapid onset of post-surgical PG A. Yes. Proliferating chronic ulcers of PG without the
often reminds of acute deep skin infection such presence of an inflammatory halo, and slow
as erysipelas or gangrene . Late syphilis has evolution with marked tissue neutrophilia and
become more common again and may develop usually without coexisting diseases.
suppurative ulcerations. Deep viral herpetic
infections can resemble PG. Q. Can the ulcer be caused by necrobiosis lipoidica
diabeticorum?
(e) Exogenous tissue injury. Facticious panniculitis
as a part of the Münchausen syndrome may A. Ulcerative type of necrobiosis lipoidica diabeticorum
masquerade as PG .. Insect or spider bites can can present with ulcers resistant to therapy and
cause necrotizing skin ulcers. squamous cell carcinoma has been reported.
Degenerative disease of collagen due to diabetic
(f) Drug reactions. Pustular drug reactions
microangiopathy/immune complex disease /
Q. How do you treat PG? defective collagen synthesis .
A. - Systemic Glucortiocoids (0.5-1mg/kg of The immune reaction precedes and initiates the release
prednisolone)
of cytokines that finally cause the degeneration of
- Methyl prednisolone (1gm within 3-5days) collagen synthesis and granulomatous alterations.
- Immunosuppressants Q. How do you differentiate such ulcers?
- Sulfasalazin (1-4gm) A. Presence of yellow-brown, atrophic, telangiectatic
- Clofazamine plaques surrounded by raised, violaceous rim in the
- Clochicine legs. Giant ulcers with necrotic material and
- Infliximab cribriform base with erythematous raised border
have been reported.
Q. What is the prognosis?
Q. What are the factors causing NLD to ulcerate?
A. - 70% better prognosis after steroids
A. Trauma, the poor circulation over the pretibial area
- 66% after cyclosporine
and an exaggerated immune response have been
- 30% mortality suggested as the causes of ulceration.
Q. What is pathergy and in which diseases do you get
pathergy? h
Exaggerated,uncontrolled inflammatory respone to
a nonspecific stimulus as seen in:
- Pyoderma gangrenosum
- Behcets’ disease
DYP SURVIVAL GUIDE FOR POST GRADUATES 173

Q. Describe the stages of LGV?


9. LYMPHOGRANULOMA
A. The disease progresses through three stages:
VENERUM 1] PRIMARY STAGE- Initial infection of the genital
mucosa- characterized by the appearance of a
Q. What is Lymphogranuloma Venereum (LGV)? transient herpetiform ulcer, heals rapidly leaving
A. Lymphogranuloma Venerum is a STD caused by no scar.
Chlamydia trachomatis serovars L1, L2, L3 that 2] SECONDARY STAGE- Occurs after 2-6 weeks.
primarily affects lymphatics. Classically described as the inguinal syndrome,
Q. LGV is also known as- and is characterized by tender, unilateral inguinal
lymphadenitis resulting in suppurative buboes
A. 1] Durand-Nicolas-Favre disease-initially described
with fistulae and associated constitutional
in 1833 by Wallace, defined as a distinct clinical symptoms. Enlargement of the femoral and
entity by Durand, Nicolas and Favre in 1913 inguinal lymph nodes separated by the inguinal
2] Climatic bubo ligament produces the 'sign of the groove'.
Q. How does the organism enter the skin and establish (Greenblatt Sign)
infection? Anorectal syndrome- Patients complain of
mucopurulent anal discharge, rectal pain and
A. LGV affects mainly the lymphatic tissue in the
bleeding, tenesmus, constipation and Ulcerative
genitorectal area. The organisms enter the body via proctocolitis.
microscopic defects in the mucosa or the skin. The
3] TERTIARY STAGE- may occur many years after
microorganisms then enter the lymphatics, leading
the initial stages. The manifestations result from
to lymphangitis, perilymphangitis and infection of
fibrosis and lymphatic obstruction. Elephantiasis
lymph nodes. Over a period of many weeks to of the external genitalia causes polypoid
months, the inflammatory process expands and growths, fistulae, chronic ulceration, scarring and
results in periadenitis, involvement of a few deformity.
neighboring lymph nodes, the development of
Q. Describe the clinical manifestation in each stage of
abscesses (which can rupture), and the formation of LGV?
fistulae and strictures. Latent persistence of the
A. Initial manifestations: 3-12 days
microorganism within involved tissues may last for
many years. • Papule
• Erosion or ulcer
Q. What is the incubation period of LGV?
• Herpetiform vesicle
A. Incubation period is 3-12 days.
• Non-specific urethritis or cervicitis
172
174 DYP SURVIVAL GUIDE FOR POST GRADUATES DYP SURVIVAL GUIDE FOR POST GRADUATES 175

Uncommonly, Balanitis and nodular ulcerations. Q. What is Esthiomene?


Inguinal syndrome: 10-30 days up to 6 months A. Esthiomene (Greek- eating away); a primary infection
• Regional lymphadenopathy (mostly inguinal affecting the lymphatics of the scrotal, penis or vulva
and femoral, also perirectal, deep iliac) which may cause chronic progressing lymphangitis,
chronic oedema and sclerosing fibrosis of the
• Overlying erythema subcutaneous tissues of these structures. This results
• Constitutional symptoms in induration and enlargement of the affected parts
• Eruption of buboes and destructive ulceration.
• Pelvic inflammatory disease. Q. What is Fitz-Hugh-Curtis Syndrome?
Ano-genito-rectal syndrome: months to years A. Fitz-Hugh-Curtis syndrome is inflammation of the
liver capsule associated with genital tract infection
• Proctocolitis
associated with LGV. Classically presenting as sharp,
• Hyperplasia of intestinal and perirectal pleuritic right upper quadrant pain.
lymphatic tissue
Q. What is the difference between Bubo and
• Perirectal abscesses pseudobubo?
• Ischiorectal and rectovaginal fistulas A.
• Anal fistulas
BUBO PSEUDOBUBO
• Rectal strictures and stenosis
Enlarged lymph node Subcutaneous nodules
• Frozen pelvis and infertility
Seen in Chancroid, LGV Seen in Donovanosis
• Esthiomene
Other manifestations Q. What is the difference between bubo of chancroid
Urethro-genito-perineal syndrome and bubo of LGV?
Peno-scrotal elephantiasis A.
Erythema nodosum BUBO OF CHANCROID BUBO OF LGV
1] Matted 1] Non matted
Submaxillary or cervical lymphadenopathy
associated with oropharyngeal lesions. 2] Unilocular 2] Multilocular
3]More tender 3]Less tender
Q. What is Bubonulus?
4] accompanied by genital 4] not accompanied by genital
A. Primary LGV associated with lymphangitis of the
dorsal penis and formation of a large tender lesion. lesion
lymphangial nodule which is called Bubonulus. CHANCROID LGV
176 DYP SURVIVAL GUIDE FOR POST GRADUATES DYP SURVIVAL GUIDE FOR POST GRADUATES 177

90% unilateral 2/3 cases unilateral of C. Trachomatis from the site of infection.
Involves only one lymph Involves multiple nodes,
Specimens may be obtained from ulcers and the
rectum; or bubo aspirate.
node, hence unilocular hence multilocular
1] Nucleic acid amplification tests and confirmation
Round or oval in shape Sausage shaped
by real-time PCR assays for LGV-specific DNA.
Same consistency throughout Variable consistency
2] The Frei intradermal test, which was based on a
Overlying skin erythematous Overlying skin violaceous and positive hypersensitivity reaction to a purified
and thinned out wrinkled and thickened chlamydial antigen, is now only of historical
Associated with genital lesion Genital lesion heals by the interest.
in most cases time bubo is formed 3] Imaging studies using CT or MRI scan may be
Ruptures with single opening Ruptures with multiple openings useful if retroperitoneal adenitis or intra-
Discharge is thin, purulent Discharge is thick, rope like abdominal abscess is suspected, and a barium
enema may reveal the characteristic elongated
Giant chancroid develops at Multiple sinuses form without
stricture in rectal LGV.
site of rupture ulceration
4] Screening for other sexually transmitted
Groove’s sign (Greenblatt's sign) -ve Groove’s sign (Greenblatt's sign) +ve infections, including HIV, and viral hepatitis B
Q. What is lymphorrohoids or perianal condylomas? and C, should be undertaken.
A. Obstruction of the lymphatics and venous drainage Q. What are the differential diagnosis of LGV?
of the lower rectum produces perianal outgrowths A. Inguinal syndrome- Syphilis, Chancroid, Granuloma
of lymphatic tissues (that resemble haemorrhoids) are inguinale, Cat-scratch disease, Tularaemia, Plague,
called lymphorrohoids or perianal condylomas. Mycobacterial disease and Lymphoproliferative
Q. Which is the most sensitive method for diagnosis of disorders.
LGV? In women, Pelvic Inflammatory Disease and tubo-
A. Polymerase chain reaction. ovarian abscess.
Q. What are the diagnostic modality of choice used Anogenital syndrome-enteric infections; other STIs
in diagnosis of LGV? caused by gonorrhoea, oculogenital strains of
C.trachomatis, herpes simplex virus and
A. The diagnostic method of choice is by nucleic acid cytomegalovirus; and actinomycosis and
amplification tests and confirmation by real-time PCR schistosomiasis.
assays for LGV-specific DNA.
Q. What is treatment of LGV?
Q. What are the diagnostic modalities used in diagnosis
A. Recommended Regimen
of LGV?
Doxycycline 100 mg orally twice a day for 21 days
A. Definitive diagnosis is by detection of the L serovar
178 DYP SURVIVAL GUIDE FOR POST GRADUATES

Alternative Regimen
10. PELLAGRA
Erythromycin base 500 mg orally four times a day
for 21 days. HISTORY TAKING
Q. How will you treat sex partners of LGV?
Name/Age/ Sex/Occupation/Residing at... /
A. Persons who have had sexual contact with a patient Hailing from.../Marital status
who has LGV within the 60 days before onset of the
- more common in middle age persons
patient's symptoms should be examined, tested for
urethral or cervical chlamydial infection, and treated - more common in low socio economic people,
with a chlamydia regimen (azithromycin 1 gm orally courier boys, salesma, farmer
single dose or doxycycline 100 mg orally twice a day - Endemic in Deccan Plateau of India because they
for 7 days). eat sorghum/jowar as staple food
Q. How will you treat LGV in pregnancy? Chief complaints:
A. Pregnant and lactating women should be treated with History of presenting complaints
erythromycin. Did the rash start with redness/bullae/scaling ?
Q. How will you treat LGV in HIV? Onset, Duration, Sites involved,(extensors/front and
A. Persons with both LGV and HIV infection should side of neck, back, buttocks, axillae, groins, pressure
recieve the same regimens as those who are HIV points.
negative. Prolonged therapy might be required, and Progression - Whether facial rash on and off(lupus
delay in resolution of symptoms might occur. erythematosus) or continuous(pellagra)
h History of Diarrhoea - Duration, Frequency, Large/
small volume, with or without blood.
h/o episodes of sleeplessness/fear/anxiety
History of redness in mouth/burning sensation in the
mouth while eating spicy food. Difficulty in eating
On enquiry
History of Photosensitivity - increased redness/
burning/itching while exposure to sunlight. History
of bloating, weight loss, aggravation of symptoms
after taking specific food(wheat, barley, oat),
Steatorrhoea, anorexia, fatigue, abdominal pain,
vomiting, dyspepsia (Malabsorption)
179
180 DYP SURVIVAL GUIDE FOR POST GRADUATES DYP SURVIVAL GUIDE FOR POST GRADUATES 181

History of any abdominal surgery. Income


History of any recurrent oral ulcers,joint pain, On and Wife works or not
Off rash over malar area, Seizures, Chest Pain, Size of the room how much square feet?
Difficulty in breathing (SLE)
QOL:
History of blistering, after minimal trauma,easy
Do fights lead to increased alcohol intake
fragility, excessive hair over body (Porphyria cutanea
tarda) Has the disease affected professional and sexual life?
History of flushing, palpitations, giddiness and black History of palpitations, loss of weight despite good
out(low B.P.), aggravation of symptoms after eating appetite, intolerance to heat or cold, anxiety,
cheese, chocolate.(Carcinoid syndrome) insomnia, hairfall, muscle weakness, irregular
mensus.(Thyrotoxicosis)
History of tremors, inability to balance while
walking,swaying to one side(Hartnup's disease) Drug History:
Past history anti-epileptic/ anti-tubercular/ oral contraceptive/
anti-hypertensive
h/o any skin disease/major illness/sexual exposure
Family History:
Diet:
Any family member having similar disease(inborn
Has he foregone meals
errors in metabolism).
Bouts of vomiting
Menstrual and Obstetric history(females):
Type of food (junk food/food fads)
Menses regular/irregular.
Breakfast/lunch/dinner
on-going pregnancy. if any
Veg/nonveg
General Examination-
Staple diet-rice/maize
general condition
Fruits and veg
Conscious, Cooperative, Oriented to time, place and
Total calorie intake person,
Protein intake any evidence of dehydrated.
Personal history: Temperature/ BP/ PR/ RR
Habits- Alcohol/Smoking/ Recreational drugs PR is significant for sign of dehydration.
Sleep /Bowel/Bladder habits Are extremities cold and clammy ?
Socioeconomic history: Pallor-
Earning members Icterus-(sign of liver failure)
182 DYP SURVIVAL GUIDE FOR POST GRADUATES DYP SURVIVAL GUIDE FOR POST GRADUATES 183

Clubbing-(sign of liver failure) Q. what does the word pellagra mean?


Cyanosis- A. Pelle is derived from Italian meaning skin and agra
Oedema feet-(sign of liver failure) signifying rough .
Lymphadenopathy Q. What are the four D's of pellagra?
Cutaneous Examination- A. Diarrhoea , dermatitis , dementia , death.
Rash- Symmetrical/Asymmetrical, Sites involved, Q. Who described pellagra first ?
present over photo-exposed area or not. A. Gasper Casal described pellagra in 1762 in the
Well-demarcated or ill demarcated from normal skin. Asturias region of northern spain.
Examine genitals. Are genitals involved? If yes, He termed the disease as "mal de la rose" because
describe the rash. all the affected patients had typical reddish and
Oral cavity and perioral area examination- glossy rash on the dorsum of hands and feet.
Dry, cracked and fissured lips; angular chelitis, Oral Q. Who termed the word pellagra?
ulcer, Tongue(Tip and margin of tongue, Papillae),
A. Frapolli named this disease as vulgo pelegrain and
Gingivitis/Nasolabial dyssebacia
first used the term pellagra in 1771.
Systemic Examination-
Q. Which regions is pellagra commonly found in?
CNS Examination-
A. Pellagra chiefly occurs in developing nations where
Higher functions-Consciousness(as clouding of
corn and corn products are the major food source in
consciousness can occur)
ethnic populations with diets deficient in niacin/
Memory(short term/Recent memory) tryptophan.
Orientation
Q. What are the causes of pellagra?
Cerebellar functions- Incoordination, Ataxia.
Q. Primary - tryptophan deficiency and niacin dietary
Per Adominal Examination-
deficiency.
Any organomegaly(Liver- Palpable/Not, If palpable-
Secondary-
liver span, tender or non tender). Any other sign of
liver failure(Flapping tremors, spider nevi, Caput 1. Prolonged diarrhoea
medusa). 2. Anorexia nervosa
CVS Examination: 3. Chronic alcoholism
Heart sounds. 4. Chronic colitis
RS Examination:
5. Severe ulcerative colitis
Any adventitious sound.
6. Hepatic cirrhosis
Diagnosis: Pellagra.
7. Carcinoid tumour
184 DYP SURVIVAL GUIDE FOR POST GRADUATES DYP SURVIVAL GUIDE FOR POST GRADUATES 185

8. Hartnups syndrome Q. What are pellagroids ?


9. Tuberculosis of g.i tract. A. Abortive forms of pellagra are known as pellagroids,
10. Medications - 5-FU, pyranzinamide, also described as erythema pellagroids.
6-mercaptopurine, Hydantoins, Ethionamide, Q. What are the skin findings of pellagra?
Phenobarbital, Azathioprine . A. The dermatitis caused by pellagra is bilaterally
11. HIV disease. symmetrical eruption at cutaneous sites of solar
exposure. The dermatitis begins as erythema
How does alcohol cause pellagra?
changing to an exudative eruption on the dorsa of
1. They cant afford the right type of food hands, face, neck, and chest with pruritus and
2. They don't buy the right food.Increased calories burning, the initial bright red demarcated erythema
because of junk food increase the need of nicotinic becomes brown and dull.
acid Acute pellagra resembles sunburn in initial stages,
3. Poor appetite because of gastritis vesicles and bulla can also be present ( pemphigus
pellagrosus).
4. Poor absorption intestinal mucosa
The back of hands are the commonest site . it may
5. Decreased absorption of pyridoxine
exend upward to form the glove or gauntlet of
6. Improper metabolism of niacin due to liver pellagra .
dysfunction.
Characteristic well marginated eruption on the front
Q. Describe the pathogenesis of pellagra? of the neck ( casals necklace ) on the C3 and C4
A. Primary pellagra results in deficiency of niacin, dermatome.
resulting from inadequate dietary nicotinic acid or On the face an erythematous rash extending from the
its precursor essential amino acid tryptophan . niacin nose to the cheeks, chins, and lips may resemble
is required for adequate cellular function and lupus erythematosus- butterfly rash.
metabolism as an essential component in co-enzyme
1 and co-enzyme 2 , these compounds are important Pellagra may also present as perineal lesions. Scrotal
co-enzymes for glycolysis, protein and amino acid and perineal erythema with erosions are common .
metabolism, pyruvate metabolism, as cellular Q. What are the G.I. features of pellagra?
functions in multiple organs and tissues are affected A. Soreness of tongue and mouth, loss of appetite,
by niacin deficiency, the clinical expression of indigestion and painful abdomen. Diarrhoea may
pellagra is diverse. lead to nausea, vomiting and diarrhoea - which may
Q. what are the early clinical symptoms of pellagra? inturn lead to malabsorbtion forming a vicious cycle.
A. Lassitude, weakness, loss of appetite, mild digestive Q. What are the neurologic features of pellagra?
disturbance and psychiatric and emotional stress. A. They occur late in the course of disease, early
186 DYP SURVIVAL GUIDE FOR POST GRADUATES DYP SURVIVAL GUIDE FOR POST GRADUATES 187

symptoms are insomnia, fatigue, nervousness, plexus. Older lesions may have epidermal
depression. hyperkeratosis and parakeratosis, with variable
Neurologic manifestations include sensorimotor acanthosis. There is often increased epidermal basal
neuropathy, parkinsonism, retinitis and optic layer melanin. Eventually, there may be epidermal
atrophy. atrophy overlying dermal fibrosis, sebaceous gland
atrophy and a chronic lymphohistiocytic perivascular
Q. Enumerate the differential diagnosis of pellagrous
infiltrate.
skin lesions?
Q. How can pellagra be prevented ?
A. Porphyria cutanea tarda and variegate porphyria,
hartnups syndrome and drug induced pellagroid A. Pellagra can be prevented by intake of a protein-rich
reactions. Photoallergy reactions can be similar. diet. Food sources of niacin, and/or tryptophan
include nutritional yeast, eggs, bran, peanuts, meat,
Q. How is the diagnosis of pellagra established ?
poultry, fish, red meat, whole-grain cereals, legumes
A. Levels of urinary metabolites of N- methyl and seeds. Recommended daily allowance are as
nicotinamide and N- methyl - 2 - pyridine -5- follows: for infants 5-6 mg, children 9-13 mg, adults
carboxamide are decreased in pellagra. Urinary levels 13-20 mg, and pregnant and lactating mothers 17 mg
of NMN below 0.8mg indicate niacin deficiency. and 20 mg, respectively.
Q. What is the treatment of pellagra? Optimal supplementation is 20-30 mg daily.
A. Oral administration of nicotinamide or niacin 100- 300 Nutritional education can be important.
mg/day in three separate doses. Supplementation of grain cereals with niacin may be
Mental changes disappear within 48 hours, but skin desirable. For secondary prevention one should
lesions take 4 weeks to respond, most patients need avoid sun exposure during the active phase of the
to be given riboflavin and pyridoxine also, and a diet disease. Close dietary follow up of the patient upon
rich in calories and proteins to address malnutrition. recovery helps prevent recurrence of pellagra.
Q. Histopathology of pellagra?
h
A. Pathological changes in the skin are relatively
nonspecific.
Vesicles, if present, may arise subepidermally as a
result of vacuolar degeneration of the basal layer, or
intraepidermall as a result of intense spongiosis. The
upper half of the Malphigian layer may show striking
vacuolar alteration. There is also perivascular
lymphocytic infiltrate of the superficial vascular
DYP SURVIVAL GUIDE FOR POST GRADUATES 189

On enquiry:
11. PSORIASIS
What was the odour/type/amount of scales
Vital Data : Loose(fall off easily)scales/adherent scales
Name Age/Sex/Religion/Occupation/Staying at/ History for etiology/ trigger factors:
Hailing from/Marital status H/o trauma at the site of the initial site or did the
Chief Complaints subsequent lesions occur at sites of injury?
Skin : H/o drug intake prior to onset of initial lesions or
Red raised scaly/nonscaly Lesions _______________ were the subsequent lesions associated with drugs
days/ weeks/months/years on ….(knees/elbows) (topical or systemic)?
(In psoriasis the lesions are usually present for a H/o any infections prior to onset of initial lesions or
number of years but the patient usually comes when were subsequent lesions associated with any fever,
there is a flare…. So one can say that lesions are
sore throat,dental, GI infections or other infections?
since______ years but flared up since __________days
Symptoms ; Usually a patient with psoriasis has mild History of sexual exposure/diarrhoea (protected or
to moderate itching or is asymptomatic. unprotected)(Reiters disease,Secondary syphilis)
Joints : H/O recurrent oral ulcers and photosensitivity(for
Which joints / pains-swelling-blocked movements / subacute lupus erythematosus)
morning or evening stiffness. H/o stressful events before initial lesions
Onset Duration Progress History for complications
Patient was apparently all right _____ years back Joint problems: History of low back/hip/finger pain/
when he noticed reddish, scaly rash(peasized/ swelling/morning or evening stiffness/difficulty in
coinsized/palmsized/dewdropsized), initially over movement
the ________. These then spread to involve…
(sequence of spread). There was h/o waxing and GI problems: H/o recurrent abdominal pain,
waning, with new lesions occurring in (seasonal swelling. Bloating or irregular bowel movements (for
variations). Some lesions healed on their own or with IBD)
treatment to leave behind pale areas. The lesions History of any chest pain, palpitations or
slowly increased over the past few years but have
breathlessness , rapid gain in weight (metabolic
rapidly increased since the past few days.Mention
whether lesions are static/progressive/regressing. syndrome)
Pruritus H/o spread of lesions to entire body with or without
pus filled lesions and fever
continous/intermittent.
mild/moderate/severe (erythroderma or pustular psoriasis)
188
190 DYP SURVIVAL GUIDE FOR POST GRADUATES DYP SURVIVAL GUIDE FOR POST GRADUATES 191

If female, Cutaneous Examination


Any relation with present or previous pregnancies? Primary Lesion: Papules/Plaques/Pustules
H/o treatment taken Secondary Lesions: Scales/Crusts /Any erosions
Topical agents Distribution:Symmetrical/Asymmetrical
Maximum involved sites:
Oral therapies/Phototherapy
Relative sparing:
Past history
Body Surface Area involved: (1 palm = 1%)
History of similar episodes earlier in the past
Description of primary lesion:
History of any skin problem
Plaque: Well defined, bilaterally symmetrical,
Drug history erythematous(if salmon coloured say so), scaly over
Personal extensors/flexors/scalp/palms/soles/face. Do the
scaly plaques extend beyond frontal margin of scalp?
Sleep,Bowel,Bladder disturbances
Scales:
Addictions: Alcohol,Smoking
Dry/greasy
Medical history: Hypertension/Cardiovascular
Silvery White/Yellowish
diseases,liver pathology/kidney diseases/
hyperlipedimia/pain/morning stiffness/loss of Hyperkeratotic/Ostraceous/Rupoid
range of motion usually of thoracic spine. Do the scaly plaques extend beyond frontal margin
Family History-similar complaints/history of Auspitz sign+/-
rheumatoid arthritis If positive:
Quality of life: interfering with function in On removal of scales(grattage) with the blunt end of
workplace/relation with spouse/outdoor activities a pin,the scales are loose and easily removed because
of parakeratosis. On further grattage, a glistening red
ON EXAMINATION membrane(Duncan bulkeley)is seen because of
General constitution: fair/poor suprapapillary thinning of epidermis. On further
Afebrile/febrile scraping, pinpoint bleeding is seen because of
rupture of dilated and tortuous capillaries in
Pulse/Bloodpressure/respiratory rate/Abdominal papillary dermis.
girth/Weight
Candle Wax sign(Signe de la tache de bougie)
Pallor/Clubbing/Cyanosis/Lymphadenopathy/ On removal, scales are loose and the coherence of
Edema Feet scales is like when you remove the wax of a candle.
192 DYP SURVIVAL GUIDE FOR POST GRADUATES DYP SURVIVAL GUIDE FOR POST GRADUATES 193

Woronoffs ring: How will you manage this patient?


Presence of hypopigmentation around a healing - First confirm diagnosis by skin biopsy
psoriatic plaque because of PgE2 deficiency - Investigate patient for underlying associated
because of inhibitor of prostaglandin synthesis. metabolic disorder.
(vasoconstriction) • BMI
Deck Chair sign: Sparing of abdominal folds • Blood sugars
(characteristic of papuloerythroderma of Ofuji)
• Serum lipids
Pavithran Nose sign: Sparing of tip of nose in severe • ECG
cases of erythroderma
• USG Abdomen for nonalcoholic fatty infiltration
If papules, are they guttate/scaly/on removal check
- Investigate for treatment
whether they leave behind normal/shiny red
surface(wafer like oblaten scale of pityriasis • CBC
lichenoides chronica). • Hb
Deep dermal tenderness (Buschke Olendorf sign)+/ • ESR
- (for secondary syphilis) • LFT
Do the papules have collarette of scaling and are in • RFT
Christmas tree pattern?(for pityriasis rosea) • CXR
Are the scales greasy and in seborrhic areas? • MT
(seborrhic dermatitis) • Serum uric acid
MUSCULOSKELETAL SYSTEM Treatment
Joints: Hands/Knees/Elbows/Wrists/Ankles/ - Topical- Corticosteroids- Superpotent for 2 weeks
Sacroiliac /lumbosacral
Maximum 45 gms/week
Check for pain/swelling/range of motion/
- Calcipotriol
deformities
Maximum should ot exceed 100gms/week
Diagnosis:
- Emollients
A patient of age…with mild/moderate/severe
- Systemic- If patient is not responding to topical
psoriasis(morphological type/stable)involving
treatment/impaired life style/more than 20%BSA/
BSA…… with/without Auspitz sign+/ Koebners /
pustular/erythrodermic
Woronoffs ring with PASI score with nail/scalp
involvement/joint involvement/treated partially or Methotrexate/Oral retinoids/Narrow band therapy/
untreated) PUVA/Combination therapies/Cyclosporin
194 DYP SURVIVAL GUIDE FOR POST GRADUATES DYP SURVIVAL GUIDE FOR POST GRADUATES 195

Q. Why is it psoriasis vulgaris? positivity,epitrochlear lymphadenopathy,mucous


A. In view of bilateral symmetrical well demarcated patches, condyloma lata, pigmentation or
scaly erythematous (salmon coloured) plaques on the papulosquamous lesions in the palms and soles, with
extensors of extremities/trunk/lumbosacral with collarette of scaling (Biette’s collar).
silvery white scaling…… Auspitz positive. Q. Why is it not pityriasis lichenoides chronica?
Q. What will a skin biopsy show? A. Lesions are not reddish brown papules with wafer
A. Parakeratosis, Munros microbscesses, Hypogranulosis like (oblaten) scales which on removal leaves behind
Spongiform pustule of kogoj, Suprapapilary thinning glistening shiny surface.Sometimes deep dermal
of epidermis, regular elongation of clubshaped / tenderness test may be positive.No scalp
camelfoot rete ridges, dilated and tortous capillaries
involvement/pitting nails.
in papillary dermis
Q. Difference between seborrhic dermatitis and
Q. What is the differential diagnosis?
psoriasis?
A. Reiters disease / Reactive arthiritis
A.
Seborrheic dermatitis
Seborrhic Psoriasis
Secondary Syphilis
dermatitis
Pityriasis Lichenoides Chronica
Parapsoriasis Morphology Greasy yellow Silvery White
Drug induced papulosquamous eruptions scales scales, usually dry
Subacute Lupus erythematosus Distribution Flexural areas extensor areas.
Lichen Planus(guttate papulosquamous lesions)
Lupus vulgaris Scalp lesions Scalp lesions do They extend beyond
But it will vary according to patient extension not extend frontal scalp margin.
Q. Why is it not Reiters/SARA/Reactive Arthritis? beyond frontal (corona psoriatica)
A. There is no preceding history of diarrhea/urethritis, margin. (corona
limpet like crusts, circinate balanitis,keratoderma seborrhica)
blenorrhagicum,annulus migrans, tendon friction
rubs and ocular complaints(uveitis) with Auspitz Negative Positive
arthritis.(reactive) Joint Pains - +
Q. Why is it not Secondary syphilis?
Metabolic - +
A. There is no history of sexual exposure, chancre in the
disease
genitals,absence of pruritus, deep dermal tenderness
196 DYP SURVIVAL GUIDE FOR POST GRADUATES DYP SURVIVAL GUIDE FOR POST GRADUATES 197

Q. Does psoriasis remit in pregnancy? Q. Which diseases show Auspitzs sign positive?
A. Yes except impetigo herpetiformis which occurs in A. Psoriasis
pregnancy. Dariers disease
Q. Which drugs aggravate psoriasis? Actinic keratosis
A. Naproxen, Lithium, Antimalarials.( So in psoriatic Q. In a case of psoriasis,can you get Auspitz negative?
arthritis indomethacin is preferred) A. Yes.(Guttate/Flexural/Pustular/Scalp/Partially
Q. Why do you ask history of smoking? treated psoriasis)
A. Smoking increases psoriasis by two fold. Q. Which diseases show koebners phenomenon?
Q. Why do you ask about alcoholism? A. Psoriasis
A. Facial psoriasis is more common in alcoholics and if Lichen planus
we want to start patient on methotrexate and Verruca vulgaris
retinoids. Lichen nitidus
Q. What is the relation between HIV and psoriasis? Q. Which conditions show Woronoffs ring?
A. The frequency of psoriasis is same but severity will A. Psoriasis
be more and there is increased incidence of arthritis,
Halo nevus
dactylitis or enthesitis
Lichen planus
Q. What is the relationship between psoriasis and
pregnancy? Q. What is Boyd and Neldner classification of
koebners phenomenon?
A. There can be decreased fertility and increased fetal
A. True Koebners - Psoriasis,Lichen planus,vitiligo
loss (more in pustular psoriasis).
Pseudokoebners - Molluscum contagiosum,Warts
Q. What is the risk of malignancy in psoriasis?
Pyoderma gangrenosum, Sweets
A. Psoriatic skin is intrinsically resistant to malignancy
because of low levels of aryl hydrocarbon hydroxylase. Syndrome.
Q. What is Renbeok phenomenon? Reverse Koebners - Granuloma annulare
A. Psoriasis disappears in the area of wrist band. Kyrles disease
(Negative koebners renbeok-opposite of koebner) After systemic infections like enteric fever, psoriasis
may disappear.
Q. What is Brocq’s phenomenon?
Q. In which disease sacral pinking is prominent?
A. In lichen planus patients rub instead of scratching-
diffuse bleeding and pain. A. Psoriasis (called as abrahamowitz sign)
198 DYP SURVIVAL GUIDE FOR POST GRADUATES DYP SURVIVAL GUIDE FOR POST GRADUATES 199

Q. What is the classic Xray finding in psoriatic arthritis? 0 - No involvement


A. Opera glass deformity and sausage shaped fingers. 1 - 1-9%
Q. How do you calculate PASI score? 2 - 10-29%
A. On the basis of desquamation,induration and 3 - 30-49%
erythema
4 - 50-69%
Plaque Rating score Head Upper ) Trunk Lower
characteri (A1) limbs (A3) limbs 5 - 70-89%
stics (A2) (A4) 6 - 90-100%
Erythema PASI= 0.1(Eh + Ih +Dh) Ah + 0.2(Eu + Iu + Du ) Au +
(E) 0-None 0.3(Et + It + Dt ) At + 0.4(El + Il + Dl ) Al
1-Slight
PASI varies between 0 and 72 in steps of 0.1
2-Moderate
Q. How do you calculate BMI?
3-Severe
A. Weight in kg/(height in metres)2
4-Very severe
Induratio Same as above Normal-18.5-25 kg /m2
n(I) Overweight-25-30 kg /m2
Scaling Same as above Obesity-from 30 to over 40 kg /m2
(D-desqua- Q. In which diseases geographic tongue is seen?
mation) A. Pustular psoriasis
Add together each of the 3 scores for each of the body Reiter’s
regions to give 4 separate subtotals Physiological
Head and neck-represent 10% of body surface Atopy
area(BSA)
Q. What are types of mucous membrane involvement
Upper extremity-20% OF BSA in psoriasis?
Trunk(includes axilla,groin,genitals)-30% of BSA
A. Geographic tongue
Lower extremity(including buttocks)-40% of BSA
Silvery white micaceous scales
So they are given corresponding weighage in scoring
Solitary patch on glans penis
by multiplying their scores by 0.1,0.2,0.3 and 0.4
respectively. Gingival lesions
200 DYP SURVIVAL GUIDE FOR POST GRADUATES DYP SURVIVAL GUIDE FOR POST GRADUATES 201

Q. Nails in psoriasis? - Sarcoidosis


Nail matrix involvement Nail bed involvement - Trauma
Pitting/ Crumbling Subungual hyperkeratosis - Idiopathic
Red lunula Onycholysis Q. Which are the drugs causing psoriasis?
Leuconychia Salmon patch/Oil drop A. - HCQS
sign.?splinter haemorrhages - Anticonvulsants
Q. How do you differentiate pitting of alopecia areata - Beta blockers and ACE inhibitors
and psoriasis? - OHAs
Alopecia Areata Psoriasis - Nsaids
Scotch grid pattern- Irregular arrangement Q. Which drugs cause exclusively nail changes in
psoriasis?
arranged at regular
A. - Lithium
intervals.
- Beta blockers
Shallow Deep and shallow
Q. Which NSAID is preferred in psoriatic arthropathy?
Smaller Larger
A. Indomethacin
Q. In how many patients nail pitting is seen? Q. What is Caspar criteria for psoriatic arthritis?
A. 10-50% A. - Psoriatic nail dystrophy
Q. what does the number of nail pits signify? - Past or family history of psoriasis
A. More than 20 pits-psoriasis - Current or past history of dactylitis
More than 30 pits-psoriatic arthropathy - X-ray: New bone formation(juxtaarticular)
Q. Causes of nail pitting - Negative Rheumatoid factor test
A. - Psoriasis Q. How will you differentiate psoriasis from RA?
- Reiters / SARA PSORIATIC RHEUMATOID
- Alopecia Areata ARTHRITIS ARTHRITIS
- Lichen planus Usually distal inter- Proximal interphalangeal
- Eczema phalangeal joint joint
- Pityriasis Rubra Pilaris Asymmetrical Symmetrical
- SLE Oligoarthritis Polyarthritis
202 DYP SURVIVAL GUIDE FOR POST GRADUATES DYP SURVIVAL GUIDE FOR POST GRADUATES 203

No morning stiffness Morning stiffness present 2%salicylic acid


Sausage shaped deformity Swan neck deformity 24%zincoxide and starch each
Enthesitis Not common 2.5%hard paraffin
Xray: no osteopenia Osteopenia seen White soft paraffin qs
Q. Do you get hair loss in psoriasis? Q. What is Goeckerman regimen?
A. No hair loss except erythrodermic psoriasis and A. 1-5%coal tar paste to be applied and kept for 12-24
corona psoriatica.(Hair loss is present in tinea capitis) hours- then UVB.
Q. What is white psoriasis? Q. What is Miami university modification of
A. Interdigital psoriasis Goeckerman regimen?
Q. How will you treat? A. Keep coal tar paste(1-5%) only for 2 hrs then have a
bath followed by UVB.
A. First line : Indomethacin
Q. What is Stanford regimen?
Second line : Methotrexate, Acitretin, Sulphasalazine
A. Anthralin at night and topical steroids in the morning.
Oral steroids(GPP of pregnancy)
Q. What is holiday and weekend therapy?
Ciclosporin
Leflunomide A. Calcipotriol on weekdays and topical steroids in the
weekends.
Third line : Infliximab
Q. What is SCAT?
Etanercept
A. Short contact anthralin therapy 2-4% for 20 mins.
Q. What is the indication of systemic steroids in
psoriasis? Q. What is the maximum dose of superpotent TCS?
A. - Pustular psoriasis A. 45 gms/week
- Erythrodermic psoriasis Maximum 14 days it should be applied
- Psoriatic arthritis Q. What is the maximum dose of calcipotriol?
- Psoriasis with metabolic disturbances A. Should not exceed 100gms/week
Q. What is ingram coal tar bath? Q. What do you mean by metabolic disorder?
A. First give UVB then apply anthralin(0.05-0.1%) in A. Hypertriglyceredimia(TGL>150,HDL<40 in males
ointment or Lassars paste. and<50 in females)
Q. What is Lassars paste? Diabetes mellitus(FBS>100)
0.2-0.8%dithranol Hypertension
204 DYP SURVIVAL GUIDE FOR POST GRADUATES DYP SURVIVAL GUIDE FOR POST GRADUATES 205

Obesity(Waist circumference>102cm in males Q. What are the noninvasive tests for MTX toxicity?
and>88cm in females) A. Aminoterminal propeptide of Type 3 collagen levels
Atherosclerosis (p III np) > 4.2ng/ml indicate hepatotoxicity.
Cardiac disease(coronary insufficiency) HMRS-Proton magnetic resonance spectroscopy-
measure fat content
Non alcoholic fatty infiltration
PMRS-to estimate cell membrane turnover and
Q. What caution you should take when giving fibrosis.
methotrexate to elderly people? Q. How about MCV ?
A. Start in low dose and look for non-alcoholic fatty liver A. MCV doesn't decrease inspite of folic acid treatment
disease (NAFLD). If WBC < 3500, Platelet <1 lakh persisting for more
Q. How much time does it take for methotrexate, than 1 week then need to decrease dose.
cyclosporine and acitretin to start to act and when is Q. Risk factors for cytopenia?
the action near complete? A. Daily MTX
A. Methotrexate- Starts within 6-8 weeks. Some show No folic acid
improvement in 72 hrs. Use of NSAIDS
Cyclosporin - starts within 3-4 weeks Sulphomethaxozole Trimethoprim
Acitretin - starts within 4-8 months for psoriasis Renal impairment
vulgaris. Q. How can you prevent MTX toxicity?
Takes 10 days for pustular psoriasis. A. By giving folic acid on all days except the days of
Q. What is the permissible maximum dose? MTX
A. 20/25 mg weekly Q. What is the dose of leucovorin and how is it
administered?
Q. How does acute toxicity of MTX manifest?
A. 20 mg IV or higher than last dose of methotrexate
A. Apthous stomatitis because of thrombocytopenia within 4-6 hours and repeated after 6 hours if renal
56. What is the alarming level of serum MTX ? parameters abnormal to avoid precipitation of MTX
The risk of potentially fatal toxicity is significantly in renal tubules and till serum MTX level
<0.01micromol/min.
increased with high-dose methotrexate in the setting
of renal impairment and is related to prolonged Q. If renal impairment what will you do?
periods of exposure due to elevated serum levels. A. - Reduce by half dose
At 24 and 48 h, toxicity is greater if the serum levels - Adequate hydration
are above 20 and 2 µmol/l, respectively - Alkanization of urine with soda bicarbonate
206 DYP SURVIVAL GUIDE FOR POST GRADUATES DYP SURVIVAL GUIDE FOR POST GRADUATES 207

Q. Why is folic acid given? - Subungual hyperkeratosis and onycholysis without


A. - Decreases myelosuppression evidence of fungus.
- Nail Pitting
- Decreases macrocytosis seen with methotrexate
- Sterile multiple paronychia
- Decreases nausea and vomiting and so increases
tolerance. - Recalcitrant scaly otitis externa
Q. What are the main side effects of MTX? - Keratolysis like lesions of palms and soles
- Eczematous lesions on palms and soles
A. - Thrombocytopenia
- Penile sharply marginated areas of erythema
- Oral ulcers
without evidence of fungus.
- Ulcerations on psoriatic plaques - Intertrigo with sharp margination of erythema.
Q. What is the cumulative dose which can be given? Q. What is the hallmark of psoriatic lesions?
A. 4gm A. Sharply demarcated margins with clear cut borders.
66. What is Type 1 and Type 2 psoriasis? Q. What does scaling indicate?
Type 1 Type 2 A. Stable chronic plaque psoriasis
Onset before 40 yrs Onset after 40 yrs Q. What does erythema indicate?
Bimodal onset Not bimodal A. Unstable progressing lesions
Severe and recurrent Mild Q. Name morphological types of psoriasis?
First degree relatives affected Not affected A. - Plaque/Pustular
Guttate psoriasis Chronic plaque psoriasis - Nummular/Guttate/Gyrate
- Rupioid/Elephantine/Ostraceous
Q. How many patients have koebners in psoriasis?
Q. What are the different patterns of psoriasis?
A. 30% of psoriasis patients have koebners. It is
A. - Extensors
associated with early onset of psoriasis.It is an
indicator of disease activity and may have a - Inverse
prognostic value. - Sebopsoriasis
Q. What are signs of borderline psoriasis? - Widespread
A. - Hyperkeratotic plaques without scaling on extensor Q. Based on inflammation what are the types?
surfaces. A. - Inflammatory
- Severe dandruff - Hyperkeratotic
208 DYP SURVIVAL GUIDE FOR POST GRADUATES DYP SURVIVAL GUIDE FOR POST GRADUATES 209

Q. Types based on time of first onset? Q. What can you find in the oral cavity in GPP?
A. Early and Late A. Acute geographic tongue
Q. Types based on velocity of propagation? Q. What is impetigo herpetiformis?
A. - Stable A. Generalised Pustular psoriasis in pregnancy
- Unstable Q. What is mechanism of action of methotrexate?
- Eruptive A. MTX inhibits dihydrofolate reductase, an enzyme
Q. What is psoriatic leukoderma? that reduces folic acid to tetrahydrofolic acid. This
inhibition interferes with DNA synthesis and cell
A. Hypopigmentation usually associated with clearing
reproduction.
Q. What is the most common type of psoriasis?
Q. What is the drug of choice of pustular psoriasis?
A. Chronic Plaque psoriasis
A. Oral retinoids(acitretin) except in pregnancy where
Q. What is the most common exanthematic type of oral steroids are used.
psoriasis?
Q. What is the role of cyclosporine in pregnancy?
A. Guttate psoriasis
A. Cyclosporin is a selective immunosuppressant that
Q. What is the risk of developing chronic plaque has been effectively used in the treatment of
psoriasis after the first episode of guttate psoriasis? generalised pustular psoriasis and is grouped into
A. 40% category C for pregnancy drug risks.
Q. What is the age group of patients with guttate Q. What is the therapeutic wavelength of UVB in
psoriasis? psoriasis?
A. Children and young adults with family history of A. 311 nm
psoriasis usually following streptococcal infections/ Q. What is narrow band UVB?
stress.
A. Wavelengths ranging from 308-311 nm are called
Q. How many patients of psoriasis vulgaris can get
NBUVB
pustular psoriasis?
Q. Advantage of NBUVB?
A. 2-5%
A. Shorter wavelengths of UVB are not very effective in
Q. How many patients of plaque type psoriasis can get
psoriasis, also they carry a higher risk of phototoxicity
occasional pustular lesions?
& skin cancers. These risks are minimized with
A. 20% NBUVB
Q. What are the nail changes in GPP? Q. Which type of lesions respond poorly?
A. Subungual pustules A. Thick lesions over palms and soles & scalp lesions
210 DYP SURVIVAL GUIDE FOR POST GRADUATES DYP SURVIVAL GUIDE FOR POST GRADUATES 211

Q. How is the therapy scheduled? Q. Dosing of acetretin in psoriasis?


A. Increasing suberythemogenic doses of UVB given A. 25-50mg/day with food for 3-6 months
thrice a week for 2-3 months followed by once a week Q. Indications of using retinoids in psoriasis?
for next 2-3 months. Initial dose is about 80% of MED,
A. Erythrodermic psoriasis, pustular psoriasis, psoriasis
and is increased by 20% at each exposure till erythema
is seen associated with HIV infection
Q. Contraindications to UVB phototherapy? Q. Indications of cyclosporine in psoriasis?
A. Photosensitivity, patient on phototoxic medications, A. Erythrodermic psoriasis, pustular psoriasis, psoriatic
unstable psoriasis, past history of skin cancer arthropathy, nail psoriasis, scalp psoriasis
Q. Oral psoralens used in PUVA therapy? Q. How to monitor a patient on cyclosporine?
A. 8-methoxy psoralen, 5-methoxy psoralen, 4,5,8- A. Blood pressure monitoring, S.creatinine levels,
trimethyl psoralen S.potassium levels
Q. Mechanism of action of PUVA in psoriasis? Q. Ennumerate the biological that can be used in
treatment of psoriasis?
A. Interferes with DNA synthesis thereby decreasing
cellular proliferation. Also causes localized A. A) Biologicals targeting T-Lymphocytes & APCs:
immunosuppression by causing apoptosis of Efalizumab, alefacept.
cutaneous lymphocytes. B) Biologicals targeting TNF-alpha: etanercept,
Q. Most commonly employed regimen? infliximab, adalimumab
A. On alternate days 8-MOP in the dose of 0.6mg/kg/ Q. Which of these biological is used most commonly?
dose followed 2 hrs later by UVA exposure A. Etanercept
Q. What is RE-PUVA? Q. Dose & side effects of etanercept?
A. Combination of etretinate or acetretin with PUVA A. 25-50mg twice weekly subcutaneously for 12 weeks.
Q. Which oral retinoid is commonly used in psoriasis? Most common side effects include injection site
A. Acetretin reaction, reactivation of TB, CNS demylination,
congestive heart failure.
Q. Advantage of acetretin over etretinate?
A. Etretinate is more teratogenic, longer half-life and h
stored in fat for a longer duration
Q. Mechanism of action of acetretin in psoriasis?
A. Binds to nuclear retinoid receptors, regulates gene
transcription, modulates epidermal cell differentiation,
decreases inflammation
DYP SURVIVAL GUIDE FOR POST GRADUATES 213

h/o drug intake prior to symptoms


12. SYSTEMIC LUPUS h/o dryness in eyes, dry mouth,watering or burning
ERYTHEMATOSUS tightness of skin/dysphagia
h/o Raynauds phenomenon/ proximal muscle
Name/Age/ Sex/Occupation/Residing at... / weakness/ swelling around eyes.
Hailing from.../Marital status h/o drug intake for the above complaints
Chief complaints h/o response to drugs
Jount pains h/o present complaints
Malar rash Past history:
oral ulcers h/o T.B/H.T/IHD/Bronchial asthma/drug allergy
gradual onset h/o medical and surgical illness
c/o joint pains in…….joints which increased to Personal history:
involve ….joints with/without morning stiffness Sleep/Bowel/Bladder/Appetite/Addiction
Associated with swelling/deformity Menstrual history
Relieving/Aggravating factors Obstetric history
h/o photosensitivity(how severe,since when) Family history
Any relieving factors Quality of life:
h/o blistering/rash/redness/scaling and face h/o interfering with work/social/interpersonal/
swelling sexual life
h/o burning/pruritus/pain General examination:
h/o oral ulcerations(painful/painless) Patient is conscious cooperative welloriented in time
difficulty in eating/swallowing with/without splace and person.
burning sensation BP/PR/RR
h/o hyperpigmentation and scaling lesion on face Pallor/Cyanosis/Clubbing/Icterus/Edema feet/
h/o psychosis/epileptic fits/mood changes/ Lymphadenopathy
depression/hemiplegia/neuropathy/ataxia Cutaneous examination
h/o chest pain/dyspnea/palpitations Malar rash: Symmetrical erythema,edema,scaling on
h/o fever/malaise/myalgia/tiredness both cheeks, bridge of nose and forehead with
h/o oliguria/pedal edema/hematuria illdefined borders
h/o nausea/vomiting/diarrhea/vague abdominal involving/sparing ears
pain involving/sparing V area of the neck
212
214 DYP SURVIVAL GUIDE FOR POST GRADUATES DYP SURVIVAL GUIDE FOR POST GRADUATES 215

Rest of the skin 9. Vision perimetery


Any ulcers/telangiectasia/purpura/gangrene/ 10. Slit slamp fundoscopy
Erythema multiforme/Abnormal pigmentation/ If any skin lesions biopsy a) Histopathology
Calcinosis cutis/Acanthosis nigricans/Anetoderma/
b) DIF
Lichen planus/Annular erythema/Thrombophle-
bitis associated with APLA. c) Lupus band test
Evidence of raynauds phenomena? 11. Serological: ANA in high titre
Fingers- Swelling/Deformity/Ulcers DsDNA, anti sm, antiU1RMP
Nail changes Complement level c3 c4, APLA (if h/s/o
thrombosis or bad obstetric history)
Mucous membrane changes(Oral and genital)
Q. In how many % of pts do you get raised ANA and
Systemic examination :
DsDNA?
RS- Crepts/Any effusion/Breath sounds
A. - 98%ANA
CVS- S1,s2 normal,murmurs
- 60%DsDNA
CNS- Ataxia/Hemiplegia/Motor deficit
Q. What are the ARA criteria?
P/A-soft/nontender
A. 1. Discoid rash
Diagnosis:
2. Malar rash
Acute/Subacute SLE with/without complications,
3. Recurrent oral ulcers
with/without flare on treatment well controlled/
poorly controlled 4. Photosensitivity
Q. How will you manage this patient? 5. Non erosive arthritis
A. First confirm diagnosis clinically, histologically, 6. Serositis: pleuritis/pericarditis
serologically 7. Haematological
1. Based in clinical features-ARA fulfillment Thrombocytopenia, leucopenia/anaemia/
2. Hb/CBC/ESR/CRP lymphopenia
3. LFT (increased SGOT/SGPT) 8. CNS:
4. RFT (increased BUN, Creatinine) Psychosis
5. Urine >0.5g proteinurea, granular RBC casts Seizures
6. Blood sugar 9. Renal- Proteinuria>0.5g in 24 hours or +3
7. Usg abdomen Rbc casts and epithelial casts
8. Chest xray, ecg 10. ANA positivity
216 DYP SURVIVAL GUIDE FOR POST GRADUATES DYP SURVIVAL GUIDE FOR POST GRADUATES 217

11. Immunology: II. LE nonspecific skin diseases with no interface


Anti ds DNA antibody, antismith Ab, dermatitis
False positive VDRL A. Cutaneous vascular disease
Q. What is Gillians classification? 1. Small vessel cutaneous leucocytoclastic
A. Gillians classification: vasculitis secondary to LE
I. LE specific skin diseases characterized by a. Dependable palpable purpura
interface dermatitis b. Urticarial Vasculitis
A. Chronic cutaneous LE(CCLE) 2. Vasculopathy
1. Classic discoid LE
a. Degos disease like lesion
a. Localised DLE
b. Secondary atrophic blanche
b. Generalised DLE
3. Periungual telangiectasia
2. Hypertrophic or verrucous DLE
4. Livedo reticularis
3. Lupus panniculitis/Lupus profundus
4. Mucosal DLE 5. Thrombophlebitis
a. Oral DLE 6. Raynauds phenomenon
b.Conjunctival/ 7. Erythromelagia
c. Nasal B. Nonscarring Alopecia
d. Genital 1. Lupus hair
5. LE tumidus/Papulomucinous LE 2. Telogen effluvium
6. Chilblain LE 3. Alopecia areata
7. Lichenoid DLE(LE/LP overlap) C. Sclerodactyly
B. Subacute cutaneous Lupus erythematosus(SCLE) D. Rhematoid nodules
1. Annular E. Calcinosis cutis
2. Papulosquamous/Psoriasiform
F. LE nonspecific bullous lesions
3. Vesicobullous annular SCLE
G. Urticaria
4. TEN like SCLE
H. Papulonodular mucinosis
C. Acute cutaneous LE(ACLE)
I. Cutis laxa
1. Localised ACLE(malar rash)
J. Acanthosis nigricans
2. Generalised ACLE
3. TEN like K. Erythema multiforme
218 DYP SURVIVAL GUIDE FOR POST GRADUATES DYP SURVIVAL GUIDE FOR POST GRADUATES 219

L. Leg Ulcers Q. In which scenario can one get false negative lupus
M. Lichen planus band test?
Q. What is the most common vascular lesion in SLE? A. It is seen when high levels of extra vascular dermal
IgG is present
A. telangiectasia
Q. Which are the conditions in which lupus band test
Q. How will you describe the malar rash? is positive?
A. A butterfly blush or discrete maculopapular eruption A. - SLE
with fine scaling on the butterfly area of the cheeks - Drug induced lupus
or elsewhere.
- PLE
Q. D/D’s of malar rash?
- LP
A. - DLE - Rosacea
- SLE - Facial telangiectasias
- Heliotrope rash of DMS - BP
- Rosacea - Porphyria
- Seborrheic dermatitis Q. What are the types and staining patterns of lupus
- Photodermatitis band test?
- Erysipelas/cellulitis A. TYPES
Q. In how many patients can you get a generalized rash? - Lesional
A. 5-10% - Nonlesional
Q. What are the clinical variants of LE? PATTERNS:
- Under low power staining pattern is described as
A. Morbilliform/Exanthematous/Bullous/TEN like
being granular
Q. Why does a patient get bullous lesions in LE? - Under high power it has been described as
A. Severe interface dermatitis and basal cell vacuolar homogenous fibrillar stippled shaggy lumpy-
degeneration. bumpy, linear or thready, in a continuos fashion
Q. What is lupus band test? Q. In which condition discontinuos or interrupted LBT
A. Deposition of one or more immune reactants (IgG, is seen?
IgM, IgA, c3 and complement components) found as A. - Actinic keratosis
a linear band at the basement membrane zone by DIF - Rosacea
termed as lupus band test. Sensitivity 95% and - PLE
specificity 87% and is a prognostic indicator.
- Non lesional sun exposed skin (80%)
220 DYP SURVIVAL GUIDE FOR POST GRADUATES DYP SURVIVAL GUIDE FOR POST GRADUATES 221

Q. What are the clinical corelations of various patterns Q. What is LE cell test?
of LBT? A. LE cells are polymorphonuclear leucocytes which
A. Stippled pattern-clinically normal skin of have ingested nuclear material from degenerative
SLE(multiple small round points of fluorescence white cells in the presence of antibody to deoxy ribo
nucleic acid(LE cell factor)
Pattern Description Condition
Stippled Multiple small Clinically normal Q. What is rosette phenomenon?
round points of skin of SLE A. Large masses of nuclear material are found
fluorescence extracellularly and with surrounding leucocytes form
Homogenous Solid Chronic atrophic rosettes.
/homogenous and hypertrophic Q. In which conditions LE cell test is positive?
band of well lesions A. - SLE
demarcated - Drug induced lupus
bright
- Chronic DLE
fluorescence
Thready Short closely set Acute - Systemic sclerosis
bright threads or erythematous - Rheumatoid arthritis
fibrils and edematous Q. Name the specific antibodies for various CTDs?
lesions
ANTIBODIES CTDs
Q. What are the uses of LBT?
Anti dsDNA SLE(renal disease)
A. 1. To diagnose LE
Anti SM SLE
2. To differentiate LE from other cutaneous Anti Ro, Anti La SCLE/Neonatal lupus/Drug
disorders(PLE/jessners lymphocytic infiltrate/ induced lupus
lymphocytoma cutis) Anti ss DNA, Anti Ro, Anti Sjogrens syndrome
3. To diff systemic LE from discoid LE LA
4. Making a diagnosis of SLE in patients without Anti U1RMP MCTD
cutaneous lesions APLA Antiphospholipid
5. To diff from other ANA positive diseases(drug syndrome(recurrent
induced LE, rheumatoid arthritis, scleroderma, thrombosis)
DMS, MCTD) Anti topomerase1(anti Systemic sclerosis
6. Prognostic significance(positive LBT from sun
scl70)
protected normal skin indicates decreased long Anti jo,antit RNA DMS
term survival it correlates positively with risk of synthetase
developing nephritis) Anti centromere CREST syndrome
222 DYP SURVIVAL GUIDE FOR POST GRADUATES DYP SURVIVAL GUIDE FOR POST GRADUATES 223

What are the different ANA patterns seen in various STDs? - Psoriasis
ANA Conditions - Vitiligo
Homogenous pattern Drug induced lupus/SLE - Nail patella syndrome
Speckled pattern SLE/MCTD/SCLE - Glomus tumour
Nucleolar pattern Systemic sclerosis/SLE - Subungual myxoid tumour
Peripheral/membranous SLE with renal - Repeated trauma
pattern involvement
- Systemic causes
Q. What are the trigger factors?
(renal/ cvs/ endocrinology)
A. UV radiation(discos/fluorescent light/UVA from Q. D/D of blue/azural lunulae?
photocopiers)
A. - Phenophthalein (in laxatives)
Q. Differential diagnosis of edema face?
- Silver
A. - Dermatomyositis
- Zidovudine
- SLE
- Cytotoxic drugs
- Hansens in reaction
Q. D/D’s of cribriform scars?
- Contact dermatitis
A. - Pyoderma gangrenosum
- Seborrhoeic eczema
- DLE
- Erysipelas
Q. In which condition do you get black lunulae?
Q. Differential diagnosis of follicular plugging? A. Subungual haemorrhage(black)
A. - SLE Q. Where do you look for telangiectasia?
- DLE A. Reticulate telangiectatic erythema on thenar and
- Acne hypothenar eminences of the palms, pulp, dorsum
- Phrynoderma of fingers, toes and over the lateral borders of feet
and heels
- Follicular LP
Q. Where do you look for vascular necrosis?
- PRP
A. Tips of fingers and alongside the nails.
- Keratosis pilaris
Q. What are the nail changes in SLE?
- Seborrhaeic keratosis
A. Splinter hemorrhages, Pitting, Ridging, onycholysis,
Q. D/D of red lunulae? striate leuconychia, red lunulae , hyperkeratotic nail
A. - SLE folds and ragged cuticles,dilated nail fold capillaries,
- Alopecia areata clubbing.
224 DYP SURVIVAL GUIDE FOR POST GRADUATES DYP SURVIVAL GUIDE FOR POST GRADUATES 225

Q. What are the hand changes in SLE? - Penicillamine


A. Sclerodactyly/Palmar erythema/telangiectasias/ - Phenytoin
above nail changes - Valproate
Q. Differentiate Rheumatoid nodules and SLE? - Infliximab
A. SLE nsubcutaneous nodules will be superficial and - Procainamide
nonerosive
- Chlorpromazine
Q. What are the types of alopecia seen in lupus
erythematosus? - Terbinafine
A. 1. Scarring alopecia Q. How do you differentiate drug induced LE from
2. Alopecia areata spontaneous LE?
3. Diffuse alopecia A. The drug induced SLE differs from spontaneous SLE
in that
4. Lupus hairs
Q. What is the most common musculoskeletal 1. Older age group
manifestation? 2. Renal and cns involvement(infrequent)
A. Arthralgia/myalgia 3. Anti histone bodies are frequent
Q. What is the most common joints involved? 4. Anti DNA antibodies are absent
A. Non erosive arthritis in PIP,MCP,wrist,knee,elbow 5. Serum complements are absent
Q. What are the causes of myopathy? 6. Pulmonary and joint involvement is common
A. 1. Inflammatory 7. Skin lesion is uncommon
2. Corticosteroids Q. How does hydralazine cause lupus?
3. Antimalarials A. It inhibits binding of complement component c4 with
Q. What are lupus hairs? subsequent lack of control of complement activity
A. Short, unruly, coarse, fragile, dry, broken off hair Q. What is Rowells syndrome?
mainly on frontal margin probably because of slowed
A. Lesions of SLE like erythema multiforme with
anagen growth
Speckled ANA pattern
Q. Which are the drugs inducing systemic lupus
erythematosus-like syndrome? Q. What are the other cutaneous features of SLE?
A. - Hydralazine A. - Facial edema
- Minocycline - SCLE
- Isoniazid - Chronic discoid LE
- Penicillin - Scarring DLE Alopecia
226 DYP SURVIVAL GUIDE FOR POST GRADUATES DYP SURVIVAL GUIDE FOR POST GRADUATES 227

- Non scarring alopecia Q. Which are the other skin diseases seen with SLE?
- Chilblain lupus A. Acanthosis nigricans
- Bullous eruptions Herpes Zoster more frequent
- Raynaud's phenomenon Scabies more severe and crusted
- Chronic urticaria Eruptive dermatofibromas
- Urticarial vasculitis (hypocomplementemic) Hyperkeratotic erythematous follicular papules on
- Livedo reticularis trunk and limbs(Chinese)
- Papular and nodular mucinosis Q. What are the features of bullous SLE?
- Episcleritis A. Bullous lesions occur on face, neck, upper trunk
- Cheilitis mainly which heal with milia and photosensitivity/
Q. What is lupus erythematosus telangiectoides? mouth ulcers/EM like lesions may be present with
A. Disseminated variety of DLE results in a persistent glomerulonephritis. A form resembling EBA has been
blotchy reticulate telangiectasia, seen on face, neck, described.
ears, dorsa of hands, breasts, heels and on sides of Q. What are the drugs that can precipitate Bullous LE?
feet
A. Hydralazine
Q. What is lupus erythematosus gyratus repens?
Gamma interferon
A. An annular variant consists of a migratory gyrate
annular erythema with the histological features of LE Q. What is Senear Usher syndrome?
although LBT is negative. There may be an A. Erythematous,scaly crusted or hyperkeratotic lesions
underlying carcinoma. on seborrheic areas and butterfly area having
Q. What are the connective tissue changes in SLE? immunological features of both SLE and Pemphigus
A. 1. Hardbound/thickened skin with pigmentation foliaceous. It occurs spontaneously or induced by
on face/limbs drugs
2. Subcutaneous nodules because of vasculitis and Q. What are the types of LE nephritis?
thrombosis(elbows/knees/back of proximal A. I: Minimal mesangial lupus(normal glomeruli by
phalanges/wrists/occiput/flexor aspects of light microscopy but immune deposits on
fingers) immunofluorescence)
3. Calcinosis Cutis II: Mesangial proliferative lupus nephritis(purely
4. Panniculitis mesangial hypercellularity of any degree or
5. Relapsing nasal and aural chondritis(cartilage mesangial matrix expansion onlight microscopy
not involved) with mesangial immune deposits)
228 DYP SURVIVAL GUIDE FOR POST GRADUATES

III: Focal lupus nephritis


13. SCLERODERMA
A- Active or inactive,segmental or global,endo
or extracapillary
Name/Age/ Sex/Occupation/Residing at... /
A/C- Glomerulonephritis<50% of all glomeruli
typically with focal Hailing from.../Marital status
C- Subendothelial deposits with or without Chief complaints:
mesangial alteration Tightness of skin/darkening of skin since……..
Focal proliferative and sclerosing glomerulonephritis Painful bluish discolouration of fingers on exposure
IV: Diffuse lupus nephritis to cold since…..
?50% of all glomeruli involved
Breathlessness/chestpain since….
V: Membranous lupus nephritis: global or
segmental subepithelial immune deposits.may Joint pains since…
show advanced sclerosis Difficulty in swallowing since …..
VI: Advanced sclerotic lupus nephritis more than Abdominal bloating since….
90% glomeruli globally sclerosed.
Onset,duration and progress:
Q. What is normal creatinine clearance?
A. Creatinine clearance- 80-120ml/min Skin Tightness:
Q. How much % of patients get renal involvement? Gradual/sudden
A. 50% Starting from ….progressing to …
Q. How will you manage lupus? History of preceding swelling of hands(pitting/
A. - Strict sunscreen nonpitting)
- Topical: Thickening/pruritus
• Steroids
Difficulty in movement/any deformity?
• Tacrolimus
• Retinoids Difficulty in opening mouth/eating/swallowing
- Hydroxychloroquine(200mg) twice a day Any facial changes patient has noted.(thinning of lips/
- Oral steroids at 1mg/kg/day shiny face)
- Other immunosuppressants like cyclophosphamide, Any discoloration/pigmentary changes?
azathioprine Is there a history of painful discoloration to blue of
- Oral retinoids fingers on exposure to cold which reverts back on
rewarming?
h
229
230 DYP SURVIVAL GUIDE FOR POST GRADUATES DYP SURVIVAL GUIDE FOR POST GRADUATES 231

Since when? How long does it last? h/o oliguria/edema feet/swelling around eyelids
How frequently it occurs? h/o leg cramps/frothy urine
Any blackish discolouration(for gangrene) h/oheadache/visual disturbances/
History of digital ulcerations/ loss of digits/tapering seizures(malignant hypertension)
of digits/nail changes? For Sjogrens syndrome
History of frothy urine-proteinuria History of difficulty in speaking or taking frequent
Joint pains since …. sips of water,cracking and fissuring at angles of
mouth(s/dry mouth)
Which joints? How was the progression?
History of grittiness in eyes increased at the end of
Aggravating/relieving factors?`` the day(dry eyes)
Any morning stiffness? h/o tingling numbness in hands and feet(peripheral
Swelling/Range of motion/deformities/weakness of neuropathy)
muscles/impairment of work For SLE:
History of fever,bodyache,myalgia h/o photosensitivity/rash on the face/seizures/
History of any hard raised lesions with chalky psychosis
discharge/ulcerations h/o oral ulcers
For RS: For DMS
h/o difficulty in breathing on exertion/at rest h/o swelling around the eyes/difficulty in standing
h/o dry cough from squatting position/combing hair.
h/o chest pain/palpitations h/o any drugs taken in the past(ergot/OCs)
For GIT h/o treatment taken for the above complaints
h/o difficulty in swallowing/reflux/vomiting Did symptoms worsen/improve with treatment
h/o bloating/postprandial h/o any complications of treatment
abdominal pain h/o headache(for migraine)
h/o constipation/incontinence/diarrhea Quality of life:
Has it impaired work/interpersonal relationships
h/o acute abdominal pain
(sexual/social)
h/o stools floating in water/passing gas more
Past medical history:
frequently(suggestive of malabsorption)
Any h/o DM/HT/TB/Bronchial Asthma/IHD/
For Renal
Drug allergy/surgical illnesses.
232 DYP SURVIVAL GUIDE FOR POST GRADUATES DYP SURVIVAL GUIDE FOR POST GRADUATES 233

Personal History: Peripheral pulsations (to r/o thromboangitis


Sleep/Bowel(constipation,rectal incontinence)/ obliterans (TAO))
Bladder (decreased urine output or frothy urine)/ Pallor/Cyanosis/Icterus/Lymphadenopathy/
Appetite(malnutrition) Edema feet
Smoking/Alcohol/Any other addictions Cutaneous Examination
Menstrual history: Face: Tight, taut shiny skin on face-induration(inability
Regular monthly cycles/duration/mild,moderate or to pinch skin)/forehead with decreased wrinkling/
heavy flow?(secondary amenorrhea) shininess/pigmentation.(salt and pepper)
Obstetric history: Inability to retract lower eyelid (Ingram sign)
Gravida…Para…Live Births..Abortions…. Parrot beak/pinched beak like nose with loss of
nasolabial folds.
Normal/LSCS mode of delivery
Difficulty in mouth opening/purse string appearance
Any complications?(miscarriage/premature labour)
Rhagades
Any improvement(Raynauds improves) or
Mat like telangiectasias on cheeks/nose
worsening(GERD worsens) during previous
pregnancies if female? Tongue protruding/dental caries
h/o impotence if males? Ridging and tightness/salt and pepper pigmentation
of neck (Barnett's neck sign)
h/o contraception
Hands:
Family history
Tautness of skin on hands/sclerodactyly extending
Any similar complaints in family?
till what level?
On the basis of history,my diagnosis is
Sausage shaped digits/Flexion deformities of
Limited/diffuse systemic sclerosis in hands/Loss of pulp of finger/stellate scars with/
…..stage(edema/induration/atrophic) with….. without
organ involvement/treated or untreated
active digital ulcers,
General examination nailfold changes(periungual telangiectasia), pallor,
Patient is conscious,cooperative,welloriented in nail plate changes
time,place and person. pterygium inversum unguis
Averagely well built and well nourished. Hair loss on skin
Pulse/BP/RR Loss of appendages
Weight/Height/BMI Any pigmentation of skin of hands.
234 DYP SURVIVAL GUIDE FOR POST GRADUATES DYP SURVIVAL GUIDE FOR POST GRADUATES 235

Calcinosis cutis Q. Should you give nifedipine or losartan in CREST


Tendon friction rubs+/-(Juxtaarticular tendon fibrosis syndrome?
(Do full movement of the joint and keep finger on A. Nifedipine worsen oesophageal symptoms so
joint-crepitations felt at wrist,MCP, and losartan is preferred.
interphalangeal joints) Q. What is the normal chest expansion ?
Raynauds elicitable/nonelicitable A. 3-5cm at level of T4
Peripheral nerves: Sensations/ claw hand/deformity Q. How do you test for microstomia?
Hair-Telogen effluvium A. Measure oral aperture-4 fingers insertion is normal.
Nails-Pterygium inversum unguis
Grade 1-3 fingers
Nailfold capillary dilatation and dropouts
Grade 2-2 fingers
Telangiectasia
Grade 3-1 finger
Mucosa-Oral(dental caries/difficulty to protrude
Q. How do you test for tendon friction rubs?
tongue)
Sensations A. Do full movement of joint and keep finger on joint-
crepitations felt at wrist,metacarpophalangeal joint
Diagnosis:Limited/diffuse systemic sclerosis in
and interphalangeal joints because of juxtaarticular
edema/induration/atrophic stage with organ
tendon fibrosis & edema.
involvement…treated/untreated
Q. What are bowed fingers?

QUESTIONS A. It is a helpful sign in the diagnosis of early systemic


sclerosis characterized by cold flexed fingers but no
Q. Why is occupation important in systemic sclerosis?
acrosclerosis or systemic abnormality.Seen in 2nd to
A. A light job - warm atmosphere and to avoid minor 4th decade and Raynauds phenomenon is not
trauma for the hands (e.g. typist, computer operators, invariable.
pneumatic drills) for Raynaud’s phenomenon.
Q. On which sites are pitted scars seen in systemic
Q. Is there any treatment to decrease perioral and facial
sclerosis?
tightening?
A. Tips of digits, ulnar border of thumb, radial border
A. 0.025 to 0.5% tretinoin
of index and middle finger,dorsa of hands over joints.
Q. How do you treat digital calcinosis?
Q. Where do you check for telangiectasia?
A. CO2 laser, surgical excision .But the average healing
time was found to be six weeks after laser. A. Face,lips,mouth,palms,upper trunk,hands,upper
thighs.
236 DYP SURVIVAL GUIDE FOR POST GRADUATES DYP SURVIVAL GUIDE FOR POST GRADUATES 237

Q. What are the sites you should look for pigmentary Q. Difference between primary Raynaud’s (Raynaud’s
changes? disease) and secondary Raynaud’s phenomenon?
A. Face, legs, thighs. A. Feature Primary Secondary
Q. What are the other skin findings of systemic raynauds raynauds
sclerosis? Sex (f:m) 20:1 4:1
A. Besides skin thickening and hardening on the fingers, Age at onset Puberty >25yrs
hands, and face which are generally the earliest areas Frequency of <5 per day >5-10 per day
of the body involved, Edematous swelling and attacks
erythema may precede skin induration. Precipitants Cold, emotional Cold
stress
Other prominent skin manifestations include:
Ischemic injury No Yes
• Pruritus in the early stages Abnormal Absent Present
• Edema in the early stages capillaroscopy
• Sclerodactyly Other vasomotor Yes Yes
• Digital ulcers phenomenon
ANA Absent or Low 90-95 % present
• Pitting at the fingertips and atrophy of finger
titres
pulp (Mizutani’s sign)
Anti centromere Absent 50-60% present
• Telangiectasia antibodies
• Calcinosis cutis Anti SCL-70 Absent 20-30% present
Q. What is pterygium inversum unguis? (Topoisomerase I
A. Fusion of hyponychium with proximal nail fold antibody)
because of fibrosis. In vivo platelet Absent >75% present
activation
Seen in
- SLE Q. What is the difference between diffuse and limited
- Scleroderma systemic sclerosis?
- Neurofibromatosis A. Disease Limited Diffuse
- Leprosy characteristics scleroderma scleroderma
- Raynauds Skin involvement Distal to Mcp Distal and
Q. Why is grip poor? joints, knees, face
proximal
extremities, face,
A. If induration severe then grip will be poor. trunk
Q. What is Barnett’s neck sign? Raynauds May precede skin May occour
A. Ridging and tightness with salt and pepper phenomenon changes by many simultaneously
pigmentation . years or a year or two
238 DYP SURVIVAL GUIDE FOR POST GRADUATES DYP SURVIVAL GUIDE FOR POST GRADUATES 239

prior to or after phospholipid antibody


the onset of skin syndrome rare
disease
Anti- SM , Anti Rare Overlap syndrome
Internal organ G.I., pulmonary Interstitial lung
involvement hypertension disease, renal, U-1 : RNP with features of
gastrointestinal, scleroderma
cardiac
Nail fold Dilatation Dilatation with
capillaries without dropouts drop outs Q. What is the treatment of digital scars?
ANA Anticentromere Anti- A. Laser
antibodies topoisomerase Q. What are the phases of scleroderma?
Disease course Slowly Aggressive
and prognosis progressive, course in A. -edema, induration and atrophy
better prognosis majority with Q. Why do you get edema?
except if rick f visceral A. i) dermal glycosoamino glycan deposits
pulmonary involvement
hypertension early in the ii) vascular changes
occours. disease course. iii) hydrostatic changes
iv) inflammation
Q. What are the various antibodies in scleroderma?
Q. What is the differential diagnosis of mask like
Antibodies Prevalence (%) Clinical association facies?
Anti-centromere 20-30 Limited scleroderma, A. -i) systemic sclerosis
CREST , pulmonary
ii) hypothyroidism
hypertension
iii) myxedema
Anti- SCL-70 15-20 Diffuse scleroderma,
iv) parkinsonism
interstitial lung disease
v) b/l UMN facial palsy
Anti- PM- SCL 2-3 Poly-myositis-
scleroderma overlap vi) depression
Anti- RNA 20 Diffuse scleroderma vii) myasthenia gravis
polymerase viii)dementia
Anti- cardiolipin 20-25 Diffuse subsets or Q. D/D of deformed or mutilated fingers.
antibodies limited subset, features A. i) Lepromatous leprosy
of secondary anti ii) systemic sclerosis
iii) raynaud disease or phenomenon
240 DYP SURVIVAL GUIDE FOR POST GRADUATES DYP SURVIVAL GUIDE FOR POST GRADUATES 241

iv) frost bite Q. What are the bony changes in systemic sclerosis?
v) vasculitis A. i) Osteoporosis
vi) Buergers disease (thrombo-angitis obliterans) ii) Osteopoikilosis
vii) trauma iii) Avascular necrosis
viii)syringomyelia iv) Penciling of the digits
ix) Dermatomyositis v) If acro-osteolysis : bone resorption
Q. How is the GIT involved?
Q. What are the types of telangectasia's seen?
A. - Macroglossia
A. i) Mat
- Oesophageal reflux/dysphagia/strictures
ii) Punctuate
- Hiatus hernia /Bleeding from telangiectasia in
iii) Linear stomach.
iv) Stellate - Intestinal obstruction/malabsorption
Q. What is the treatment of telangectasias? - Paralytic ileus
A. i) Pulse dye laser - Dilated atonic colon/telangiectasia in colon
ii) Argon laser - Duodenal strictures and ulcers
iii) ND-YAG laser Q. Why does malabsorption occur?
Q. MCTD has combination of which disease? A. Abnormal peristalsis causes stagnation which results
in bacterial overgrowth cause malabsorption and it
A. i) Scleroderma
can be corrected with tetracycline.
ii) SLE Q. What is steatorrhea?
iii) Polymyositis / dermatomyositis A. Foamy looking bulky stools due to fecal fat 4 gm/24
iv) RA hrs
and presence of Anti U-1 RNP antibody positive Q. What are the endoscopic findings in systemic
sclerosis?
Q. How often do u get calcinosis in systemic sclerosis?
A. i) Barretts' esophagus
A. - 25%
ii) Stricture oesophagus
Q. What are the types of calcification?
iii) Hiatus hernia
A. i) Dystrophic
iv) Stack of coin appearance
ii) Metastatic (CRF)
Q. What are the symptoms of pulmonary hypertension?
iii) Iatrogenic A. It is either asymptomatic or exertional breathlessness.
iv) Idiopathic Rarely chest pain or syncope.
242 DYP SURVIVAL GUIDE FOR POST GRADUATES DYP SURVIVAL GUIDE FOR POST GRADUATES 243

Q. What are the symptoms of pulmonary fibrosis? Q. How does systemic sclerosis in childhood differ?
A. i) Dyspnoea- exertional A. Raynauds phenomenon is less frequent and renal
ii) Dry cough involvement is rare. Course is slower disability and
visceral involvement is less severe.
iii) Chest pain
Q. What are the various associations of systemic
iv) Hemoptysis (rarely) sclerosis?
Q. What is Scleroderma renal crisis? A. 1. Dermatomyositis/polymyositis
A. Occurs in 5-10% of scleroderma patients and 2. Sjogren's syndrome
characterized by malignant hypertension with rapidly 3. Myasthenia grevis
progressing renal failure.
4. Temporal arteritis
Q. What are the risk factors for renal crisis?
5. Malignant atopic papulosis
A. i) Rapidly progressive diffuse skin thickening
Q. What is Kahn's criteria and Alarcon-Segovia criteria
ii) High dose corticosteroids (so should be avoided for MCTD?
in early diffuse disease)
Alarcon Segovia criteria Kahn's criteria
Q. What are the symptoms of renal crisis? A.Serological criteria A.Serological criteria
A. Headache(seizures, encephalopathy), vision Anti RNP>=1:1600 High titre Anti RNP
disturbances (retinopathy), malignant hypertension, corresponding to speckled
LVH, cramps, decreased urine output, hematuria, ANA>=1:1200 titre.
proteinuria, edema of feet.
B.Clinical criteria B.Clinical criteria
Q. Explain the Relationship between pregnancy and Swollen Hands Swollen fingers
systemic sclerosis.
Synovitis Synovitis
A. Systemic sclerosis usually remains unchanged during
Myositis(biologically Myositis
pregnancy but it may also precipitate the disease in
some cases, and, improves raynauds. Pregnancy proven)
worsens esophageal reflux, renal crisis and Raynauds phenomenon Raynauds phenomenon
pulmonary and cardiac manifestations. Acrosclerosis absent
There is a High risk of miscarriage in systemic MCTD is present if criteria MCTD is present if criteria
sclerosis or premature birth because of decidual A is accompanied by 3 or A is accompanied by
vasculopathy.
more clinical criteria one Raynauds phenomenon
Peripheral gangrene may occour. Fertility may be of which must include with>= 2out of the
impaired.
myositis or synovitis. remaining 3
244 DYP SURVIVAL GUIDE FOR POST GRADUATES DYP SURVIVAL GUIDE FOR POST GRADUATES 245

Q. How will you manage a case of systemic sclerosis? - GIT: Esophageoscopy/ proctoscopy/
A. - Thorough history and clinical examination will colonoscopy/USG/LFT/Ba swallow(stiff glass
clinch the diagnosis. tube appearance)/Esophageal manometry
- First confirm the diagnosis of scleroderma - CVS: ECG/Echo/Chest X-Ray (Cardiomegaly/LVH)
- Document the extent of Systemic Involvement - Lung: Chest X-Ray/PFT/CT/HRCT
- Investigations to find out the general status of Q. What will be seen in skin biopsy?
the patient A. Early- less inflammatory infiltrate around dermal
For confirmation : vessels,eccrine coils and around the subcutaneous
tissue.Vascular changes mild.
- A skin biopsy should be done for purpose of
accurate documentation and for excluding Late- Paucity of blood vessels, thickening and
scleroderma mimics hyalinization of walls and narrowing of
lumen.Thickened collagen bundles
- Skin ultrasonography to determine the extent and
Q. What can be the changes in X-ray,PFT and HRCT?
depth of sclerosis.
A. Chest X-Ray - Ground glass appearance Bilateral
- Antinuclear antibody and its patterns should be
reticular or nodular basilar shadows fibrosis/
tested at the time of presentation
cardiomegaly/LVH
- Specialized antibodies can be checked like anti
PFT- DLCO decreased/FLVC decreased
SCL-7 and anti U1 RNP (to rule out MCTD)
HRCT-Narrow illdefined crescent in the lower lobe
- For systemic involvement and complications
Reticular nodular shadows:
- X Ray Hands for periarticular osteoporosis , joint
space narrowing, erosive arthropathy with Honey comb pattern/large cystic airspace/
"pestle and mortar" deformity of the DIP joints, mediastinal involvement and pleural involvement.
resorption of terminal phalanges, tufting of the Q. What do you expect in the blood tests?
tip of the fingers, presence of calcinosis. A. CBC- anaemia(renal failure/GI bleed/malabsorption)
- Lungs: X ray Chest PA, Lung function tests ESR- increased
including 6 minute walk test, DLCO, FEV1, VC LFT- primary biliary cirrhosis often associated
- Skin USG to know extent and depth of sclerosis 24 hrs Urinary protein- proteinuria(0.5gm/day)
- Anti Scl 70/urine/PFT/6min walk test/FEV VDRL- False positive in 5%patients
repeated every 6 months.
Creatinine clearance-depends on renal involvement
- SKIN: sclerosis, pigmentation, telangiectasia
Q. What can happen to the heart in scleroderma?
- MSK: motor examination/Xray Hands
A. Conduction disturbances/arrhythmia/myocardial
246 DYP SURVIVAL GUIDE FOR POST GRADUATES DYP SURVIVAL GUIDE FOR POST GRADUATES 247

involvement/cardiomegaly/LVH/AV block/ Q. Can you predict which patients of Raynauds


secondary to lung and renal involvement phenomenon will go on to get systemic sclerosis?
Q. What is watermelon stomach? A. Yes. Nailfold capillarascopy shows enlarged or
A. It is a striped appearance of the stomach on distorted capillaries/loss of capillaries/disruption
endoscopy because of gastric antral vascular ectasia of capillary bed may indicate.
seen in systemic sclerosis. Q. In Which conditions is Cobblestoning seen?
Q. What are eye changes in systemic sclerosis? A. 1. Darier White disease
A. Tightness of lids, diminished tear secretion, kerato- 2. Systemic sclerosis(Small papules of
conjunctivitis sicca, retinopathy. lymphangiectasia)
Q. What are principles of treatment? Q. What are the advantages of ACE Inhibitors?
A. i) Assess which stage the patient is presenting in? A. 1. Controls bloodpressure in most and relieves
(edematous/infiltrated/atrophic) encephalopathy.
ii) If any overt impending risk of internal organ 2. May improve renal function and if given prevents
involvement renal failure and death.
Q. What is the treatment of Raynaud's phenomenon 3. doesnt aggravate oesophageal symptoms
and digital ulcers? Q. What is the role of LMW dextran?
A. i) Avoid cold and smoking. A. In occasion patients there is healing of ulcer tips. But
ii) Simply warming the hands in hot water for 5 to there is risk or renal failure.10% dextran in
10 mins regularly reduces the number and 5%dextrose or in normal saline solution iv.First inject
severity of attacks. 20m l over 3 to 5 mins to decrease the risk of
ii) Calcium channel blockers like Nifedipine anaphalaxis. Then infuse 20 to 30 ml /hr. Dose is 500-
10mgtds or sustained released preparations, 1000 ml over 4 hrs on day 1.500ml on day 2. Then on
Amlodipine, Nicadipine, Diltiazem alternate days for 10 days.
iii) Losartan (angiotensin type II receptor blockers) Q. What are the recent advances in the treatment of
50mg per day raynauds and digital ulcers?
iv) i.v. prostacycline analogue (iloprost) A. i) endothelial receptor antagonists. (bosentan)
v) Other drugs: aspirin, dipyridamole, pentoxfylline, ii) phospho di-esterase inhibitors (sildenafil,
topical nitroglycerine. tadanafil)
vi) Sympathetic blocks: chemical or surgical iii) i.v. iloprost (high cost, inconvenient method of
sympathectomy administration)
248 DYP SURVIVAL GUIDE FOR POST GRADUATES DYP SURVIVAL GUIDE FOR POST GRADUATES 249

Q. How will you administer Sildenafil? v) i.v. immunoglobulin,Plasmapheresis


A. Start with half tablet of 25 mg twice a day gradually vi) Halofuginone
increase to 25 mg thrice a day, check blood pressure vii) Infliximab
in supine and sitting position to rule out hypotension viii)Ximedon
if hypotension SSRI like fluoxetin 20mg per day. ix) UVA phototherapy
Q. What is the dose of Bosentan? x) PVAC(derived from deglycolipidated
A. Bosentan 125 mg twice daily, helps in preventing new Mycobacterium vaccae)
digital ulcers by 50%,specially in patients with more xi) Urokinase therapy
than or equal to 3 digital ulcers at baseline.
5. Isotretinoin 1mg/kg/day,Factor 13 IV (cutaneous
Q. Which is the best drug for pulmonary hypertension? sclerosis)
A. Bosentan because of oral dosaging and lack of serious Q. Why is penicillinamine no longer used?
injection related adverse effects. A. Penicillamine causes blood and platelet dysplasia
Q. Role of cyclophosphamide? and hepatotoxicity, pemphigus, lupus like syndrome
A. Oral or Pulse IV Improves lung function, dyspnoea, etc limit its use.
skin thickening and health related QOL. Q. How will you monitor skin fibrosis?
Q. What is QUINS trial? A. a. Modified Rodnan score(mRSS) for skin thickness
A. Quinapril 80mg/day in scleroderma for 3 years,which b. 20MHz ultrasound determination of skin
holds promise to replace ACE inhibitors in PSS. thickness.
Q. SCOT trial? c. Plicometer(medical tool with pair of tongs to
pinch skin which measures skin thickening)
A. Scleroderma-Cyclophosphamide or Autologous stem
cell transplantation. d. Durometer(handheld device which measures the
hardness of the affected skin by applying
Q. How can you reduce the skin tightness? pressure/indentation on the skins surface.
A. i) d-penicillamine 125mg every alternate day- e. Cutometer, Elastometer for skin elasticity
doubtful effect.
f. Vesmeter for skin hardness,elasticity and
ii) interferon- modest role.( flu like syndrome is a viscosity.
side effect) g. MRI measures skin thickness but
iii) Cyclosporine(no visceral improvement) and expensive.mostly used to determine and score
Tacrolimus musculoskeletal improvement (myositis/
iv) Recombinant human relaxin arthritis/calcinosis cutis)
250 DYP SURVIVAL GUIDE FOR POST GRADUATES DYP SURVIVAL GUIDE FOR POST GRADUATES 251

Q. How will you treat arthritis associated with Interstitial lung Breathing
scleroderma? disease exercises, Antibiotics (infection)
A. i) NSAIDS Immunosuppressives (steroids,
ii) Low dose steroids 5-10mg/day cyclophosphamide)
iii) Methotrexate (for severe polyarthritis) Imatinib
iv) Physiotherapy, splinting, surgical correction in Lung transplant
contractures and disability occours. Advanced stage Immunosuppressives
Q. Explain the management of myositis in scleroderma? multisystem disease (Azathioprine 150mg/day, ATG,
ALG and MMF)
A. High dose steroids with addition of immuno-
suppressants (methotrexate or azathioprine) Autologous stem cell transplant
52. Management of organ damage Q. How does D-penicillamine work?
Manifestations Treatment A. Interferes with intermolecular cross linking of
collagen.
Gastrointestinal Proton pump inhibitors, H2
Q. When patient is on D-penicillamine what should
blockers you look out for?
involvement Prokinetic agents A. - Hematological abnormality
Calcium channel blockers - Proteinuria
Scleroderma ACE inhibitors - Autoimmune phenomenon (pemphigus/
myasthenia gravis)
renal crisis Antihypertensives
Q. Where will you use steroids?
Dialysis
A. 1. Early edematous phase.
Renal transplant
2. Arthritis and serositis in low dose.
Pulmonary Calcium channel blockers 3. Myositis and myocarditis in higher doses.
Hypertension Prostacyclin/analogues 4. IV methyl prednisolone pulse therapy for ILD.
Endothelial receptor blockers Q. Would you give cyclosporine in scleroderma?
Phosphodiesterase inhibitors A. Limited use because cyclosporine(2.5-4mg/kg)
Combination therapy causes arterial hypertension and nephrotoxicity.
Q. What is Modified Rodnan Score ?
Imatinib
A. Usually 17 sites are assessed and skin thickness is
Lung transplant
categorized to grade 0, 1 , 2 or 3 corresponding to
252 DYP SURVIVAL GUIDE FOR POST GRADUATES DYP SURVIVAL GUIDE FOR POST GRADUATES 253

normal,mild moderate and severe. 4. Chronic venous hypertension and varicosities.


Areas examined are : 5. Malignancies, mycosis fungoides, lymphomas.
- Fingers (2) - Hands (2) 6. idiopathic or primary.
- Forearms (2) - Upperarms (2) Q. What is Sneddon's syndrome?
- Thighs (2) - Lower leg (2) A. Idiopathic livedo reticularis with transient ischaemic
attacks / strokes with hypertension and /or
- Feet (2) - Face (1) dementia.
- Anterior Chest (1) - Abdomen (1) Q. Where is nail fold capillary dilatation seen?
Q. How do you treat salt and pepper pigmentation? A. 1. Systemic sclerosis(limited and diffuse)
A. 1. Tacrolimus with narrow band uvb and topical 2. Dermatomyositis
Tacrolimus alone for hypopigmentation. 3. Raynaud's disease
2. Hydroquinone, Retinoids,Salicylic acid and 4. SLE
chemical peeling for hyperpigmentation with Q. Difficulty in walking?
sunscreens.
A. Dyspnoea
3. Topical Calcipotriol for both hyper and DMS
hypopigementation.
Joint pains
Q. What is atrophie blanche?
Q. Which is the most common CVD (Collagen vascular
A. This is a livedoid vasculitis also called as livedo disease) ?
reticularis with summer ulceration.favours distal A. Rheumatoid arthritis
lower extremities especially the ankles characterized
Q. What is the importance of wounds in toes/fingers?
by painful punched out ulcers, white round or
stellate scars with peripheral telengectasias, A. Non heeding may cause auto amputation, bony
reticulated violaceous erythema may extend from the involvement, gangrene.
ulcers. Q. What are scleroderma mimics?(pseudoscleroderma)
Q. In which condition do you see atrophie blanche? A. 1. Scleredema adultorum of Buschke
A. 1. Collagen Vascular Disease(SLE, APLA Syndrome) 2. Scleromyxedema
2. Vasculitis (Wegners granulomatosis, 3. Eosinophilic fasciitis/Eosinophilia myalgia
syndrome
cryoglobulinemia)
4. Toxic oil syndrome
3. Coagulation defects(decreased protein C,
decreased f actor 12 and decreased 5. Amyloidosis
Antithrombin 3) 6. Drugs: Bleomycin/Pentazocine/Vinylchloride
254 DYP SURVIVAL GUIDE FOR POST GRADUATES DYP SURVIVAL GUIDE FOR POST GRADUATES 255

Q. How will you differentiate scleroderma from linear depressions where veins appear to be
scleredema, scleromyxedema, eosinophilic fasciitis? sunken within the indurated skin.)
Systemic Scleredema scleromyxedema Eosinophilic Q. What is Nephrogenic systemic fibrosis?
sclerosis fasciitis
Major clinical -Limited -Post - -
A. Seen commonly in dialysis dependant CRF .
variants -diffuse infectious Exposure to gadolinium based contrast medium/
(type I) thrombotic events /surgical procedure.
-monoclonal
gammopathy Ill defined thick indurated plaques symmetrically on
associated extremity and trunk with an ameoboid appearance.
(type II)
-diabetes
Q. What are the predictors of bad prognosis of
mellitus Scleroderma?
(type III) A. - Males
Raynauds ++ - - -
phenomenon - Extensive sclerosis of skin and visceral involvement
Symmetric ++ ++ ++ ++ - Rapidly progressive disease
induration
Sclerodactyly ++ - - - - Late onset in life
Facial + ± ( in type I + - - Low Hb
involvement and II)
- (in type III) - High ESR
Systemic ++ - ++ + - Proteinuria
involvement
ANA ++ - - - - Decreased DLCO
Anti- + - - - - Decreased complement
centromere (in
antibodies limited) - Lung Disease
Anti- + ( in - - - - Arrythmia
topoisomerase diffuse)
I antibodies - Decreased CMI
- HLA-B8
Q. What is scleroderma sine scleroderma?
Q. What is the disease activity index of systemic
A. It is characterized by internal organ involvement and
sclerosis?
serological abnormalities without skin involvement.
A. - Skin thickening-0.5
Q. What is Groove sign?
- Digital necrosis- 0.5
A. It is seen in
- Arthritis-0.5
- lymphogranuloma venerum, and
- DLCO < 80% IE 0.5
- Shulmans' syndrome: (eosinophilic fasciitis,
256 DYP SURVIVAL GUIDE FOR POST GRADUATES

- Hypocomplementimia-1
14. VESICOBULLOUS DISEASES
- ESR > 30 - 1.5
- Any deterioration as per patient within last month
Name/Age/ Sex/Occupation/Residing at... /
in skin- 2, vessels- 5, heart, lung- 2
Hailing from.../Marital status
- Disease in considered active if value of items
detected in patients is > 3. But it has low sensitivity CHIEF COMPLAINTS:
Q. What is the diagnostic criteria for renal involvement Fluid filled blisters / raw areas / crusted areas on
in systemic sclerosis? the body _________ days / years associated with
A. GHAP CRITERIA : GFR reduced itching / pain / burning
Hypertension > 140/90 Painful oral ulcers _________ days / years.
Azotemia -BUN > 25mg/dLS
Difficulty in swallowing since…….days
Proteinuria > 1g/24 hours
Q. What are the causes of death in systemic sclerosis? ODP:
A. • inter-current infection Pt. was a/a_______ days / months ago.
• scleroderma renal crisis PEMPHIGUS VULGARIS:
• cardiac failure 1. Pt. developed raw painful areas in the mouth since
• perforation of GIT ______ days. The lesions had little tendency to heal
• occasionally CA-lungs. and gradually increased in size and number making
Q. Can ergots be given in a patient of systemic sclerosis swallowing difficult.
with migraine?
2 Pt. then developed multiple clear fluid filled lesions
A. No,since they will increase Raynauds phenomenon.
on ______ parts of body.
Q. What is ACR criteria?
A. 1. Major: The blisters burst on it own to form raw areas within
___ days.
Skin tightening proximal to MCD joints
2. Minor: These lesions showed little tendency to heal and
A. Digital pitted scars or tissue loss of Volar gradually increased in size and number.
Pads of finger tips. BULLOUS PEMPHIGOID:
B. Sclerodactyly 1. Pt. developed itching over the ______ areas of the
C. Poibasilar Pulmonary fibrosis. body, associated with red raised lesions.
1 Major and 2 minor criteria should be present. 2. This was followed by development of tense, clear
h fluid filled blisters
257
258 DYP SURVIVAL GUIDE FOR POST GRADUATES DYP SURVIVAL GUIDE FOR POST GRADUATES 259

3. These lesions remained intact and eventually burst H/O aggravating factors
to form raw areas. These raw areas showed a tendency - Sun exposure
to heal. Seasonal exacerbation
4. The lesions healed with post inflammatory - drugs
hyperpigmentation without tendency for scarring. - Ingestion of excess garlic
H/o. Systemic features : - Ingestion of iodides/fish
Fever H/O relieving factors
Joint pain - diet avoidance of wheat
Malaise - avoidance of trauma
H/O taken
Negative history to rule out other blistering disorders:
1. Describe the oral / topical treatment taken
1. H/O itching prior to the onset of lesions and
2. H/O any monthly 3 day admission for IV Rx-
associated with diarrhea -DH
Pulse
2. H/O lesions over the other mucosae i.e. eyes / 3. Effect of the Rx - decrease no and size of lesions
genitals also with scarring - CP
- No new lesions
3. H/O raw areas at trauma prone area without a 4. Rx taken - since-----months / years
positive family history of blistering disorders- EBA
- last dose taken on ______
4. H/O palm and sole involvement and H/O drug Personal History :
ingestion prior to onset of symptoms- Bullous EM
1. H/O- DM/HT/ TB/Bronchial Asthma/IHD/
5. H/O drug ingestions / topical application of any Drug allergy
irritants 2. H/O addictions :
6. H/O photo sensitivity / joint pain /oral ulcers - Alcohol - which type,
Bullous SLE/pemphigus erythematosus/Porphyria - how many pegs,
cutanea tarda
- how often
7. H/O weight loss /rash/ history to rule out any Smoking - Bidi / Cigarette,
underlying malignancy - Paraneoplastic pemphigus
- how many / day
8. H/O exacerbation on alcohol consumption - PCT
- Tobacco chewing
9. H/O insect bite
- Using Mishri
260 DYP SURVIVAL GUIDE FOR POST GRADUATES DYP SURVIVAL GUIDE FOR POST GRADUATES 261

3. H/O - Food - Spicy since___months ,followed by development of


- Onion / garlic consumption multiple fluid filled blisters and raw areas with little
tendency to heal on the body since___months/years
- Tomato / Banana (Phenol & tannins)
BULLOUS PEMPHIGOID:
- Wheat
Mr./Mrs. ____ aged ___, ___by occupation comes with
- Fish / Salt (iodides)
multiple red raised itchy lesions over the ___areas
4. Sleep followed by development of tense fluid filled blisters
5. Appetite - difficulty in eating due to raw lesions which have a tendency towards healing.
in the mouth leading to decreased food intake. O/e :
QUALITY OF LIFE • Pallor
How the disease has affected the patient's life: • Icterus
Personal life: relation with spouse/other family • Cyanosis
members and friends
• Clubbing
Professional life: Number of days lost from work due
• Edema
to the disease
• Lymphadenopathy
MENSTRUAL
LMP: • Weight

4-5 days regular cycle with normal bleeding • BMI = weight (Kg)

Cycle every 28 days Height (cm²)


OBSTETRIC • T, P, R, BP
G_P_A_L_ Cutaneous Examination :
Family completed or not? 1. Distribution of the lesions
Contraceptive used • B/L Symmetrical involvement
Tubal ligation done or not? • involving ------areas of body
SUMMARY ON HISTORY • Sparing--------areas of body
PEMPHIGUS VULGARIS: • With approximately-------% BSA involved
Mr./Mrs. ____ aged ___, ___by occupation comes with 2. Arrangement :
multiple raw painful areas in the mouth • Tending towards Symmetry
262 DYP SURVIVAL GUIDE FOR POST GRADUATES DYP SURVIVAL GUIDE FOR POST GRADUATES 263

3. Morphology : • Pseudo Nikolskys sign


Vesicle • Pear sign
• Multiple • Sheklekoffs sign
• Clear fluid filled vesicles / Bullae • Microscopic Nikolskys sign
• Measurement: smallest being -----cm diameter & 5. Oral :
largest ------cm diameter • Tongue / Buccal mucousa / palate- i.e which are
• On a clear base or erythematous base the areas involved
• Which Burst / do not burst easily • Multiple erosions measuring -----cm covered
with whitish slough
• Wheals ±
• Oral hygiene - good / poor
Raw areas
• gums hygiene - good / poor
• Multiple
6. Other Mucosae :
• erosions / ulcers
• Genitals- erosions + /-
• Covered with crusts (corn flake like / thick/
adherent) -Scarring +/ -

• Having musty / mousy / foul • Eyes - erosions / redness


-Scarring /no scarring
• Measurement: smallest measures--------cm &
largest measuring -------cm. 7. Nails -Paronychia + /-
• Lesions heal : with or without post inflammatory -Scarring + /-
hyper pigmentation 8. Palms & soles
No scarring/scaring Involved/spared
no milia formation 9. Any other lesions :
4. Signs : Nevi / Sebaceous cyst / vitiligo (unrelated skin
• Nikolskys sign- direct and marginal lesions)

• Nikolskys phenomenon 10. Bulla Charting :

• Modified Nikolskys( Bulla spread sign / Asboe No. of Vesicles, Bullae


Hansen's sign /Leutz sign) No. of Erosions ,crusts
264 DYP SURVIVAL GUIDE FOR POST GRADUATES DYP SURVIVAL GUIDE FOR POST GRADUATES 265

Summary : Q. What are the bed side examinations for pemphigus


Pemphigus vulgaris: vulgaris?
Mr. /Mrs. --------,aged-- years,-----------by occupation A. • Bulla - Tzanck smear is done which shows:-
comes with multiple raw painful areas in the mouth Acantholytic cell seen
---------months , followed by multiple fluid filled A B
_____ & raw areas with --------- tendency to heal on
body ------months _________scalp / trunk & sparing Pemphigus foliaceous Pemphigus vulgaris
palms & soles. Oval Round
O/e. - Multiple, Symmetrically distributed clear fluid Condensed nucleus hypertrophic nucleus
filled _______ on erythematous base /normal Basophilic cytoplasm Hyalinised cytoplasm
appearing skin with tendency / little tendency to heal
and form erosions with crusting
• Streptocytes - are filamentous lines of leucocytes
Nikolskys sign + /-
• Mourning edged cells
sheklekoffs sign +/ -
• Sertoli rosette
Pt. is treated / not treated with------------medicines.
Q. D/D of oral lesions in Pemphigus vulgaris?
VIVA QUESTIONS A. i. Aphthous ulcer
PEMPHIGUS VULGARIS ii. EM
iii. Primary herpetic gingivostomatitis
Q. Why is this a case of pemphigus vulgaris ?
iv. Erosive lichen planus
A. • age (4-5th decade)
v. Trauma
• History of - asymptomatic blisters
vi. leucoplakia
- are flaccid Q. Which other mucosa can be involved in pemphigus
- are clear fluid filled vulgaris?
- on an clear or on erythematous base A. any stratified squamous like oesophagus, larynx,
- with tendency to burst pharynx, conjunctiva, urethra , cervix and anus.
- without tendency to heal Q. What are H/P features of Pemphigus vulgaris?
• Distribution and morphology - Tendency to A. Tomb stone appearance with a supra basal split.
symmetry with involvement of mouth / other Q. What does DIF & IIF show in pemphigus?
mucosa A. The DIF shows fish net pattern of Ig G and C3
• O/e - BSS + , Nikolsky + IIF shows increase in dsg 1 and dsg 3 titres.
266 DYP SURVIVAL GUIDE FOR POST GRADUATES DYP SURVIVAL GUIDE FOR POST GRADUATES 267

Q. What are the reasons for a false negative DIF? ix. Urine R
A. In 10% of specimens, it may result from technical error M
(eg, by using wrong or weak antisera), the presence
of clinical or subclinical inflammation and early x. Stool R
blister formation within the biopsy specimen (this is
M
especially true in cases with PNP), or the use of a
limited panel of antisera that does not include IgA Q. How do you do Tzanck?
antisera (for cases with IgA pemphigus).
A. In the case of blistering disorders, the intact roof of a
Q. In which pemphigus DIF may be "truly" negative? blister is opened along one side, folded back and the
A. in a rare case with drug induced pemphigus. floor gently scraped. The material thus obtained is
smeared onto a microscopic slide, allowed to air dry,
Q. Tests to check disease activity / progress of disease? and stained with Giemsa stain. The commercially
A. • IIF - 2 fold increase in titre is a marker for relapse available Giemsa stain solution is diluted 1:10 with
• ELISA titers distilled water, and the diluted solution is poured over
the smear and kept for 15 minutes. Then it is washed
Q. Mx of PEMPHIGUS VULGARIS ? with water and examined under the microscope. The
A. • To confirm sis :. Tzank smear, stained nuclei may vary in color from reddish blue to
• Skin biopsy H/P If facilities available purple to pink. The cytoplasm stains bluish.
Q. Describe a Tzanck cell?
:. DIF /IIF
A. A typical Tzanck cell is a large round keratinocyte
• In view of Rx
with a hypertrophic nucleus, hazy or absent nucleoli,
i. Hb, CBC and abundant basophilic cytoplasm. The basophilic
ii. ESR staining is deeper peripherally on the cell membrane
("mourning edged" cells) due to the cytoplasm's
iii. LFT tendency to get condensed at the periphery, leading
iv. RFT to a perinuclear halo.
v. BS f Q. What is sertoli rosette and streptocytes?
pp A. A "Sertoli rosette" consists of cell aggregates with an
epithelial cell at the center surrounded by a ring of
vi. lipid profile leucocytes. "Streptocytes" are adherent chains of
vii. ECG leukocytes formed by filamentous, glue like substances.
viii. chest X- ray PA view Q. How will you take a biopsy?
A. Shave biopsy from a new blister
268 DYP SURVIVAL GUIDE FOR POST GRADUATES DYP SURVIVAL GUIDE FOR POST GRADUATES 269

Q. What are the types of Nikolskys sign? • Pear sign : is pooling of the fluid at the base of
A. • Nikolskys sign: The sign is elicited by applying the lesion due to gravity
lateral pressure with the thumb on the skin over • Sheklekoffs sign/ false nikolsky's sign: involves
a bony prominence. pulling of the remanant of the roof of the blister..
- direct : is over normal appearing skin at a - It is positive in subepidermal blisters like
distant site BP,MMP, Herpes gestationis, liner IgA
- marginal: is close to an existing lesion dermatosis, EBA, junctional and dystrophic EB,
Nikolsky sign is positive in pemphigus porphyrias and bullous SLE.
(active disease) • Microscopic Nikolskys sign: is rubbing the
• Nikolskys phenomenon : i.e the superficial layer surrounding normal appearing skin with an
of epidermis is thought to move over the the eraser to cause an microscopic split.this is useful
deeper layer , instead of immediately forming for taking a biopsy.
an erosion like in nikolsky's sign. Q. Treatment of Pemphigus group :
• Modified Nikolsky's sign: is peripheral Control Consolidation Maintenance
extension of blister on applying pressure on its ( intensity of (Maintain same (intensity of Rx
surface.this is useful in patients in whom there treatment to get intensity of treatment gradually ed to the
is no new blister for biopsy disease control =? 2 until 80% of lesions lowest level to prevent
• Bulla spread sign / Leutz sign : unidirectional weeks) have healed =? few new lesions)
weeks)
pressure applied by a finger causes peripheral
extension of the bulla beyond the marked Q. How is disease severity graded ?
margin.
Mild Moderate Severe
• Bulla spread sign / Ashoe Hansen's sign: < 10 skin lesion 10-30 skin >30 skin
modified bulla spread sign where pressure is lesion lesions
applied to the centre of the lesion. < 5 Mucosal 5-15 >15
Mucosal mucosal
- Bulla spread sign is positive in all varieties of lesions lesions
pemphigus and subepidermal blisters like BP, Rx:
DH, EBA, MMP,dystrophic EB, SJS, TEN Topical Steroids Oral steroids
• Pseudo Nikolskys sign / epidermal peeling sign: 1.5-2mg/kg body
this is nikolsky's sign elicited on erythematous weight
.
skin. DCP
- It is positive in SJS,TEN, Burns, bullous ILS
ichthyosiform erythroderma T Tetracycline
T. Dapsone
- It is due to necrosis of epidermal skin and not Azathiprine,Cyclophosphamide,Mycophenolate
due to acantholysis. mofetil
270 DYP SURVIVAL GUIDE FOR POST GRADUATES DYP SURVIVAL GUIDE FOR POST GRADUATES 271

* Newer advances in Rx :- Q. Prognosis of pemphigus foliaceous?


• Immunoablative doses of cyclo phosphamide A. • Better prognosis than pemphigus vulgaris
• Rituximab (Ab against CD 20) • General health not affected
• Immunoadsorpation • Benign course with intermittent crops of lesions
* Ix done in view of long term steroids Rx? Q. Presentation of pemphigus foliaceous?
a) Blood Pressure A. • Seborrheic dermatitis like
b) Blood Sugar • Erythroderma
c) Weight Record Pemphigus vegetans
d) Abdominal; girth record
Q. Why is this case of Pemphigus vegetans?
e) Rapture eye examination
A. Chronic VBD with
f) Dexa scan,Xray pelvis both hips
Primary presentation pustule(scalp)/vesicle
Ø Causes of death in pemphigus?
• Tumid , vegetating hyper trophic plaques in
• Mortality has decreased from 73 % to less than intertreginous areas
5% after the use of systemic steroids
• Ceribriform tongue.
• Causes of death - Septicemia due staphylococci
Q. Types of P. vegetans?
and streptococci infection
• Plumonary embolism A. Neumann type Hallopeau type
Early type with Later vegetating type
Pemphigus foliaceous
vesiculopustular lesions
Q. Why is this a case of P. foliaceous?
A. Chronic VBD with Q. D/D of P. vegetans?
• No oral involvement A. • Seborrheic dermatitis
• Seborrheic areas of body involved • Hailey & Hailey
• Flaccid Vesicles which have tendency to heal • HS
spontaneously • donovanosis
• Erosions with corn flake like crusts & muosy odour • Halogenodermas
Q. Can pemphigus foliaceous become pemphigus • amoebiasis
vulgaris? Q. What is the Premlata's sign?
A. • Yes, rarely due to epitope spreading A. • Characteristic cerebriform tongue in P. vegetans
272 DYP SURVIVAL GUIDE FOR POST GRADUATES DYP SURVIVAL GUIDE FOR POST GRADUATES 273

Q. D/D scrotal tongue ? Q. What is the Neonatal pemphigus?


A. • Melkerson Rosenthal syndrome A. • A pregnant woman with pemphigus vulgaris
• Familial may deliver a child with vesiculobullous lesion
• Downs syndrome • Self limited
• Pachyonychia congenita • Regresses in 3 to 4 weeks
• 50% of pts with scrotal tongue have geographic • To be distinguished from pemphigus neonatorum
tongue.
(severe impetigo)
Q. What are D/D of Geographic tongue?
Q. What is pemphigus herpetiformis ?
A. • Atopic dermatitis
A. • Papulovesicular lesions with central clearing
• Pustlar psoriasis
arranged in annular form
• Reiter's syndrome
• Pruritus is severe
• Li therapy
• H/e shows: eosinophils and neutrophils in
• AIDS
addition to the features if Pemphigus.
Pemphigus erythematosus
• May develop into pemphigus foliaceous or
Q. What is Pemphigus erythematosus / senear usher pemphigus vulgaris
syndrome? • Rx Dapsone /low doses of prednisolone.
A. • Localised variant of PEMPHIGUS FOLIACEOUS Q. What is IgA pemphigus ?
• features of both pemphigus & LELupus band
A. • vesicular pustule eruption with neutrophillic
test + in 80%
infiltration
• ANA + in 30 %
• Types are Sub corneal pustular dermatosis type
Q. What is endemic Pemphigus foliaceous or Fogo
and intra epidermal neutrophillic type.
salvagum?
• Morphologically : flaccid bullae on erythematous
A. • Distictive form of pemphigus foliaceous
skin arranged in annular / flower like pattern
• With burnt appearance on sun exposed areas
Paraneoplastic Pemphigus
• May be an infection cause by a virus transmitted
by black fly (Simulium prurinosum) Q. What is the other name for Paraneoplastic
Q. What is Vieira Sign? Pemphigus?
A. nail is initially yellow them becomes dark in colour - A. Paraneoplastic autoimmune multiorgan syndrome.
seen in pemphigus foliaceous
274 DYP SURVIVAL GUIDE FOR POST GRADUATES DYP SURVIVAL GUIDE FOR POST GRADUATES 275

Q. What is diagnostic criteria of Paraneoplastic • BP Ag 230 KDA- Hemidesmosome


Pemphigus?
A. Camisa et all criteria for diagnosis of Paraneoplastic
Pemphigus is: • Plectin
Major Criteria -
1. Polymorphous muco cutaneous eruption • DSG 1 - 160 KDA
2. Concurrent internal Carcinoma
3. Characteristic serum immunoprecipitation Lower epidermis
findings
• DSG 3 - 130 KDA
Minor criteria -
1. Positive staining of rat bladder by IIF
2. Intercellular & BMZ reaction on D/F Q. What are D/D’s of annular lesions in VBD ?
3. Acantholysis an H/e A. • Pemphigus herpetformis
Q. What are theTumour associated with PNP? • linear IGA bullous dermatosis
A. • NHL • Sub corneal pustular dermatosis
• CLL
• Hailey & Hailey disease
• Castleman Tumour
• Walderstorm macroglobulinemia Bullous Pemphigoid
• Thymoma Q. Why is this a case of BP?
• Retroperitoneal sarcoma
A. • Older age group 60-80 years
Q. Which are the auto Ab associated with PNP ?
A. Autoantibodies are directed against • Transient / no oral lesions
• Desmoplakin I 250 KDA • multiple well defined erythematous plaques
with itching associated with tense clear fluid
desmosome filled bullae with little tendency to burst.After
• Desmoplakin II 210 KDA bursting the ulcers have a tendency to heal.
• Lower abdomen & LL areas of involved
• Envoplakin 210 KDA • absence of head & neck involvement
desmosome • absence of mucosal involvement

• Periplakin 190 KDA • absence of atrophic scar


276 DYP SURVIVAL GUIDE FOR POST GRADUATES DYP SURVIVAL GUIDE FOR POST GRADUATES 277

Q. Differentiate between pemphigus vulgaris & BP Q. What are the drugs causing bullous pemphigoid ?
Pemphigus vulgaris BP A. - Topical drugs like benzyl benzoate , 5FU anthralin
40-60 years 60-80 years
- Penicillin, Penicilliamine, PUVA
M : F = 3:2 M : F = 1: 1
Oral Mucosal Involvement ++ transient oral mucosal invol in 40% - anti diabetic,
trunk & seborrheic areas invol lower abdo & LL involved
Nikolskys + Nikolskys +/- - Diuretic like furosemide,
BSS + BSS +/- - Diclofenac
Pruritis usually absent Present
Flaccid busters which ruptures tense buster on eryth plaque which Q. What are the diseases associated with BP?
easily & have no tendency to heal rupture to form erosions that heal
easily
A. 1. lichen planus - lichen planus pemphigoides
H/e suprabasal split with tomb stone H/e Split in L. lucida with eosinophil 2. Malignancy - renal cell carcinoma, gall bladder
appearance +
ca, ca colon, breast ca, leukemia
DIF- IgG & C3 in fish net pattern DIF- Granular Linear IgG & C3
deposition 3. Diabetes
Nikolsky + Nikolsky -
4. Psoriasis
Q. What are the factors ppt BP?
5. ulcerative colitis
A. • UV light
6. multiple sclerosis
• Surgery
7. nephropathy
• adhesive dressing
Q. Types of BP? 8. pernicious anemia
A. 1. Localised BP- Pretibial 9. eosinophilia
- P & S dyshydrosiform 10. Elevated IgE
- vulval in young females Q. Ix to confirm bullous pemphigoid
2. Child hood BP A. 1. H/P 2. DIF
3. Vesicular 3. IIF 4. Saline split DIF
4. Vegetating 5. ELISA assay
5. Nodular
6. Newer Ix- Immunoblotting / western blot
6. Drug induced
- Immunoprecipitation
7. Erythro dermic
- Immunohisto chemistry
8. urticarial plaques of vesicle
9. 105 KDA Ag anti P 105 Pemphigoid - widespread Q. What is the course & prognosis of BP?
erosions like TEN or Pemphigus vulgaris. A. • Benign self limited course from months & years
278 DYP SURVIVAL GUIDE FOR POST GRADUATES DYP SURVIVAL GUIDE FOR POST GRADUATES 279

• mortility rate reduced from 25 % in pre steroid 2. Non- scarring type - bullus pemphigoid like.
era to 6 % in the current steroid Rx era. However itching is either absent or mild.
Q. What is the D/D of bullous pemphigoid ? Q. DD of Desquamative gingivitis
A. 1. pemphigus vulgaris A. 1. Pemphigus vulgaris
2. mucus membrane pemphigoid
2. Bullus pemphigoid
3. Linear IgA bullus dermatosis
3. Mucus membrane pemphigoid
4. Other VBDs
4. Lichen planus
Mucus membrane pemphigoid
5. Contact stomatitis
Q. How do you diagnose a case of mucus membrane 6. Linear Iga bullus dermatosis in adults
pemphigoid?
7. Erythema multiforme
A. 1. Age group between 60 to 80 years
2. Chronic inflammatory vesiculo bullous diseases Q. What is ocular pseudo pemphigoid?
involving the mucus membrane with scarring A. • Mucus membrane pemphigoid like involvement
3. Histo pathology shows lesions & eosinophils of the eye associated with use of drugs like
with lymphocytes, in later lesions fibrosis seen pilocarpine, epinephrine or timolol eye drops
4. DIF shows linear deposition of IgG, IgA, Cg • Here DIF is negative
Q. What are the types of mucus membrane pemphigoid? Q. Which drug can induce mucus membrane pemphigoid?
A. a) Localized Involvement - A. • Penicillamine
1. Oral pemphigoid - desquamative gingivitis Q. Rx of MM pemphigoid?
2. Occular pemphigoid
A. Pt. are divided as
3. Nasopharyngeal involvement
1. low risk pt.- i.e patients with oral diseases or both
4. laryngeal involvement - hoarseness, chonic oral & skin diseases
sore throat, laryngeal stenosis
2. High risk pt.- i.e patients with ocular, genital,
5. Genital pemphigoid
nasopharyngeal, esophageal or laryngeal
b) Extensive involvement of skin & mucosa mucosal involvement
Q. what are the types of skin lesions in m.m pemphigoid?
• The low risk pt. are treated with -
A. 1. Scarring type i.e Bursting Perry type which is
localized, asso. with atrophic scaring . There is i. Oral prednisolone/ tetracycline + Nicotinamide
no mucosal involvement. / dapsone along with the topical cortico steroids
280 DYP SURVIVAL GUIDE FOR POST GRADUATES DYP SURVIVAL GUIDE FOR POST GRADUATES 281

• The high risk pt. are treated with • Papular uriticaria


i. Oral prednisone (1 to 1.5 mg/kg/day)with or • Scabies
without cyclophosphamide or azathioprine ( 1to
• neurotic excoriations
2 mg/ kg/ day)
Q. What are Associations of DH?
ii. care of eyes and other mucosa to prevent
development of synechia and strictures is a must. A. • DH as associated with
Q. Newer Rx of MMP? 1. Gluten sensitive enteropathy
A. • IVIg 2. Diabetes mellitus
• mycophenolate mofetil 3. Collagen vascular diseases like LE, Sjogren
• etanercept, daclizumab 4. Pernicious anemia
Dermatitis Herpetiformis 5. Biliary cirrhosis
Q. How will you diagnose a c/o dermatitis 6. Ulcerative colitis.
herpetiformis?
7. Splenic atrophy
A. i. usually in younger adults - 2nd & 3rd decade
Q. Which Carcinomas may be associated with DH?
ii. Intensely pruritic symmetrical papular or papulo
vesicular eruption on Bilateral extensors of limbs A. i. T-cell lymphoma
shoulders & buttocks ii. Oesophageal adenocarcinoma
iii. Characteristics H/o of subepiderimal blisters iii. Non hodgkin's lymphoma
with neutrophilic inflammatory infiltrate.
iv. Small bowel and stomach cancer.
iv. granular IgA deposition at tips of dermal papilla
on DIF(classically called picket fence pattern) Q. What is the natural H/o DH?
v. Prompt relief in symptom on Rx with dapsone. A. Majority of the patients have a life long course with
remissions & exacerbations
Q. In there mucosal involvement in DH?
A. • Rarely the oral mucosa may show maculopapular 10% to 15 % of patients have spontaneous remission
vesicles, ulceration which bleed and heal slowly Q. How will you treat a c/o DH?
• Laryngeal lesion may lead to hoarsenses. A. Ø the mainstay of treatment induces
Q. What are D/D’s for DH? i. Dapsone
A. • bullous pemphigoid ii. Avoidance of gluten in diet
• erytherma multiforme 1. Dapsone is started in the dose of 100mg / day.
• Transient acantholytic dermatosis If there is no improvement, then the dose is
282 DYP SURVIVAL GUIDE FOR POST GRADUATES DYP SURVIVAL GUIDE FOR POST GRADUATES 283

increased by 50 mg every 2 weeks to a maximum •. Painful erosions or ulcers


of 300 mg /day • Desquamative gingivitis
2. Gluten free diet - avoidance of Barley, Rye, oats, • Other mucosa may be involved like
wheat, millet.
i. Eye - irritation, redness, dryness, photosensiti-
3. Rice, & corn do not contain gluten & can be vity blurring of vision rarely conjunctival scarring
consumed & blindness
4. Other drugs like sulfasalasine, colchicin, ii. Nasal- Stuffiness ,crusting ,bleeding
cyclosporine, azathioprine & prednisolone have
iii. Laryngeal- hoarseness
been tried.
Q. Association with LABD?
A. • Other auto immune disorders like SLE,
Linear IgA Bullous Dermatosis
Dermatomyositis.
Q. How will you diagnose a c/o LABD?
Q. D/D for LABD?
A. i. Bi modal onset adult onset i.e 5th decade and Childhood Adults
childhood i.e children <5 years age Bullous impetigo Bullous pemphigoid
Epidermolysis Bullosa Dermatitis herpetiformis
ii. sudden onset of tense bullae associated with Porphyria Erythema multiforme
mild pruritis. Eye involvement must be
iii. In children there is a "cluster of jewels" differentiated from mucus membrane
pemphigoid
appearance i.e annular arrangement of new small
tense blisters around a crusted healing Q. How can one differentiate between BP & LABD?
erythematous plaque
iv. In adults the morphology may be : BP LABD
Tense blisters on an erythematous lesions may be BP like or DH like
1) Bullous pemphigoid like. base
2) Dermatitis herpetiformis like H/e shows sub epidermal split with H/e shows subepidermal split.
v. H/e showing subepidermal split with a sparse intact epidermis. Dermal infiltrate Dermal infiltrate is a spare and
consist of eosinophils, mononuclear consist of neutrophils & few
neutrophilic & eosinophillic infiltrate in upper cells & few neutrophilis eosinophils
dermis DIF shows linear IgG & C3 deposits DIF shows linear IgA & C3 deposits
vi. DIF shows linear IgA deposition along the DEJ along BMZ along BMZ
In skin split D/f the IgG is found In skin split D/f the IgG is found
vii. IIF shows circulating IgA anti- BMZ anti bodies epidermal side epidermal side
Q. What is the mucosal involvement in LABD? IIF shows circulating anti BMZ IgG IIF shows circulating anti BMZ IgG
A. • Oral mucosa is involved in 80% of pt. is found on epidermal side is found on epidermal side
284 DYP SURVIVAL GUIDE FOR POST GRADUATES DYP SURVIVAL GUIDE FOR POST GRADUATES 285

Q. How will you differentiate between DH & LABD? cyclosporine


DH LABD tetracycline + Nicotinamide
Group intensely pruritic BP like or DH like
papulovesicles on extensor surface of
mycophenolate mofetil
extremities Erythromycin
H/e –Subepidermal split, neutrophilic Subepidermal split,
micro abscess with few eosinophils sparse dermal infiltrate consisting of SCPD (SUBCORNEAL PUSTULAR DERMATOSES/
seen in adjoining dermal papillae neutrophils & eosinophils SNEDDON-WILKINSON DISEASE)
giving a multi locular appearance.
In older lesions the tips of dermal
Q. How will you diagnose a C/o SCPD ?
papillae separate and the blister A. i. Adult onset usually >40 yrs age
becomes unilocular.
DIF granular IgA deposits giving a IgA linear deposits ii. Female preponderance M:F = 1:4
picket fence appearance iii. Clinically multiple small flaccid pustules or
Dramatic response to DDS Steroid + DDS required vesicles that rapidly become pustular lesions on
Q. Course & prognosis of LABD? an erythematous base which usually occur in
crops. The pustules may coalesce to form
A. • In 2/3 of children remission occurs in 4 years
annular, circinate or serpiginous patterns. Lesions
rarely disease may persist into adult hood.
seen over flexural areas
• In adults diseases has a smoldering course but
iv. DIF & IIF are negative
usually remits after 3 to 6 years.
Q. Associations of SCPD?
Q. Pregnancy & LABD?
A. i. elevated serum IgA
A. • LABD usually remits after 2nd trimester
ii. IgA gammopathies - IgA myeloma ,IgA
• no fetal abnormality associated paraproteinemia
Q. Rx of LABD? iii. Collagen vascular diseases like SLE, RA
A. • Mild cases can be managed with the topical Q. What are culture reports form pustule ?
cortico steroids
A. • Sterile pustule
• Dapsone 1 to 2 mg/kg/day with or without
Q. H/p Finding of SCPD?
prednisolone
A. • Subcorneal blister with neutrophils
• Other sulphonamides like
Sulfamethoxipyridazine (0.5 to 1.5 g/day) • Absence of acantholysis
Sulfapyridine (1 to 2 gm/day) Q. D/D
• Other immunomodulators like : A. i. Impetigo
Colchicine (2mg/day) ii. Dermatitis herpetiformis
286 DYP SURVIVAL GUIDE FOR POST GRADUATES DYP SURVIVAL GUIDE FOR POST GRADUATES 287

iii. Subcorneal type of IgA pemphigus Q. How will you diagnose a c/o Hailey & Hailey
iv. Pemphigus foliaceus disease?
v. Pustule psoriasis A. 1. Disease is more common in the 3rd decade
vi. Glucagonoma syndrome 2. Autosomal dominant inheritance
vii. Acute generalized exanthematous pustulosis 3. Clinically characterized by recurrent vesicullo
bullous eruption over the flexures & groins.
Q. Course & prognosis of SCPD
4. Nikolsky's sign is usually negative
A. • SCPD is a benign condition with exacerbations,
5. aggravated by :-
remissions, usually remits in 5 to 8 years.
i) heat
• Appearance of myeloma is associated with poor
prognosis ii) UV light
Q. Rx of SCPD iii) Mechanical friction
iv) Infections like staph, strepto, proteus,
A. i. Sulfones such as -
pseudomonas, candida albicans
1) Dapsone 50 to 1500 mg / day
6. Histological characterized by dilapidated brick
2) Sulfapyridine 1 to 3 gm /day
wall appearance i.e separated epidermal cells
ii. Systemic corticosteroids due to acantholysis with lymphocytic infiltrate
iii. Acetretin 7. Immino histologically - Negative
iv. Infliximab Q. Is there a clinical diagnostic test for Hailey & hailey?
v. PUVA A. Yes, exposing the normal appearing skin of the back
vi. Milder cases may be managed worth topical to UV A causes characteristic h/p change in 24 hours
corticosteroids or topical tacalcitol. Q. What are the D/D for hailey & hailey?
A. D/D for crusted lesions - Impetigo
Hailey - Hailey Disease D/D for annular lesions - Tinea circinata
Q. What are the causes of Hailey- Hailey disease? D/D for vegetating lesions - Pemphigoid vegetans
A. H & H is an Autosomal dominant inherited disease - Dariers diseases
however only 70 % patients have a family history. Q. What are the course & prognosis of hailey & hailey
Q. Which is the gene involved in Hailey & Hailey diasease?
diseases? A. It has prolonged course of several weeks
A. SERCA - 1 i.e sarco endoplasmic reticulum calcium characterized by excerbations & remissions
ATPase 1 shows heterozygous mutations. The disease severity usually decreases with age .
288 DYP SURVIVAL GUIDE FOR POST GRADUATES DYP SURVIVAL GUIDE FOR POST GRADUATES 289

Q. What is the treatment ? Q. What are Clinically variants of TAD?


A. The acute flare of hailey & hailey can be treated with A. 1. Transient eruptive i.e multiple itchy lesions that
1. Antibiotics like tetracycline - 2gm /day followed settle in a few weeks
by maintenance of 500 mg/day other systemic 2. Persistent pruritic- i.e cases with less severe
antibiotics like penicillin, erythromycin or pruritus than the earlier type but lesions persist
minocycline have also been tried for months to years showing only partial
2. Systemic & topical corticosteroids response to treatment
3. Steroids sparing immunomodulatory drugs like 3. Chronic asymptomatic i.e persistant trunkal
dapsone in a loading dose of 100- 200 mg /day papules specially in inframammary region .
till the flare subsides followed by maintenance These cases are more commonly seen in
of 50 mg /day oncology patients
4. Topical tacrolimus, calcitriol ,tacalcitol
Q. Grover's diases is associated with which other skin
5. UVA with psoralens conditions ?
The persistant recalcitrant cases can be treated with A. A. Eczema - asteaotic & seborrhea
Ø Split skin graft
B. Allergic contact dermatitis
Ø Erbium Yag & Co² lesions
C. Psoriasis
Q. Who were hailey - hailey ?
D. solar keratosis & other condition of actinic
A. Two dermatologist brothers Howard & Hugh hailey. change
They described the condition.
Q. What are the H/P findings of Grover's disease?
Trascient Acantholytic Dermatosis
A. acantholysis with vesicle formation is the
Q. What is the etiology of TAD or grovers disease?
characteristic epidermal change with 5 main patterns
A. 1. Sun exposure
1. Pemphigus vulgaris like suprabasal acantholysis
2. heat & sweating
2. Pemphigus foliaceus like superficial acantholysis
3. febrile or post operative bed ridden patients
3. H & H like dilapidated brick wall appearance
4. Immuno deficiency like Leukamia & lymphoma
4. Darier's like acantholytic dyskeratosis with
I.e all these implicate an acrosyringeal involvement
corps ronds & grains
Q. What is it clinically characterized by?
5. Eczema like spongiotic pattern
A. 1. Usually seen in pt.> 40 years M:F is 3:1 to 7:1
Q. What is the DIF & IIF?
2. Papules, Papulovesicles & nodules occur in crops
which have variable pruritus A. Negative
290 DYP SURVIVAL GUIDE FOR POST GRADUATES DYP SURVIVAL GUIDE FOR POST GRADUATES 291

Q. What is D/D for Grover's disease? 1. Mild Diseases


A. 1. Acne - Avoidance of sun exposure & sweating
2. Acute febrile neutrophilic dermatosis - Using potent topical corticosteroids
3. Insect bite reaction - Symptomatic Rx for pruritus
4. Bullous impetigo 2. Moderate & severe Disease -
5. Candidiasis When pruritus is severe and there is no
6. Dermatitis herpetiformis improvement with topical line of management :
7. Drug erruption 1. Oral vitamin A -i) 50000 IU TDS for 2 weeks
8. Folliculitis ii) f/b 50000 IU Od for 12weeks
9. seborrheic dermatitis 2. Retinoids - i ) Acetretin 0.5 mg/kg/day
10. Scabies ii) Isotretinoin - 40 mg /day tapered to
Q. What are the histological D/D 10mg/day. Gradually over 12 weeks.
A. 1. Pemphigus vulgaris 3. Dapsone
2. Pemphigus foliaceus 4. PUVA & NB UVB
3. Hailey & Hailey diseases 5. Oral corticosteroids
4. Darier's diseases
5. Eczema Epidermolysis Bullosa
Q. Where is focal acantholytic dyskeratoses seen as a Q. what is the cause of epidermolysis bullosa?
secondary feature? A. It is genetically transmitted mechano-bullous disorder
A. Local secondary acantholysis is seen in Q. How is EB classified?
Ø herpes virus diseases
A. Depending upon the level of spilt
Ø Impetigo
1. EB simplex -Intraepidermal split at the level of
Ø Solar keratosis basal keratinocytes
Ø Squamous cell carcinoma 2. EB functional - Blister with in the BMZ
But in Grover's disease it is seen as a primary pattern 3. Dystrophic EB- Blister beneath the BMZ
Q. What is treatment for Grover's diseases? Q. Where is the mutation in EB ?
A. The patients of Grover’s diseases can be divided as A. 1. In EB simplex the mutation is at the genes
mild ,moderate, severe disease depending upon the
encoding for keratin 5& 14, plectin.
clinical involvement & can be treated accordingly.
292 DYP SURVIVAL GUIDE FOR POST GRADUATES DYP SURVIVAL GUIDE FOR POST GRADUATES 293

2. In dystrophic EB, the mutation is in the laminin - Biopsy needed to differentiate from
5, collagen XVII, plectin Junctional and Dystrophic forms.
3. In dystrophic EB, the mutation is in collegen vii 4. EBS with muscular dystrophy- rare
gene - Autosomal recessive.
Q. What are the types of EB simplex? - starts in early infancy.
A. 1. Weber Cockayne - Blisters seen at trauma prone sites which heal
- Most common type with scarring & milia formation.
- autosomal dominant - Muscular dystrophy can cause immobility
and eventually death.
- Manifests in infancy or early childhood
5. EB with mottled pigmentation- Autosomal
- Hyperhidrosis of palms & soles dominant.
- Thick walled blisters on palm & sales - Manifests at birth or infancy.
- callosities - Blisters may be localized or generalized and
- hair /teeth/nail/mucous membrane are normal heal with mottled pigmentation.
2. Koebner EB 6. EB Simplex Ogna- Heamorrhagic blisters on
hands and feet with Onychogryphosis.
- Sporadic cases
7. EB Simplex Superficialis- Autosomal dominant.
- Autosomal dominant
- Manifests at birth.
- Manifests at birth a soon thereafter.
Q. What are Juctional EB types?
- Tense blisters which often heal with atrophic
A. 1. Herlitz Junctional EB/EB lethalis.
scaring seen on pressure &friction areas.
- Autosomal recessive.
- Nail involvement present but minimal.
- Manifests at birth.
- Blistering tendency persists throughout life
but tends to improve after puberty. - Skin is fragile. Skin involvement heals with
minimal scaring.
3. Dowling - Meara EBS or EB herpetiformis.
- Oral and pharangyeal involvement may be
- Autosomal dominant. severe.
- Onset at birth or soon thereafter. - Painful corneal erosions.
- Mucosal involvement, nail shedding & - Teeth and nail involvement present.
transient milia formation.
- 40% patients die at 1st yr and most patients
- Blistering tendency decreases after 6-7 yrs. succumb in 1st 5 yrs.
294 DYP SURVIVAL GUIDE FOR POST GRADUATES DYP SURVIVAL GUIDE FOR POST GRADUATES 295

2. Generalized non Herlitz Junctional EB/ - Lesions heal with scarring and milia
Generalised trophic benign EB/ Wolf-Hunter. formation.
- Manifests at birth - Nail dystrophy present.
- Generalized fragility. 2. Pretibial dystrophic EB and EB pruriginosa.
- Significant nail, teeth and mucosal - EB pruriginosa
involvement but less severe than Herlitz. - Autosomal dominant / recessive.
3. Atrophic hair loss present. JEB with pyloric - Presents at birth.
Atresia. - Intractable pruritis.
- Presents at birth. - Violaceous papules and plaques.
- Non bilious vomiting on attempt to feed - Pretibial dystrophic EB
should arouse suspicion.
- Autosomal dominant.
- Teeth and nail involvement.
- Late onset.
- Death in few months.
- Itching bullae, atrophy and scaring on shins.
4. Inverse JEB. - Mucosal involvement absent.
- Fragility and generalized blisters in neonate 3. Autosomal Recessive Dystrophic EB/ Hallopeu-
that localizes to intertrigenous areas within Siemens.
several months.
- At birth or infancy.
5. Progressive JEB.
- Cutaneous as well as mucosal involvement.
- Delayed onset at 5-8yrs.
- Nikolsky's sign positive.
- Manifests as nail dystrophy and defective
- Syndactyly leading to Mitten like appearance.
tooth enamel.
- Flexural contactures.
- Partial deafness.
- Nail dystrophy.
- Blistering over the hands, feet, knees and
elbows. - Scarring alopecia.
Q. What are the types of dystrophic EB. - Microstomia and Ankyloglossia.
A. 1. Dominant dystrophic EB (Cockayne- Touraine - Hoarseness and aphonia may develop due
to laryngeal involvement.dysphagia due to
type)and Albopapuloid (Pasini type).
oesophageal involvement.
- Autosomal dominant.
- Aspiration pneumonia.
- Generalized blistering at birth at trauma
- Anal stenosis and faecal retention.
prone sites.
296 DYP SURVIVAL GUIDE FOR POST GRADUATES DYP SURVIVAL GUIDE FOR POST GRADUATES 297

- Ocular involvement. Laryngo- Onycho- Cutaneous syndrome (Shabir's


- Urethral stenosis. syndrome).
- Anaemia Q. How will you do the biopsy if there are no fresh
blisters?
- Growth retardation.
A. The skin that is clinically uninvolved but adjacent to
- Multiple sq cell carcinomas in areas of
a site where the patient usually blisters is selected(
repeated blistering.
The palms and soles should be avoided because the
Other types involve. increased skin thickness in these areas makes the
- Non Hallopeu Siemens recessive DEB. identification of the cleavage site difficult). After
- Transient bullous dermolysis of the new preparation in a sterile manner a cotton-swab, pencil
born eraser, or gloved finger or thumb is used to apply a
firm downward pressure to the skin and a traction
- Inverse recessive DEB.
is then exerted by twisting (at least 180 degrees each
Q. In which types of Eb there is premature death? way), until erythema is produced. The development
A. - Herlitz type JEB. of erythema at the site is a good endpoint. For
- Hallopeu -Siemens RDEJ newborns or infants, a reasonable rule of thumb is to
rub the selected area at least 20 times. Biopsies are
- Dowling Meara type of EBS.
performed at least five minutes after the induction of
Q. What are the D/D for EB? erythema to allow the development of a
A. - herpes simplex. microscopically identifiable blister. The biopsy site
- Congenital syphilis. is cleaned with an antiseptic solution (povidone
iodine stings less than alcohol) and draped. Two
- Staphylococcal scalding skin syndrome.
separate biopsies from adjacent sites are taken. Most
- Gunther's disease. laboratories prefer punch biopsies over shave
- Neonatal phemphigus. biopsies because punched specimens are easier to
- Incontinentia pigmentii. orient and embed for frozen sections.
- Kindler's syndrome. Q. What are the contents of michel’s media?
- Ichthyosis Bullosa of Siemens. A. ammonium sulfate, N-ethyl-maleimide, magnesium
sulfate, potassium citrate buffer, and distilled water;
- Bullous congenital ichthyosiform erythroderma.
the correct pH buffering is important. It is essential
- Linear IgA disease. to avoid contamination of the IFM sample with
- Bullous phemphigoid. formalin or TEM fixative to preserve antigen
- Cicatricial pemphigoid. expression.
298 DYP SURVIVAL GUIDE FOR POST GRADUATES DYP SURVIVAL GUIDE FOR POST GRADUATES 299

Q. What are the E.M.findings of EB? • Chorionic villus sampling (10-12 weeks of
A. Transmission electron microscopy (TEM) is time- gestation).
consuming, labor-intensive, expensive, operator- • Amniocentesis (16- 20 weeks of gestation) for
dependent, potentially more subjective in its successive pregnancies.
interpretation, and available only in a few specialized - Skin biopsy and blood sample from affected
centers. However, TEM may be helpful for patient to confirm mutation.
determining the level of skin separation and is used
- Fetal DNA- PCR.
in some instances to confirm or refine the diagnosis
obtained by immunofluorescence mapping (IFM). As Treatment involves:-
an example, in milder forms of EB, particularly those - Counselling the parents.
inherited in a dominant manner, IFM may be normal,
- General skin care measures i.e. avoid trauma,
whereas TEM can identify microsplits and other
gentle handling of baby, prevent infections.
ultrastructural abnormalities at the dermal-epidermal
junction. - General measures like air/water bed, loose soft
clothing, soft leather shoes, air conditioned low
TEM utilizes a magnification of 3000 to 30,000X, which
humidity cool environment, avoid band aids.
allows the visualization of the level of skin cleavage
and morphology of the structural components of the - Symptomatic treatment for painful blisters:-
dermal-epidermal junction, including keratin • Amitryptyline
intermediate filaments, hemidesmosomes, and • Phenytoin therapy
anchoring fibrils. TEM is of particular use in
• Vit E
diagnosing EB simplex Dowling-Meara, a subtype
of EB simplex. In this form of EB, IFM may show no • Tetracyclines
abnormality, except an intraepidermal split, whereas • Retinoids
TEM demonstrates the characteristic aggregation of
• Cyclosporine
keratin intermediate filaments in the basal
keratinocytes. • PUVA
Q. What is the Management of EB? - Surgical management:-
A. After confirming the diagnosis the following must be • Split thickness skin grafting.
done:- • Keratinocytes cultured in sheets.
- Genetic counseling. • Tissue engineering.
- Prenatal diagnosis with
300 DYP SURVIVAL GUIDE FOR POST GRADUATES DYP SURVIVAL GUIDE FOR POST GRADUATES 301

EPIDERMOLYSIS BULLOSA ACQUISITA Q How to differentiate between bullous SLE and EBA
Q. How will you diagnose a case of EBA? Bullous SLE EBA
A. EBA is a diagnosis of exclusion. The Roegnic's criteria No skin fragility Generalized skin fragility
Photosensitivity present No evidence of photosensitivity
include: Sun exposed areas involved in bullous Trauma prone areas involved in EBA
1. cutaneous lesions i.e. erosions, blisters, milia, SLE
Dramatic response to Dapsone Dramatic improvement seen with
scars at trauma prone sites corticosteroids
2. Adult onset
3. H/E showing subepiderma split Q. Course and prognosis of EBA?
4. Absence of family history of bullous disorders A. It has a chronic course with exacerbations and
remissions. Rarely the disease may remit completely
5. Family history like bullous pemphigoid.
6. DIF: linear Ig G and C 3 deposition along DEJ Q. Treatment of EBA?
7. Exclusion of all other blisrering diseases A. 1 Oral Steroids: 0.5 to 1.5 mg/ kg/ day
Q. what are the clinic variants of EBA? 2 Dapsone
A. 1. non inflammatory i.e dermolytic pemphigoid 3 Others: azathioprine, IV Ig, cyclophosphamide.
which heals with scarring mycophenolate mofetil,
2. Inflammatory i.e generalized eruption similar to 4 Rituximab has been tried.
bullous pemphigoid. h
Q. Differential diagnosis for EBA
A. 1. Bullous pemphigoid
2. Cicatrical pemphigoid
3. Dystrophic Epidermolysis pemphigoid
4. Linear IgA disease
5. Dermatitis herpetiformis
6. Porphyria cutanea tarda
7. Bullous SLE
1. ACANTHOSIS
NIGRICANS(AN)
Q. What is this acanthosis nigricans?
A. Presence of bilaterally symmetrical, hyperpigmented,
velvety plaques present in the axillae, neck, groins.
Q. What is the differential diagnosis?
A. 1. Dowling-Degos’ disease – soft fibromas, pitted
acneiform scars periorally
2. Gougerot-Carteaud Syndrome
3. Granular Parakeratosis – hyperkeratotic
papules/plaques, mid-age women
SHORT CASES Q.
A.
Classify AN
1. Hereditary-Autosomal Dominant (FGFR3
mutation)
2. Benign acquired(pseudo-AN)-higher fasting
insulin levels
3. Malignant- mucosal involvement common
4. Drug-induced
5. Nevoid
6. Autoimmune-SLE
Q. What are the common sites of affection?
A. Acanthosis nigricans is commonly seen on axillae,
neck, and groins. Other sites include the face, external
genitalia, flexor and extensor surfaces of elbows and
knees, dorsum of the hands and fingers,sub-
mammary, umbilicus, and perianal area.
Q. Enumerate a few endocrinopathies associated with
AN.
A. 1. Insulin-Resistant Diabetes Mellitus
302 303
304 DYP SURVIVAL GUIDE FOR POST GRADUATES DYP SURVIVAL GUIDE FOR POST GRADUATES 305

2. Acromegaly Q. What is difference between benign and malignant


3. Gigantism AN?
A. Malignant AN involves mucosa, occurs relatively
4. Cushing Syndrome
suddenly and is often pruritic. Unusual sites like
5. Addison’s disease palms (tripe palms) involved.
6. Polycystic Ovary Syndrome Q. What is the Leser-Trelat sign?
7. Hypothyroidism A. Leser-Trelat sign is the rapid appearance or growth
Q. What is HAIR-AN syndrome? of multiple seborrheic keratoses along with AN. Its
existence is controversial because eruptive seborrheic
A. Hyper-Androgenism Insulin Resistance – Acanthosis keratoses can also develop in erythrodermic patients
Nigricans in women who do not have an underlying malignancy.
Q. Which drugs can give rise to A N? Q. Describe the histopathological findings
A. 1. Nicotinic Acid 2. Niacinamide A. - hyperkeratosis
3. Testosterone 4. Diethylstilbestrol - irregular spiky papillomatosis
5. Oral Contraceptives 6. Protease inhibitors - slight hyperpigmentation in the basal layer.
7. Triazinate 8. fusidic acid - no acanthosis.
Q. What are the malignant conditions associated with Q. What is the pathogenesis of Acanthosis Nigricans?
AN? A. In benign forms it probably results from keratinocyte
A. Most cases are associated with adenocarcinoma, and dermal fibroblast proliferation stimulated by
especially of the gastrointestinal tract, particularly of insulin or insulin-like growth factor and stimulation
the stomach; others being the lung, breast, of tyrosine kinase growth factor receptor. In
malignant forms, transforming growth factor-a is
gallbladder, pancreas, esophagus, liver, prostate,
involved.
kidney, colon, rectum, uterus, and ovaries. Rarely,
Hodgkin’s disease. Q. What is the treatment of Acanthosis Nigricans?
Q. What are tripe palms? A. 1. If obese, then weight loss
2. If malignancy is the cause, then removal of the
A. Tripe (villous lining of ruminant stomach) palms, also
tumor required
known as acanthosis palmaris, are a manifestation of
malignant AN and are characterized by thickened, 3. Treatment of the underlying endocrinopathy
velvety palms with thickening of the palmar 4. Keratolytics- Salicylic acid ointment, retinoic acid
dermatoglyphics (pachydermatoglyphy). Serum cream, oral isotretinoin or acitretin
epidermal growth factor is elevated. 5. CO2 or Alexandrite Laser ablation
306 DYP SURVIVAL GUIDE FOR POST GRADUATES

Q. Is AN a misnomer?
2. ACROCHORDON
A. Yes, there is neither acanthosis nor hyperpigmenta-
tion histologically. The excess melanin is in the
Q. What is acrochordon?
stratum corneum.
A. It is a benign fibrous and fibrohistiocytic soft tissue
h tumour. It is the most common fibrous lesion of the
skin and presents as a soft skin-colored to slightly
hyperpigmented pedunculated papule,
predominantly on the neck, axilla and groin.
Q. What are the differential diagnosis of acrochordon?
A. - Seborrheic keratosis
- Nevus lipomatosus superficialis
- Angiofibroma
- Intradermal melanocytic nevus
- Pedunculated neurofibroma
Q. What are the synonyms of acrochordon?
A. - skin tag
- fibroepithelial polyp
- soft fibroma
- papilloma colli
- fibroma molluscum
- fibroma pendulum
Q. What are the Clinical features of acrochordon?
A. It presents as a soft skin-colored to slightly
hyperpigmented, pedunculated papule, predominantly
on the neck, axilla and groin, as well as scattered
elsewhere. They are usually asymptomatic
Q. What are the common sites for acrochordon?
A. Sides of neck, axilla, groins, eyelids

307
308 DYP SURVIVAL GUIDE FOR POST GRADUATES

Q. Which are the conditions associated with


acrochordon?
3. ACNE
A. Colonic polyps, Diabetes, Acromegaly. Most patients
Q. What are open and closed comedones?
do not have any associated disease except obesity.
They may also be associated with acanthosis A. Open comedones- they are 1-2mm in diameter. They
nigricans. represent follicles with a widely dilated orifice. The
black - head does not represent dirt, but it is due to
Q. What are the histopathological findings? oxidation of melanin and presence of certain lipids
A. Polypoid with variably loose to dense collagenous in the sebum.
stroma with thin-walled, dilated blood vessels in the Closed comedones- they are 1-3 mm diameter
center papular lesions without surrounding erythema. They
Q. What is the treatment for acrochordon? represent follicles that have become dilated with
A. Unless irritated or infected, skin tags are more of a cellular and lipid debris. They possess only a
cosmetic issue than a clinical concern and can be microscopic opening to skin surface. The lesions
easily removed by remain undetected unless the skin is stretched
between the fingers to reveal it.
a] simple scissor excision
Q. What are types of comedones?
b] electrodesiccation
A. Sandpaper comedones: white heads, distributed on
c] cryosurgery the forehead, produces gritty feel to the skin.
h Macro-comedones: these are closed or open
comedones more than 1mm in diameter.
Submarine-comedones: They are deep seated and
are the source of recurrent inflammatory nodular
lesions.
Secondary comedones: produced after exposures to
dioxins (chloracne), topical steroids, pomades
Q. What are grades of acne?
A. The severity of acne can be graded as under-
Grade 1(mild) : Comedones and occasional
papules are present.
Grade II (moderate) : Comedones, many papules,
few pustules.

309
310 DYP SURVIVAL GUIDE FOR POST GRADUATES DYP SURVIVAL GUIDE FOR POST GRADUATES 311

Grade III (severe) : Predominantly pustules nodules 3) Box scars: they are oval to round depressions 0.1-
and abscesses. 0.5mm in depth that have sharp vertical edges,
Grade IV (nodulocystic) : Mainly cysts or abscesses they don’t taper to a point at the base.
and wide spread scarring. Q. What are variants of acne?
GLOBAL ACNE GRADING SYSTEM: A. The different variants of acne are-
Depending on the type of lesion present a score of 0-
1) Neonatal acne.
4 is given
0. No lesions 2) Infantile acne
1. For comedones 3) Acne conglobata
2. Papules 4) Acne fulminans
3. Pustules 5) SAPHO syndrome – synovitis, acne, pustulosis,
4. Nodules hyperosteosis, osteitis
The score does not change if there is more than one 6) PAPA syndrome: sterile pyogenic arthritis,
lesion and the lesion of highest score is taken and pyoderma gangrenosum, acne
multiplied by the value for respective region 7) Acne excoriee
1 for nose 8) Acne mechanica
2 for Forehead 9) Acne with solid facial edema (Morbihan’s
2 for Right cheek disease)
2 for Left cheek 10) Acne with endocrinal abnormalities
3 for back 11) Acneiform eruptions:
Q. What are different types of scars?
-Epidermal growth factor receptor
A. The different type of acne scars are- inhibitor- induced eruption
1) Ice pick scar- long narrow scar that extends deep - Steroid folliculitis
into dermis, leaves a small yet obvious hole
which looks as if it is pierced by an ice pick and - Drug induced acne
caused by inflammatory break out, such as deep - Occupational acne and chloracne
papule or cyst. Most commonly seen on cheeks. - Gram negative folliculitis
2) Rolling scars- a result of tethering of otherwise
- Radiation acne
normal appearing skin to subcutaneous tissue
below. The process gives skin a rolling or - Tropical acne
undulating appearance. - Acne aestivalis
312 DYP SURVIVAL GUIDE FOR POST GRADUATES DYP SURVIVAL GUIDE FOR POST GRADUATES 313

- Pseudoacne of nasal crease Q. Difference between neonatal and infantile acne.


A.
-Apert syndrome-acrocephalosyndactyly.
Q. What is Gram-negative folliculitis?
A. It is seen in patients with pre-existing acne vulgaris
who are treated with long-term oral antibiotics
especially tetracyclines. Patients give a history of
initial reduction followed by worsening of the
lesions. The lesions are papulopustules around the
nose or deep-seated nodules. The organisms
involved are Enterobacter, Klebsiella, Escherichia and
Proteus. Treatment includes antimicrobials with
Gram-negative coverage and resistant cases can be Q. What are differential diagnosis of acne?
treated with oral isotretinoin for 4-5 months. A. COMEDONAL:
Q. What is hormone induced acne? a) Closed:
A. Acne with hormonal disturbances characterised by - Milia
irregular menses, deepened voice, clitoromegaly, - Osteoma cutis
muscular habitus, androgenetic alopecia, increased - Syrigomas
libido, hirsutism, insulin resistance and acanthosis - Sebaceous hyperplasia
- Trichoepithelioma
nigricans. Seen in patients with PCOS (Stein-
- Eruptive vellus hair cyst,
Leventhal syndrome) and congenital adrenal
- steatocystomamultiplex Plane warts
hyperplasia and HAIR-AN syndrome. The acne in
- Colloid milium
these patients is severe and more difficult to treat and
- Follicular mucinosis
onset is usually abrupt. Thorough history including
- Trichodiscomas, fibrofolliculoma
age and pubertal status, physical examination,
b) Open comedones:
laboratory investigations like total and free
testosterone,DHEAS, 17-hydroxyprogesterone. In - Trichostasis spinulosa
patients of PCOD, FSH and LH should also be done. - Favre-Racouchot disease
314 DYP SURVIVAL GUIDE FOR POST GRADUATES DYP SURVIVAL GUIDE FOR POST GRADUATES 315

- Nevus comedonicus Q. What is adrenarche?


Basaloid follicular hamartoma syndrome A. Adrenarche is an early sexual maturation stage at
- Dilated pore of Winer around 6-8 years of age. During adrenarche, the
adrenal cortex secretes increased levels of androgens
- Follicular spines ( trichodysplasia
such as DHEA and DHEAS, but without increase in
spinulosa, type 6 PRP, multiple myeloma,
cortisol levels. It is a result of development of new
demodicosis, lithium therapy)
zone of adrenal cortex, the zona reticularis. It is a
INFLAMMATORY TYPE process related to puberty, but distinct from
- Rosacea hypothalamic pituitary gonadal maturation and
- Perioral dermatitis function.
- Folliculitis Q. What is ethiopathognesis of acne?
- Acneiform eruption A. 1) Alteration in pattern of keratinization in
sebaceous follicles
- Keratosis pilaris
2) Circulating sex hormones
- Neutrophilic dermatosis
3) Quality and quantity of sebum production
- Furuncles/carbuncle
4) Follicular microbial flora
- Pseudofolliculitis barbae, acne keloidalis nuchae
- Resident bacterial flora includes p.acnes,
- Lupus miliaris disseminatus faciae p.granulosum, pityrosporum ovale.
- Follicular mycosis fungoides - P.acne causes increase in lipase, protease,
- Neurotic excoriations/faciei lecithinase which are proinflammmatory.
NEONATAL ACNE ( neonatal cephalic pustulosis) 5) Mediation of inflammation
- Sebaceous hyperplasia - Increase in interleukin 1 activity, ductal
- Milia corneocyte produces IL- á, IL- 8 and TNF-á
which causes inflammation.
- Candidiasis
6) Enviornmental factors
- Miliaria rubra (pustular variant)
- Hot humid climate
- Papular pustular eruption of hyper IgE
syndrome - Emotional stress.
- Vesiculo-pustular eruption of transient - External application of oil and pomades.
myeloproliferative disorder - Diet with carbohydrates with high glycemic
index.
316 DYP SURVIVAL GUIDE FOR POST GRADUATES DYP SURVIVAL GUIDE FOR POST GRADUATES 317

Q. Diet in acne? Q. What are the topical agents used in treatment of


A. Avoid the food which is responsible for exacerbation acne?
or development of acne, a balanced diet should be A. Topical agents available for acne treatment are-
suggested. High glycemic food should be avoided 1] Topical retinoids.
as patients taking high protein diet, low glycemic
index have less acne. 2] Benzoyl peroxide.
Q. How will you treat acne vulgaris? 3] Topical antibacterials.
A. 4] Sulfur, Sodium sulfacetamide, resorcin and
MILD MILD MODERATE MODERATE SEVERE Salicylic acid.
Comedonal Papulopustular Papulopustular Nodular Conglobata/Fulminans
First line Topical
retinoid
Topical retinoid +
Topical
Oral antibiotic+
Topical
Oral antibiotic+
Topical
Oral isotretinoin (may
Require concurrent oral
5] Azelaic acid.
antimicrobial retinoid+/- BPO retinoid+/- BPO corticosteroid, esp. for
acne fulminans)
6] Topical nicotinamide
Second Alternative Alternative topical Alternative oral Oral Isotretinoin Dapsone
topical retinoid +Alternate antibiotic‡ alt. +Alternative oral
line retinoid topical anti- topical retinoid ± antibiotic‡ alt.
High dose oral antibiotic
+topical retinoid+BPO Q. Descride about topical retinoids?
Azelaic acid microbial BPO/azelaic acid topical retinoid ±
Salicylic acid Azelaic acid BPO/azelaic acid
Salicylic acid A. Mechanism of action-
Options for Oral Oral Oral contraceptive/ anti
female contraceptive/
anti androgen
contraceptive/
anti androgen
androgen
1] Promote desquamation of follicular epithelium
patient
Surgical Comedone Comedone Comedone Intralesional corticosteroid (reduces comedones and development of new
options extraction extraction extraction +
Intralesional lesions).
corticosteroid
Refractory
to
Exclude Gram-
negative
Exclude Gram-
negative
2] Anti-inflammatory activity.
folliculitis folliculitis
treatment
Female patient: Female patient: Female patient: exclude
Agents available-
exclude adrenal or exclude adrenal or adrenal or ovarian
ovarian ovarian dysfunction Tretinoin (0.025% and 0.05%)
dysfunction dysfunction Exclude use of anabolic
Exclude use of Exclude use of steroid or other acne-
anabolic steroid anabolic steroid exacerbating medication Adapalene (0.1%)
or other acne- or other acne-
exacerbating
medication
exacerbating
medication
Tazarotene (0.05% and 0.1%)
Maintenance Topical Topical retinoid ± Topical retinoid ± Topical retinoid ± Topical retinoid ± BPO
retinoid ± BPO BPO BPO Q. Describe about Benzoyl peroxide?
BPO

Q. What advice is to be given to a patient of acne? A. Available in concentration (2.5%, 5% and 10%) . Wash
formulations are also available which are utilized for
A. 1] Do not Scrub as scrubbing the face increases
truncal acne.
irritation which worsens acne.
Mechanism of action- Anti-bacterial and decreases the
2] Do not apply oil over scalp.
likelihood of bacterial resistance.
3] A high glycemic diet may worsen acne.
Q. What are the topical antibiotic formulations
4] Use of only prescribed medications. available for treatment of acne?
5] Do not use harsh cleansers, anti-bacterial soaps, A. Topical clindamycin, topical erythromycin, are
astringents.
318 DYP SURVIVAL GUIDE FOR POST GRADUATES DYP SURVIVAL GUIDE FOR POST GRADUATES 319

available. Use of these is not recommended alone lesions are greater than 5mm in diameter and
because of increasing antibiotic resistance. unresponsive to conventional therapy.
Q. What is acne lotion? - Gram-negative folliculitis.
A. 3g precipitated sulphur, 3g resorcinol, 3g zinc oxide, - Inflammatory rosacea.
2ml glycerine, and 10ml methylated spirit. It can be - Pyoderma faciale.
used in all grades of acne vulgaris and especially for - Acne fulminans.
the application over truncal acne.
- Hidradenitis suppurativa.
Q. What is azelaic acid?
Q. What is the dose of isotretinoin?
A. It is a 1 : 2 heptanedicarboxylic acid that has
A. 0.5-1mg/kg/day in one or two daily doses. To obtain
antimicrobial, anti-inflammatory and comedolytic effect.
the greatest chance of prolonged remission, patients
May lighten post-inflammatory hyperpigmentation.
should receive 120-150mg/kg over the treatment
Available as 10% and 20% formulations. Azelaic acid course.
reduces comedones by normalizing the disturbed
terminal differentiation of keratinocytes in the follicle Q. How will you monitor a patient on oral isotretinoin?
infundibulum. A. 1] Baseline-
Q. What are the oral antibiotics available for acne? - Examination- Careful history and physical
examination.
A. TETRACYCLINE DOXYCYCLINE MINOCYCLINE ERYTHROMYCIN
250-500 mg 1-4 50-100 mg once or 50-100mg once or 250-500mg 2-4 times - Identify those patients at increased risk of toxicity
times a day taken on
an empty stomach.
twice a day. twice a day. a day. or adverse effects.
Side effects- Side effects- Side effects- Side effects- - Laboratory-
Nausea, vaginitis photosensitivity vertigo, Gastrointestinal upset
and perianal pigmentation in and vaginal itching. • Pregnancy test (in women of child bearing age)
itching,staining of areas of
growing teeth,
nephrotoxic,
inflammation, oral • CBC with platelet count.
mucosa, post acne
intracranial
hypertension.
osteomas or scars, • Liver function tests (AST, ALT, ALP, Bilirubin)
in photodistributed
pattern, on the • Serum lipid profile.
shins, in sclera,
nailbed, ear • Renal function tests (BUN and Creatinine)
cartilage, Teeth
and lupus like - Special tests- Consider baseline X-ray of wrists,
hypersensitivity
syndrome, ankles, or thoracic spine if planning for long
intracranial term retinoid therapy.
hypertension.

Q. What are the indications of oral isotretinoin? - Consider ophthalmologic examination if patient
have history of cataracts or retinopathy.
A. - Severe recalcitrant nodular acne (Inflammatory
320 DYP SURVIVAL GUIDE FOR POST GRADUATES

2] Follow ups-
4. ACNE EXCORIÉE
- Examination- Clinical evaluation monthly.
- Assessment of patient response, improvement Q. Why is this acne excoriee ?
and complaints of adverse effects. A. Mild acne is accompanied by extensive excoriations
- Laboratory- Monthly. and hyperpigmentation.
• CBC with Platelets. Q. D/Ds ?
• LFT A. Neurotic excoriation
• Serum lipid profile. Q. What is Acne excoriée also known as?
• RFT A. Acne excoriée des jeunes filles (excoriated acne of
• Pregnancy test for women in child bearing age young girls)
- Special tests- As indicated by symptoms. Q. What is Acne excoriée?
A. – A subset of neurotic excoriations characterised
h by ritualistic picking of acne lesions. Often, mild
acne is accompanied by extensive excoriations.
– Lesions can become so deep that scarring may
occur.
Q. Is there any sex predilection for the disease?
A. Yes, it is frequently seen in young women.
Q. Who are at an increased risk of acquiring this
disorder?
A. Patients with an anxiety disorder, obsessive–
compulsive disorder or personality disorder are
particularly at risk.
Q. What is the treatment?
A. 1] Antidepressants
2] Psychotherapy
3) Trichloroacetic acid peels
4) Skin lightening peels and also azelaic acid
5) In severe cases, oral isotretinoin can be
considered.
321
DYP SURVIVAL GUIDE FOR POST GRADUATES 323

Q. How would you describe a typical melanocytic nevus?


5. ACQUIRED
A. A melanocytic nevus is well-defined, symmetrical
MELANOCYTIC NEVUS round or oval, generally measuring 2-6 mm in
diameter.
Q. What are acquired melanocytic nevi? Q. What are junctional nevi?
A. They arise due to defects in the development of A. A junctional nevus is a brown or dark brown, macular
epidermal melanocytes, known as “nevus cells” and lesion with slight accentuation of skin markings
such nevus cells may also migrate into the dermis. generally appearing on any part of the body including
Nevi are thought to be either developmental mucocutaneous surfaces.
malformations (hamartomas) or benign proliferations. Q. How are junctional nevi differentiated from freckles?
Q. What is the differential diagnosis of acquired A. Freckles have a predilection for sun-exposed areas
melanocytic nevi? and fade on sun protection, which is not a property
A. 1) Seborrheic keratosis – rough, verrucous surface, of junctional nevi.
pseudo-horn cysts Q. What are compound nevi?
2) Dermatofibroma – firm consistency, dimpling, A. Compound nevi may be elevated lesions, their colour
mostly on lower extremity being a lighter shade of brown then that of junctional
nevi. Compound nevi have multiple round ovoid
3) melanoma (look for ABCDE rule: A asymmetry, globules, sometimes forming a cobblestone pattern
B border irregularity C colour variegation D or they may be dome-shaped or papillomatous .
diameter > 7 mm and E evolution changing They may contain coarse long hair in the center.
morphology)
Q. What are dermal nevi?
4) Pigmented basal cell carcinoma: rolled out
A. Dermal nevi seen after adolescence are dome-shaped,
border
smooth-surfaced, hemi-spherical nodules appearing
5) Congenital nevus: history, often hairy on the face and the neck. Telangiectases may be seen
Q. What is the routine word for acquired melanocytic and impart an agiomatous appearance. The adult
nevus? variety of dermal nevi consists of sessile or soft,
wrinkled sac located in the flexures, which
A. Acquired melanocytic nevus is commonly known as
characteristically demonstrates the presence of
a “mole”
cholinesterase in histochemical reactions.
Q. What are the common varieties of acquired Q. Do acquired melanocytic nevi have a potential for
melanocytic nevi? malignancy?
A. 1) Junctional Nevus 2) Compound Nevus A. Yes, they have a tendency to develop malignant
3) Intradermal Nevus
322
324 DYP SURVIVAL GUIDE FOR POST GRADUATES DYP SURVIVAL GUIDE FOR POST GRADUATES 325

melanoma but very rarely(1:100,000). Most 13) Post-op fever


melanomas arise de novo. Pointers toward Q. What are the indications for removing acquired
development of malignant melanoma are : diameter melanocytic nevi?
>7 mm, irregular edge, irregular pigmentation,
A. 1) changing lesion-size, shape and color
inflammation, bleeding, crusting, and oozing.
2) atypical appearance in suspicion of melanoma
Q. Which cytokines have a protective effect against
malignant transformation of acquired melanocytic 3) cosmetic
nevi? 4) repeated irritation and itching
A. Interleukin-1a, IL-6 and TNF-a Q. What is abtropfung phenomenon?
Q. What is pseudomelanoma? A. Migration of nevus cells from the basal layer into the
dermis.
A. Incomplete removal of the nevus by shaving or
excision stimulates the remaining tissue to proliferate h
in a manner resembling melanoma. It has a “fried
egg” appearance.
Q. What are the triggers for the development/growth
of melanocytic nevi?
A. 1) Sun exposure
2) Neonatal phototherapy
3) Blistering process- TEN/SJS, epidermolysis
bullosa, severe sunburn
4) Scarring process – Lichen sclerosus
5) Chemotherapy
6) Allogenic bone marrow transplantation
7) Renal transplantation
8) Chronic Myelogenous Leukemia
9) Increased hormone levels in pregnancy
10) Addison’s disease
11) Thyroid hormone
12) Atopic dermatitis in children
DYP SURVIVAL GUIDE FOR POST GRADUATES 327

6. ACROKERATOSIS POROKERATOSIS

VERRUCIFORMIS OF HOPF Q. Morphology of porokeratosis?


A.. It presents as a hyperkeratotic papule or plaque, with
Q. What is acrokeratosis verruciformis of HOPF? an annular appearance due to thread like elevated
border that expands centrifugally.
A. Multiple skin coloured, small, warty papules on the
dorsal aspect of the hands and feet. Q. Differential diagnosis of porokeratosis?
Q. Differential diagnosis? A.. · Pityriasis rosea
A. – Verruca plana · Dermatophyte infection
– seborrheic keratosis · Hansen’s disease
– stucco keratosis · Annular LP
– solar keratosis · Sarcoidosis
– arsenical keratosis (rarely) · Syphilis
Q. Etiology of acrokeratosis verruciformis of HOPF? · BCC
A. Autosomal dominant disorder due to mutation in the · Granuloma annulare
gene ATP2A2. · Basal cell epithelioma
Q. What other disease is associated? · Actinic keratosis
A. Darier disease · ILVEN
Q. Describe histopathology ? · IP (stage 2)
A. Hyperkeratosis, · Linear LP
papillomatosis and acanthosis, · Verruca vulgaris
“Church spire” confirguration (other causes of church · Porokeratotoic eccrine ostial and dermal duct
spire appearance are verruca vulgaris, seborrheic nevus
keratosis, acanthosis nigricans) Q. Types of porokeratosis?
Q. Treatment? A.. – Porokeratosis of mibelli
A. – Electrodessication – Disseminated superficial porokeratosis
– Cryosurgery – Disseminated superficial actinic porokeratosis
– Urea, lactic acid, a-hydroxy acid and retinoid – Linear porokeratosis
preparations have also been tried. – Punctate porokeratosis
326
328 DYP SURVIVAL GUIDE FOR POST GRADUATES

– Porokeratosis Palmaris et plantaris disseminata


(PPPD)
7. ACTINIC KERATOSIS
– Porokeratosis ptychotropica
Q. What do actinic keratosis represent?
Q. Epidemiology of porokeratosis?
A. They represent in situ dysplasias resulting from sun
A.. – Autosomal dominant inherited disorder exposure.
– Porokeratosis of mibelli affects boys more than Q. Differential diagnosis?
girls except DSAP
A. – Seborrheic keratosis
– PPPD is more common in males
– Squamous cell carcinoma
– DSAP is more common in Caucasians and rare
in blacks – Bowen’s disease
– Linear porokeratosis has been reported in – Superficial basal cell carcinoma (BCC)
monozygotic twins Q. Where are they found?
Q. Pathology of porokeratosis A. Found chiefly on chronically sun exposed surfaces
A. Cornoid lamella is the sine qua non for histological of face, ears, balding scalp, dorsal hands and
diagnosis. forearms.
Thin column of tightly packed parakeratotic cells Q. Describe morphology of these lesions?
extending from an invagination of the epidermis A. Multiple, discrete, flat or elevated verrucous or
through the adjacent statum corneum often protruding keratotic red pigmented or skin coloured surface
above the surface of the adjacent skin,under the covered by adherent scale but sometimes smooth and
lamella. Granular layer is absent or thinned, shiny.
dyskeratotic and pyknotic kerationocytes with
perinuclear edema in the spinous layer beneath the Q. How does it feel on palpation?
lamella are seen. Dermal lymphocytic infiltrate is seen. A. Surface is rough like sandpaper more easily felt than
Q. Treatment of porokeratosis? seen.
A. – Unsatisfactory Q. Which are the most common epithelial precancerous
– Topical retinoids, 5 FU, imiquimod lesions?
– Cryotherapy, CO2 laser local excision have been tried A. Actinic keratosis
– In widespread lesions, oral acitretin may be tried Q. Histological types of Actinic Keratosis
Q. Dermoscopy of porokeratosis? A. – Hypertrophic
A. Dots and globules in the border ( white tract – Atrophic
structures) which corresponds to cornoid lamella – Bowenoid
329
330 DYP SURVIVAL GUIDE FOR POST GRADUATES DYP SURVIVAL GUIDE FOR POST GRADUATES 331

– Acantholytic Q. Differential diagnosis of seborrheic keratosis


– Pigmented A. – Verruca plana
– Lichenoid – Basal cell carcinoma
– Epidermis may be – Acantholytic or atrophic with – Squamous cell carcinoma
overlying overlying occasional parakeratosis – Melanoma
– Basal cells are dysplastic - Bowen’s disease
Q. How can it be prevented? Q. Where else are they seen?
A. Regular application of sunscreen and diet which is A. Instep, dorsa of feet, forearms, dorsal hands
low in fats
Q. Which areas are never affected?
Q. Describe the hypertrophic type of actinic keratosis?
A. Palms and soles
A. It leads to cutaneous horn formation most frequently
Q. What is seen on histopathology of stucco keratosis?
present on dorsal forearms and hands.
A. Hyperkeratotic type of seborrheic keratosis with no
Q. Difference between seborrheic and actinic keratosis?
hypergranulosis and no wart particles seen by
A. Seborrheic keratosis are more pigmented, have a electron microscopy.
stuck -on appearance and are sharply marginated as
Q. What is Leser–Trélat sign?
compared to actinic keratosis.
A. Sudden appearance of numerous seborrheic keratosis
Q. Treatment?
in an adult which may be the cutaneous finding of
A. electrodessication, cryotherapy, excision. 5 FU, internal malignancy. (commonly adenocarcinoma,
imiquimod. primarily of the stomach). Acanthosis nigricans and
STUCCO KERATOSES/SEBORRHEIC KERATOSES tripe palms accompany seborrheic keratosis.
Q. How is seborrheic keratosis present over palms?
Q. Why is this seborrheic keratosis?
A. Eccrine poromas
A. Oval, slightly raised, tan/light brown to black,
sharply demarcated papules or plaques rarely more Q. What are dermatosis papulosa nigra?
than 3 cm in diameter ‘stuck- on’ lesions occurring A. A variant of seborrheic keratosis seen over the central
on the lower legs especially in vicinity of Achilles face common in Africans, Americans and Asians.
tendon. Q. Six types of histopathology of seborrheic keratosis?
Q. What are the other names? A. Types are:
A. Stucco keratosis or Kerato-elastoidosis Verrucosa of 1. Hyperkeratotic
extremities.
2. Cloral
332 DYP SURVIVAL GUIDE FOR POST GRADUATES

3. Adenoid
8. ALOPECIA AREATA (AA)
4. Acanthotic reticulated
5. Irritated Q. Why is this alopecia areata ?
6. Melanoacanthoma A. There is a well-defined area of hair loss, no scaling,
Q. What is the recent concept of etiology of seborrheic no scarring, no breakage of hair at different length.
keratosis? Q. Differential diagnosis?
A. HPV – hence some believe it to be contagious A. • Trichotillomania
Q. Can seborrheic keratosis turn malignant? • Tineacapitis (non - inflammatory)
A. No, if there is malignancy it has to have been there • Syphilis
from the beginning. • Androgenetic Alopecia
Q. Treatment of seborrheic keratosis • Pseudopelade of Brocq
A. Liquid nitrogen, curettage, electro- fulguration, CO2 • Telogen Effluvium
laser vaporization and shave excision. • Traumatic Alopecia
Q. Histopathology of seborrheic keratosis? • Alopecia Mucinosa
A. Hyperkeratosis, acanthosis, papillomatosis • Cong Triangular Alopecia
Q. Dermascopy of seborrheic keratosis? • Metastatic malignancies
A. Comedone- like lesions, milia- like lesions, network- Q. Classify alopecias?
like pattern A. Non-scarring
• Diffuse
– AGA
– Telogen effluvium
– Post partum alopecia
– Neonatal alopecia
– Anagen effluvium
– Drug induced
– Hair shaft disorders
– Ocassional AA
• Focal
– AA
333
334 DYP SURVIVAL GUIDE FOR POST GRADUATES DYP SURVIVAL GUIDE FOR POST GRADUATES 335

– Fungal, bact or viral infection • “Alopecia areata” was first used by Sauvages in
– Trichotillomania his “Nosologica Medica”, published in 1760 in
Lyons, France.
– Traction alopecia
• Also known as Pelade
– Tick bite alopecia
Q. Epidemiology of AA?
Scarring
A. • the prevalence of alopecia areata is 0.1 to 0.2%,11
• lymphocyte associated
– LPP • with a calculated lifetime risk of 2%
– Central centrifugal cictricial alopecia • Family history positive in 10 % AA patients
– Graham Little Piccardi Lassueur syndrome Q. Explain Pathogenesis of AA?
– Pseudopelade of Brocq A. • “Immune privilege” becomes defective
– DLE • Genetic predisposition-
• Neutrophil associated – HLA haplotype; DR4, DQ3, DR11
– Perifolliculitis capitis abscedens et suffodiens – Concordance seen in identical twins
– Folliculitis decalvans – Family history positive in 10%
• Mixed type Q. Animal models for AA?
– Acne Keloidalis Nuchae A. • Dundee experimental bald rat (DEBR)
– Acne necrotica • C3H/HeJ mouse
– Pustular dermatosis of scalp Q. IKEDAS classification?
Q. History of AA? A. • Common type (81%)
A. • Hippocrates called alopecia, “Fox’s disease” – Patchy pattern with short duration
• Alopecia areata was first described by Cornelius • Atopic type(10%)
Celsus in 30 AD. – Ophiasis and reticular, 50% may go into
• Cornelius Celsus actually described two types totalis
of alopecia, alopecia totalis, and alopecia • Prehypertensive type (4%)
Ophiasis, which gives a “snake” like pattern. – reticular
Celsus actually thought that alopecia Ophiasis
• Autoimmune type/endocrine type (5%)
only occurred in children
– Alopecia totalis
Q. What are the other names of alopecia areata?
Q. Can we adapt it in Indian setting?
A. • Alopecia areata is sometimes known as “area
celsi” in tribute to Cornelius Celsus. A.. • No, as the percentage of patients with pre-
336 DYP SURVIVAL GUIDE FOR POST GRADUATES DYP SURVIVAL GUIDE FOR POST GRADUATES 337

hypertensive group is very few, they should be • Coudability test- try to push hair towards scalp
classified as adult and childhood AA and it bends
• And further subdivided into- classical, atopic and • Dystrophic hair
autoimmune in both groups Q. Explain exclamatory mark hair?
Q. Classification by pattern A. • Anagen hair suffers defective keratinization
A. - Generalized: universalis &thinning and later turns into telogen, which
causes narrowing of the shaft
- Totalis: Restricted to scalp
• As this hair grows out it appears broad distally
• Patchy • Ophiasis and thin proximally, appearing like an
• Sisaphio • Reticulate exclamatory mark
• Diffuse • Subtotal • This is seen in most severe form of injury
• totalis Q. What are dystrophic hair?
Q. What are the symptoms of AA? A. • If least form of injury, the anagen bulb does not
A. • Asymptomatic go into telogen but only changes to dystrophic
anagen hair
• Patches of hairloss on scalp or beard or anywhere
• In moderate form of injury, we notice only a
on body usually noticed by a relative or the
telogen bulb on hair mount
barber
Q. Changes in nails?
• Overnight greying of hair
A. • Scotch plaid nail pitting
• Brittle nails with pitting
• Trachyonychia(sandpaper like roughness
Q. Signs in hair? because of excessive longitudinal ridging)
A. • Hair pull test- positive (take 20 hair and more • Red or mottled lacunae
than 10 % hair get pulled out)
• Nail thinning and ridging
• Exclamation mark hair
• Longitudinally arranged punctate leukonychia
• Cadaver hair-broken hair seen over scalp as • Dystrophy, onycholysis
comedones (black dots)
• Few reports of cataracts have been reported
• Follicular ostia- ostia are well preserved in
Q. What is the difference in nail pitting in psoriasis and
alopecia areata, in contrast to the findings in
AA?
scarring alopecia
A. • AA: Regular pitting, superficial pitting, scotch
• Depigmented hair plaid pattern
• Frayed rope appearance of free ends • Psoriasis: Irregular pitting, deep pitting
338 DYP SURVIVAL GUIDE FOR POST GRADUATES DYP SURVIVAL GUIDE FOR POST GRADUATES 339

Q. Dermoscopy findings of AA? • Loss of eyebrows and eyelashes


A. • Yellow dot- Sebum collection in ostia • Alopecia totalis
• Black dots- broken hair • Alopecia universalis
Q. What are the scoring system in AA? • Recurrent episode
A. - Severity of alopecia tool score (SALT SCORE) • Pattern-ophiasis, sisaphio, reticular
Q. What are the associations of AA? • Nail changes
A. • Down’s syndrome • Asso systemic disorders
• Hashimoto’s thyroiditis • Associated genetic disorders-Down’s
• Addisons disease • Patchy regrowth of terminal hairs
• Pernicious anemia • Family history
• Vitiligo Q. What are the lab tests for AA?
• LP A. • Fungal culture, skin biopsy, serology for lupus
• Morphea erythematosus or serology for syphilis.
• Lichen sclerosis et atrophicus • CBC for anemia
• Pemphigus foliaceus • Thyroid Function Tests
• SLE Q. What is the histopathology of AA?
• Sjogren Syndrome A. • Swarm of bees appearance
• Ulcerative colitis • Decrease in total number of follicles
• Myasthenia gravis • Increase in number of vellus hair
• Autoimmune hemolytic anemia Q. What if we don’t treat AA?
• Diabetes mellitus A. Spontaneous remission occurs in up to 80% of
• Autoimmune testicular and ovarian disease patients with limited patchy hair loss of short duration
• Chronic mucocutaneous candidiasis with (< 1 year). Such patients may be managed by
endocrinopathy reassurance alone, with advice that regrowth cannot
be expected within 3 months of the development of
Q. What are the bad prognostic signs of AA?
any individual patch. The prognosis in longstanding
A. • Early age of onset extensive alopecia is poor and a wig may be an
• Extensive scalp involvement option.
340 DYP SURVIVAL GUIDE FOR POST GRADUATES

Q. What are the treatment options for AA?


A. TOPICAL :
9. APHTHOUS STOMATITIS
• Immunosuppresants - ILS
• Immunomodulators - tacrolimus Q. Why is it aphthous stomatitis?
• Contact sensitizers - SADBE, DPCP, DNCB A. Presence of multiple, painful, round or ovoid ulcers
• VEGF stimulators - minoxidil 5-10% with circumscribed margin, erythematous halo and
• Phototherapy - PUVA a yellow or grey floor on oral mucous membrane,
• Cryotherapy - liquid nitrogen h/o recurrence, no other sites are involved.
• LASER theraphy - Excimer 208, P. diode Laser
Q. What is differential diagnosis of aphthous
904nm
stomatitis?
SYSTEMIC :
• Immunosuppresants - Cyclosporine A. Aphthous stomatitis must be differentiated from the
• Immunomudulators - Alefacept, Isoprinosin following:
• Photoshemotherapy - Whole body PUVA · Herpetic gingivostomatitis, Recurrent labialis
• Misc. - Sulfasalazine, IVIg herpes
OTHERS : · Pemphigus
• Psychotherapy - Hypnotherapy, systemic
desensitization · Lichen planus
• Supportive - tattooing, hair pieces, wigs · Erythema multiforme
• PRP, IL candida antigen · Cicatricial pemphigoid
Q. Explain contact immune therapy?
· Behcet’s syndrome
A. • Contact immunotherapy was introduced by
Rosenberg and Drake in 1976. · Candidiasis
• The contact allergens used include: · Avitaminosis like pellagra and scurvy
– 1-chloro,2,4,dinitrobenzene (DNCB); [Banned · Drug - induced
because mutagenic]
– squaric acid dibutylester (SADBE) Q. What is etiology of aphthous ulcer?
– 2, 3-diphenylcyclopropenone (DPCP) A. Exact cause not known. The various etiologic agents
– Initially patient senstitzed with higher are-
concentration of allergen applied on forearm. · Trauma such as self biting, tooth brush injury,
– Later patient challenged with very low dental procedures
concentration of allergens on the alopecic areas
· Spicy foods, citrus fruits
to maintain a brisk erythema over the lesion.
· Cessation of smoking (smoking is relatively
beneficial in pemphigus, ulcerative colitis,
h though it is injurious to general health)
341
342 DYP SURVIVAL GUIDE FOR POST GRADUATES DYP SURVIVAL GUIDE FOR POST GRADUATES 343

· Allergy · It begins with red discrete papules


· Stress · They present as 1-5, round, painful, shallow
· Heredity (1/3rd patients ) ulcers measuring 2-4 mm and surrounded by
erythematous halo and edema. Ther ulcer floor
· Hormonal changes in women such as
is covered with yellow gray pseudo-membrane.
menstruation, pregnancy, menarche and
menopause · They tend to heal within 7-10 days without
scarring
· Haematological abnormality
2) Major aphthous ulcer :
· Infections and immunologic manifestations
· Also called as Sutton ulcers, mostly affects lips,
Q. What is the pathogenesis behind aphthous ulcer?
soft palate and tongue
A. The cause of recurrent aphthous ulcer is unknown
· They are larger (more than 1cm), deeper and
but believed to be due to T- cell mediated localized
longer duration, more frequent in recurrence and
destruction of oral mucosa associated with increased
more painful
relative ratio of CD 8+ : CD 4+T cell.
· They are round and oval with a raised irregular
Q. What are most common sites of aphthous ulcers?
border with surrounding edema
A. It mostly occurs as recurrent painful ulcers on buccal
· They appear as group of ulcers 1-6 in number on
mucosa, labial mucosa and tongue, buccal and
any part of oral mucosa and heal in 10-20 days
lingual sulci and soft palate. Involvement of heavily
with scarring
keratinized mucosa of the hard palate and gingiva is
much less common. 3) Herpetiform ulcer :
Q. What is the classification and clinical features of · It is seen in females of older age
aphthous ulcers? · Inumerable pinpoint vesicles about 1-3mm,
A. Aphthous ulcers are classified into three forms: extremely painful and appear on any oral site
1. Minor aphthous ulcer (80%) · They enlarge to form large, irregular ulcers and
heal in 10 days or more
2. Major aphthous ulcer (10%)
· Recurrence is frequent
3. Herpetiform aphthous ulcer (10%)
Q. What are other diseases associated with aphthous
1) Minor aphthous ulcer :
ulcer?
· Most common, also called Mikulicz ulcer
A. Other conditions in which aphthous ulcer is seen are :
· It occurs mostly at age of 10-40 years
· Familial Mediterranean Fever
· Prodromal stage of tingling and burning for 1 to
· Hyperglobulinemia D
2 days
344 DYP SURVIVAL GUIDE FOR POST GRADUATES DYP SURVIVAL GUIDE FOR POST GRADUATES 345

· Deficiency of interleukin- 1 antagonist · Lignocaine 5% viscous solution held in mouth


· Behcet’s Syndrome for 5 minutes before meals is soothing
· HIV Infection · Triggers like spicy foods, citrus fruits and dry
fruits must be avoided
· Malabsorption Syndrome
· Mechanical protection with a plastic guard
· Ulcerative Colitis
· Topical steroids like triamcinolone acetonide
· Crohn’s Disease
0.1% in orabase or hydrocortisone hemisuccinate
· Pernicious Anemia pellets four times a day. If no response, stronger
Q. What is histopathological findings of aphthous topical steroids or topical tacrolimus may be
ulcers? used
A. - Lymphocytic infiltration · Topical tetracycline mouthwash or orally 250 mg
- Occasional plasma cells and eosinophils which tablet, dissolved in 10 ml of water and gargled
suggest delayed hypersensitivity for 3 minutes. Topical minocycline can also be
used
Q. Investigations to be done to confirm diagnosis
· Rinses of hydrogen peroxide/water solution 1:4
A. Most of the times diagnosis is clinical
strength
- CBC
· Soothing applications of old tea bags to erosions
- Haemoglobin assay
· Supplementation of folate, vitamin B12 and iron
- Differential WBC count
· Oral corticosteroids for severe recurrent
- RBC indices aphthous ulcer; e.g. prednisolone 40 mg for 5
- Iron studies days and then tapered by 5mg every 2 days
- Red cell folate levels · Thalidomide 300mg /day, levamisol, dapsone,
- Serum B12 levels azathioprine, sucralfate, pentoxifylline,
colchicine 0.5 mg tds, thalidomide(50-300mg
- Serum anti-endomysial antibodies and and IgA
daily) has been used
anti-tissue transglutaminase antibodies
Q. What is the management and treatment of aphthous h
ulcers ?
A. · No permanent cure is available
· Good oral hygiene: use of hot saline
mouthwashes and 0.2% aqueous chlorhexidine
gluconate, 0.1% benzydamine mouthwash
DYP SURVIVAL GUIDE FOR POST GRADUATES 347

dryness can be present. Finger pulps become dry,


10. ASTEATOTIC ECZEMA cracked and parchment-like. Eczematous changes,
fissuring, excoriations can occur as secondary changes
Q. Synonyms?
Q. Histopathological features?
A. Winter eczema, senile eczema, eczema craquele
A. Mild hyperkeratosis, spongiosis, dermal infiltrate
Q. Meaning of the word “asteatotic”?
Q. How do you manage this condition?
A. Decrease in lipid content
A. - General management- Comfortable room
Q.. Define asteatotic eczema? temperature, Decreased usage of soaps and
A. Asteatotic eczema is defined as a condition occurring detergents, avoid contact with allergens.
in older age groups or in people with some - Medical management- Treat the underlying
underlying systemic disease or malignancy, condition, sedative antihistaminics, topical
characterized by dry scaly skin sometimes associated immunomodulators, sedatives, UVB therapy,
with fissuring and excoriations emulsifying ointment (emulsifying wax 20%,
Q. Factors implicated in etiology? liquid paraffin 30% and white soft paraffin 50%)
A. Age related decrease in skin lipid content, dry can be used as an emollient. In severe cases a mild
weather, malnutrition, drugs, exposure to allergens steroid like triamcinolone can be added to it.
& sensitizers, systemic diseases & malignancies DIAPER DERMATITIS
Q. Which systemic diseases are associated with
Q. Synonyms?
asteatotic eczema?
A. Napkin dermatitis, napkin rash
A. Hepatitis, cholestasis, uremia, hypothyroidism
Q. Definition?
Q. Which drugs are implicated in etiology?
A. Its a type of irritant contact dermatitis developing in
A. Diuretics, cimetidine, rarely topical corticosteroids
the diaper area in a newborn, due to various
(therapeutic paradox)
etiological factors
Q. Which internal malignancies are associated? Q. What are the etiological factors?
A. Malignant lymphoma, gastric adenocarcinoma, A. A) Friction- Friction between skin and fabric.
angio-immunoblastic lymphadenopathy, spheroidal
cell carcinoma of breast B) Water- Hydration of skin increases susceptibility
to frictional damage.
Q. Describe the clinical features?
C) Urine- Ammonia and urinary degradation
A. Dry, scaly skin. On the shin it gives “pavement stone” products cause irritation.
appearance called eczema craquele. Mostly extensor
D) Feces- Various proteases and lipases have an
surfaces are involved. Sometimes generalized
irritant effect.
346
348 DYP SURVIVAL GUIDE FOR POST GRADUATES DYP SURVIVAL GUIDE FOR POST GRADUATES 349

E) Microorganisms- Like candida albicans and - Seborrhoeic dermatitis


certain bacteria have a role. - Acrodermatitis enteropathica
F) Topical antibiotics- Broad spectrum antibiotics - Langerhans cell histiocytosis
enhance growth of candida.
Q. How do you manage this condition?
G) Chemical irritants- Like soaps, detergents,
antiseptics have irritant effects A. A) General management: Stop oil massage, stop
Q. What are the clinical features? using occlusive diapers, stop using irritant soaps
and antiseptics.
A. Lesions appear as areas of confluent erythema over
the napkin area i.e. buttocks, genitalia, lower B) Medical management: Topical antibiotics/
abdomen, pubic areas and upper thighs. Deep groins antifungals, topical steroids, emollients
and flexures are spared h
Q. Comment on incidence?
A. Most commonly in late infancy between 6-12 months
of age. Less common in India. More common in
affluent societies
Q. What is meant by ‘tide mark’ dermatitis?
A. When lesions appear only on the margins of the
napkin area
Q. What is Jacquet’s dermatitis?
A. Sometimes severe contact dermatitis occurs in the
form of vesicles, erosions & ulcers, in a pattern and
distribution, similar to diaper dermatitis
Q. Can there be any variations in the clinical
presentation?
A. Vesicles and pustules resembling herpes simplex,
purple brown nodules along with rash known as
infantile gluteal granuloma, sometimes
dissemination to trunk and extremities as an id
reaction to candida albicans
Q. What is the differential diagnosis?
A. - Napkin psoriasis
DYP SURVIVAL GUIDE FOR POST GRADUATES 351

B] Minor criteria-(3 of 23 present)


11. ATOPIC DERMATITIS
· Xerosis
Q. Why is this a case of atopic dermatitis? · Ichthyosis/palmar hyperlinearity/keratosis
A. - Presence of itchy papules which become pilaris
excoriated and lichenified · Immediate (type I) skin test reactivity
- Flexural distribution(in adults or older children) · Elevated serum IgE
or facial and extensor in infants.
· Early age of onset
Q. What does “Atopy” mean?
· Tendency toward cutaneous infections/impaired
A. Atopy is derived from the Greek word ‘atopos’,
cell-mediated immunity
meaning strange or unusual. The word atopy was
coined by Coca. · Tendency toward non-specific hand or foot
Q. What is the triad of atopy? dermatitis
A. Atopy is a triad of Atopic dermatitis, Allergic rhinitis · Nipple eczema
and Bronchial asthma. · Cheilitis
Q. What is Atopic Dermatitis? · Recurrent conjunctivitis
A. - Atopic dermatitis is an itchy, chronic, or · Dennie–Morgan infraorbital fold
chronically relapsing, inflammatory skin
condition characterized by itchy papules which · Keratoconus
become excoriated and lichenified · Anterior subcapsular cataract
- Typically, have a flexural distribution, frequently · Orbital darkening
associated with other atopic conditions in the
· Facial pallor/erythema
individual or other family members
Q. What are the diagnostic criterias for atopic · Pityriasis alba
dermatitis? · Anterior neck darkening
A. Hanifin & Rajka proposed the following criteria for · Pruritus when sweating
atopic dermatitis-
· Intolerance to wool and lipid solvents
A] Major criteria-(3 of 4 present)
· Perifollicular accentuation
• Pruritus
· Food intolerance
• Typical morphology and distribution of skin
lesions · Course influenced by environmental/emotional
• Chronic or chronically relapsing dermatitis factors
• Personal or family history of atopy · White dermographism/delayed blanching

350
352 DYP SURVIVAL GUIDE FOR POST GRADUATES DYP SURVIVAL GUIDE FOR POST GRADUATES 353

Q. What is refined Hanifin and Rajka’s diagnostic exposed surfaces, especially the extensor aspect of
criteria? the knees, are most involved. The lesions consist of
A. The UK refinement of Hanifin and Rajka’s diagnostic erythema and discrete or confluent oedematous
criteria for atopic dermatitis. Scabies should be papules which are intensely itchy, may become
excluded. exudative and crusted as a result of rubbing.
Secondary infection and lymphadenopathy are
In order to qualify as a case of atopic dermatitis with the UK
common.
diagnostic criteria, the child must have:
Q. What are the clinical features of Childhood atopic
• An itchy skin condition (or parental report of
dermatitis?
scratching or rubbing in a child)
A. Age 2 to 12 years, involves most characteristically the
Plus three or more of the following:
elbow and knee flexures, sides of the neck, wrists and
1] Onset below age 2 years (not used if child is ankles. The sides of the neck may show a striking
under 4 years) reticulate pigmentation (atopic dirty neck). The
2] History of skin crease involvement (including erythematous and oedematous papules tend to be
cheeks in children under 10 years) replaced by lichenification.
3] History of a generally dry skin Q. What are the clinical features of adult atopic
4] Personal history of other atopic disease (or dermatitis?
history of any atopic disease in a first degree A. In adults, AD is seen predominantly in a flexural
relative in children under 4 years) distribution, but extensive areas of skin may be
5] Visible flexural dermatitis (or dermatitis of involved. Patients may present with head and neck,
cheeks/forehead and outer limbs in children often with severe eyelid involvement OR may present
under 4 years) with a chronic hand dermatitis components. In any
stage of AD, the most severely affected individuals
Q. What are the three clinical stages of atopic
may evolve to generalized exfoliative erythroderma.
dermatitis?
Q. What is Senile atopic dermatitis?
A. 1] Infantile Phase
A. Occurs in age>60years, characterised by marked
2] Childhood Phase
xerosis.
3] Adult/Adolescent Phase
Q. Name the regional variants of atopic dermatitis?
Q. What are the clinical features of Infantile atopic
A. 1] Cheilitis sicca
dermatitis?
2] Lip-lickers eczema
A. Age- from birth to 2 years Lesions most frequently
start on the face. When the child begins to crawl, the 3] Ear eczema
354 DYP SURVIVAL GUIDE FOR POST GRADUATES DYP SURVIVAL GUIDE FOR POST GRADUATES 355

4] Eyelid eczema - Breast-feed the baby as long as possible.


5] Hand and neck dermatitis - Avoidance of triggering factors like certain specific
6] Juvenile plantar eczema foods (egg white, shell fish, red meat, cow’s milk,
peanuts), dust, woollen clothes, carpets.
7] Atopic hand eczema
Bathing: DO NOT USE HARSH SOAPS AND
8] Dyshidrotic eczema
DETERGENTS.
Q. What are Dennie–Morgan folds?
- DO NOT ADD ANY ANTISEPTIC TO THE
A. Symmetric, prominent fold (single or double) just BATH WATER
beneath the margin of the lower eyelid is noted in - IN ACUTE PHASE- USE NO SOAPS
many AD patients during infancy or childhood. The
fold originates in or near the inner canthus and - Mild soap with pH near skin (Syndet) is advised
extends to affect one-half to two-thirds of the lower - Apply oil before bathing
lid. - Bath with lukewarm water
Q. Who are Allergic Shiners? - Pat dry immediately
A. In patients of atopic dermatitis darkening under the - Use of moisturiser immediately
eyes can be seen. Such patients are allergic shiners. - Toys- Should not be furry
Q. What is lichenification? - Avoid pets.
A. Lichenification refers to thickening and accentuation B]Local measures:
of skin markings due to epidermal proliferation. It is
- Emollients
the late manifestation of atopic dermatitis because of
chronic rubbing and scratching. - Topical steroids
Q. What is Hertoghe’s sign? - Topical Tacrolimus/Pimecrolimus
A. Absence or thinning of lateral eyebrows. - Topical antibiotics
Q. What is White dermographism? - Phototherapy (UVA+UVB or NB-UVB)
A. Stroking of skin leads to a white streak that reflects C]Systemic measures:
excessive vasoconstriction, most apparent on - Systemic corticosteroids
forehead. - Oral cyclosporine (5mg/kg then tapered to 2mg/
Q. How will you treat atopic dermatitis? kg)
A. Treatment can be categorised into 2 broad groups- - Azathioprine (2-3.5mg/kg)
A] General Measures- - Mycophenolate mofetil (1-2.5mg/kg)
- Antihistaminics (Adjunctive therapy)
356 DYP SURVIVAL GUIDE FOR POST GRADUATES

Q. What are the complications associated with atopic


dermatitis?
12. BACTERIAL INFECTIONS
A. 1] Infections- Bacterial (impetiginisation) QUESTIONS
Viral infections (Eczema herpeticum, Molluscum
contagiosum) Q. Classification on basis of depth of infection?
2] Ocular complication-Atopic keratoconjunctivitis, A. · Superficial infection in epidermis: Bullous and
Vernal keratoconjunctivitis, Subcapsular cataract, non- bullous impetigo, folliculitis, impetigo of
Keratoconus. Bockhart
Q. What are the various scores to assess the severity of · Infection at dermo- epidermal level: ecthyma
atopic dermatitis? · Infection in dermis: erysipelas, furuncle,
A. 1] EASI(Eczema Area Scoring Index) carbuncle
2] SCORAD (SCORing Atopic Dermatitis) · Deeper infection: cellulitis, abscess, necrotizing
3] POEM(Patient Oriented Eczema Measure). fasciitis, pyomyositis
Q. What is the normal bacterial flora of the skin?
h
A. Bacteria that are found regularly on skin form the
normal flora. The normal flora has been classified in
the following groups.
1. Resident flora: the bacteria that grow on the skin
and relatively stable in number and composition
at particular sites. Increase in their numbers
results from multiplication of those already
present and not due to addition from outside
2. Transient flora: the organisms that lie free on skin
without attachment and are derived from
exogenous sources. They are unable to multiply
on skin. They disappear from skin within short
time. e.g. staphylococci, gram-negative bacilli
3. Aerobic bacteria: form microcolonies within the
outer layers of the stratum corneum, whereas
anaerobes are found in the deeper portions of
hair follicle. The normal flora of moderately moist
areas consists mainly of staphylococci. Lipid-rich
357
358 DYP SURVIVAL GUIDE FOR POST GRADUATES DYP SURVIVAL GUIDE FOR POST GRADUATES 359

areas contain mainly propionibacterium acnes. Very GRAM-NEGATIVE BACILLI


wet areas (axilla, toe webs) contain mainly · They die quickly on desiccation and are not part
lipophilic bacteria and other coryneforms.
of resident flora
Certain areas like face may be contaminated by
organisms from the nostrils and mouth (Staph. · The perianal region and perineum get
aureus, beta hemolytic streptococci), but these are not contaminated from the gastrointestinal tract and
part of normal facial flora. they are transient flora.
STAPHYLOCOCCI AND MICROCOCCI STREPTOCOCCI
· Gram positive , catalase-positive cocci · They have been classified by Lancefield into 18
· They are divided into coagulase-positive staph. groups. Of these, group A, group B and C have
aureus and coagulase negative dermatological significance.
· Coagulase-positive are pathogenic and transit Q. What are the degerming properties of skin?
residents on skin. They are found in the anterior
A. Dessication
nares and perineum
· Hydration of the skin helps in bacterial growth
· Micrococci are much less frequently present than
staphylococci. They are differentiated from the on the skin surface
latter by their inability to produce acid · A person has fewer bacteria on exposed parts
anerobically from glycerol in the presence of such as the hands and forearms than on the axilla
erythromycin Resident Flora
CORYNEFORM BACTERIA
· These are the first line of defense against
· Recently these bacteria have been classified on colonization and subsequent infection with
the basis of presence and type of amino acid, pathogenic organisms.
mycolic acid and neutral sugars into following
groups: · The phenomenon of Bacterial Interference is that
the colonization of a site by one strain of bacterial
a. Corynebacterium species
interference that the subsequent colonization by
b. Brevibacterium species: These are usually large another strain. A possible explanation is that
colony coryneforms present in abundance in
these bacteria may be competing for the same
hyperhidrotics and in moist areas of skin.
nutrients, although it may also be due to
They are implicated in the production of
odor in feet and resistant to penicillin. production of antibiotics by resident skin bacteria
which are capable of inhibiting other flora.
c. Proprionbacterium species: These are anerobes.
The commonest organism is P.acnes, which · Bacterial Interference has been used therapeutically
is abundant on scalp, forehead and back. in patients with recurrent furunculosis.
360 DYP SURVIVAL GUIDE FOR POST GRADUATES DYP SURVIVAL GUIDE FOR POST GRADUATES 361

Bacterial Adherence and ostia of the hair follicle, creating a follicular-


· The potential of an organism to colonize skin is based pustule.
related to its ability to adhere to the surface • The inflammation can be either limited to the
which varies in different individuals and at superficial aspect of the follicle with primary
various skin sites. involvement of the infundibulum or the
inflammation can affect both the superficial and
· Adhesins of pathogenic staphylococci and
deep aspect of the follicle.
streptococci are teichoic acids. If the skin or nasal
mucous membrane is treated with teichoic acid, Q. Clinical features of folliculitis?
it blocks the epithelial receptors thus inhibiting A. • Bacterial folliculitis frequently involves the face,
colonization by Staph. Aureus. chest, back, axillae or buttock .
Lipids • The appearance depends on the depth of
· Sebum has a mild inhibitory effect on normal follicular involvement.
skin micro-organisms in vitro. • Lesions of superficial folliculitis (Bockhart’s
impetigo) are small, 1–4 mm pustules or crusted
Desquamation
papules on an erythematous base.
· Surface bacteria that are unable to multiply (i.e. • The lesions are frequently clustered and heal
transients) are lost with the exfoliation of without scarring.
corneocytes.
• Sycosis, a deep folliculitis, appears as large,
Immune System tender, erythematous papules, often with a
· In normal skin, IgG and IgA antibodies are central pustule. The lesions may be pruritic and
secreted through eccrine sweat. slightly tender.
· Secretory IgA in sweat may explain the absence Q. D/D of folliculitis?
of bacterial colonization in eccrine ducts. A. - other forms of folliculitis, fungal or viral
Acid Mantle (molluscum can rarely cause folliculitis)
· Higher colonization by bacteria in patients with - acne vulgaris
hemiplegia and diabetes was associated with - rosacea
high skin pH and this may contribute to - chloracne
endogenous infection in the ward. When the skin
- pseudofolliculitis barbae
was cleaned with acidic mineral water, the skin
surface pH was reduced and the frequency of - keratosis pilaris
bacterial infection was markedly decreased. Q. Other name for impetigo?
Q. What is folliculitis? A. Synonyms:
A. • Folliculitis is defined histologically as the · Non-bullous impetigo
presence of inflammatory cells within the wall · Impetigo contagiosa
362 DYP SURVIVAL GUIDE FOR POST GRADUATES DYP SURVIVAL GUIDE FOR POST GRADUATES 363

· Bullous impetigo Presents as small red papules similar to insect


· Localized staphylococcal scalded skinsyndrome bites. These lesions rapidly evolve to small
· Pemphigus neonatorum blisters and then to pustules that finally scab
over with a characteristic honey-colored crust.
· Staphylococcal impetigo These lesions often start around the nose and on
Q. What is impetigo? the face, but less frequently they may also affect
A. Impetigo(latin word ,means’ to attack’) is a common, the arms and legs. At times, there may be swollen
highly contagious, superficial skin infection that but non-tender lymph nodes (glands) nearby. It
primarily affects children. The condition presents in has no constitutional symptoms.
both non- bullous and bullous forms. The primary · Bullous impetigo: This form of impetigo is
pathogens in non-bullous and bullous impetigo are
caused only by staph bacteria. These bacteria
Staphylococcus aureus and, less commonly, group A
produce a toxin that reduces cell-to-cell adhesion
β-hemolytic Streptococcus (Streptococcus pyogenes).
causing separation between epidermis and
Q. D/D of impetigo? dermis. This leads to the formation of a blister.
A. - Insect bites Bullae can appear in various skin areas,
- Eczematous dermatoses especially the buttocks and trunk. These blisters
- Infections-Tinea corporis are fragile and contain a clear yellow-colored
fluid. The bullae are delicate and often break and
- Candidiasis
leave red, raw skin with a ragged edge. The
- Herpes simplex viral infection lesions may spread peripherally and produce
- Varicella annular lesion (impetigo circinata). A dark crust
- Infestations -Scabies will commonly develop during the final stages
- Pediculosis of development. With healing, this crust will
resolve.
- Vesiculo-bullous disorders-Pemphigus foliaceus
Q. Will impetigo leave scars?
- Bullous pemphigoid
- Thermal burn A. Because the crusts and blisters of impetigo are
superficial, impetigo generally does not leave scars.
- Drug reactions-Bullous erythema multiforme
Affected skin looks red for a while after the crusts go
- Stevens–Johnson syndrome away, but this redness fades in a matter of days to
Q. What are the types of impetigo? weeks.
A. There are two forms of impetigo: Q. What is the treatment of impetigo?
· Non-bullous impetigo: This is the more common A. The treatment is divided into topical and systemic.
form, caused by both staph and strep bacteria. Topical:First line – mupirocin, fusidic acid.
364 DYP SURVIVAL GUIDE FOR POST GRADUATES DYP SURVIVAL GUIDE FOR POST GRADUATES 365

Systemic : Q. What are the risk factors for furuncle?


First line : A. Risk factors for furunculosis include bacterial
- Dicloxacillin 250-500 mg qid for 5-7 days. carriage in the nostrils, diabetes mellitus, obesity,
- Amoxicillin plus clavulanic acid lymphoproliferative neoplasms, malnutrition, and
use of immunosuppressive drugs.
- Cephalexin 25 mg/kg tid, 250 - 500 mg qid
Q. Differential Diagnosis:
Second line :
A. - Epidermal and pilar cysts (infected)
- Azithromycin 500 mgfor 1 day then 250 mg daily
for 4 days. - Furuncular myiasis (due to maggots)
- Clindamycin 15 mg/kg/day tid Q. What is carbuncle?
- Erythromycin 250-500 mg q.i.d. for 5-7 days A. Carbuncle in Latin means “small piece of coal”. It is
an infection of multiple adjacent hair follicles and
Q. What is a furuncle?
intervening tissue. It is a necrotizing swelling with
A. Furuncle, is a deep folliculitis (furunculosis means high local temperature and pain. It is a tender firm to
petty thief). It is most commonly caused by infection hard lump. Sites of involvement are neck, shoulder,
by the bacterium Staphylococcus aureus, resulting in a hips, thighs. It has multiple sinuses in sieve pattern
painful swollen area on the skin caused by an
discharging pus. It is associated with diabetes
accumulation of pus and dead tissue. Individual
mellitus, prolonged steroid therapy, immune-
boils clustered together are called carbuncles. Most
compromised status and exfoliative dermatitis
human infections are caused by coagulase-positive S.
aureus strains, notable for the bacteria’s ability to Q. What is the management of recurrent furunculosis?
produce coagulase, an enzyme that can clot blood. A. · Careful evaluation of the underlying causes.
Almost any organ system can be infected by S. aureus. · Treatment of predisposing factors: poor hygiene,
Q. What are the signs of furuncle? obesity, hyperhidrosis, ingrown hair, tight
A. They are pustules around a hair follicle that clothing.
are tender, warm, and very painful. They range from · General care of skin: this is to reduce the number
pea-sized to golf ball-sized. In a severe infection, an of staphylococcus aureus on the skin. General
individual may experience fever, swollen lymph care of both hands and body with water and soap
nodes and fatigue. A recurring lesion is called chronic is important. The patient should avoid tauma to
furunculosis. Skin infections tend to be recurrent in the skin as well as the potential skin irritant such
many patients and often spread to other family as strong soaps and deodorant.
members. Systemic factors that lower resistance
commonly are detectable, including: diabetes, · Care of the dressing: Dressing should be changed
obesity, and hematologic disorders. frequently if purulent drainage collects.
366 DYP SURVIVAL GUIDE FOR POST GRADUATES DYP SURVIVAL GUIDE FOR POST GRADUATES 367

· Measures to eliminate nasal and skin carriage of of 2 cm. Cultures of axilla, perineum and skin
staphylococcus aureus: surface are made by scrubbing of these areas with
· Local use of ointment in nasal vestibule dry cotton.
reduces the nasal carriage of staphylococcus Q. What is ecthyma?
aureus and secondarily the shedding of the A. Ecthyma is an ulcerative pyoderma of the skin caused
organism on skin. Intranasal application of by bacteria such as Pseudomonas, Streptococcus
2%mupirocin contribute to reduce the pyogenes, and Staphylococcus aureus. Because
recurrent furunculosis. Prophylaxis with ecthyma extends into the dermis, it is often referred
fusidic acid ointment in nares twice a day to as a deeper form of impetigo.
every fourth week for patients and the family
members have shown success. Q. What is epidemiology of ecthyma?
· Oral antibiotics:Rifampin 600 mg orally A. Ecthyma has a predilection for children, elderly and
daily for 10 days. Other drugs used are diabetic individuals. Outbreaks have also been
reported in young military trainees. Ecthyma usually
dicloxacillin for methicillin susceptible
S.aureus, trimethoprim-sulfamethaxole, arises on the lower extremities.
ciprofloxacin, or minocycline for methicillin Q. What is the etiology ?
resistant S.aureus. A. - Ecthyma can be seen in areas of previously
Q. What is the interference therapy for treatment of sustained tissue injury (e.g., excoriations, insect
recurrent furunculosis? bites, dermatitis), in patients who are
immunocompromised (e.g., diabetes,
A. - Bacterial interference therapy employs use of
502A strain of staph aureus in treatment of neutropenia, HIV infection). Important factors
recurrent furunculosis. By colonization of nose, contributing to the development of streptococcal
it is hoped to interrupt the transfer the pathogenic pyodermas or ecthyma: High temperature and
staph from nasal mucosa to skin. Nasal humidity, Crowded living conditions, Poor
colonization with 502A is achieved only in hygiene
presence of the original resident staph aureus. - Untreated impetigo that progresses to ecthyma
- The 502 A strain is transferred from a stock most frequently occurs in patients with poor
trypticase soy agar slant to trypticase soy broth hygiene.
and incubated at 37 degree Celsius for eighteen - Some strains of Streptococcus pyogenes have a high
hours. The dry sterile cotton swab dipped in affinity for both pharyngeal mucosa and skin.
diluted culture before implantation. A separate Pharyngeal colonization of S. pyogenes has been
swab is used for each implantation site. A 502A documented in patients with ecthyma.
implantation is performed every second or third Q. What are the clinical features of ecthyma?
day for 4 applications. Subsequently A. - Ecthyma begins as a vesicle or pustule overlying
implantation is followed at weekly intervals. an inflamed area of skin that deepens into a
- Dry sterile swab rotated in each nostrils to depth dermal ulceration with overlying crust
368 DYP SURVIVAL GUIDE FOR POST GRADUATES DYP SURVIVAL GUIDE FOR POST GRADUATES 369

- The crust of ecthyma lesions is gray-yellow and Q. What is acne keloidalis nuchae?
is thicker and harder than the crust of impetigo A. - It is common on the nape of neck in males with
- A shallow, saucer-like ulceration is apparent curly hair or who shave hair often and close to
when adherent crust is removed scalp. Initially, itchy round small bumps appear
- The dermal ulcer has a raised and indurated associated with folliculitis. There is an acute
surrounding margin perifollicular inflammation with weakening of
- Ecthyma lesions can remain fixed in size follicular wall at lower infundibulum. Naked hair
(sometimes resolving without treatment) or can shaft is released in dermis acting as a foreign
progressively enlarge to 0.5-3 cm in diameter body, leading to acute to chronic granulomatous
inflammation. Then fibroblasts deposits new
- Ecthyma heals slowly and commonly produces
a scar. Regional lymphadenopathy is common, collagen and fibrosis develops forming
even with solitary lesions hypertrophic scar.
Q. What are the D/D? - It presents as a firm, dome shaped, follicular
papule on occipital area with papules and
A. · Ecthyma Gangrenosum pustules. These coalesce to form large plaque,
· Insect Bites ultimately forming keloid in band-like distribution.
· Leishmaniasis The lesions heal with scarring alopecia.
· Lymphomatoid Papulosis Q. Describe erysipelas?
· Mycobacterium Marinum Infection of the Skin A. Erysipelas: also known as St. Anthony fire/Ignis sacer.
· Papulonecrotic Tuberculids It is caused by beta hemolytic group A streptococcus.
· Pyoderma Gangrenosum It is acute infection of superficial dermal lymphatics.
Local redness, swelling with raised border. Margins
· Sporotrichosis are sharply demarcated. Prodromal symptoms of
· Tungiasis malaise, chills with fever, headache and vomiting are
Q. What is botryomycosis? common. The spectrum of disease ranges from
A. Greek word-”Botyrs” means bunch of grapes. It is transient hyperemia with slight desquamation to
caused by staphylococcus aureus. Two types of skin intense inflammation with vesicles and bullae.
lesions are seen- cutaneous and subcutaneous Common over face and legs. Predisposing factors are
nodules. There may be suppurative plaques, ulcers, operative wounds, fissure, obesity, abrasion, venous
abscess and sinus. Predisposing factors are trauma, insufficiency, leg ulcers. Complications are septicemia,
alcoholism and diabetes mellitus. On histopathology cellulitis, necrotizing fascitis.
granular bodies (club) are seen. It has basophilic Q. How to treat erysipelas?
centre composed of cells, debris and bacteria. A. Treatment of Erysipelas
Eosinophilic rim of immune response is seen. This
phenomenon is called Splendore Hoeppli’s Erysipelas is treated with antibiotics including
phenomenon. penicillin, dicloxacillin, cephalosporins, clindamycin,
370 DYP SURVIVAL GUIDE FOR POST GRADUATES DYP SURVIVAL GUIDE FOR POST GRADUATES 371

and erythromycin. Most cases of erysipelas can be cancer patients, transplant recipients, severe
treated with oral antibiotics. However, cases of asthmatics, etc.)
sepsis, or infections that do not improve with oral • Diabetics
antibiotics require IV antibiotics administered in the
hospital. • Intravenous drug users
Q. What is cellulitis? • Users of quinolone antibiotics
A. It is a suppurative inflammation of subcutaneous • Young children
tissue by streptococcus pyogenes and rarely • Elderly
staphylococcus aureus. It presents as a local
• College students living in dormitories
erythema, tenderness, ill-defined borders, associated
with malaise, chilly sensation. It can have vesicles or • People staying or working in a health care facility
nodules that discharge pus with necrotic material. for an extended period of time
Complications are gangrene, abscess and sepsis. • People who spend time in coastal waters where
Associated with diabetes. MRSA is present.
Q. What is MRSA? • People who spend time in confined spaces with
A. - Meth ici lli n- resist an t Staphylococcus other people, including occupants of homeless
aureus (MRSA) is a bacterium responsible for shelters and warming centers, prison inmates,
several difficult-to-treat infections in humans. It is military recruits in basic training, and
also called multidrug-resistant Staphylococcus individuals who spend considerable time
aureus and oxacillin-resistant Staphylococcus in changing rooms or gyms.
aureus (ORSA). MRSA is any strain of Staphylococcus
aureus that has developed resistance to beta-lactam • Veterinarians, Livestock handlers, and Pet
antibiotics, which include the penicillins owners.
(methicillin, dicloxacillin, nafcillin, oxacillin, etc.) Q. How is the diagnosis of MRSA made?
and the cephalosporins. Strains unable to resist these A. - The bacterium generally must be cultured via
antibiotics are classified as methicillin- blood, urine, sputum, or other body fluid
sensitive Staphylococcus aureus, or MSSA. cultures, and cultured in the lab in sufficient
- MRSA is especially troublesome in hospitals, quantities to perform these confirmatory tests
prisons, schools, and nursing homes, where first. These techniques include Real-time
patients with open wounds, invasive devices, and PCR and Quantitative PCR and are increasingly
weakened immune systems are at greater risk of being employed in clinical laboratories for the
infection than the general public. rapid detection and identification of MRSA
Q. What are the risk factors for acquiring MRSA? strains.
A. • People with weak immune systems (people - Another common laboratory test is a rapid latex
living with HIV/AIDS, people living with lupus, agglutination test that detects the PBP2a protein.
372 DYP SURVIVAL GUIDE FOR POST GRADUATES DYP SURVIVAL GUIDE FOR POST GRADUATES 373

- PBP2a is a variant penicillin-binding protein that · There have been claims that bacteriophage can
imparts the ability of S. aureus to be resistant to be used to cure MRSA.
oxacillin. · Cannabinoids, chemicals found in the Cannabis
Q. Treatment of MRSA? plant, are also suspected of being highly effective
A. · MRSA are resistant to traditional anti- at killing MRSA.
staphylococcal beta-lactam antibiotics, such · The psychedelic mushroom Psilocybe semilanceata
as cephalexin. has been shown to strongly inhibit the growth
· MRSA has a greater spectrum of antimicrobial of Staphylococcus aureus.
susceptibility, including to sulfa drugs (like co- · Hydrogen peroxide, tobramycin, chlorhexidine
trimoxazole/trimethoprim-sulfamethoxazole), digluconate, chlorhexidine gluconate,
tetracyclines (like doxycycline and minocycline) levofloxacin, and silver sulphadiazine effective
and clindamycin, but the drug of choice for against MRSA topically.
treating CA-MRSA is now believed to Q. What are the infections caused by corynebacterium ?
be vancomycin. A. Erythrasma, Pitted keratolysis, Trichomycosis
· Linezolid is now felt to be the best drug for axillaris
treating patients with MRSA pneumonia. Q. What is Trichomycosis axillaris?
· Teicoplanin is a structural congener of A. It is caused by Corynebacterium tenuis. It is a
vancomycin that has a similar activity spectrum superficial bacterial colonization of hair shaft in
but a longer half-life. Because the oral absorption sweaty hair bearing areas in armpits, pubic areas. It
of vancomycin and teicoplanin is very low, these presents as yellow, orange, black or red granular
agents must be administered intravenously to concretions coating hair shaft with malodour. Simple
control systemic infections. clipping of hair effective.
· Several newly discovered strains of MRSA Q. Describe pitted keratolysis?
show antibiotic resistance even to vancomycin A. It is an acquired, chronic, asymptomatic, superficial
and teicoplanin. These new evolutions of the bacterial infection of the foot and occasionally of
MRSA bacterium have been dubbed Vancomycin palms, characterized by discrete superficial
intermediate-resistant Staphylococcus aureus crateriform pits and erosions with surrounding
(VISA). maceration. The lesions are shallow, circular with
· Linezolid, quinupristin/dalfopristin, punched out appearance and coalesce to produce 0.5
daptomycin, ceftaroline, and tigecycline are to 7.0 mm in diameter pits and associated with
used to treat more severe infections that do not hyperhidrosis and malodour. The patient may give
respond to glycopeptides such as vancomycin. a history of frequent immersion of hands and feet in
water.
374 DYP SURVIVAL GUIDE FOR POST GRADUATES DYP SURVIVAL GUIDE FOR POST GRADUATES 375

Q. What are the differential diagnosis? Q. How can one prevent pitted keratolysis?
A. • Intertrigo • Tinea pedis A. Avoid using occlusive footwear, using absorbent
• Hand eczema • Keratolysis exfoliativa cotton socks, avoid sharing footwear, cleaning the
Q. Causes of pits on palms and soles. foot with antibacterial at the end of the day to remove
A. • Pitted keratolysis dirt and debris from settling down.
• Plantar warts Q. How can we treat pitted keratolysis?
• Punctate keratoderma A. • Topical antibiotics such as erythromycin 1% gel
• Pits of basal cell nevus syndrome twice daily, fusidic acid, mupirocin, etc.
• Keratosis punctata Treatment of hyperhidrosis :
• Punctate porokeratosis • Benzoic and salicylic acid ointment, castellani’s
• Darier’s Disease paint, clotrimazole cream, 2% miconazole
• Acropigmentation of Kitamura nitrate cream, Whitfield’s ointment .
• Arsenic poisoning • Orally, macrolide antibiotics can be used.
Q. Name the organisms causing pitted keratolysis? Q. How does erythrasma present?
A. Micrococcus sedentarius (Kytococcus sedentarius), A. Dark discoloration of the skin folds which are moist
Dermatophilus congolensis and Corynebacterium and occluded. The duration is from months to years,
minutissimum. mostly asymptomatic. Areas of involvement- axillae,
Q. What is the other name of pitted keratolysis genitocrural crease, web spaces in toes,
A. Keratoma plantare sulcatum, Ringed keratolysis inframammary area.
Q. How does the infection occur? Q. Differential diagnosis of erythrasma?
A. Under prolonged hyperhidrosis and occlusion and A. · Acanthosis Nigricans
contact with wet surface, the bacteria proliferate and
release proteinases that destroy the stratum corneum. · Pityriasis versicolor
The malodour is caused by production of sulphur · Hailey-Hailey disease
compounds such as thiols and thioesters.
· Candidiasis
Q. What do we see on histopathology of pitted
keratolysis? · Contact Dermatitis
A. Presence of crater limited to stratum corneum, · Intertrigo
filaments and coccoid organisms can be seen at the · Flexural psoriasis
base and margin, special stains such as Gram stain,
PAS stain and methenamine silver stain can be used. · Seborrheic Dermatitis
Q. How will you confirm the diagnosis ? · Tinea Corporis
A. Wood’s lamp is negative. Scrapping of the lesions- · Tinea Cruris
gram staining, culture on brain heart infusion agar. · Tinea Pedis
376 DYP SURVIVAL GUIDE FOR POST GRADUATES DYP SURVIVAL GUIDE FOR POST GRADUATES 377

Q. Name the organism causing erythrasma? Q. Which bacterial infections are not pyoderma?
A. Corynebacterium minutissimum A. Rhinoscleroma, bacillary angiomatosis, erythrasma,
Q. How does corynebacterium cause erythrasma? trichomycosis axillaris
A. The organism proliferates under heat and humidity. Q. Which pyodermas are not bacterial infections?
They are present between the intercellular spaces and
A. Pyoderma gangrenosum, pyoderma fasciale,
dissolve keratin fibrils. The coral red fluorescence,
pyoderma vegetans
seen under wood’s light, is created by the porphyrins
produced by the diphtheroid and is diagnostic. Q. Difference between primary and secondary
Q. Name dermatological and systemic associations in pyoderma?
erythrasma? A. Primary: pre-existing skin is normal; caused by single
A. Patients with erythrasma should be screened for organism; single antibody present
pitted keratolysis and trichomycosis axillaris Secondary: pre-existing skin disease present; caused
(corynebacterial triad). If extensive involvement of by multiple organisms; multiple antibody present
the body is noted, then diabetes mellitus should be
excluded. Obesity and immune-suppressed states Q. What is DCAP?
are predisposing factors for occurrence of erythrasma. A. Dermatosis chronica atrophicans et pustulosa. It is
Q. Diagnosis of erythrasma on Wood’s lamp? chronic folliculitis of legs
A. Wood’s lamp examination of erythrasma lesions Q. What is hypopyon sign?
reveals coral-red fluorescence of lesions. Results may A. When a patient is made to sit or stand, semicircular
be negative if the patient has had a bath prior to
meniscus of pus is formed in the lower part of bullae
presentation. The cause of this colour fluorescence
has been attributed to excess coproporphyrin III Q. What is follicular triad?
synthesis by these organisms, which accumulates in A. Hidradenitis suppurativa, nodulocystic acne (acne
cutaneous tissue and emits a coral-red fluorescence conglobata), dissecting cellulitis of scalp
when exposed to a Wood’s lamp.
Q. What is follicular tetrad?
Q. Name the antibiotics used in treatment of
erythrasma? A. All of the above plus pilonidal sinus
A. • Oral-Erythromycin, clarithromycin Q. Name the infections caused by pseudomonas?
• Topical – fusidic acid , miconazole , clindamycin A. Ecthyma gangrenosum, green nail syndrome, toe-
are effective . web infection, blastomycosis-like pyoderma, hot-foot
Q. Prognosis? syndrome, pseudomonas folliculitis (hot-tub
folliculitis or swimming pool rash, malignant otitis
A. Often recurrent if underlying factors like obesity are
not controlled externa.
378 DYP SURVIVAL GUIDE FOR POST GRADUATES DYP SURVIVAL GUIDE FOR POST GRADUATES 379

Q. Write about green nail syndrome? Q. What is hot foot syndrome?


A. It is onycholysis of distal portion of nail with greenish A. Painful, erythematous plantar nodules or pustules
discoloration. It is caused by excessive contact of on foot occurring after swimming in a pool
hands with water, use of detergents, soap, nail containing P.aeruginosa. Systemic symptoms seen
trauma and other causes of onycholysis. There is only in those who are severly infected. It is self-
limiting and requires only symptomatic treatment.
increased growth of pseudomonas aeruginosa which
produces a blue-green pigment called pyocyanin. Q. What is rhinoscleroma?
Diagnosis can be confirmed by Gram stain and A. It is a chronic inflammatory granulomatous
culture. Differential diagnosis includes sub-ungual inflammation of upper respiratory tract, characterized
hematoma, melanocytic nevus, melanoma and by sclerosis and deformity. It is caused by Klebsiella
aspergillus infection. Treatment is avoidance of pneumonia rhinoscleromatis.
predisposing factors and treatment with There are three stages:
fluroquinolones and tobramycin orally and 1%acetic 1) nasal catarrh, 2) infliltrative stage, 3) sclerotic
acid or Neosporin topically. stage
Q. What is hot tub folliculitis or swimming pool rash? Q. What is the treatment of rhinoscleroma?
A. It is follicular maculopapular or vesicular or A. · Surgery combined with antibiotic therapy is
beneficial in patients with granulomatous
pustular lesion occurring 1-4 days after bathing in
disease and nasal or pharyngeal obstruction or
hot tub or public swimming pool. The organism nasal sinus involvement due to the proliferation
gains entry through hair follicle or through break in of lesions.
skin. It resolves in 1-2 weeks. As the water
· Surgery:
temperature increases the chlorine content of water
decreases and the number of bacteria increases. - Tracheostomy in patients with laryngeal
obstruction of the second degree
Diver’s suit colonizes the bacteria, causing folliculitis
(granulomatous stage) and above (sclerotic
on sides of trunk, axillae, buttocks, proximal stage).
extremities. Face and neck are usually spared.
- Plastic surgery in patients with cicatricial
Associated symptoms are painful eyes, earaches, sore
stenosis or when imperforation remains in
throat, headache, fever, painful swollen breasts, the nasal cavity, pharynx, larynx, or trachea.
malaise, rhinorrhoea, nausea, vomiting and
- Surgery and laser therapy are required to
abdominal pain. Differential diagnosis include Staph
treat airway compromise and tissue
aureus folliculitis, insect bite, papular urticaria, deformity.
Majocchi’s granuloma, eosinophilic folliculitis,
- Palatal symptoms may be relieved by means
miliaria and acne vulgaris.
of uvulo-palato-pharyngoplasty.
380 DYP SURVIVAL GUIDE FOR POST GRADUATES DYP SURVIVAL GUIDE FOR POST GRADUATES 381

· Antibiotics : Q. D/D of infundibular folliculitis?


- Tetracycline is the drug of choice. A. • Keratosis pilaris
- Other antibiotics include ciprofloxacin and • Lichen nitidus
rifampicin, clindamycin and third-generation • Milia
cephalosporins.
• Pityriasis rosea
- Sclerotic lesions respond well to treatment • Pityrosporum folliculitis
with ciprofloxacin. Ciprofloxacin has the
following advantages: Its oral administration • Folliculitis
is convenient, it achieves good tissue • Hypervitaminosis A
penetration, it is concentrated in Q. Treatment of infundibular follicilitis?
macrophages, and it may prove useful in the A. Cooling, soothing agents like calamine and
treatment of patients with rhinoscleroma emollients. Topical steroids, PUVA and Isotretinoin
· Steroids : may be used in resistant cases.
- These agents have anti-inflammatory Q. What are the topical antibacterials used in
properties and cause profound and varied dermatology?
metabolic effects. Corticosteroids modify the A. · Aminoglycosides: Genatmycin 1%cream,
body’s immune response to diverse stimuli. Neomycin B and 3% neomycin C, Framycetin 1%
- May decrease inflammation by reversing cream, Sisomycin 0.1% cream
increased capillary permeability and · Macrolides: Erythromycin 1%, 2 % gel, cream,
suppressing PMN activity. lotion; Clarithromycin 1% cream
Q. What is infundibular folliculitis? · Lincosamides: Clindamycin 1%gel, lotion,
A. Called as disseminate and recurrent infundibular solution; Lincomycin 2% gel
folliculitis described in 1968. · Fusidic acid 2% cream
Clinical presentation is like miliaria or keratosis · Mupirocin 2% cream
pilaris. Generalised flesh-colored papules on chest, · Fluoroquinolones: Nadifloxacin 1% cream
upperarm, back and the lesions are monomorphous.
In a given area, all follicles are involved, without · Sulfonamides: silver sulfadiazine 1% cream
skipping of any follicle. · Metronidazole: 0.75% gel, 1% cream
Q. What are the causes ? · Benzoyl peroxide : 2.5%, 4%, 5%, 10% gel or lotion
A. Hot, humid, tropical environment. Also common in · Azelaic acid: 20 % cream
atopics. · Polymyxin B
382 DYP SURVIVAL GUIDE FOR POST GRADUATES DYP SURVIVAL GUIDE FOR POST GRADUATES 383

LUPUS MILIARIS DISSEMINATUS FACIEI (LMDF) Q. Treatment of LMDF?


A. Reported therapy include:
Q. Describe LMDF?
· Oral prednisolone
A. Uncommon, chronic, inflammatory, smooth surfaced,
skin-coloured papules, 1-3 mm in size, distributed · Isotretinoin
symmetrically across the face, nose, eyelids and upper · Dapsone
lip. Occasionally the lesions can appear on the trunk
· Tetracyclines
and extremities also. Most commonly seen in young
adults. · Antimalarials.
Q. Differential diagnosis of LMDF? · Scar revision procedures (laser resurfacing,
A. · Granulomatous rosacea dermabrasion, chemical peel) may benefit
patients after the disease has run its course.
· Papular sarcoidosis Treatment with the 1450-nm diode laser has been
· Papular acne reported to be effective.
· Acneiform drug eruption Q. What is FIGURE?
Q. What is the etiology? A. Facial Idiopathic Granuloma With Regressive
A. Exact etiology is unclear but many possible factors Evolution
have been postulated to play a role. Some suggest a h
role of Demodex folliculorum, while others suggest
it to be a granulomatous reaction to hair follicle
destruction or ruptured epidermal cyst.
Q. Does scarring occur in LMDF?
A. Yes, the lesions regress over 12-18 months followed
by scarring.
Q. Histopathology of LMDF?
A. Early lesions show superficial perivascular and peri-
appendiceal lymphocytic infiltrates with a few
histiocytes and neutrophils. Fully developed lesions
show round granulomas, often with caseation
necrosis. The changes mimic miliary tuberculosis.
(One of the few conditions showing caseation necrosis
in skin is LMDF. Other tuberculosis lesions do not
show it)
DYP SURVIVAL GUIDE FOR POST GRADUATES 385

Q. What is the age and sex predisposition


13. BECKER’S NEVUS
A. - Adolescence
Q. What is the morphology of Becker’s Nevus? - Male predominance[5:1] (due to androgen
receptors being present on the surface)
A. Irregular, hyperpigmented, smooth macule on
Q. What are the other co-existing conditions?
shoulder, anterior chest or scapular region with
hypertrichosis. A. - Connective tissue nevi
- Smooth muscle hamartomas
Q. Differential diagnosis of becker’s nevus.
- Structural abnormalities of bone and soft tissues
A. - Café - Au - Lait Macule (CALM)
Q. What is the pathogenesis?
- Congenital melanocytic nevus
A. - Unclear
- Plexiform neurofibroma - Believed to be an organoid hamartoma of
- Congenital smooth muscle hamartoma ectodermally and mesodermally-derived tissues
Q. Synonyms of Becker Nevus - An increase in androgen receptors and probable
heightened sensitivity to androgens have been
A. - Becker’s melanosis
postulated
- Pigmented hairy epidermal Nevus Q. Histopathological findings:
Q. What is Becker’s Nevus? A. - Papillomatosis, acanthosis, hyperkeratosis
A. - It is a benign condition - Increased melanin content of melanocytes
- Organoid epithelial Nevus - Melanophages in papillary dermis
Q. What are the clinical features? - Increased number of melanosomes

A. - It presents as a macule, grows in size to reach - Inreased number of smooth muscles (hamartoma)
the size of a palm while acquiring thick dark hair Q. Treatment:
A. - Not required
- The surface becomes thick and corrugated
- Electocautery
- Firmness to palpation may point to an associated
- Waxing for hair
smooth muscle hamartoma
- Camouflage, make-up
- Color fades with time
- Lasers: NdYag and Q-switched have been tried
- It is a benign lesion for cosmetic benefits
384
386 DYP SURVIVAL GUIDE FOR POST GRADUATES

Q. What is Becker Nevus Syndrome


14. CAFÉ–AU–LAIT
A. • It is characterized by presence of a particular
type of organoid epithelial nevus showing MACULES (CALM)
hyperpigmen-tation, increased hairiness and
hamartomatous augmentation of smooth muscle Q. Classical Café – au – lait macule?
fibres and other ipsilateral hypoplasia of breast A. A discrete, well circumscribed, uniformly light
and skeletal anomalies. brown, round or oval macule of variable size, having
h characteristic irregular margins
Q. Differential diagnosis of CALM
A. – Lentigenes
- Pigmentory mosaicism
- Becker’s Nevus
- Postinflammatory hyperpigmentation
- Epidermal nevus
- Junctional nevus
- Nevus spilus
- Congenital melanocytic nevus
- Congenital smooth muscle hamartoma
- Mastocytoma
Q. What do you mean by Café – au – lait?
A. It means Coffee with milk (light brown). If very dark,
it is called café noire (black coffee), which is
sometimes seen in Watson syndrome.
Q. Age of appearance?
A. Usually soon after birth
Q. What is the frequency of solitary CALM in the
normal population.
A. Found in 10-20% of normal population
387
388 DYP SURVIVAL GUIDE FOR POST GRADUATES

Q. Types of CALM?
15. CALLUS
A. A) Isolated CALM
B) Multiple CALMs Q. Why is it a callus?
Q. Difference between isolated and multiple CALMs? A. Lesion is circumscribed hyperkeratosis of the skin
A. Isolated CALM is an isolated phenomenon while with skin markings intact, occurring at the site of
multiple CALM is a marker of multi-systemic disease. constant pressure/friction
The number and size is larger in multiple CALM Q. Differential diagnosis of callosity?
(>3 in number and > 1.5 cms in size). A. Corns and warts
Q. Conditions associated with CALM? Q. Histology of callosity?
A. Remember the pneumonic “BIT-CAFÉ” A. Massive hyperkeratosis with thinning of the
B – Blooms syndrome epidermis
I – Idiopathic Q. What is the morphology of callus?
T – Tuberous Sclerosis A. - The central part is thickest, gradually tapering
C – Dyskeratosis Congenita at the periphery and merging with the normal
skin
A – Ataxia Telangiectasia, Albright syndrome
- The skin markings continue over the callus unike
F – Fanconi’s Anaemia
both corn and wart when they stop at the border
E – Elephantiasis of the lesion
h - Colour varies from light yellow to brownish
black. When in contact with water for a long time,
they appear white due to maceration.
- Produced by repeated or constant friction or
pressure at a particular site.
- Usually asymptomatic but may become painful
when fissured.
Q. What are the sites of predilection for the
development of callosities?
A. The occupation determines the site and size of callus
e.g.
- Tree climbers (forearm and instep of the feet)

389
390 DYP SURVIVAL GUIDE FOR POST GRADUATES

- Labourers like carpenter, blacksmith, gardeners


and farmers (palms)
16. CARCINOMAS
- Violinist (the fingertip)
Q. Why is this BCC?
Fiddlers (chin)
A. Presence of pearly, waxy papules with central
- Taxi drivers ( on the buttocks) depression with rolled out or thready translucent
- Desk worker (elbow) border with erosion or ulceration, bleeding on slight
- Various ill-fitting garments and shoes can injury, crusting and telangiectasia over the surface
develop callosities Q. D/D of ulcerative lesion on face?
- In diabetics and leprosy patients with A. - Basal cell carcinoma
neuropathic feet, callosities may preceed the - Squmaous cell carcinoma
development of perforating ulcer
- Keratoacanthoma
Q. How will you treat callosity?
- Melanoma
A. - Palliative treatment- 1. Application of
- Syphilitic gumma
keratolytics (40% salicylic acid)
- Deep fungal infections
- Advice- avoid precipitating factors like occlusive
and ill-fitting foot wear, avoid repeated pressure Q. Which is the commonest skin malignancy?
at the bony prominences A. Basal cell carcinoma
h Q. Which is the commonest skin malignancy in
immuno-compromised?
A. Squamous cell carcinoma
Q. What are the clinical types of BCC?
A. - Nodular/nodulo-ulcerative BCC
- Pigmented
- Superficial
- Morphea-like
- Fibroepithelioma of Pinkus
Q. What is the fibroepithelioma of Pinkus?
A. Elevated, skin-coloured, sessile lesion on lower
trunk, lumbosacral areas, groin or thigh.

391
392 DYP SURVIVAL GUIDE FOR POST GRADUATES DYP SURVIVAL GUIDE FOR POST GRADUATES 393

Q. Commonest BCC in HIV infected person? - Immunosupression


A. Superficial type - BCC syndrome
Q. Common syndromes associated with BCC? Q. Histopathology of BCC?
A. - Xeroderma pigmentosum
A. Nest of basaloid tumor cells, retraction artefact,
- Albinism hyperchromatic nuclei, palisading of cells at
- Nevoid BCC (Gorlin syndrome) periphery
- Bazex syndrome Q. What is Jacobi ulcer?
- Rombo syndrome
A. - Also known as rodent ulcer
- Rasmussen syndrome
- Neglected BCC that has ulcerated
- Linear unilateral basal cell nevi
Q. What is the etiology of BCC? Q. Treatment of BCC?
A. - Ultraviolet radiation (UVB)—causes pyrimidine A. - Surgery: curettage, electrosurgery, cryotherapy,
damage excision
- ionizing radiation - Moh’s micrographic surgery
- Arsenic exposure - Radiotherapy
- Immuno-suppression - Lasers: Co2, Nd: YAG laser
- H/o of previous non-melanoma skin cancer
- Photodynamic therapy
Q. What is Gorlin syndrome?
- Medical treatment: topical and intralesional
A. Diagnostic criteria:
5- FU, 5% imiquimod, interferon a 2b.
- Multiple BCC >5 or BCC before age of 30.
Q. Do BCC metastasize?
- Odontogenic keratocysts of jaws
- Palmo-plantar pits A. Rarely metastasis, incidence is 0.0028%-0.55%
- Lamellar calcification of falx Q. Which variants of BCC metastasize?
- First degree relative with NBCC A. - Morpheiform
Minor criteria: - Infiltrative
- Medulloblastoma, macrocephaly, cleft lip and - Micronodular
palate, ocular abnormalities, rib abnormalities
- Basosquamous
Q. What is associated with multiple basal cell
epithelioma? Q. What is is basosquamous BCC?
A. - Arsenic keratosis A. BCC transition to squamous cell carcinoma, which is
- Chronic sunlight exposure highly metastatic
394 DYP SURVIVAL GUIDE FOR POST GRADUATES DYP SURVIVAL GUIDE FOR POST GRADUATES 395

Q. What is collision tumor? Q. What is marjolin ulcer?


A. - Two separate and distinct neoplasms which are A. SCC on site of chronic ulcer or scar
in close proximity Q. Etiology of SCC?
- Nevus and BCC, nevus and seborrheic keratosis, A. - Cumulative lifetime sun exposure (UVB)
BCC and melanoma - Ionizing radiation
Q. What are the risk factor for metastatic BCC? - Syndromes like xeroderma pigmentosum, bloom
A. Large size >10 cms, long standing, recurrent lesion, syndrome, ataxia telangiectasia fanconi
aggressive histological pattern syndrome
Q. What is retraction artefact? - Immunosuppresive therapy
A. While processing the sample, there is separation of - HPV Infection
the tumor mass from the dermis - Chronic arsenicism
Q. Why is this squamous cell carcinoma ? - Industrial, chemical
A. Raised, firm, hard, pink to flesh-colored papules or - Thermal exposure
plaque with telangiectasia, smooth or verrucous or - Chronic inflammation
papillated surface and indistinct margin and elevated
base. It appears on normal skin and on pre-malignant Q. Histopathology of SCC?
skin conditions like actinic keratosis and arsenic A. Keratinocyte dysplasia, nuclear atypia, loss of
keratosis. There is fixity to underlying structure. polarity, abnormal intercellular bridges, atypical
Surface becomes friable and undergoes central mitosis, horn or keratin pearls
ulceration and hemorrhage and is painful. Common Q. What are premalignant skin condition causing SCC?
sites are face and back of hand. A. - Actinic keratosis
Q. Different types of Squamous cell carcinoma? - Tar keratosis
A. - Keratotic invasive SCC - Arsenic keratosis
- Nodular SCC - Thermal keratosis
- Arsenic induced SCC - Chromomycosis
- Thermal SCC - Bowen’s disease (intraepidermal SCC)
- Radiation SCC - Erythroplakia of Queyrat
- Oral SCC Q. What are skin condition causing SCC?
- Genital SCC A. - Scars
- SCC of lip - Ulcerative oral lichen planus
396 DYP SURVIVAL GUIDE FOR POST GRADUATES

- Chronic osteomyelitis
17. CONGENITAL
- EBDR
- Discoid lupus erythematosus MELANOCYTIC NEVUS
- Hidradenitis suppurativa
Q. Broder’s system of classification of SCC? Q. Why is it a congenital melanocytic nevus?
A. Based on % of undifferentiated cells: A. A pigmented, well defined plaque with or without
hypertrichosis present since birth
- <25% —grade 1
Q. Differential diagnosis?
- <50%—grade 2
A. - Acquired melanocytic nevus
- <75%—grade 3
- Congenital blue nevus
- >75% grade 4
- Nevus spilus
Q. What are the histopathological variant of SCC?
- Becker’s nevus
A. - Keratoacanthoma-like
- Spindle cell - Pigmented epidermal nevi
- Adenoid - Café au lait macules
- Mucinous SCC - Plexiform neurofibroma
- Papillary SCC - Congenital smooth muscle hamartoma
- Pseudovascular SCC - Melanoma
- Verrucous carcinoma Q. What are the synonyms?
Q. Treatment of SCC? A. Giant pigmented nevus, garment nevus, nevus
A. - Electrodessication and curettage :<10mm and pigmentosus et pilosus, giant hairy nevus, bathing
superficial SCC trunk nevus
- Mohs micrographic surgery Q. Pathogenesis?
- Cryotherapy A. It is a result of a developmental defect in neural crest-
derived melanoblast occurring after 10 weeks in-
- Radiotherapy
utero but before 6 th uterine month
- Laser therapy
Q. Clinical features?
- Photodynamic therapy.
A. - It is present at birth with both sexes having equal
Q. Other malignancy associated with SCC? predilection seen in all races
A. Respiratory cancer, buccal cavity, pharynx, small - A plaque with or without coarse terminal dark
intestine, non-hodgkin lymphoma and leukemia brown or black hair, sharply demarcated or
397
398 DYP SURVIVAL GUIDE FOR POST GRADUATES

merging with surrounding skin, regular or


irregular contour seen at any site
18. CONTACT DERMATITIS
- Large nevus are wormy with smooth skin surface
Q. Name two pathogenic types of contact dermatitis.
or pebbly, mamillated, rugose or cerebriform
- On dermascopy- globules, a fine speckling of a A. - Non-immunologic irritant contact dermatitis
darker hue with a lighter surrounding brown hue - Immunologic allergic contact dermatitis
seen with follicular pigmentation, milia-like cyst Q. Name two types of irritant contact dermatitis.
Q. Histopathology? A. - Acute toxic eruptions
A. - Melanocytes occur as well ordered clusters
- Chronic cumulative insult reactions
(theques) in the epidermis and dermis as sheets,
nests and cords. Nevus cells are seen Q. Describe acute type of contact dermatitis .
surrounding and within the blood vessels, A. Acute, toxic eruptions occur from a single exposure
appendages and nerves to a strong toxic chemical (acid/alkali) inducing
- A diffuse infiltration of strands of erythema, vesicles, bullae or skin sloughing. The
nevomelanocytes in the lower one third of the actions occur within minutes to hours after exposure,
reticular dermis and subcutis is seen localized to the areas of maximum contact and have
Q. Treatment? sharp borders. Healing occurs soon after exposure.
A. - Surgical excision of the nevus. Q. Describe chronic cumulative insult subtype of ICD.
- If less than 1 .5 cm, observe the lesion since it has A. These are the more common type of ICD and are due
practically no chance of becoming malignant to multiple exposures of many low level irritants such
- If 1.5-20 cm, slightly higher chance of malignancy, as soaps and shampoos over time. This may take
hence close and periodic observation and if on weeks, months or even years to occur. It is
an amenable area, surgical excision is preferred characterized by erythema, scaling, fissuring,
- If > 20 cm, serial excision is advised (chances of pruritus, lichenification and poor demarcation from
malignant melanoma are around 5-13%) surrounding skin.
- Even at present, dermabrasion or laser therapy Q. Why is the distribution of contact dermatitis rash
is done, but these can lead to scarring, moreover, important ?
the deeper components of the nevi are not
A. The location and distribution of the dermatitis are
removed so the chances of malignancy are NOT
reduced vital clues to the underlying culprit. For example, an
eczematous dermatitis on the dorsum of the feet
h possibility of shoe dermatitis.
399
400 DYP SURVIVAL GUIDE FOR POST GRADUATES DYP SURVIVAL GUIDE FOR POST GRADUATES 401

Q. Differences between irritant and allergic contact Q. Which are the different agents which can cause
dermatitis eyelid dermatitis?
A. A. Nailpolish, eye make-up and airborne allergens
IRRITANT ALLERGIC
Examples soap Nickel, fragrance, hair dye Q. Suspicious agents involved in dermatitis of different
Number of compounds Many Fewer
Distribution of reaction Localized May spread beyond the area of areas?
maximal contact and become
generalized A.
Concentration of agent needed High Can be minute LOCATION SUSPICIOUS AG ENT
to elicit reaction
Time course Immediate to late Sensitization in 2 weeks, Earlobes or neck Metal jewel lary
elicitation takes 24-72 hours Forehead, scalp m argins Hair dye
Immunology Nonspecific Specific type IV delayed
Face Cosmetic fragrances and pres ervatives,
hypersensitivity reaction
Diagnostic test None Patch test airborne all ergens
Prevalence 75% 25% Axilla Deodora nts
Hands Gloves, occupationa l contact
Q. How many percent of cases of contact dermatitis are Waist band Ela stic
ICD and how many ACD? Dorsa of feet Shoes

A. 80% of ICD, 20% of ACD Q. What is the allergen causing contact dermatitis in
the bindi?
Q. Which is the only way to distinguish between ACD
and ICD? A. Paratertiary butylphenol (PTBP)
A. Patch testing Q. What causes parthenium dermatitis?
Q. Which is the most common allergen overall ? A. Parthenium hysterophorus or photosensitizing weed
Q. Which is an irritant and allergic sensitizer?
A. Poison ivy (most common type IV allergen)
A. Turpentine
Q. Which is the most common allergen used in the tray
of patch testing? Q. Allergy of what causes oral lichen planus?
A. The metal nickel which is found commonly in A. Mercury in dental filling and gold.
costume jewellery Q. Differential diagnosis of contact dermatitis.
Q. Why is nickel dermatitis more common in females? A. Contact dermatitis with its scaling, erythema,
lichenification, and/or vesicles, belongs in the group
A. There is a high rate of sensitization secondary to ear
of eczematous disorders. Other such conditions –
piercing hence it is more common in females
atopic dermatitis, nummular eczema, neurodermatitis,
Q. What causes nickel dermatitis in males? stasis dermatitis, seborrheic dermatitis,
A. In men, nickel dermatitis is predominantly of photodermatoses, dermatophyte infections, drug
occupational origin eruptions, and dyshidrotic eczema (pompholyx)
402 DYP SURVIVAL GUIDE FOR POST GRADUATES DYP SURVIVAL GUIDE FOR POST GRADUATES 403

should always be considered when evaluating a thoroughly avoided. Sources of the allergen as well
prospective patient for contact dermatitis. A complete as cross-reacting agents should be explained to the
history including previous skin diseases, drug and patient. An acceptable non-sensitizing substitute
exposure histories, location and course of the should be offered. For ICD, avoidance of as many
eruption, patch testing, and potassium hydroxide irritants as possible is crucial. Frequent water
tests should help point to the diagnosis of contact exposure, which desiccates and chaps the skin should
dermatitis. be kept to minimum. Frequent moisturization and
Q. If a change in skin care product does not lead to hand protection with gloves are important. With
clearing of a patient’s rash, does it mean that original contact dermatitis, systemic steroids should be used
product was not the culprit? only in acute situations. Compresses may be helpful
if vesicles are present. When the condition is chronic,
A. Not necessarily. Many commercial cosmetic and
topical steroids of appropriate strength and
toiletry products contain the same allergens usually
moisturizers are the mainstay of the therapy.
fragrances and preservatives. Many products contain
Recently, the newer nonsteroidal macrolide
cross-reacting agents that can exacerbate the original
immunosuppressive agents, tacrolimus and
problem. For example, patients who are allergic to
pimercrolimus, have been used increasingly with
the hair dye allergen paraphenylenediamine will
good results. Lastly phototherapy and Grenz ray
need to avoid over-the-counter topical anaesthetic
therapy has also been used in difficult cases.
benzocaine. Both compounds belong to the para-
amino group and can cross-react with one another. h
Q. What substances are tested in the standard
‘screening’ patch test?
A.
ALLERGEN SOURCES
Benzocaine 5% Topical anaesthetic
Nickel sulphate 2.5% Metal jewellery
Neomycin sulphate 20% Topical antibiotics
Balsam of peru 25% Perfume, medications
Formaldehyde 1% Preservative, fabric finishes

Q. Management of contact dermatitis


A. If the patient has ACD the allergen should be detected
by patch testing and subsequently it should be
DYP SURVIVAL GUIDE FOR POST GRADUATES 405

plantar and the dorsal surface, but also occurs on


19. CORN (CLAVUS) distal ends of 4th and 5th toe. It is a modified hard corn
in which superficial capillaries have involved the
Q. What is a corn? epidermis subjacent to the hyperplastic mass.
A. It is a circumscribed, cone shaped, hyperkeratosis of Vascularity is presumed to be the result of repeated
the skin. trauma to the corn. These corns are painful.
Q. What are the clinical features of corn? Q. What is neurofibrous corn?
A. - The size varies from 1mm to 2 cm in diameter. A. It is an advanced type of hard corn. There is underlying
- The apex of this conical keratotic mass is pointed fibrosis and hypertrophy of nerve filaments that is
inwards and presses upon the subjacent associated with severe spasmodic pain.
structures Q. What is a seed corn?
- Pressure upon nerves can cause severe pain A. Also known as clavi miliarae. They appear as
- Dermatoglyphics are lost on the surface of the corn multiple, asymptomatic, discrete, tiny (1-3mm ) sized,
yellowish white papules on the plantar surface of the
Q. What are the types of corn? heel and the fifth toe joint.
A. Hard corn, soft corn, vascular corn/ clavus vasculare, Q. What are the predisposing factors for corn?
neurofibrous corn, seed corn
A. - Bony deformities like spurs
Q. What is a hard corn?
- Ill-fitting occlusive footwares
A. They are well circumscribed usually occurring on the
- Diabetes
dorsa of the toes(pressure area, found in diabetics)
and are painful. Surface is dry, smooth and slightly - Improper walking
convex. When the upper portion is shaved off, the Q. How will you differentiate corn and callosity from
central white core is noted which is the hardest part. plantar warts?
It is the apex of this core that presses on the nerves to A. 1. Callosities have no defined edge, while corns
cause severe pain. have well defined edge; so do warts.
Q. What is a soft corn? 2. Callosities are not painful while the other two
A. It is also known as clavus molle, occurs in the are. Corns are painful on vertical pressure while
interdigital spaces of the toes, most commonly the warts are painful on horizontal pressure.
web space. Soft corns are soft, soggy, and macerated 3. On paring clavuses disapperar, corns show a
so they appear white. central white core while wart shows brown
Q. What is a vascular corn? colored spot which are thrombosed capillaries.
A. Also known as clavus vascularae. It generally occurs 4. Dermatoglyphic markings are present in
on the sides of the foot along the junction of the callosities and absent in the other two.
404
406 DYP SURVIVAL GUIDE FOR POST GRADUATES

Q. How is corn treated?


20. CUTANEOUS
A. - Palliative treatment- application of keratolytics
such as 40% salicylic acid plaster. A solution of TUBERCULOSIS
salicylic 17% and lactic acid 17% in flexible
collodion solution can be painted over the parred Q. What is part of primary tuberculosis complex?
area and allowed to dry.
A. A painless red or brown papule – indurated nodule
- Patient is adviced to avoid ill fitting shoes. or plaque – ulcerates to form tuberculous chancre.
- Surgical correction of bony or muscular defect. This with prominent regional lymphadenopathy
An intralesional injection of triamcinolone occasionally cold suppurative and draining lesions
acetonide may give relief from pain. over lymph node complete the complex.
h Q. Differential diagnosis of primary inoculation
tuberculosis
A. - Sporotrichosis
- Yaws
- Histoplasmosis
- Coccidioidomycosis
- Nocardiosis
- Syphillis
- Atypical mycobacterial disease
- Tularemia
- Blastomycosis
- Leishmaniasis
Q. What is the prosector’s wart?
A. Resulting from inoculation during an autopsy this is
the prototype of tuberculosis verrucosa cutis.
Q. Morphology of tuberculosis verrucosa cutis
A. Small papules which become hyperkeratotic
resembling a wart. Lesion enlarges by peripheral
expansion with or without central clearing. Fissuring
may occur discharging purulent exudate mostly
solitary.
407
408 DYP SURVIVAL GUIDE FOR POST GRADUATES DYP SURVIVAL GUIDE FOR POST GRADUATES 409

Q. Frequent locations of tuberculosis verrucosa cutis Q. How will you investigate?


A. Dorsa of fingers and hands in adults and buttocks in A. SPECIFIC NON-SPECIFIC
children
- Skin biopsy - Mantoux test
Q. Differential diagnosis of tuberculosis verrucosa cutis
A. - Atypical mycobacteriosis ( caused by M. Marinum) - AFB staining - Quantiferon
- North American blastomycosis Gold Test
- Majocchis granuloma - Mycobacterium culture - CBC, ESR
- Chromoblastomycosis - Mycobacterium PCR
- Verrucous epidermal nevus Q. What is the historical importance of cutaneous
- Hypertrophic lichen planus tuberculosis to a dermatologist?
- Iododerma
A. Finsen, who used ultraviolet therapy (Finsen
- Bromoderma
chambers) for treatment of cutaneous TB, is the only
- Verruca vulgaris dermatologist to have won a Nobel prize to date.
- Verrucous epidermal nevus
Q. Which is the condition in which caeseation is seen
Q. What is scrofuloderma?
in cutaneous TB?
A. It is tuberculous involvement of the skin by direct
extension usually from underlying TB A. In tuberculids, lupus miliaris disseminatus faciei
lymphadenitis. Q. Describe lupus vulgaris?
Q. Histopathology of cutaneous TB A. Typically, a single plaque composed of grouped red
A. Epidermal changes – brown papules which trend to heal slowly in one area
hyperkeratosis and progress in another minute, translucent, deeply
Acanthosis embedded, diffusely in infiltrated dermis, expanded
Pseudoepitheliomatous Hyperplasia by development of new papules in periphery
Rarely, atrophy coalescing with slightly elevated scaly plaques.
Dermal changes – multiple non - caseating granuloma Q. Differential diagnosis of lupus vulgaris
with lymphocytes, epitheliod cells, Langhans giant
cells A. - Rosacea
Q. Differential diagnosis of scrofuloderma - Tertiary syphilis
A. - North American blastomycosis - Sarcoidosis
- Majocchis granuloma - Leprosy
- Chromoblastomycosis - Systemic mycosis
410 DYP SURVIVAL GUIDE FOR POST GRADUATES DYP SURVIVAL GUIDE FOR POST GRADUATES 411

- Chronic discoid lupus erythematous - Lupoid rosasea


- Leishmaniasis - Lupoid leishmaniasis
- Acne vulgaris Q. What is the meaning of lupus vulgaris?
Q. What is seen in diascopy of plaque of lupus A. Lupus means wolf (refers to the mutilated
vulgaris? appearance of the face as if bitten by a wolf) and
A. Apple jelly nodules (yellowish brown soft nodules vulgaris means common
at the periphery of the lesions). One more test, rarely
h
performed is to probe the lesion with a sterile probe,
when it enters easily and there is a butter like
consistency.
Q. Classification of tuberculosis?
A. Name- Bravo cassification

By direct inoculation Primary inoculation TB


By continuity Scrofuloderma
MULTIBACILLARY TUBERCULOSIS Tuberculosis peri-orifacialis
By Hematogenous Spreading Acute military TB
Gumma (cold abscess)
By direct inoculation (re- Tuberculosis verrucosa cutis
PAUCIBACILLARY TUBERCULOSIS exposure) Lupus Vulgaris
By hematogenous spreading Lupus Vulgaris

Q. Other lupus’s in dermatology


A. - Lupus erythematosus
- Lupus hair
- Lupus miliaris disseminatus faciei
- Lupus band test
- Lupus profundus
- Lupus rosacea
- Lupus pernio
- Lupoid sycosis barbae
DYP SURVIVAL GUIDE FOR POST GRADUATES 413

Q. What is the other name of Darier’s disease?


21. DARIER’S DISEASE
A. Darier White disease, keratosis follicularis,
dyskeratosis follicularis.
Q. Why is it a case of Darier’s disease ?
A. Lesions are present as keratotic red brown papules Q. What is the epidemiology of Darier’s disease?
(dirty warty papules) in a ‘seborrheic’ distribution A. The disease begins between the ages 6 to 20 years
involving the trunk, scalp, face and lateral aspects of and peaks during puberty
the neck. Q. What are the exacerbating factors?
- There is mild flexural involvement. Moderate A. It is worse in summer. Sweating, heat and occlusion
itching and malodour is fairly frequent. and lithium precipitate the lesions.
- Nail changes include longitudinal red and/ or Q. What is the pathogenesis of Darier’s disease?
white lines, ‘v’ shaped nicking of distal end and
A. It is an autosomal dominant genodermatosis caused
subungual hyperkeratosis.
due to insufficient functioning of the 2b isoform of
- Flat, skin-coloured or brownish papules the sarco/endoplasmic reticulum Ca 2+ ATPase
resembling flat warts over dorsal aspect of hands (SERCA2b)
and feet. Usually skin lesions are bilaterally
Q. What are the complications of Darier’s disease?
symmetrical but rarely unilateral, arranged in a
linear pattern. A. Infections- Secondary bacterial infection, fungi,
Kaposi’s varicelliform eruption due to HSV.
- Palmoplantar involvement is in the form of
punctate keratosis. - Oral salivary gland obstruction with painful
swelling
- Whitish papules over the palate, gingiva, buccal
mucosa and tongue (cobblestone appearance) . - Ocular complications like corneal ulceration or
Q. What are the differential diagnosis of Darier’s staphylococcal endopthalmitis
disease? Q. What are the histopathology findings in Darier’s
A. - Acrokeratosis verruciformis of Hopf (flat wart disease?
like papules on the extremities). A. The prominent histologic findings are acantholysis
- Seborrheic dermatitis and dyskeratosis.
- Grovers disease (no palmoplantar involvement, - 2 types of dyrkeratotic cells are found- corp
nail changes and mucosal change) ronds and grains
- Pemphigus vegetans - corp ronds have a central homogenous
basophilic pyknotic nucleus that is surrounded
- Hailey-Hailey disease
by a clear halo. This is seen in the upper stratum
- Pyoderma vegetans malphigii layer
412
414 DYP SURVIVAL GUIDE FOR POST GRADUATES

- The grains are small oval cells found in the horny 22. DERMATOFIBROMA
layer and have elongated nuclei, eosinophilic
cytoplasm and are surrounded by homogenous
Q. Why is it a dermatofibroma?
dyskeratotic material
A. Presence of dome-shaped slightly erythematous
- Other features present are papillomatosis,
dermal nodule with a button-like firm consistency
hyperkertosis. There is mild to moderate
seen usually on the extremities, sometimes tender.
perivascular inflammatory infiltrate in the
superficial dermis Q. Differential diagnosis
Q. What is course of Darier’s disease? A. - Papulonodular lesions containing mucin
A. It follows a chronic course without spontaneous - Scar
remission. The disease severity can fluctuate over - Blue nevus
time with some patients reporting improvement and - Pilar cyst
others deterioration over time. - Metastatic cancer
Q. How will you manage a case of Darier’s disease? - Kaposi sarcoma
Q. · General measures- lightweight clothes, sunscreen. - Dermatofibromsarcoma protuberans
- Topical retinoids Q. Other names?
- Treatment of infection with antibiotics and A. - Solitary histiocytoma
antifungals
- Sclerosing haemangioma
· Systemic management:
Q. Pathogenesis?
- Oral isotretinoin and acitretin in severe
A. Many think it is a late histiocytic reaction to an
disease unresponsive to topical therapy. It
arthropod bite
is not recommended in patients with
predominantly bullous / intertriginous Q. Clinical features?
lesions as it may aggravate the disease. A. - Asymptomatic
- Cyclosporine in those who do not respond - A solitary dome shaped papulo-nodular lesion
to oral retinoids with dull to shiny texture and ill-defined borders
· Surgical management : of 3-10 mm seen with colour being variable from
skin coloured, pink, brown, tan to dark chocolate,
- It is done for focal recalcitrant lesions mainly centre being dark fading to normal skin at
in flexural and gluteal area. periphery.
h - Distribution-legs,arms,trunk.
415
416 DYP SURVIVAL GUIDE FOR POST GRADUATES

- Dimple sign/Fitzpatrick sign - lateral


compression with thumb and index finger
23. DERMATITIS ARTEFACTA
produces a depression or dimple.
Q. Dermatitis artefacta is also known as?
Q. Histopathology
A. Factitial dermatitis.
A. Whorling fascicles of spindle cells with small amounts
of pale blue cytoplasm and elongated nuclei seen. Q. What is Dermatitis artefacta?
Epidermal hyperplasia is often seen. A. Dermatitis artefacta is a psychocutaneous disorder
Q. Treatment in which patients self-inflict cutaneous lesions as a
means to satisfy a psychological need, of which they
A. Cryosurgery, excision are not always consciously aware.
h Q. What are the clinical features of Dermatitis artefacta?
A. – The lesions of dermatitis artefacta can mimic any
dermatoses.
– They can be single or multiple, unilateral or
bilateral, and although usually within easy reach
of the hands, the lesions may also be created by
an external auxiliary agent.
– Lesions can range from vesicles to purpura to
subcutaneous emphysema and can be caused by
methods such as carving with sharp instruments,
applying chemicals, injecting foreign substances,
etc.
Q. Has dermatitis artefacta any sex preponderance? In
which age group it is more common?
A. Most frequently in adolescence and young
adulthood, it can occur at any age.
Female-male ratio of 8:1.
Q. What is the histopathology of dermatitis artefacta?
A. – Hyperkeratosis
– Irregular acanthosis
– Blood vessel proliferation
417
418 DYP SURVIVAL GUIDE FOR POST GRADUATES

– Fibroplasia
24. DOWLING – DEGOS DISEASE
– Sometimes polarizing exogenous material with
a variable foreign body response is seen
Q. Why is it Dowling-Degos disease?
Q. What is the differential diagnosis of dermatitis
A. Reticulate hyperpigmented macules affecting neck,
artefacta?
axillae, submammary folds, flexors, elbows, knees
A. – Primary skin disorders. and groin. Comedone-like papules on back and neck.
– Monosymptomatic hypochondriacal psychosis. Pitted perioral and facial scars and epidermoid cysts
– Malingering. may be seen. Itching present.
– Obsessive compulsive disorder. Q. Differential diagnosis?
Q. What is the treatment? A. - Reticulated pigmentation of Kitamura and Acro-
pigmentation of Dohi (some consider both these
A. – Symptomatic
conditions as variants of Dowling- Degos
– Supportive disease).
– Anti-anxiety - Acanthosis nigricans
– Anti-depressants - Neurofibromatosis-1
– Anti-psychotics Q. Synonym?
Q. Which age-group has the most favourable A. Reticulate and pigmented anomaly of the flexures
prognosis?
Q. What is it?
A. The most favorable prognosis exists for children or
A. Rare genodermatosis characterized by acquired
adults in whom the lesions represent a response to a
reticulate pigmentation of the flexures
transient stress.
Q. Mode of inheritance?
h A. Autosomal dominant with variable penetration
Q. Genetic defect?
A. Mutation in Keratin – 5 gene located on
chromosome 17
Q. Prevalence?
A. - Most cases reported from Asian and
Mediterranean countries including India
- Both genders equally affected

419
420 DYP SURVIVAL GUIDE FOR POST GRADUATES

Q. Course? 25. EPIDERMAL NEVI


A. Starts in early adult life (3rd-4 th decade). Slowly
progressive. Q. Why is this epidermal nevus?
Q. Histopathology? A. The lesions are pigmented velvety streaks which were
A. - Increased pigmentation of basal layer. pink initially and gradually become brown and
- Filiform elongation of rete ridges with supra- verrucous (wart-like) with age. The lesions follow
papillary thinning (antler-like pattern), the Blaschko’s lines. These lesions do not cross the
- Melanocyte proliferation, no melanin midline.
incontinence Q. What is the differential diagnosis of verrucous
epidermal nevus?
Q. Relation with Acropigmentation of Kitamura?
A. 1) Nevus sebaceous
A. - In many cases, features of both are observed.
Some consider it to be varied presentation of 2) Lichen striatus- acquired condition, doesn’t persist
same entity. 2) Verruca Vulgaris – Koebner phenomenon
- In this, hyperpigmented macules appear during 3) Linear Lichen planus – violaceous, flat topped,
childhood on the dorsum of hands and feet in severe pruritus
reticulate pattern and are atrophic. They are 4) Seborrhoeic keratosis
aggravated by sunlight.
5) Linear psoriasis
Q. What is Galli-Galli disease? Q. What are Epidermal Nevi?
A. Regarded as acantholytic variant of Dowling-Degos. A. Epidermal nevi can be: congenital lesions (birthmark)
Q. Treatment? or a lesion arising early in life (1st year of life); a benign
A. - Topical: Hydroquinone, Tretinoin, Adapalene, tumor of melanocytes; a hamartoma.
corticosteroids Q. Classify Epidermal Nevi
- Lasers: erbium-YAG lasers A. 1) Keratinocytic:
h - Verrucous epidermal nevus
- Inflammatory Linear Verrucous Epidermal
Nevus
2) Appendageal:
- Nevus Sebaceous - Nevus Comedonicus
- Eccrine Nevi - Apocrine Nevi
3) Epidermal Nevus Syndromes
421
422 DYP SURVIVAL GUIDE FOR POST GRADUATES DYP SURVIVAL GUIDE FOR POST GRADUATES 423

Q. Where do epidermal nevi originate? Q. How will you treat a case of verrucous epidermal
A. Epidermal nevi are thought to originate from the nevus?
pluripotent stem cells in the basal layer of the A. 1) Cryotherapy- for small lesions
embryonic epidermis. 2) Argon/CO2 laser
Q. What is the pathogenesis of epidermal nevi? 3) Full thickness surgical excision with grafting –
A. Epidermal nevi display mosaicism: mutations of definitive
keratins 1 and 10, mutations in the ATP2A2 gene. Yet 4) topical therapy- corticosteroids,retinoic acid,
another theory is that many of the nevi are caused by tars, anthralin, 5-FU and podophyllin-limited
post-zygotic mutations in fibroblast growth factor
5) oral therapy- retinoids
receptor 3 (FGFR3), which is a gain-of-function
(activating) mutation. The hamartomatous process Q. Why is this inflammatory linear verrucous epidermal
also involves a part of the dermis. nevus ( ILVEN) ?
Q. What is nevus unius lateris and ichthyosis hystrix? A. There are pruritic, unilateral, scaly, erythematous
papules which coalesce to form a linear plaque,
A. Nevus unius lateris is a variant in which there are
mostly on a limb and at times on the trunk.
extensive unilateral plaques, often involving trunk.
Q. What are the differentials?
Ichthyosis hystrix is a variant in which bilateral
involvement on the trunk is seen. A. 1) Congenital Hemidysplasia with Ichthyosiform
Nevus & Limb Defects (CHILD)
Q. What is epidermal nevus syndrome?
2) Psoriasis superimposed on an epidermal nevus
A. Epidermal nevus seen with other developmental
– micaceous scales
abnormalities.
3) Lichenoid Epidermal Nevus – verrucous plaque
Q. What are the complications of Verrucous epidermal
nevus? Q. What are other names of Inflammatory Linear
Verrucous Epidermal Nevus?
A. The complications include: maceration, infection, and
paronychia. A. ILVEN is synonymous with Dermatitic Epidermal
Nevus and Eczematous Epidermal Nevus
Neoplasms like BCC, SCC and keratoacanthoma may
rarely occur after puberty. Q. How is the pathogenesis of ILVEN different from
that of psoriasis?
Q. What are the histopathological findings of verrucous
epidermal nevus? A. Involucrin, a structural component of mature
squamous epithelium, is seen in large numbers in
A. The histopathological findings consist of
orthokeratotic epithelium of ILVEN, but is minimally
hyperkeratosis, acanthosis, and papillomatosis –
expressed within the keratinocytes underlying areas
“church-spire” pattern.
of parakeratosis. However, in psoriasis, involucrin
424 DYP SURVIVAL GUIDE FOR POST GRADUATES

can be seen in all layers of the epidermis except for


the basal layer.
26. ERYTHEMA MULTIFORME
Q. What are the clinical features of ILVEN?
Q. What are the other names of erythema multiforme?
A. Patients with ILVEN present before the age of six with (EM)
pruritic, unilateral, scaly, erythematous papules A. Erythema multiforme von Hebra, Erythema
which may coalesce to form a linear plaque, mostly multiforme minor (also called as Erythema
on a limb and at times on the trunk. It has 4:1 exudativum multiforme) and Herpes simplex
female:male predilection and lesions resolved associated erythema multiforme.
spontaneously by adulthood.
Q. Who first described EM?
Q. Describe the histopathological findings A. Austrian dermatologist Ferdinand von Hebra in 1860.
A. The histopathological findings in ILVEN consists of Q. What are the important causes of EM?
elongation of rete ridges and broad zones of
A. · INFECTIONS: - Herpes simplex associated EM
parakeratosis which alternate with depressed regions
of orthokeratosis and hypergranulosis. Exocytosis of - Mycoplasma induced EM
lymphocytes and neutrophils into the spongiotic - Histoplasma
epidermis may be seen. · Drug induced EM
Q. How will you treat a patient of ILVEN? · Radiation induced EM
A. 1) Surgical excision – results in scarring · Exposure to poison ivy
2) Pulsed-dye laser- destruction of small blood · Systemic diseases (inflammatory bowels disease,
vessels in papillary dermis LE, Behcet’s syndrome)
3) Topical tretinoin & 5-Fluorouracil cream · Many cases are Idiopathic EM
combination therapy · Rarely seen in pregnancy
4) Topical calcipotriol Q. Describe the classical lesion of EM?
5) Patients with arthritis may be treated with A. Target or Iris of Bateman lesion: three zones are
Eternacept seen—
h · Central dusky purpura /bulla
· Elevated pale ring
· Macular erythema
Q. What are the clinical features of EM minor?
A. EM minor is a self limiting recurrent disease of young
adults lasting for 1 to 4 weeks. It is uncommon in
425
426 DYP SURVIVAL GUIDE FOR POST GRADUATES DYP SURVIVAL GUIDE FOR POST GRADUATES 427

childhood and shows slight male preponderance. - Anti tuberculosis: rifampicin, isoniazid
Lesions begin as sharply marginated erythematous - Anticonvulsants: phenytoin, barbiturates
macule, which becomes raised edematous papule - NSAIDS: acetylsalicyclic acid, ibuprofen
in 24 to 48 hrs. - Others: glucocorticoids, piroxicam, gold,
Typical target lesions (Iris lesions) are seen. Lesions cimetidine, methotrexate, digitalis
are symmetrical, acral and involve palms, soles and - Vaccinations: BCG
mostly dorsum of hands. Q. Which skin conditions can have EM as one of its
Q. Differences between EM minor, EM major and SJS manifestations?
A. A. - LE (Rowell’s syndrome)
- Paraneoplastic pemphigus
EM minor EM major SJS
Type of skin lesions Typical target Typical target Dusky macules - Sarcoidosis
Atypical Targets Macular atypical targets
Occasional bullous Bullous lesions (>10% Q. What is the histopathology of EM?
lesions BSA)
Distribution Extremities, face Extremities, face Trunk, face A. · Normal basket weave stratum corneum
Mucous involvement Absent or mild Severe Severe
Systemic involvement Absent Present Present · Vacuolar interface dermatitis with vacuoles and
Progression to TEN No No Yes foci of call necrosis
Precipitating factors Infections Infections and drugs Drugs
· Mononuclear dermal infiltrate primarily in the
Q. Is Koebner phenomenon seen in EM? upper dermis then at dermo-epidermal junction.
A. Yes · T- lymphocytes seen
· Cytotoxic cells in the epidermis and helper cells
Q. What is the classification of erythema multiforme?
in the dermis
A. · Erythema multiforme minor Q. What are histopathological D/D ?
· Erythema mutiforme major A. · FDE
· SJS · Graft versus host disease
· Overlap syndrome · Pityriasis lichenoides
· TEN · LE
Q. What are the drugs causing EM? Q. What are the clinical D/D?
A. - Antibiotics: penicillin and semi-synthetic A. · Bullous arthropod reaction
p e n i ci l l i n , s ul f on am i d e s , t e t r ac y cl i ne , · Urticarial vasculitis
c h l o r a mp h e ni c o l , s tr e p to m y ci n , · Bullous pemphigoid
griseofulvin. · Pemphigus vulgaris
428 DYP SURVIVAL GUIDE FOR POST GRADUATES DYP SURVIVAL GUIDE FOR POST GRADUATES 429

· Paraneoplastic pemphigus Q. What is the prognosis and mortality rate of EM


· LE minor, EM major and SJS?
Q. What is the treatment of EM? A.
EM Minor EM major SJS
A. - In HSV induced EM: treatment with antiviral Prognosis Good Good Variable
(acyclovir, valacyclovir, famciclovir). Mortality Low Low 1-5%

- Prevent out breaks of HSV .


- Use of sunscreen to prevent UVB-induced
outbreak in HSV induced EM h
- topical steroids and oral anti-histaminics.
- In extensive disease: systemic steroids.
Q. What is SCORTEN?
A. lt is a severity of illness score to determine prognosis
for cases of TEN based on 7 independent risk factors
for death as assessed on the first day of
hospitalization. It includes the following parameters-
1] Age > 40 years
2] Presence of malignancy
3] Epidermal detachment
4] Heart rate- >120/min
5] Bicarbonate< 20mmol/L
6] Urea>10mmol/L
7] Glycaemia>14mmol/L
• 1 point is awarded for each parameter,
SCROTEN is then derived after totaling of scores.
SCORTEN PROBABILITY OF DEATH
1) 1 3
2) 2 12
3) 3 35
4) 4 58
5) >5 90
DYP SURVIVAL GUIDE FOR POST GRADUATES 431

27. ERYTHEMA NODOSUM (EN) b) Fungal


· Histoplasmosis, blastomycosis, coccidioidomycosis
Q. Why is this EN? c) Parasitic
A. The lesions are erythematous, warm, painful nodules, · Toxoplasmosis
1-5 cms in diameter which is usually slightly elevated
above the skin surface, which occurs mostly on the d) Viral
lower extremities, particularly on anterior tibial · Hepatitis B virus
surface and lateral shin. 2. Drugs
Q. Differential diagnosis of EN?
· Sulfonamides, bromides, iodides, sulfonylurea,
· Erythema induratum- seen on posterior calves, OCPs, penicillin, gold, infliximab
chronic course, possibility of ulceration and
scaling 3. Malignancies
· Syphilitic gumma- unilateral distribution · Hodgkin disease, non -Hodgkin lymphoma,
· Subcutaneous fat necrosis associated with acute myelogenous leukemia
pancreatitis and nodular vasculitis- associated 4. Granulomatous dermatosis
clinical features and histopathology can be used
to differentiate · Sarcoidosis
· Subacute infection - helicobacterial cellulitis and 5. Inflammatory bowel disease
mycobacterium infection · Ulcerative colitis, Crohn’s disease.
Q. Define erythema nodosum? 6. Pregnancy
A. Erythema nodosum is a commonest type of septal Q. In children, what is the most common cause?
panniculitis, which could be caused by internal
infections, neoplasms, inflammatory multisystem A. The most common cause in children is streptococcal
diseases and drug induced. infection
Q. What is the etiology of erythema nodosum? Q. Why is EN common in females?
A. The various causes of erythema nodosum are A. In pregnancy and young females, predisposition to
1. Infections high levels of estrogen favours EN
a) Bacterial Q. What clinical features of EN?
· Streptococcal infections, tuberculosis, Yersinia · EN is most frequently seen in females in age
enterocolitica, mycoplasma pneumonia,
group 20-30 years
lymphogranuloma venerum, leptospirosis.
430
432 DYP SURVIVAL GUIDE FOR POST GRADUATES DYP SURVIVAL GUIDE FOR POST GRADUATES 433

· It appears as erythematous, warm, painful Q. Difference between EN and ENL


nodules, 1-5cms in diameter which is usually A.
slightly elevated above the skin surface, which EN ENL
Cause Various M. leprae
occurs mostly on the lower extremities, Onset Acute Chronic
particularly on anterior tibial surface and lateral Duration Lasts for weeks 72 hrs
Ulcers No Seen
shin Systemic involvement Mild Severe
H/P features Septal panniculitis with Neutrophilic vasculitis
· Lesions are rarely seen on posterior calves, upper lymphohistiocytic infiltrate
leg, extensor arms, neck and very rarely face
Q. In which condition does EN- like lesions are seen?
· Lesion change colour in second week from bright
A. EN- like lesion are seen in helicobacter septicemia,
red to bluish, which then fades to form the bruise
brucellosis, cat scratch disease, Behcet’s disease and
that is called erythema contusiforme which Sweet’s syndrome
disappears in 1-2 weeks
Q. What is histopathology of EN?
· The nodules usually don’t ulcerate and subside · It is a prototype of septal panniculitis
within 2-6 weeks, without atrophy or scarring, · In early stage, septal edema and
leaving behind hyperpigmentation lymphohistiocyte infiltrate with few neutrophils
Q. What are the types of EN? and eosinophilis
A. There are two types of EN · In older lesions, there is granulomatous
infiltrate.
1) Acute type: Fever, headache, episcleritis,
· Miescher’s radial granulomas are the hallmark
conjunctivitis, malaise, joint pain, arthralgia of EN. These granuloma consist of small well
occur in 50% cases. Erythema, swelling and defined, aggregation of small histiocytes,
tenderness, morning stiffness occur over joint. arranged radially around the central cleft
Any joint may be involved, but ankles, knees and · There is also thickening of septa in late lesions
wrist most commonly affected due to fibrosis
2) Chronic type : Also called erythema nodosum Q. What are the investigations to be done for EN?
migrans (Vilanova disease),is a chronic form of A. The following investigation should be done-
panniculitis, that presents as asymptomatic or · ASO titre through a throat swab culture
minimally painful nodules on leg. The lesion · Preceeding diarrhea history is suggestive of
starts as nodules and form plaques. The plaques Yersinia infection
show peripheral extension and central clearing · Travel and exposure history for endemic fungal
and last for months to years infection
434 DYP SURVIVAL GUIDE FOR POST GRADUATES

· Chest x-ray and Mantoux test for tuberculosis or


sarcoidosis
28. FIXED DRUG ERUPTIONS
· Complete blood count and bone marrow studies
for haematological disorders Q. Why is this FDE?
· Colonscopy and barium studies for patients with A. History of drug intake, typical site (mucosal, genital
gastrointestinal symptoms are common sites), typical morphology, one or a few,
Q. What is the management and treatment of EN? round, sharply demarcated erythematous and
edematous plaques are seen, sometimes with a
A. The treatment of EN consists of three components-
dusky, violaceous hue, central blister or detached
· Identifying trigger epidermis. Heals with pigmentation.
· Rest and elevation of affected extremity Q. Explain the differential diagnosis to be considered
· Specific anti-inflammatory medication in a case of FDEs.
(i) The following steps to be done A. - Single lesion - a spider bite or exaggerated
1. Bed rest in most of the cases especially arthropod bite reaction
children. - Multiple lesions - erythema multiforme or SJS
2. Reducing vigorous exercise and restriction - On the genitalia - herpes simplex viral infections
of physical activities might prevent
recurrences. - Periungual lesion - paronychia
3. Infection if any should be controlled to - Non-pigmenting variant - soft tissue bacterial
prevent prolongation of EN infections
4. NSAIDs like aspirin, indomethacin and Q. Why is it called “fixed” drug eruption?
naproxen should be given A. Fixed drug and Fixed site
5. Supersaturated solution of potassium iodide, Q. What are the various clinical variants of FDEs?
5 drops three times a day, increasing per drop
per dose everyday upto 30 drops three times A. - Classical - Bullous
a day (One drop equals to 50 mg KI). - Generalized - Linear
6. In tablet form, KI is 300 mg three times a day - Urticarial - Purpuric
7. Intralesional steroids are given for persistant - Non Pigmented
lesions
- Large lesions, heal without residue, seen in the
8. In acute lesion, colchicine 0.6 mg twice daily girdle region, usually seen with
9. Aspirin and potassium iodide are 2 drugs pseudoepihedrine and also with NSAIDs,
which cure and cause EN betahistine, cimetidine, acetaminophen and
h tetrahydrozoline.
435
436 DYP SURVIVAL GUIDE FOR POST GRADUATES DYP SURVIVAL GUIDE FOR POST GRADUATES 437

- It has two variants: pseudo-cellulitis or - pigment incontinence is usually marked.The


scarlatiniform type and baboon syndrome stratum corneum is normal.
(symmetrical drug related intertriginous and - Papillary dermal fibrosis and deep perivascular
flexural exanthema-SDRIFE). pigment incontinence are commonly present
Q. What are the various causative drugs responsible for from prior episodes.
FDEs? Q. What is the treatment advised in a case of FDE?
A. - Antibiotics-sulfonamides, tetracyclines A. - Counselling - They should be told not to use the
(classically produces genital FDE), erythromycin suspected drug again.The offending drug should
- NSAIDs- pyrazolone derivatives, paracetamol, be clearly mentioned in the case/paper and
naproxen, oxicams, mefanimic acid, acetaminophen, should be explained to the patient.
celecoxib - Symptomatic management of the disease -
- Barbiturates Antihistaminics, steroids, topical antibiotic form
the mainstay of the treatment.
- Carbamazepine
Q. What is the treatment for the pigmentation produced
- Phenolphthalein after FDE?
- Cetrizine, hydroxizine A. No treatment is known. It subsides with time.
- Dextromethorphan Q. In which drug was this phenomenon 1st described?
- Quinine A. With phenolphthalein, which was used as a laxative.
- Lamotrigine AGEP
- Phenylpropanolamine Q. Why is this AGEP?
Q. What are the predisposing factors of FDE? A. - Nonfollicular pustules, more than 100 and less
A. HLA - B22 that 5 mm in diameter occurring on face and
Q. What is the pathogenesis? major skin creases
- Nikolsky’s sign is positive
A. Supposed to be type IV hypersensitivity reaction. Can
be reproduced by patch testing in some cases. - Mucous membrane involvement is common
Q. Histological features seen in FDE lesions. Q. D/D of AGEP?
A. Interface dermatitis with subepidermal vesicle A Generalized Pustular psoriasis
formation, necrosis of keratinocytes and a mixed · Pustular psoriasis of pregnancy (impetigo
superficial and deep infiltrate of neutrophils, herpetiformis)
eosinophils and mononuclear cells. · Subcorneal pustular psoriasis
438 DYP SURVIVAL GUIDE FOR POST GRADUATES DYP SURVIVAL GUIDE FOR POST GRADUATES 439

Q. What is another name of acute generalized Lichenoid drug eruption


exanthematous pustulosis?
A. Toxic pustuloderma, pustular drug eruption Q. Why is this lichenoid eruption?
Q. What is the incidence of AGEP? A. · Looks same like idiopathic oral lichen planus
A. 1 to 5 cases per million per year. with a classic lace like reticular pattern .
Q. What is the most common cause of AGEP? · The drug eruption is mostly unilateral. Common
on buccal mucosa (inside cheeks). Can also be
A. Drugs:
seen on tongue, palate, gums .
· ampicillin, amoxicillin, quinolones,
hydroxychloroquine, Q. Drugs causing lichenoid drug eruption.
· sulfonamides, antibiotics, terbinafine, imatinib, A. · Gold, hydroxycholoquine, furosemide, NSAIDS,
corticosteroid, macrolides aspirin
· oxicam, NSAIDS and anti-epileptics · ACE inhibitors, beta blockers, CCB’s
· Mercury exposure, bacterial and viral infections · Anti convulsants, anti-tuberculosis, ketoconazole
(EBV,CMV) · Antimalarials
Q. What are the associated abnormalities with AGEP? · Hepatitis B vaccinations
A. Fever, facial edema and scarlatiniform rash. Q. Site of lichenoid drug eruption?
Laboratory abnormalities: leukocytosis, neutropenia,
eosinophilia. A. - Photodistributed or generalized
Q. what is the % of mortality in AGEP? - Lower lip involved
A. 5% - Mucous membrane and nails are spared
Q. Histopathology of AGEP? Q. When does lichenoid eruption occurs?
A. - Early lesion: marked papillary edema, A. Months to years of starting the offensive drug
neutrophilic clusters in dermal papillae and Q. Histopathology?
perivascular eosinophilis
A. · Parakeratosis
- Well developed lesion: intraepidermal or
subcorneal pustulosis · Band-like lymphocytic infiltrate along the
Q. Treatment of AGEP? dermo-epidermal junction, with necrosis of
keratinocytes
A. Stop the offending drug
· Dermal infiltrate of mainly lymphocytes. If
- Mostly topical steroids and antihistaminics.
eosinophils and plasma cells are seen, then it is
Sometimes, systemic steroids in the dose of 1mg/
kg/day. photo-lichenoid eruption
- In severe cases: infiximab,etanercept. Q. Metal causing oral lichenoid eruption?
Cyclosporine may be used. A. Mercury, gold, cobalt and nickel
440 DYP SURVIVAL GUIDE FOR POST GRADUATES

Q. What is the unusual form of eruption?


29. FRECKLES
A. Drug-induced ulceration of lower lip
Q. What is the treatment of lichenoid drug eruption? Q. What are freckles?
A. STOP THE OFFENDING DRUG. A. Freckles are small, <0.5 cm, well-circumscribed light
· Oral antihistaminics to dark brown macules that occur profusely on the
· Topical steroid sun exposed skin of the face, neck, shoulders and back
of the hands appearing in the 1st three years of life in
· Tacrolimus
fair colored people. Autosomal dominant inheritance
· Oral steroid like prednisolone may be likely. Not found on mucous membranes.
· Good oral hygiene to prevent secondary Decrease later in life
candidiasis Q. What is the pathogenesis of freckles?
Q. How does lichenoid drug eruption differ from lichen A. It results from sun-induced melanogenesis and
planus ? transport of an increased number of fully melanized
A. · Lichenoid drug eruption has a later age of onset, melanosomes from melanocytes to keratinocytes
more generalized involvememt without flexural Q. What are the predisposing factors for freckles?
predilection on photoexposed areas.
A. Exposure to sun and positive family history
· Morphologically they are more eczematous,
Q. What is the other name for freckles?
psoriasiform or pityriasis rosea like and
Wickham’s striae absent, mucous membrane are A. Ephelides
spared. Q. Which population is more susceptible to freckles?
h A. Blonds and red heads with blue eyes and of celtic
origin
Q. What are the differential diagnosis ?
A. Simple lentigines, solar lentigines, café-au-lait
macules (solitary and larger) and junctional nevi
Q. How will you differentiate ephelides from lentigo
simplex?
A. Lentigo are benign, persistent, discrete,
hyperpigmented macules, appearing at any age and
on any part of the body, including the mucosa. The
intensity of colour is not depend ent on sun exposure.

441
442 DYP SURVIVAL GUIDE FOR POST GRADUATES

Solar lentigo occurs at older age due to long standing


exposure to sun. The back of hands and face are
30. FOX FORDYCE DISEASE
favoured sites.
Q. Describe the condition?
Q. How do you differentiate ephelides from solar
lentigines? A. It is a rare condition occurring mostly in women,
during adolescence or post adolescence. It can be
A. They appear by 3rd-4th decade of life, are larger in size,
familial in nature
seen on areas of chronic sun –exposure especially on
dorsum of hands, not associated with any hair type, Q. What are the clinical features of Fox- Fordyce
more common in type 1 and type 2 skin, there is disease?
history of repeated sunexposure, are persistent for A. It is characterized by conical, flesh-coloured or
life, may darken in summer and not hereditary . grayish, intensely pruritic, discrete follicular papules
Q. What is the histology of ephelides and does it differ in areas where apocrine glands occur. The axillae and
from lentigo ? areolae are the primary sites of involvement but the
umbilicus, pubis, labia majora and perineum may
A. . The ephelides show increased production of
be affected. Hair density over affected area is affected
melanin pigment by a normal number of
melanocytes which are larger in size and have h
more branching dendrites . Epidermis is normal
showing increased pigmentation in the basal
layer. In lentigo,s number of melanocytes are
increased and rete ridges are broader and deeper.
Q. What is the prevention and treatment for freckles?
A. - Appropriate sun protection
- Topical retinoids, hydroquinone, cryotherapy
and pulsed- dye lasers are effective in ephelides
h

443
DYP SURVIVAL GUIDE FOR POST GRADUATES 445

to erythematous papules arranged in an annular


31. GRANULOMA ANNULARE fashion, centre of the lesion showing slight
atrophy.
Q. Definition?
- Inverse Koebnerization positive (Injury causing
A. Its a necrobiotic disorder characterized by healing).
degenerative changes in the dermis which evokes a
granulomatous response leading to a characteristic - Average duration about 3 months to 2 yrs
pathological appearance Q. Clinical variants?
Q. Pathogenesis? A. Following variants are seen:
A. Believed to be a delayed hypersensivity reaction to a) Giant form
various etiological triggering factors b) Targetoid form
Q. Various etiological factors? c) Perforating granuloma
A. Following factors have been implicated: d) Disseminated form
a) Trauma e) Generalised GA
b) Sunlight f) Subcutaneous GA
c) At sites of insect bite g) Malignancy associated GA
d) Various drugs Q. Associated condition?
e) Diabetes mellitus A. DM, AIDS, internal malignancies, Mauriac’s
f) Familial syndrome, Necrobiosis Lipoidica, Sarcoidosis,
g) BCG vaccination Mycosis fungoides
h) Various auto-antibodies (antithyroid antibodies, Q. Differential diagnosis?
antinuclear antibodies) A. Following conditions should be ruled out:
i) Viral infection- HIV a) Tuberculoid leprosy
Q. Incidence? b) Lupus vulgaris
A. - Rare disorder c) Sarcoidosis
- More in children d) Gumma
- But all age groups can be affected e) Tinea circinata
- In adults, more common in females f) Erythema multiforme (for targetoid form)
Q. Classical lesion? g) Rheumatic nodules (for subcutaneous GA)
A. - Asymptomatic or mildly pruritic, single or h) Lymphoma cutis (for disseminated GA)
multiple, well defined closely set skin coloured
444
446 DYP SURVIVAL GUIDE FOR POST GRADUATES

Q. Treatment?
32. HAEMANGIOMA’S
A. Following treatment options:
K.a skin biopsy can cause healing of lesion (inverse Q. Which is the most common benign vascular
koebnerization). neoplasm of childhood?
a) Corticosteroids A. Haemangioma of infancy (HOI-benign tumour of
b) Etretinate capillary endothelium)
c) Pentoxyfylline Q. Prevalence of haemangioma
d) Potassium iodide A. 3-10% in caucasians
e) Alkylating agents Q. Clinical subtypes of HOI ?
f) Chloroquine A. – Superficial (strawberry) 50-60%
g) Niacinamide – Deep (sub cutaneous) 15%
h) Cryotherapy – Mixed 25%
i) Photochemotherapy Q. What can a superficial HOI mimick?
j) Lasers A. Capillary malformations
k) Skin biopsy can cause healing of lesion (Inverse Q. What is the appearance/colour of superficial HOI?
Koebnerization)
A. Bright red colour with finely lobulated surface
h Q. What is the appearance of a deep HOI?
A. Soft skin coloured to blue nodule with scattered
telangiectasias
Q. Are infantile haemangiomas present at birth?
A. Usually absent at birth, precursor lesions may be
present at birth
Q. How do the precursors look?
A. They can have a bruised appearance or can look like
nevus anemicus or red telengiectatic patch
Q. HOI more common in girls or boys?
A. - Girls.
- Ratio of males: females is 1:3
447
448 DYP SURVIVAL GUIDE FOR POST GRADUATES DYP SURVIVAL GUIDE FOR POST GRADUATES 449

Q. Problem with haemangioma on beard area Q. Name complications of haemangiomas?


A. It is associated with laryngeal haemangiomas. A. 1. Obstruction of vital functions – seeing (vision),
Q. HOIs more common in which infants? breathing, eating, urinating
A. - Premature infants 2. Ulceration followed by thrombosis, bleeding and
infection
- Infants of mothers who undergo chorionic villus
sampling 3. High output cardiac failure
Q. Histopathology of proliferating type? 4. Mutilation and scarring
A. - endothelial cell hyperplasia Q. Most common complication?
- lobule formation A. Ulceration in diaper area
- mast cells Q. What is Kasabach Meritt Syndrome?
- prominent basement membrane A. Platelet trapping and consumption coagulopathy,
both associated with vascular tumour
- perineural invasion
Q. Two vascular tumours associated with KMS (most
Q. Histopathology of involuting type?
commonly)?
A. - fibrofatty tissue replacement
A. Kaposiform Haemangioendothelioma and tufted
- decreased mast cells angioma
Q. How does a haemangioma appear during Q. Port wine stains and lymphangiomas are not
involution? neoplasms. What are they?
A. Dull red or violaceous or grey colour and softening A. Vascular malformations
of haemangioma
Q. Are vascular malformations present at birth?
Q. How do residuas of haemangioma appear?
A. Yes, usually evident at birth
A. 50% have normal skin; the other 50% show
Telangiectasias, atrophy, hypopigmentation Q. Do vascular birth marks involute or persist?
fibrofatty deposition and if ulceration occurs then A. - HOI : spontaneous involution over years
scarring - Vascular malformations : persist into childhood
Q. What are the segmental patterns of haemangiomas? and adulthood
(Topographical classification) Q. Where and in whom are cherry angiomas most
A. - S1 : Frontotemporal commonly seen?
- S2 : Maxillary prominence A. Cherry angiomas : 1.5 mm red to purple papules
- S3 : Mandibular located primarily on the trunk and upper extremities
commonly seen in the middle aged and elderly
- S4 : Frontonasal
450 DYP SURVIVAL GUIDE FOR POST GRADUATES DYP SURVIVAL GUIDE FOR POST GRADUATES 451

Q. Where do you find venous lakes? Q. What is the most common presenting feature of
A. Dark blue slightly raised papules on sun exposed pyogenic granuloma?
skin surfaces of elderly patients most commonly on A. PGs are 5-10 mm soft red papules that bleed easily
ears, lips and face with minor trauma Most commonly seen on skin;
Q. Difference between vascular malformation and a may also occur on mucous membranes or rarely
vascular neoplasm? blood vessels.
A. - Vascular malformation: developmental error and Q. What is the granuloma gravidarum?
present at birth; static, grow proportionately with
A. A variant of PG that occurs on gingiva during
the patient
pregnancy.
- Vascular neoplasm : tumour, start few days after
birth, grow at a faster rate than the patient does Q. What are blue black hyperkeratotic vascular
papules?
Q. What is Klippel Trenaunay Syndrome?
A. Angiokeratomas
A. Overgrowth of an extremity due to vascular vascular
malformation : Capillary (port wine stain), varicose Q. Types of angiokeratomas
veins and soft tissue and / or bony hypertrophy of A. – Localized angiokeratomas – usually solitary
extremity found on extremity
Q. What is Klippel Trenaunay Webber syndrome? – Angiokeratoma circumscriptum – unilateral
A. Same as KTS but with arteriovenous malformation plaques on extremity present at birth
Q. What is the biological classification of vascular birth – Angiokeratoma of Mibelli – dorsal surface of
marks?
hands and feet develops in childhood or
A. adolescence
VASCULAR TUMOURS VASCULAR
MALFORMATIONS – Angiokeratoma of Fordyce – common on scrotum
- Infantile haemangioma - Capillary malformations – Angiokeratoma corporis diffusum (Fabry’s
(slow flow) disease)
- Congenital haemangioma
- Venous malformations (slow
- Kaposi form flow)
– not linked recessive disease, resulting from
haemangioendothelioma - typical / glomus cells
deficiency of lysosomal enzyme and
- Tufted angioma galactosidase. Accumulation of glycolipids in
- arterial malformations
- Spindle cell haemangioma cells of various tissues
- lymphatic malformations
- Pyogenic granuloma (slow flow) Q. Which benign acquired vascular disease is often
- Congenital - macrocystic / microcystic initially confused with Kaposi’s sarcoma?
haemangiopericytoma - A V malformations (fast flow) A. Acroangiodermatitis (gravitational purpura) –
- Combi malformations (slow eruption of purple macules, papules, plaques;
flow) usually associated with chronic venous insufficiency
452 DYP SURVIVAL GUIDE FOR POST GRADUATES DYP SURVIVAL GUIDE FOR POST GRADUATES 453

Q. What vascular tumour is associated with Sucquet Q. What is PELVIS syndrome?


Hoyer Canal? A. Large Perianal haemangioma, External genitalia
A. SHC – arterial segment of glomus body and may give malformations, Lipomyelomeningocoele, Vesicorneal
rise to glomus tumours abnormalities, Imperforate anus, Skin tags
Q. Where are angiolymphoid hyperplasia with Q. What is RICH and NICH?
eosinophilia lesions seen most commonly? A. RICH : Rapidly Involuting Congenital
A. Clusters of grapes – head and neck (red brown Haemangiomas
papules) NICH : Non- Involuting Congenital
Q. Which is the most prevalent tumour of infancy? Haemangiomas
A. Haemangiomas of infancy Q. What is local treatment?
Q. Histopathology of vascular malformation A. ILS steroid over 3 months under short GA only for
A. Superficial and deep blood vessel dilation with localised HOI- 50% efficacy
different contours of lumen Q. Most severe side effect of Interferon a
Q. Histopathology of hobnail haemangioma A. Spastic Paralysis 10-30%
A. - Dilated blood vessels Q. Commonest side effect of Interferon a
- Big endothelial projection proceeds into vessel A. Fever, flu like symptoms (+hemat /hepat toxicity)
Q. Treatment of haemangiomas Q. Which laser works in haemangioma
A. - Beta blockers: Timolol A. Pulsed dye : Superficial – Yes, Ulcerated – Yes, Deep
- Steroids: intralesional -localised lesions – No, Residual telangiectasia – Yes
Oral- large life threatening lesions Q. Mechanism, dose and side-effects of vincristine
- Lasers: pulsed-dye - Vincristine – interferes with mitotic microtubules
tumor cell apoptosis
- Excision
- Severely complicated, HOI not responding to
- Interferon a 3 million units
steroids
- Imiquimod
- I.V. weekly 0.5 mg/kg for 15 weeks
- Sclerosing agents (hypertonic saline)
- 1 mg/m2for children
Q. What is PHACES syndrome?
- Nearly 100% efficacy, regression in 3 weeks from
A. Post cranial fossa abnormalities, Haemangioma starting treatment
(segmental), Arterial abnormalities, Coarctation of
- Side effects – Fatigue, alopecia, haematological
aorta, Eye abnormalities, Spinal agensia
toxicity, peripheral neuropathy
454 DYP SURVIVAL GUIDE FOR POST GRADUATES DYP SURVIVAL GUIDE FOR POST GRADUATES 455

Q. What is the first line of treatment? Q. Will you give propranolol in fibrofatty
A. • 0.5% timolol drops twice daily apply for 14 days haemangiomas?
- Oral beta blockers can also be given A. No- only laser or excision
- Vasoconstriction of microvessels of the Q. What is SACRAL?
haemangioma A. Spinal dysraphism
- Rapid change in colour, softening Anogenital anomalies
- Lowers the rate of renin Cutaneous abnormalities
- Inhibits vasodilation causes relative Renal and urologic anomalies
vasoconstriction A and L Angioma of lumbo sacral localization
- Even works for liver haemangiomas Q. What is LUMBAR?
- Side effects – Bradycardia, hypoglycemia, A. Lower body haemangiomas
diarrhea, nightmares
Urogenital abnormalities; ulceration
• Prednisolone: 2.5 – 5 mg/day for 2 months taper
Myelopathy
slowly to cover involuntary phase
Bony deformities
• Cause vasoconstriction and saturate estrogen
receptors Anorectal malformations; arterial anomalies
Q. When will you give interferons? Renal anomalies
A. Severely complicated, H not responding to steroids. Q. What has to be done especially in perineal HOI?
Interferon a – 2A + 2B (subcut) – anti angiogenic A. If patient has perineal spreading haemangiomas, do
Pelvic MRI, spinal MRI even with no neurological
1-3 million units/m2/day (no blood vessel formation)
symptoms
Fibroblast growth factor reduces – 6-12 months
h
Q. What are the indications for surgery?
A. - Pedunculated HOI, over vital areas
- Bleeding or ulceration
- Non regressing haemangiomas
- Laryngeal HOI
Q. When do you do late surgery?
A. To repair post regression residue
DYP SURVIVAL GUIDE FOR POST GRADUATES 457

Q. What are the complications of Hailey – Hailey?


33. HAILEY - HAILEY DISEASE
A. Infections – secondary bacterial infections, fungal and
viral infections (Kaposi’s varicelliform eruption)
Q. Why is it a case of Hailey – Hailey ?
There are a few case reports of squamous cell
A. Lesions start as flaccid vesicle which ruptures and
carcinoma.
gives rise to crusted erosions. Lesions are present in
intertriginous areas (axilla and groins), perianal Q. What is the histopathology of Hailey – Hailey ?
region. Malodourous vegetations are present. Nail A. There is extensive but incomplete acantholysis in
involvement in form of longitudinal leuconychia. epidermis giving‘ Dilapidated brick wall’
Lesions heal with hyperpigmentation. No scarring appearance. Dermal papillae lined by single layer of
present. Pruritus-moderate to severe. basal cells protrude into blister cavities and are
Q. What is the differential diagnosis of Hailey – Hailey referred to as ‘villi’. There is moderate perivascular
disease? infiltrate present in the superficial dermis.
A. - Intertrigo Q. Which other diseases have flexural involvement?
- Candidiasis A. Acanthosis nigricans, erythrasma, intertrigo,
Dowling-Degos disease, inverse psoriasis,
- Inverse psoriasis (it has sharper borders, fewer
pemphigus vegetans, tinea infections.
erosions, less crusting)
Q. What is the course of Hailey – Hailey?
- Pemphigus vegetans
A. The course is characterized by exacerbations, more
- Irritant contact dermatitis
in summer and remissions. Lesions rarely, subside
Q. What is the other name of Hailey – Hailey ? on their own. The severity of the disease decreases
A. Familial benign chronic pemphigus with increasing age.
Q. Who were Hailey and Hailey? Q. How will you manage a case of Hailey – Hailey?
A. Two brothers Howard and Hugh Hailey who A. · General measures :
described the disease first. - Patient is advised to wear lightweight clothes
Q. Why do you get Hailey – Hailey ? to avoid friction and sweating
A. It is an autosomal dominant disorder with mutation - Treatment of secondary infections
in ATP2C1 gene resulting in Golgi associated · Topical therapy :
Calcium ATPase thus interfering with intracellular
- Topical corticosteroids with topical
signalling.
antimicrobials
Q. What are the exacerbating factors?
- Intralesional corticosteroids if lesions are
A. - Infections - heat refractory to topical preparation.
- sweating - Friction - Tacalcitol
456
458 DYP SURVIVAL GUIDE FOR POST GRADUATES

· Systemic therapy:
34. HALO NEVI
- Broad spectrum antibiotics, dapsone,
retinoids, methotrexate
Q. What is a halo nevus?
· Surgical Management:
A. It is a nevus with distinguishing feature of a
- Dermabrasion or vapourizaion of epidermis surrounding hypochromic or acromic feature.
with CO2 or erbium: YAG laser can be done
Q. What are the synonyms of halo nevus?
- It is done in cases not responding to general
and topical measures A. It is also known as Sutton nevus, Peri-nevoid vitiligo
and leukoderma aquisitum centrifugum.
These approaches remove diseased epidermis and
their dermal niche of fibroblasts to level of mid Q. Describe the lesion of halo nevus.
dermis. Re-epithelisation (from spared adnexal A. The lesions are characterized by pigmented nevus,
structures ) occurs within 7 to 14 days. with a surrounding depigmented zone. Halo nevi
h tend to occur most frequently on the trunk, mostly in
teenagers. The central nevus gradually loses its
pigmentation, turns pink and then disappears. Over
time, the area repigments. Darkening of the central
nevus than lightening can be seen in association with
halo phenomenon.
Q. What are the lab diagnosis seen in halo nevus?
A. The infiltrate contains cytotoxic T cells, and may
represent immunologically induced rejection. The
peripheral blood has also been shown to contain
activated adhesive lymphocytes that disappear when
the lesion is excised. Antibodies against melanocytes
and cell mediated immunity to melanoma has also
been demonstrated.
Q. What are the other lesions that can be seen with
leukoderma?
A. Blue nevi and Neurofibroma.
Q. What is the histology of halo nevi?
A. It demonstrates band of lymphocytes that extends
throughout the lesion, intimately mingling with the
459
460 DYP SURVIVAL GUIDE FOR POST GRADUATES

melanocytes. The lymphoid infiltrate associated with


melanoma tend to aggregate at the periphery of the
35. HENOCH–SCHÖNLEIN
lesion. Term Meyerson’s nevus has been applied to PURPURA (HSP)
eczematous change associated with a nevus.
Hypopigmentation may be present. Q. Why is this HSP?
h A. Palpable purpura over the lower limbs and buttocks
associated with abdominal pain, diarrhea, and
polyartharlgia
Q. D/D of HSP?
A. · Subsiding drug rash
· Erythema multiforme
· Subsiding erythema nodosum
· Viral exanthem
· Idiopathic thrombocytopenic purpura
· Disseminated Intravascular Coagulation
· Subacute Bacterial Endocarditis
Q. What is henoch’s purpura?
A. Purpura with abdominal pain
Q. What is schonlein’s purpura?
A. Purpura with arthritis
Q. What is HSP?
A. It is small vessel vasculitis, involving skin, joint,
gastrointestinal tract and kidneys. There is IgA
mediated vasculitis
Q. Triad of HSP?
A. Palpable purpura, joint pain and abdominal pain
Q. What are the clinical features of HSP?
A. · Common in children in age group of 5 to 15 yrs
· Palpable purpura over the lower limbs and
buttocks. Urticarial wheals, necrotic, bullous and
461
462 DYP SURVIVAL GUIDE FOR POST GRADUATES DYP SURVIVAL GUIDE FOR POST GRADUATES 463

hemorrhagic lesions also occur. Skin lesions heal - Vaccination: typhoid, cholera, measles, yellow
in 5 days. Associated with abdominal pain, fever
diarrhea and polyarthralgia. Edema of the hands, - Drugs: penicillin, aspirin, macrolides
feet, scalp, and ears
Q. What are other names of HSP?
· Arthritis, most commonly involving the knees
and ankles A. - Anaphylactoid purpura
· Abdominal tenderness - Purpura rheumatica
· Gastrointestinal bleeding Q. Histopathology of HSP?
· Acute scrotal edema that may mimic testicular A. - Leukocytoclastic vasculitis features
torsion - Small blood vessels (post capillary venules ) are
Q. What is the dreaded complication of HSP? infiltrated with neutrophils that show fragmented
A. Renal involvement in form of proteinuria, hematuria, nuclei
RBC casts - Swelling and fibrinoid degeneration of vascular
Q. Treatment of HSP? walls
A. - HSP is benign and self limiting - Extravasation of RBC
- Rest and observation is needed Q. What are complications of HSP?
- Dapsone 50 to 200 mg/day, colchinine 0.6 mg/ A. HSP can have complications, which generally occur
day, systemic steroids, IVIG are given for more frequently in children than in adults. These
refractory cases complications include severe abdominal pain and
gastrointestinal bleeding. Adults can have extended
- Non-steroidal anti-inflammatory drugs (NSAIDs)
kidney problems
may help joint pain and do not seem to worsen
the purpura. However, NSAIDs should be used Q. What is the prognosis of HSP?
cautiously in patients with renal insufficiency A. · Henoch-Schönlein purpura (HSP) is generally a
- For abdominal pain, H2 blockers or systemic steroid benign disease with an excellent prognosis
(prednisolone 1mg /kg.) cyclophosphamide · More than 80% of patients have a single isolated
and dipyridamole can be used episode lasting a few weeks
- Other regimens include steroids/azathioprine, · Approximately 10-20% of patients have
and steroids/cyclophosphamide (with or recurrences
without heparin and warfarin)
· Fewer than 5% of patients develop chronic
Q. What are the causes of HSP? Henoch-Schönlein purpura
A. - Infections: EBV, varicella virus, parvovirus B19, · Abdominal pain resolves spontaneously within
campylobacter, B hensalea 72 hours in most patients
464 DYP SURVIVAL GUIDE FOR POST GRADUATES DYP SURVIVAL GUIDE FOR POST GRADUATES 465

Q. What is the variant of HSP in infancy and early buttocks. Urticarial wheals, necrotic, bullous and
childhood? hemorrhagic lesions also occur. Skin lesions heal
A. Acute hemorrhagic edema of infancy (Finkelstein’s in 5 days. Associated with abdominal pain,
disease). Here there is facial edema and also targetoid diarrhea and polyarthralgia. Edema of the hands,
lesions on extremities feet, scalp, and ears
Q. Why is this HSP? · Arthritis, most commonly involving the knees
A. Palpable purpura over the lower limbs and buttocks and ankles
associated with abdominal pain, diarrhea, and · Abdominal tenderness
polyartharlgia · Gastrointestinal bleeding
Q. D/D of HSP? · Acute scrotal edema that may mimic testicular
A. · Subsiding drug rash torsion
· Erythema multiforme Q. What is the dreaded complication of HSP?
· Subsiding erythema nodosum A. Renal involvement in form of proteinuria, hematuria,
· Viral exanthem RBC casts
· Idiopathic thrombocytopenic purpura Q. Treatment of HSP?
· Disseminated Intravascular Coagulation A. - HSP is benign and self limiting
· Subacute Bacterial Endocarditis
- Rest and observation is needed
Q. What is henoch’s purpura?
- Dapsone 50 to 200 mg/day, colchinine 0.6 mg/
A. Purpura with abdominal pain day, systemic steroids, IVIG are given for
Q. What is schonlein’s purpura? refractory cases
A. Purpura with arthritis - Non-steroidal anti-inflammatory drugs (NSAIDs)
Q. What is HSP? may help joint pain and do not seem to worsen
A. It is small vessel vasculitis, involving skin, joint, the purpura. However, NSAIDs should be used
gastrointestinal tract and kidneys. There is IgA cautiously in patients with renal insufficiency
mediated vasculitis - For abdominal pain, H2 blockers or systemic steroid
Q. Triad of HSP? (prednisolone 1mg /kg.) cyclophosphamide
A. Palpable purpura, joint pain and abdominal pain and dipyridamole can be used
Q. What are the clinical features of HSP? - Other regimens include steroids/azathioprine,
A. · Common in children in age group of 5 to 15 yrs and steroids/cyclophosphamide (with or
· Palpable purpura over the lower limbs and without heparin and warfarin)
466 DYP SURVIVAL GUIDE FOR POST GRADUATES DYP SURVIVAL GUIDE FOR POST GRADUATES 467

Q. What are the causes of HSP? · Fewer than 5% of patients develop chronic
A. - Infections: EBV, varicella virus, parvovirus B19, Henoch-Schönlein purpura
campylobacter, B hensalea · Abdominal pain resolves spontaneously within
- Vaccination: typhoid, cholera, measles, yellow 72 hours in most patients
fever Q. What is the variant of HSP in infancy and early
- Drugs: penicillin, aspirin, macrolides childhood?
Q. What are other names of HSP? A. Acute hemorrhagic edema of infancy (Finkelstein’s
disease). Here there is facial edema and also targetoid
A. - Anaphylactoid purpura lesions on extremities
- Purpura rheumatica
h
Q. Histopathology of HSP?
A. - Leukocytoclastic vasculitis features
- Small blood vessels (post capillary venules ) are
infiltrated with neutrophils that show fragmented
nuclei
- Swelling and fibrinoid degeneration of vascular
walls
- Extravasation of RBC
Q. What are complications of HSP?
A. HSP can have complications, which generally occur
more frequently in children than in adults. These
complications include severe abdominal pain and
gastrointestinal bleeding. Adults can have extended
kidney problems
Q. What is the prognosis of HSP?
A. · Henoch-Schönlein purpura (HSP) is generally a
benign disease with an excellent prognosis
· More than 80% of patients have a single isolated
episode lasting a few weeks
· Approximately 10-20% of patients have
recurrences
DYP SURVIVAL GUIDE FOR POST GRADUATES 469

genital herpes) is less common , seroprevalence, as it


36. HERPES SIMPLEX VIRUS appears at the age of onset of sexual activity,
antibody to HSV2 rarely found prior to adolescence.
Q. Enumerate all the herpes viruses. Q. Epidemiology of HSV infection?
A. ALPHA HERPESVIRIDAE: A. Children less than the age of 10 years get
- HSV-1, HSV-2, Varicellovirus(HSV-3) asymptomatic infection of HSV1 type (80-90%). 90%
GAMMA HERPESVIRIDAE: of adults have HSV1 (20 -40 Years) and 70-90% of
genital herpes infection in HSV2.
- EBV(Lymphocryptovirus ; HSV-4), HSV-8
Q. What is the mode of HSV transmission?
BETA HERPESVIRIDAE:
A. HSV1 infection occurs through contaminated saliva
- Cytomegalovirus (HSV-5), HSV-6, HSV-7 and through direct contact and HSV2, through sexual
Q. What are the clinical features of herpes simplex contact. Virus replicates at the site of infection (
infection? mucocutaneous) travels to dorsal root ganglia and
A. The characteristic lesions of herpes infection are establishing latency until reaction.
grouped vesicles on an erythematous base filled with Q. What is primary HSV infection?
clear fluid which break down to form superficial A. Primary infection refers to individuals who are
erosions with characteristic scalloped borders. exposed to the HSV virus for the 1st time, either type
Sometimes only erosions or ulcerations are seen 1 or 2 at any site. these patients are seronegative
especially in herpetic gingivostomatitis initially but subsequently develop HSV-specific
Q. What is the differential diagnosis of herpes antibodies.
infection? Q. What about recurrent infection?
A. Genital herpes has to be differentiated from chancre, A. Recurrent HSV infection represents reactivation of the
chancroid, mucus membrane pemphigus, candidiasis latent virus in the sensory ganglia “reactivated” virus
particles migrate back along the nerves to the site in
Oral herpes has to be differentiated from erosive or the skin where the primary infection occurred, with
ulcerative lichen planus, erythema multiforme, subsequent viral replication and the development of
streptococcal infections, mucus membrane the clinical lesions. The most common sites for
pemphigus, aphthae, oral candidiasis, behcets recurrent herpes simplex infections are the
disease. lip(herpeslabialis,coldsores),genitalial and sacral area.
Q. What are the type of HSV virus ? Q. What is the difference between a primary and an
A. Types of HSV viruses are HSV1 and HSV2. initial HSV infection?
HSV1(Herpes labialis) is more common ,30-95% A. When an individual without preexisting antibodies
adults are sero positive for HSV1 whereas HSV2( to either HSV1 Or HSV2 develops an infection with
468
470 DYP SURVIVAL GUIDE FOR POST GRADUATES DYP SURVIVAL GUIDE FOR POST GRADUATES 471

Herpes (either type 1or 2), it is referred to as primary adults and result from autoinoculation from another
infection.When an individual with preexisting site of infection. In children the etiologic agent is
antibodies to one type HSV then experiences an generally HSV1 while in adults its HSV2.
infection with the other HSV type, it is referred as the Q. What is kaposi’s varicelliform eruption?
initial (or initial, non primary) infection.
A. Eczema herpeticum, or Kaposi’s varicelliform
Q. Define asymptomatic shedding. eruption, is an extensive, disseminated cutaneous
A. In some cases recurrent infection, clinical lesions may infection with HSV occurs most commonly in patients
not be visible, but virus can be covered if the skin or with atopic dermatitis, although it may also be
mucosal site is cultured and the virus can be associated with other dermatoses such as dariers
transmitted to another person, the host immune disease, pemphigus, severe seborreheic dermatitis,
response is believed to eliminate the focus of virus and psoriasis. Antiviral therapy is beneficial in this
before full-fledged recurrence develops. Patient process.
should be counselled regarding this. Q. Can a baby get herpes? Is it a serious problem?
Q. How long is HSV incubation period, the time from A. Neonatal herpes is one of the most critical problems
initial infection to appearance of the vesicles? associated with the increased incidence of genetial
A. The time between exposure and development of herpes . Herpes infection is the neonatal period can
primary disease is estimated to be 3-14 days. be, and often is, devastating because of the
inadequate immune response seen in neonates. In
Q. How is Tzanck smear performed ?
most cases, transmission of HSV to the neonate occurs
A. In a tzanck smear, the base of the suspected herpetic by delivery through an infected birth canal.
lesion is gently scraped, and the skin or mucosal cells Postpartum acquisition occurs less commonly.
removed are placed on a glass slide. The cells are Development of primary or initial non primary
stained and then examined by light microscopy for genital herpes by the mother at or near the time of
evidence of viral –induced cytologic change, delivery poses a significant risk for the infant,
including the characteristics multinucleated giant however most cases of neonatal herpes are the result
cells.The technique cannot distinguish HSV from of asymptomatic shedding in women with no history
VZV. of genital herpes. The usual onset of neonatal herpes
Q. Who gets herpetic whitlow? is 2-12 days following exposure . Approximately 80%
A. Herpetic whitlow (HSV infection of the hand or of infected neonates have the least some characteristic
fingers) was previously considered a disease of health skin lesions.
care professionals, which occurred as a result of Q. What is herpetic sycosis?
contact with infected patients. In fact, as few10% of A. It is the infection of the hair follicle. There are few
individuals with this disorder fall in that category. eroded follicular papules or extensive lesions
The remaining cases are seen in children and young involving the beard area.
472 DYP SURVIVAL GUIDE FOR POST GRADUATES DYP SURVIVAL GUIDE FOR POST GRADUATES 473

Q. What is herpes gladiatorum? Herpes zoster


A. It is seen in wrestlers and boxers who are susceptible Q. Why herpes zoster?
to HSV1 infection and who come in contact with the
A. It is a localized eruption characterized by unilateral
infected individual with an active disease. The sites
are lateral side of the neck ,side of the face and radicular pain and vesicles on an erythematous base
forehead. Any wrestler with history of orolabial limited to the dermatome innervated by a single
herpes should be put on suppressive therapy. spinal or cranial nerve sensory ganglion.
Q. Treatment of primary episode of HSV Q. What is the differential diagnosis?
A. Acyclovir 200mg PO 5 times a day or 400mg tid for A. a) HSV infection
10 days. - h/o recurrent lesions at the same site
Valacyclovir 1gm PO bid for 10 days b) Contact dermatitis
Famcyclovir 250mg PO tid for 5-10 days
- DFA is negative and no multinucleated epithelial
Q. Treatment for recurrent episodes of HSV infection. giant cells on Tzanck smear
A. ACV 200 mg PO 5 times/dayor 400 mg tid for 5-7 c) Bacterial infection
days
- DFA is negative and no multinucleated epithelial
VAL 500 mg PO for 5 days
giant cells on Tzanck smear.
FAM 125 mg PO bid for 5 days
d) Insect bite and blister beetle bite(pedarus)
Q. What is the suppressive therapy for HSV infection? dermatitis
A. ACV 200-400 mg bd
Q. Causative organism?
VAL 500-1000 mg qd
A. It is caused by reactivation of varicella zoster virus.
FAM 125-250 mg bid
Q. What is the meaning of herpes and zoster?
Q. What is the treatment of HSV infection in
immunosuppresssed patients? A. Herpes - to creep and Zoster - girdle
Primary /initial infection ACV 5-10 mg/kg iv q 8 hrs. Q. How will you identify the level of involvement?
If not severe 400 mg PO 5 times/dsy for 7-14 days A. C4 - at the clavicle
VAL 1gm PO tid for 7 days T4 - at the nipple
FAM 500 mg PO bid for 7 days T10 - at the umbilicus
Suppressive therapy L1 - at the genitalia
ACV 400-800 mg PO tid Q. What is the etiopathogenesis?
VAL 1 gm PO tid A. During the course of the disease, varicella zoster virus
FAM 500 mg PO bid passes from the lesions in the skin and mucosal
474 DYP SURVIVAL GUIDE FOR POST GRADUATES DYP SURVIVAL GUIDE FOR POST GRADUATES 475

surface into ending of sensory nerves and is 2-3 days of thoracic disease. Closely grouped red
transported centripetally up the sensory fibers to papules, appearing in continuous or interrupted
sensory ganglia where it remains latent in ganglia. band, rapidly become vesicular and then
Subclinical reactivation occurs in immune- pustular. New vesicles continue to appear for
compromised individuals. Clinical manifestation days.
arises when the virus is reactivated. The newly - The mucous membrane of affected dermatome
synthesized virus is transported to the skin. There is is also involved. The lymph nodes in the affected
unilateral vesicular eruption of one or two area are also enlarged and tender.
dermatomes along distribution of sensory nerves.
- The area supplied by trigeminal area,
Reactivation is triggered by trauma, sunburn, stress
particularly the ophthalmic division, and trunk
and old age.
from T3 to L2 are mostly affected. The dermatome
Spread of infection along the posterior nerve root to involved are thoracic 53%, cervical 4-20%,
the meninges and the cord, results in local trigeminal including ophthalmic and lumbo-
leptomeningitis, cerebrospinal fluid pleocytosis and sacral 11%
segmental myelitis.
- Ramsay- Hunt syndrome: from reactivation of
Q. What are the clinical manifestations? the infection of geniculate ganglion leading to
A. - Prodromal symptoms such as fever, malaise, involvement of facial and auditory nerves
paresthesia, dysesthesia before lesion erupt. leading to facial palsy in the combination of
- The pre- eruptive pain of herpes zoster simulates vesicles in external ear with or without tinnitus,
pleurisy, myocardial infarction, duodenal ulcer, tongue ( causes taste loss in anterior 2/3rd) and
cholecystitis. Pruritus, tenderness, tingling or hard palate, dry mouth and eyes, vertigo and
hyperesthesia are also seen. deafness.
- A few patient experience acute segmental - Immuno-compromised patients may have usual
neuralgia without ever developing a cutaneous course of disease or they may develop recurrent
eruption.(zoster sine herpete). This is common and severe disease which is prolonged and
with HIV patients. atypical, eg. hyperkeratotic papules, bullous,
ecthymatous lesions, lichenoid papules and
- The rash is usually unilateral, dermatomal and
follicular papules. Disseminated lesions (defined
does not cross the midline (but about 1 finger
as more than 20 lesions outside the area of
overlap can be seen in midline due to
involvement) or visceral involvement is seen in
overlapping of nerves).
10% of patients.
- The time between start of pain and onset of rash
- Chronic herpes zoster is characterized by long
averages 1-3 days in trigeminal herpes zoster and
standing single or multiple pox- like lesions with
476 DYP SURVIVAL GUIDE FOR POST GRADUATES DYP SURVIVAL GUIDE FOR POST GRADUATES 477

periods of exacerbation and regression. It is The pain has 2 forms: a continuous and spasmodic
usually associated with acyclovir resistance. shooting type. The healed lesions may become
- Transplacental transmission of varicella zoster hyperaesthetic, causing severe pain on light touch and
virus does not occur with maternal herpes zoster. even on contact with clothes (allodynia). In addition
to pain, patient may complain of severe pruritis,
Q. What are the complications associated with herpes
dysesthesia, anesthesia.
zoster?
Q. What are the ocular complications?
A. Most patients recover from it without any
complication. A. Involvement of nasociliary branch of fifth cranial may
lead to ocular complication. This may be suspected
- The complication may be cutaneous, ocular,
when Hutchinson’s sign is positive, which is
visceral and neurologic.
involvement of the tip or the sides of nose with herpes
- The cutaneous complications include bacterial zoster. Corneal damage and retinal artery occlusion
infection , scarring, Ramsay-Hunt syndrome, leading to blindness.
zoster gangrenosum and cutaneous
Q. What are the dermatological complications?
dissemination
A. Bacterial infection in affected dermatome is the
- Visceral complication such as pneumonitis,
commonest complication. Necrotizing fasciitis,
hepatitis, eosphagitis, pericarditis, gastritis,
herpes gangrenosum, keloid and pigmentary changes
cystitis, arthritis.
of dermatome are also seen.
- Neurological complications include post-
Q. How do you diagnose herpes zoster?
herpetic neuralgia, meningo-encephalitis,
transverse myelitis, peripheral cranial nerve A. History, physical examination and Tzanck smear
palsies, deafness and ocular complications. (multinucleated giant cell) are usually done. DFA,
viral culture, serology, PCR and biopsy may also be
- Multidermatomal, disseminated or trigeminal
done to rule out herpes zoster.
herpes zoster tend to have higher complication
rate. Q. What is treatment of herpes zoster?
Q. What is post herpetic neuralgia? A. Antiviral agents like Acyclovir 800 mg five times a
day, valacyclovir(1 gm thrice a day) or famcyclovir
A. It’s the commonest complication defined as
500mg tds for 7-10 days.
recurrence or persistence of pain in the affected area
for a month after the lesion have healed. It frequently, The duration of treatment is longer in immune
affects patients above 60 years. Other risk factors are compromised patient and depends on clinical
presence of severe pain, greater rash severity. It response. The drug should be administered within
remits spontaneously. 48 hours to 72 hours after appearance of rash.
478 DYP SURVIVAL GUIDE FOR POST GRADUATES

For post herpetic neuralgia, temporary relief by


application of cold water or ice cubes. Topical
37. HIDRADENITIS
lignocaine patches or local infiltration of lignocaine, SUPPURATIVA
topical capsaicin, gabapentin, pregabalin, oral
analgesics and anti inflammatory drugs may provide Q. What is hidradenitis suppurativa?
relief in mild to moderate cases. In severe or persistant
pain, systemic therapy with tricyclic antidepressants A. It is a chronic inflammatory disease believed to be
like amytriptyline 25-75 gm day, antiepileptics like due to occlusion of hair follicles (not of apocrine
carbmazepine(600mg-1000g) or phenytoin(300mg glands) characterized by double (sister) comedones,
day). Combination of antiepileptics and deep sinuses and abscesses and clinically
antidepressants are also useful. The live attenuated characterized by painful subcutaneous nodules. It is
varicella vaccine has been approved by FDA as a more common in females and improves in pregnancy
prophylaxis for persons of 60 years and above without Q. What are the other names of HS?
history of herpes zoster. A. Apocrine acne, acne inversa, apocrinitis, Verneuil’s
disease, pyoderma fistulans significa
h
Q. Differential diagnosis
A. Chronic furunculosis
Scrofuloderma
Cutaneous Crohn’s disease
Mycetoma
Donovanosis
Elephantiasis nostras verrucosa cutis
Q. Can HS be seen in children?
A. HS starts at or after puberty. Children not affected
unless they develop precocious puberty.
Q. What are the sites of HS?
A. Axilla, groins, inframammary, inguinal, perianal,
peri-areolar and posterior auricular.
Q. What is Hurley’s classification?
A. - Stage 1: abscess formation but without sinuses
or scarring.
479
480 DYP SURVIVAL GUIDE FOR POST GRADUATES DYP SURVIVAL GUIDE FOR POST GRADUATES 481

- Stage 2: one or more widely separated abscess • Approach:


with sinuses and scarring. - Stage1: systemic antibiotics + ILS
- Stage 3: multiple interconnected abscess with - Stage 2: long term immunosuppressive
sinuses involving the entire area. therapy and limited surgical treatment
Q. What is follicular occlusion triad and tetrad? - Stage 3: wide surgical excision of skin,
A. - Triad : Hidradenitis suppurativa, acne radiation therapy and immunosuppressive
conglobata and dissecting cellulitis of the scalp therapy
- Tetrad : HS, acne conglobata, dissecting cellulitis h
of scalp and pilonidal sinus.
Q. What are the conditions associated with HS?
A. Common : Obesity, Crohn’s disease
Rare : Acanthosis nigricans, Fox Fordyce, KID
syndrome, pachyonychia congenita, Dowling Degos,
SAPHO and PAPA syndrome
Q. What are the complications of HS?
A. Strictures, fistulae, scarring, anaemia, malnutrition,
cutaneous SCC, sacral osteomyelitis, lumbosacral
abscess and cutaneous amyloidosis.
Q. What is the treatment of HS?
A. Topical: topical antibiotics like clindamycin
Systemic treatment: oral tetracycline, rifampicin,
minocycline, dapsone, isotretinoin, acitretin(high
doses), cyproterone acetate and ethinyl estradiol,
finasteride (high doses), low dose steroids,
cyclosporine, biologicals like infliximab, eternacept,
botulinum toxin, radiotherapy and light therapy
Surgical:
a) Radical excision: perianal and axillary areas
shows good results
b) Incision and drainage: opening of sinuses and
allowing them to heal by secondary intention
DYP SURVIVAL GUIDE FOR POST GRADUATES 483

e) Connective tissue disorder: SLE, dermatomyositis


38. ICHTHYOSIS f) Granulomatous disorder: Sarcoidosis
g) Drug induced: Statins, clofazimine, cimetidine
Q. Definition?
Q. Name ichthyosiform syndromes?
A. Disorder of keratinisation resulting from abnormal
epidermal differentiation or metabolism A. BIDS syndrome, KID syndrome, CHILD syndrome,
Netherton’s syndrome, Refsum’s disease
Q. Word derivation?
Q. Differentiate between three major types of
A. Ichthys = fish (greek); because disease is
congenital ichthyosis?
characterized by fish-like scales
Q. Other name for ichthyosis? A.
Fe ature Ichthyosis X-linked Lamella r
A. Saurosis Vulgaris Ichthyosis Ichthyosis
Q. Classification: Pathogenesis: Histidine Steroid sulfatase Tg 1
reduced, deficiency, mutation,
A. a) Congenital ichthyosis defective filaggrin corneocyte deficient cell to
retention cell cross linking
b) Acquired ichthyosis
Onset: After 3 months of Within 3 months At birth
c) Ichthyosiform syndromes life of life
Q. Classify Congenital ichthyosis? Sites Extensors more Flexors involved Flexors, palms
involved: involved and soles
A. Classified as follows: Type of scale: Dry, fine, pasted Smaller darker Large
a) Autosomal dominant : on large scales Scales, dirty quadrilateral,
neck appearance free at edges,
a. Ichthyosis vulgaris stuck in centre
b. Bullous ichthyosiform erythroderma Associated/ Hyperkeratosis of Extra cutaneous Dry shiny facial
special palms and soles, features like skin, ectropion,
c. Ichthyosis bullosa of Siemens feature: Associated cryptorchidism, sometimes
keratosis pilaris corneal maceration
b) Autosomal recessive : opacities over flexures,
a. Lamellar ichthyosis nails involved
Course/ Improves with Does not Remains
b. Non - bullous ichthyosiform erythroderma Prognosis: age improve with lifelong
age
c) X-linked :
Histopathology: Hyperkeratosis, Stratum Hyperkeratosis,
a. X-linked recessive ichthyosis follicular corneum hypergranulosis,
Q. Classify acquired ichthyosis: plugging, thickening, mitotic figures
hypogranulosis hypergranulosis
A. a) Malignancy associated: Hodgkin’s lymphoma, infiltrate
multiple myeloma, solid organ malignancy
Treatment: Emollients, urea, Oral retinoids, MTX,
b) Infections: Leprosy, AIDS salicylic acid, emollients, urea, isotretinoin,
ammonium propylene glycol calcipotriol,
c) Metabolic: Malnutrition, renal failure lactate salicylic acid,
d) Endocrine disorders: Diabetes, hypothyroidism emollients

482
484 DYP SURVIVAL GUIDE FOR POST GRADUATES

Q. Synonym of Bullous ichthyosiform erythroderma?


39. IDIOPATHIC GUTTATE
A. Epidermolytic hyperkeratosis
Q. BIDS syndrome? HYPOMELANOSIS
A. B – Brittleness of hair
Q. What is idiopathic guttate hypomelanosis?
I – Ichthyosis
A. It is a common acquired condition in among elderly
D – Decreased fertility
of both the sexes most common in females,
S – Short stature
characterized by asymptomatic hypomelanotic to
Q. KID syndrome? amelanotic macules all over the body. It is common
A. K – Keratosis sign of aging.
I – Ichthyosis Q. What is the other name for idiopathic guttate
D – Deafness hypomelanosis?
Q. CHILD syndrome? A. Leukopathia symmetrica progressiva
A. C – Congenital Q. Describe the clinical appearance of idiopathic
H – Hemidysplasia guttate hypomelanosis.
I – Ichthyosiform erythroderma A. Individual lesions are small hypopigmented to
L – Limb depigmented macules averagely 2 to 5 mm in size
D – Defects distributed chiefly on the shin and forearms. The
average number is between 10 to 30. They spare face
h and trunk regions. The lesions are irregularly shaped
and very sharply defined like depigmented ephilides.
Q. What is the histology of idiopathic guttate
hypomelanosis?
A. Epidermal atrophy and reduced number of
hypoactive melanocytes.
Q. What are the differential diagnosis for idiopathic
guttate hypomelanosis?
A. - Pityriasis versicolor
- Post inflammatory hypopigmentation
485
486 DYP SURVIVAL GUIDE FOR POST GRADUATES

Q. What are the factors responsible for idiopathic 40. INCONTINENTIA PIGMENTI
guttate hypomelanosis?
A. - Chronic sun exposure Q. What is Incontinentia pigmenti also known as?
- Genetic influence A. Bloch–Sulzberger syndrome; Bloch–Siemens
- Trauma syndrome
Q. What is the pathogenesis of idiopathic guttate Q. What is Incontinentia pigmenti?
hypomelanosis? A. X-linked dominant disease, lethal in male foetuses
A. - DOPA reaction is decreased or absent and is caused by mutations in the NEMO gene (NF-
kB essential modulator) on Xq28
- Deficient epidermal melanin
Q. Describe the cutaneous lesions of Incontinentia
- Decreased number of melanocyte and pigmenti?
incompletely melanised melanosomes have
A. Cutaneous lesions tend to follow the lines of
been seen.
Blaschko, and occur in four classic phases:
Q. What is the treatment for idiopathic guttate 1] VESICULAR STAGE: yellow or clear vesicles in
hypomelanosis? linear and whorled streaks, most prominently
A. – Superficial dermabrasion distributed on the lower extremities
– Cryotherapy 2] VERRUCOUS PHASE: hyperkeratotic linear
plaques
h 3] LINEAR & WHORLED HYPERPIGMENT-
ATION: occurs most often on the trunk. There
are lines of reticulate hyperpigmentation, the
scalloped edges of the lesions are consistent with
growth of normal keratinocytes into the damaged
areas
4] ATROPHIC, HYPOPIGMENTED THIN
STREAKS: may persist indefinitely
Q. What are the other cutaneous lesions of Incontinentia
pigmenti?
A. 1] Patchy scarring alopecia
2] Woolly hair nevi
487
488 DYP SURVIVAL GUIDE FOR POST GRADUATES DYP SURVIVAL GUIDE FOR POST GRADUATES 489

3] Nail dystrophy Q. How will you manage Incontinentia pigmenti?


4] Anhidrosis in atrophic streaks A. The cutaneous lesions generally do not require
5] Asymmetric breasts and supernumerary nipple therapy.Treatment is symptomatic. All affected
infants should be referred for a baseline
Q. What are the extracutaneous manifestation of
ophthalmologic examination and should have
Incontinentia pigmenti?
periodic neurodevelopmental and dental evaluation.
A. 1] Dental abnormalities: partial anodontia and
conical or peg-shaped teeth h
2] CNS abnormalities: seizures, delayed
psychomotor development, mental retardation
and spastic hemiplegia
3] Ocular disease: retinal vascular abnormalities,
microphthalmia, pseudoglioma, cataract, optic
atrophy and eventually blindness
4] Skeletal: Skull anomalies, Scoliosis
5] Pulmonary hypertension
Q. What is the pathology of Incontinentia pigmenti?
A. Stage 1 : Eosinophilic spongiosis and scattered
dyskeratotic keratinocytes
Stage 2: The epidermis is acanthotic with
hyperkeratosis and foci of dyskeratosis
Stage 3: Pigmentary incontinence
Stage 4: Thinned epidermis and dermis devoid of
appendages
Q. What are the differential diagnosis of Incontinentia
pigmenti?
A. 1] Neonatal Incontinentia pigmenti- infectious
causes (e.g.herpes zoster, varicella or Herpes
simplex or other viral infections)
2] Linear and whorled nevoid hypermelanosis and
hypomelanosis of Ito
DYP SURVIVAL GUIDE FOR POST GRADUATES 491

41. KELOID AND (earlobes), extremities, trunk, rarer on face, palms and
soles, genitals
HYPERTROPHIC SCAR Q. Keloids are common in which race?
A. Blacks (and in them, they are larger in proportion)
Q. What is a keloid?
Q. Histopathology of keloid ?
A. A firm irregularly shaped fibrous hyperpigmented,
pink or red papule or plaque. A. Dense, sharply defined nodular growth of
myofibroblasts and collagen with a whorl like
Q. Differential diagnosis of keloid
arrangement central thickened hyalinised bundles of
A. - Hypertrophic scar
collagen and paucity of elastic tissue. By pressure
- Carcinoma en cuirasse tumour causes thinning of normal papillary dermis
- dermatofibrosarcoma protuberans and atrophy of adjacent appendages mast cells are
- Keloidal blastomycosis (lobomycosis) present.
- Keloidal scleroderma (a type of morphea) Q. Treatment of keloid
Q. What is the origin of keloid A. - Intralesional injection of triamcinolone Initially
A. Growth occurs over a cut laceration or burn or less 40 mg/ml, later 10-20 mg/ml
often an acne pustule over chest or upper back, - Cryotherapy
spreads beyond limits of original injury and often - Flash lamp pulsed dye laser
sends out claw like projections (keloid = chela= claw)
- 5- Flurouracil
Q. Describe a typical lesion?
- Calcium channel blockers
A. Overlying epidermis is smooth glistering and
thinned from pressure. Can be surrounded by - Surgical excision
erythematous halo, have consistency of rubber mostly - Verapamil
linear with bulbous ends. Keloid may be Q. Difference between keloid and hypertrophic scar
telangiectatic.
A. Keloids:
Q. What are the symptoms?
itch and pain.
A. Lesions may be tender, painful, pruritic may rarely
ulcerate or develop draining sinus tracts. Do not cross joint creases.

Q. Common locations of keloids Need not have history of injury.

A. Commonest – chest (sternal region), neck, ears Is racial/familial.


490
492 DYP SURVIVAL GUIDE FOR POST GRADUATES

Hypertrophic Scar:
42. KERATOLYSIS
asymptomatic.
May cross joint creases. EXFOLIATIVA
Have history of injury.
Q. What are the other names of Keratolysis Exfoliativa?
Is not familial and racial
A. Lamellar dyshidrosis, recurrent palmar peeling.
Itching present
Q. What is Keratolysis Exfoliativa?
Pain present
A. - It is a superficial exfoliative dermatosis of the
Occurs over the lines of tension palms and soles. Clinically there is no
Does not cross skin creases inflammation, there is occurrence of white spots
There need not be any h/o injury and these white spots spread toward the
periphery . The lesions rupture to produce an
Often racial and familial annular adherent collarette . The lesions are
Invariably recurs after excision asymptomatic
h - The condition is exacerbated by environmental
factors. the patients may also have atopy. The
condition can also be identified as sub-clinical
eczema
Q. How can the condition be differentiated from
dermatophyte infection?
A. Because of the scaling, the lesion has to be
differentiated from dermatophyte infection, hence a
KOH mount is useful
Q. How can we treat Keratolysis Exfoliativa?
A. Treatment is by use of urea, ammonium lactate and
salicylic acid. Topical corticosteroids can be used
when dermatitis is present
h

493
43. KERATOSIS PILARIS 44. LENTIGO
Q. What is the mode of inheritance? Q. What are different types of lentigo?
A. Autosomal dominant, sporadic and acquired in some A. Types:
cases
] a) Lentigo simplex
Q. Classical lesions of keratosis pilaris?
b) Solar lentigo(lentigo senilis)
A. Multiple, discrete, skin coloured to hypopigmented,
goose-flesh like, horny follicular micro-papules. They c) PUVA lentigines
are better felt than seen d) Ink spot lentigo (sunburn lentigo)
Q. Commonly involved sites? e ) Labial, penile and vulvar melanosis (melanotic
A. Shoulders, upper arms, lateral aspect of thighs, macules, mucosal lentigines)
buttocks, upper back, sometimes legs
Syndromes associated with lentigo:
Q. Age group affected?
a) Multiple lentigines syndrome
A. - Manifests by 2-3 yrs of age.
b) Moynahan syndrome
- Remains until adulthood; subsides gradually
after that c) Generalized lentiginosis
Q. Associations? d) Centrofacial lentiginosis
A. - Ichthyosis vulgaris e) Carney complex
- atopic diathesis (one of the minor criterias) f) Inherited patterned lentiginosis in black
Q. Comment on the general health of the patient? g) Partial unilateral lentiginosis
A. Usually unaffected h) Peutz-jeghers syndrome
Q. Histopathology? i) LEOPARD syndrome
A. Widened follicular orifices with keratinous plug
Q. What is Lentigo Simplex?
Q. Differential diagnosis?
A. - These lesions occur as sharply defined, round
A. - Lichen spinulosus - Lichen striatus
to oval, brown or black macules
- Phrynoderma - Pityriasis rubra pilaris
- Seen in childhood but may appear at any age
- Lichen nitidus
- No predilection for areas of sun exposure,
Q. Treatment?
A. Mild topical keratolytics : 3% salisylic acid 0.1% - No racial predilection
tretinoin - Increase in number during childhood or puberty
h - Eruptive form is called as lentiginosis
494 495
496 DYP SURVIVAL GUIDE FOR POST GRADUATES DYP SURVIVAL GUIDE FOR POST GRADUATES 497

Q. What are oral melanotic macules? Q. What is the treatment of solar lentigines?
A. They are found mainly in the 5th decade on the A. - sun protection
vermilion border, gingival, buccal mucosa, and - bleaching creams containing 4% or 5%
palate hydroquinone
Q. What is the histology of lentigo simplex? - chemical peels, local dermabrasion, topical
tretinoin, adapalene.
A. - hyperpigmentation of the basilar keratinocytes
- combination of 2% 4-HA and 0.01% tretinoin.
- increase in number of melanocytes in the basal
- liquid nitrogen
layer.
- cryotherapy
- melanophages are present in upper dermis.
- Argon, Q-switched Nd:YAG, frequency doubled
Q. What is the other name for Solar Lentigo? Nd:YAG laser
A. lentigo senilis / liver spots Q. What is PUVA lentigines?
Q. Describe the morphology of solar lentigines. A. Individuals receiving oral methoxsalen
A. - Persistent, benign, discrete, hyperpigmented photochemotherapy may develop persistent
pigmented macules in which there may be
round to oval macules occurring on the sun
melanocytic atypia. These lesion occur site normally
damaged skin protected from sunlight
- Most commonly occur on back of the hands, Q. Ink spot lentigo?
cheek and forehead A. - synonym- sunburn lentigo.
- They may be accompanied by depigmented - These lesions commonly occur on the shoulders
macules, actinic purpura, and other chronic as small markedly irregular, reticulated, grey
degenerative changes in the skin black macules resembling spots of ink on skin
- They may evolve into benign lichenoid keratosis Q. What is the histology of ink spot lentigo?
and reticulated seborrhoeic keratosis A. Histologically there is mild increase in number of
Q. What is the histology of solar lentigines? melanocytes and increased melanin in both the basilar
keratinocytes and stratum corneum
A. - Rete ridges show club shaped or narrow, budlike
Q. What is labial, penile and vulvar melanosis?
extension.
A. - Melanocytic macules are usually light brown on
- Marked increase in pigmentation in the basal cell oral labial mucosa, but may be strikingly
layer especially at the tip of bulbous rete. irregular and darkly pigmented in the genitalia.
- Number of melanocytes is increased. - Sites of predilection in females is labia minora,
- Upper dermis contains melanophages. and in males is glans.
498 DYP SURVIVAL GUIDE FOR POST GRADUATES DYP SURVIVAL GUIDE FOR POST GRADUATES 499

Q. What is the histology of labial, penile, vulvar dystrophicus, multiple skeletal anomalies and
melanosis? central nervous system disorder.
A. Histologically these lesions demonstrate broad “box - Mucous menbranes are spared.
car” rete ridges with prominent basilar - Onset is in the first year of life.
hyperpigmentation and a normal to slightly increased
Q. What is Carney Complex?
number of melanocytes. The melanocytes are
morphologically normal. A. Synonyms are:
Q. Multiple lentigen syndrome: - NAME (Nevi, Atrial myxomas, Myxoid
A. These lesions appear shortly after birth and develop neurofibroma and Ephilides) syndrome
a distinctive speckled appearance - LAMB (Lentigines, Atrial myxoma,
Q. What is LEOPARD syndrome? Mucocutaneous myxoma and Blue nevi)
syndrome
A. Lentigines
Electrocardiagraphic abnormalities Q. What is Inherited patterned lentiginosis in black
person?
Ocular hypertelorism
A. - These are light complexion black patients with
Pulmonary stenosis
autosomal dominant lentigines beginning in
Abnormalities of genitalia infancy or early childhood, without internal
Retardation of growth abnormalities
Deafness - Distribution over central face and lips, with
- It is due to mutation in PTPN11 gene variable involvement of dorsal hands and feet,
- Clinically Cafe noir spots are black spots noted elbow, buttocks
in these patients Q. What is partial unilateral lentiginosis.?
Q. What is Moynahan Syndrome? A. - There is presence of simple lentigines, wholly
A. It consists of multiple lentigens, congenital mitral and partially involving half of the body
stenosis, dwarfism, genital hypoplasia and mental - Conjunctival involvement present
deficiency Q. What is Peutz-Jeghers syndrome?
Q. What is centrofacial lentiginosis? A. - It is an autosomal dominant syndrome consisting
A. - It is characterized by lentigenes on the nose and of pigmented macules on the lips, oral
adjacent cheeks, associated with status mucosaand perioral and acral areas
500 DYP SURVIVAL GUIDE FOR POST GRADUATES

- Gastrointestinal polyps especially prominent in 45. LICHEN PLANUS (LP)


jejunum are frequently associated
- It can present with bleeding PR Q. Why is this LP?
Q. What is Laugier- Hunziker syndrome? A. - Presence of violaceous, flat topped, papules and
A. Distribution similar to Peutiz-Jeger, but also includes plaques with scant, adherent scales.
melanonycha and genital melanosis - On the surface, gray or white puncta or streaks
(Wickham’s striae) cross the lesion.
h
Q. What are the differentials?
A. - LE
- Lichenoid eruption
- Lichen nitidus
- Lichen striatus
- Lichen sclerosus
- PR
- Ashy dermatosis
- Psoriasis
- Lichenoid GVHD
- Secondary syphilis
- Paraneoplastic pemphigus
Q. Explain the primary lesion of LP:
A. The primary lesion consists of (The “P’s” of LP):
- purple (violaceous)
- plane (flat topped)
- polygonal
- pruritic
- papules
- plaques
- persistent
501
502 DYP SURVIVAL GUIDE FOR POST GRADUATES DYP SURVIVAL GUIDE FOR POST GRADUATES 503

- Found on penis Q. Enumerate the various types of LP:


- Responds to prednisolone (steroids) A i. Actinic LP
- Heals with pigmentation ii. Acute LP
- Associated with pterygium iii. Annular LP
- The lesion is initially erythematous pin point iv. Atrophic LP
papules but the well developed lesions become
v. Bullous LP and LP pemphigoid
violaceous 0.5-1.5 cm plaques and the resolving
lesions are often hyperpigmented. The surface vi. Hypertrophic LP
is glistening and dry, with scanty adherent scales. vii. Inverse LP
On the surface, gray or white puncta or streaks
viii. LP pigmentosus
(Wickham’s striae) cross the lesion.
ix. Lichen Planopilaris
- Areas of prelidiction: flexor wrists, trunk, medial
thighs, shins, dorsal hands, glans penis. They x. Linear LP
rarely occur on face (on the eyelids or lips) xi. LP- lupus erythematosus overlap syndrome
- Koebner phenomenon occurs in LP xii. Nail LP
Q. Other name? xiii. Oral LP
A. Lichen ruber planus (named by Hebra) or Erasmus xiv. Ulcerative LP
Wilson’s disease
xv. Vulvovaginal LP
Q. What is meaning of “lichen”?
xvi. Lichenoid drug eruption
A. Lichen: moss, planus: flat topped.
Q. Explain Actinic LP.
Q. What is Broq’s phenomenon?
A. - Other names LP actinicus, LP subtropicus, LP
A. A patient with LP reacts to the itching by rubbing
rather than scratching and consequently scratch tropicus and lichenoid melanodermatitis.
marks are usually not present. - Most patients are young adults or children. There
Q. Explain the nail changes in LP: is no predilection for either sex. The onset of this
variant is during the spring and summer, and the
A. - Longitudinal ridging and splitting
lesions primarily involve sun-exposed skin of
- Onycholysis and the lunula may be red the forehead and face, followed by the dorsal
- Pterygium formation is very characteristic (the surfaces of the arms and hands and the neck. The
proximal nailfold fuses with the proximal portion lesions usually consist of red–brown plaques
of the nail bed). with an annular configuration, but melasma-like
- In nail bed LP there may be onychopapilloma, a hyperpigmented patches have been observed.
localized distal subungual hyperkeratosis. Summertime actinic lichenoid eruption has
504 DYP SURVIVAL GUIDE FOR POST GRADUATES DYP SURVIVAL GUIDE FOR POST GRADUATES 505

sometimes been regarded as a variant of or - The clinical appearance of atrophic LP is likely a


synonymous with actinic LP. result of thinning of the epidermis rather than
Q. Explain Acute LP: degeneration of elastic fibers
A. - Lesions are usually widely distributed and - The epidermal atrophy may be accentuated by
disseminate rapidly. the use of potent topical corticosteroids
- This form is also known as exanthematous or - The most common site of involvement is the
eruptive LP. The commonly affected areas lower extremity
include the trunk, the inner aspects of the wrists Q. Explain Bullous LP and LP pemphigoids:
and the dorsal aspect of the feet. A. Bullous or vesiculobullous lesions can develop just
- The clinical course is usually self-limited and, within pre-existing LP lesions or more randomly,
in general, lesions resolve with including on previously uninvolved skin. The former
hyperpigmentation within 3 to 9 months. is called bullous LP, while the latter is referred to as
Q. Explain Annular LP: LP pemphigoides.
A. - This form is thought to occur when papules Q. Explain Hypertrophic LP:
spread peripherally and the central area A. - Also referred to as LP verrucosus .
resolves. - Extremely pruritic, thick hyperkeratotic plaques
- The annular edge is slightly raised and typically are on the shins or dorsal aspect of the foot and
purple to white in color, while the central may be covered by a fine adherent scale. The
portion is hyperpigmented or skin- c o l o r e d lesions are usually symmetric and tend to be
associated scale in annular LP). chronic because of repetitive scratching. The
- The most common site of involvement is the average duration of hypertrophic LP reported to
axilla, followed by the penis, extremities and be 6 years. Chronic venous stasis frequently
groin. contributes to the development of this condition.
Squamous cell carcinoma, which must be
- Most patients are asymptomatic, while some
distinguished from pseudoepitheliomatous
have pruritus.
hyperplasia, has been reported to arise within
Q. Explain Atrophic LP. these lesions.
A. - Atrophic LP may represent a resolving phase of Q. Explain Inverse LP:
LP, given the history of the lesions, papules
A. - In this unusual variant, an inverse distribution
coalesce to form larger plaques that often, over
pattern is observed. Violaceous papules and
time, become centrally depressed and
plaques appear in intertriginous zones (axillae>
atrophic with residual hyperpigmentation
inguinal and inframammary folds) and less often
506 DYP SURVIVAL GUIDE FOR POST GRADUATES DYP SURVIVAL GUIDE FOR POST GRADUATES 507

in the popliteal and antecubital fossae. of the scalp with or without atrophy. These features
Occasionally, LP lesions occur elsewhere on the need not be present simultaneously.
body. Q. Explain linear LP:
- Hyperpigmentation is usually present as well A. Although linear lesions frequently occur in sites of
and it may be the sole manifestation, leading scratching or trauma in patients with LP (as a result
to overlap with LP pigmentosus. of the Koebner phenomenon), the term linear LP is
Q. Explain LP pigmentosus: usually reserved for lesions that appear
A. LP pigmentosus typically presents in individuals spontaneously within the lines of Blaschko. Although
linear LP is usually seen in patients in their late 20s
with skin types III and IV as brown to gray–brown
or early 30s, the initial presentation can be in the first
macules in sun-exposed areas of the face and neck,
to eighth decades. Presumably this pattern reflects
usually with no preceding erythema and often somatic mosaicism but how the involved and
evolving into diffuse or reticulated pigmentation. uninvolved skin differ is not known.
Involvement of intertriginous sites is occasionally
observed, and a linear distribution following Q. Explain LP- lupus erythematosus overlap:
Blaschko’s lines has also been described. A. Patients whose lesions have overlapping features of
Q. Explain Lichen Plano Pilaris: both LP and lupus erythematosus (LE) have been
reported. These lesions are preferentially located in
A. In lichen planopilaris, involvement of the hair follicle acral sites. Histologic and direct immunofluorescence
is observed, both clinically and histologically. This (DIF) microscopic findings show features of both LP
variant is also called follicular LP and LP cuminatus. and LE.
Multiple, keratotic plugs surrounded by a narrow Q. Explain nail LP:
violaceous rim are observed primarily on the scalp,
although other hairbearing areas can also be affected. A. The nails are affected in approximately 10% of
The inflammatory process may result in scarring and patients with LP; usually, several nails are affected.
loss of follicular structure, i.e. a permanent alopecia. The characteristic nail abnormalities include lateral
Over time, the central areas of the scalp often ‘burn thinning, longitudinal ridging, and fissuring. These
changes are manifestations of matrix damage, which
out’ and are indistinguishable from other causes of
can lead to scarring and dorsal pterygium formation
‘end-stage’ cicatricial alopecia. However, examination
if left untreated. Non-specific changes in the nail bed
of the periphery may reveal the primary lesions. include yellow discoloration, onycholysis and
Women are more frequently affected than men, and subungual hyperkeratosis. Insome patients, twenty-
this form may occur alone or with typical LP lesions nail dystrophy may represent a variant of LP. Nail
elsewhere. A variant of lichen planopilaris known as LP presenting as twenty-nail dystrophy is much more
Graham Little–Piccardi– Lassueur syndrome is common in children than in adults.
characterized by the triad of: (1) non- cicatricial loss
of pubic and axillary hairs and disseminated spinous Q. Explain Oral LP:
or acuminated follicular papules; (2) typical A. Oral LP can appear in at least seven forms, which occur
cutaneous or mucosal LP; and (3) scarring alopecia separately or simultaneously: atrophic, bullous,
508 DYP SURVIVAL GUIDE FOR POST GRADUATES DYP SURVIVAL GUIDE FOR POST GRADUATES 509

erosive, papular, pigmented, plaquelike and reticular. as was once thought andusually appears between
The most common and characteristic form of oral LP the third and fifth decade of life. Although
is the reticular pattern. It is characterized by slightly palmoplantar LP is more common in men than in
raised whitish linear lines in alace-like pattern or in women, ulcerative LP prevails in female patients.
rings with short radiating spines. This form is usually Typical LP lesions may be present on other parts
asymptomatic and the most common site of of the body. The ulcers are intensely painful and
involvement is the buccalmucosa; lesions are often often recalcitrant to conventional therapy. Chronic
bilateral and symmetric. Gingival involvement is ulcerative lesions are at risk of developing
common, and oral LP affecting the gingivae squamous cell carcinoma.
exclusively is seen inapproximately 10% of cases. It
typically presents as chronic desquamative Q. Explain Vulvovaginal LP:
gingivitis. Atrophic, erosive and bullous lesions are A. LP of the vulva can present with several clinical variants,
associated with symptoms ranging from mild but the most common appears to be erosive disease.
discomfort to severe pain. There is a higher Vaginal involvement occurs in up to 70% of women
incidence of plaque-like lesions amongst tobacco with erosive vulvar LP, and because there is often oral
smokers. For unknown reasons, oral LP is very mucosal involvement as well, the term ‘vulvovaginal
uncommon in young patients and, in some studies, gingival syndrome’ has been introduced.
women have been affected about twice as often as men.
Patients with oral LP should be questioned about Q. What is the difference between LP and Lichen nitidus.
symptoms related to esophageal involvement and A.
examined for other mucosal lesions, particularly
genital lesions, and vice versa, because ~70% of the Lichen Nitidus LP
patients with mucosal vulvovaginal LP have clinical Size
Colour
Pin head
Skin colored
Pin head to plaques
Erythematosus-violaceous
signs of oral LP. It has been reported that the Number Multiple Multiple
erosive or ulcerative type of oral LP is less frequently Sites All over except oral cavity, nail Oral cavity also involved. Nail
involved in 5-10% involvement in 10%
associated with cutaneous LP than are all other types Wickhams striae Absent Common
of oral LP. Such mucous membrane lesions are more Hyperpigmentation Unusual Common
Hypopigmentation Common Unusual
therapy-resistant and less likely to spontaneously Koebnerisation Common Common
remit than cutaneous lesions. Malignant Pruritus Uncommon Common
transformation of longstanding, non-healing oral LP Histology Parakeratotic cap, absent or thin
granular layer, lymphocytes,
Hypergranulosis, band like
infiltrate of lymphocytes,
has been reported. Several studies have reported a epitheloid cells, occasional deposits of IgG, IgA, IgM and C3
relationship between oral LP and chronic liver disease, langhans giant cells, Ball and
claw appearance. No
in the epidermis

particularly that due to HCV infection3,4. immunoreactants seen.

Q. Explain Ulcerative LP:


A. Also called as Cram’s disease. Ulcerations can occur
within palmoplantar lesions of LP, particularly
those on the soles. Palmoplantar LP is not as rare
510 DYP SURVIVAL GUIDE FOR POST GRADUATES DYP SURVIVAL GUIDE FOR POST GRADUATES 511

Q. Explain the treatment of lichen Planus. Q. Histology?


A. - Chronic - Topical Steroids, Intralesional Steroids A. A. Orthokeratosis
injections, Biologicals (alefacept, efalizumab) B. Wedge shaped hypergranulosis
- NB-UVB C. Saw toothed rete ridges with irregular acanthosis
- UVA1 D. Diffuse vacuolar degeneration of basal layer
- PUVA E. Civatte bodies/Colloid bodies (degenerated
- Isotretinoin, Acitretin are also found to be basal cells)
affective in some cases F. Max Joseph space (artifact created due to
- LMW heparin 3mg injected subcutaneously once degeneration of basal layer)
a week led to remission of cutaneous and G. Band like lymphocytic infiltrate at DEJ
reticulate oral LP H. Melanin incontinence
- For erosive skin lesions tacrolimus, Q. Drugs causing LP?
pimecrolimus is effective A. A. Antitubercular drugs (rifampicin)
- Mycophenolate Mofetil can induce remission in B. Anticonvulsants (phenytoin)
some cases
C. Antipsychotics (carbamazepine)
- For oral lesions super potent steroids in gel form
D. Diuretics (thiazide diuretics)
are used
E. NSAIDs (ibuprofen)
- In patients with vulvovaginal LP topical
clobetasol is used. ILs may be used in non F. Antidiabetics (sulfonylureas)
responsive cases G. Antimalarials (chloroquine)
- Along with this supportive therapy like anti- H. Heavy metals (gold)
histaminics for itching can be used I. Sildenafil
- Acute-systemic steroids, cyclosporine J. Therapeutic paradox (Griseofulvin & antimalarials)
Q. Why violaceous hue in LP? Q. Which viruses are implicated in LP?
A. Due to Tyndall effect (diffraction of light-the same A. - Hepatitis C
effect which makes the sky look blue). - Hepatitis B
Q. Why Wickham’s striae are seen? - Various herpes viruses(HHV6,7, HSV and VZV)
A. Due to focal hypergranulosis - Transfusion transmitted viruses(TTV)
Q. Why is there itching in LP? - Vaccines for viruses( HBV)
A. Due to band like infiltrate which realeases cytokines h
DYP SURVIVAL GUIDE FOR POST GRADUATES 513

46. LICHEN PLANUS Q. Difference between erythema dyschromia perstans


and LPP.
PIGMENTOSUS A.

Q. Definition?
A. Its a type of lichen planus characterized by slate grey
pigmented macules and patches over sun exposed
areas, trunk and flexures.
Q. Differential diagnosis?
A. Ashy dermatosis, frictional dermatosis, other causes
of hyperpigmented macules.
Q. Racial variation ?
A. More common among darker races
Q. Classical lesion?
A. - Asymptomatic, single or multiple, well defined,
slate grey macules coalescing together to form
larger patches.
- Pigmentation may be patchy, follicular or
diffuse.
Q. Treatment?
A. – Not available.

512
DYP SURVIVAL GUIDE FOR POST GRADUATES 515

Q. Epidemiology?
47. LICHEN SIMPLEX
A. - More in women
CHRONICUS(LSC) - Rare in children
- More between 30-50 yrs of age
Q. Why is this lichen simplex chronicus? Q. Clinical features?
A. Single or multiple well defined plaques showing A. Excessive itching, more at night and in free time,
features of lichenification in areas easy to scratch, like: relieved only when lesion starts paining
Occipital & nuchal areas, wrists, legs, perineum, Q. Giant lichenification of Pautriers?
scrotum & vulva.
A. Solid tumor like lichenified plaques are formed due
Q. Differential diagnosis? to chronic scratching in areas where subcutaneous
A. Following conditions should be kept in mind: tissues are lax, e.g. Genitocrural regions
a) Psoriasis Q. Histopathology?
b) Hypertrophic lichen planus A. - Hyperkeratosis
c) Prurigo nodularis - Acanthosis
d) Atopic dermatitis - Irregular elongation of rete ridges
e) Chronic allergic contact dermatitis - Wedge shaped hypergranulosis
f) Chronic eczema - Broadening of dermal papillae
Q. What is meant by lichenification? - Mild perivascular dermal infiltrate
A. It is characterized by: - Increased number of fibroblasts
- Vertically oriented collagen bundles in dermis
a) Thickening of the skin
Q. Treatment?
b) Increased skin markings
A. Following options considered:
c) Hyperpigmentation
a) Topical steroids
Q. Etiology?
b) Intralesional steroids
A. Itching and subsequent chronic scratching (itch-
c) Emollients
scratch cycle) leads to lichenification of the skin.
Following causes of itching are suspected: d) Sedative anti-histaminics
a) Atopic disorders e) Anxiolytics
b) Psychological factors f) Botox injections
c) Site of pre-existing dermatosis g) Topical aspirin-dichloromethane
d) Allergic contact dermatitis h
514
48. LICHEN SPINULOSUS 49. LICHEN SCLEROSUS ET
Q. Why is this lichen spinulosus?
ATROPHICUS
A. Asymptomatic small skin coloured grouped keratotic Q. Why the lesion is of lichen sclerosus?
follicular papules with a central spinous process.
Lesions are distributed symmetrically over trunk, A. - White polygonal, flat topped papules, plaques
limbs & buttocks. or atrophic plaques surrounded by erythema or
a violaceous halo.
Q. What are the differential diagnosis?
- In atrophic lesion, skin is smooth wrinkled, soft
A. - Keratosis pilaris, phrynoderma and white.
- Lichen nitidus - Bulla, purpura or telangietasia can appear on the
- Lichen striatus patch.
- Pityriasis rubra pilaris - Mostly seen on genitalia
Q. What is the underlying pathology? Q. What are the genital and extra-genital D/D ?
A. There is abnormal keratinisation of the pilosebaceous A. Genital D/D
unit resulting in keratotic follicular papules · Morphea
Q.3 Etiology? · Erosive Lichen Planus
A. Unknown · Sexual abuse (in children)
Q.5 What are the histopathological features? · Erythroplasia of Queyrat (Bowen Disease of the
A. - Dilated hair follicle with keratin plug, Glans Penis)
- Mild perifollicular infiltrate · Balanitis Circumscripta Plasma-cellularis
Q.6 How do you treat this condition? · LSC
A. - Topical salicylic acid (3% oint) Extra-genital D/D:
· Lichen Nitidus
- Topical tretinoin (0.025%)
· Oral Leukoplakia
- Ammonium lactate (12% lotion)
· Vitiligo
h · Anetoderma
· Atrophoderma of Pasini and Pierini
Q. What are the synonyms?
A. - Kraurosis vulvae (in females)
516 517
518 DYP SURVIVAL GUIDE FOR POST GRADUATES DYP SURVIVAL GUIDE FOR POST GRADUATES 519

- Balanitis xerotica obliterans (in males) - Children may present with urinary and bowel
- Csillag’s Disease complaints
Q. What is the epidemiology? - Normal anatomical structure obliterated
A. - Seen in both childhood and old age Q. What are the complications of genital LS?
- More common in white A. - Fusion of the labia minora with labia majora with
- Females are 6-10 times more affected than males. burying of clitoral hood and urethral meatus
Q. What is this the etiology? - Introital stenosis with fusion
A. • Cause unknown Q. What is “figure of 8” or “hour glass” appearance?
• Possible theories are: A. In women, if the perianal area with vulvar and
- Hereditary: HLA-DQ7 perineal area is involved, it leads to stenosis and
- Autoimmune fusion which looks like figure of 8 appearance.
- Infectious: Borrelia Q. Is mucosa involved?
- Constant friction A. Vaginal and cervical mucosa not involved. This
differentiates it from lichen planus (mucosa
Q. What is the age of onset in females?
involved).
A. It has a bimodal distribution: prepubertal and post-
menopausal. Q. What are the clinical features in pre-pubertal girls?
Q. Describe the lesion of lichen sclerosus? A. Infantile perineal protrusion refers to pyramidal soft
tissue swelling covered by red or rose colored skin
A. - White polygonal, flat topped papules, plaques
along medial perineal raphe.
or atrophic plaques
- Lesion surrounded by erythema or a violaceous Q. What is the effect of pregnancy?
halo A. Pregnancy leads to complete improvement and
- In atrophic lesion, skin is smooth wrinkled, soft complete resolution.
and white Q. What is the role of OCPs ?
- Bulla, purpura or telangietasia can appear on the A. OCPs are anti-androgenic, so increase in lesions is
patch seen in pre-menopausal women.
- Mostly seen on genitalia Q. What are the histopathological findings?
- Itching is severe with erosions A. Trizonal appearance:
- Increased dysuria, urethral /vaginal discharge,
Zone 1:
dyspareunia and burning sensation are the
complications - compact orthokeratosis, thinned epidermis
520 DYP SURVIVAL GUIDE FOR POST GRADUATES DYP SURVIVAL GUIDE FOR POST GRADUATES 521

- follicular plugging Q. What is the prognosis ?


- vacuolization of the basal layer A. - The disease can last for a considerably long time.
Zone 2: Occasionally, “spontaneous cure” may
ensue, particularly in young girls.
- Remarkable edema in the papillary (upper)
dermis is replaced by a dense, homogenous - Lichen sclerosus is associated with a higher risk
fibrosis as the lesion matures of squamous cell carcinoma (12-15%-genital
LSA).
Zone 3:
Q. What is the treatment ?
- lichenoid infiltrate below the zone of edema
A. · Superpotent topical steroids daily once or twice
Q. What are the sites involved in males?
a day or weekend therapy for 3 months
A. Glans penis, penile shaft and scrotum.
· Topical tacrolimus 0.1% to 0.03%
Q. What are the complications in males?
· Topical pimecrolimus 1%
A. - Recurrent balanitis
· Gabapentin, duloxetine hydrochloride for
- Poor urinary stream vulvodynia
- Phimosis and paraphimosis · HCQs, calcitriol, calcipotriol, tretinoin,
- Dysparenuia cyclosporine, hydroxyurea in refractory cases
Q. Where are the extragenital lichen sclerosus lesions · Cryotherapy
seen? · Photodynamic therapy
A. - Mostly seen in upper back, chest, breast and · CO2 ablation
wrist. Mostly asympomatic, but can be pruritic.
· Surgical correction in females
- Associated with morphea with lichen sclerosus
like lesion. · Circumcision in males
- Lichen sclerosus usually begins as white, h
polygonal papules that coalesce into plaques.
Evenly spaced dells or comedone-like plugs
correspond to obliterated appendiceal ostia.
With time, the plugs and dells will disappear and
leave a smooth, porcelain-white plaque. Skin
color is white, often with a shiny porcelain
appearance. Telangiectases and follicular plugs
may be seen. The size of the plaque or plaques
may vary widely.
DYP SURVIVAL GUIDE FOR POST GRADUATES 523

credited for the first demonstration of these lines


50. LICHEN STRIATUS in 1901
· Blaschko’s lines are skin lines invisible under
Q. Why is this lichen striatus?
normal conditions. They become apparent when
A. There are small, erythematous, scaly, 1 to 3 mm, some diseases of the skin or mucosa manifest
papules that coalesce to form plaque, either themselves according to a specific pattern
continous or interrupted.
· They follow a “V” shape over the back, “S”
Q. What is the differential diagnosis? shaped whorls over the chest, stomach, and sides,
A. · Lichen planus and wavy shapes on the head
· Linear psoriasis · The lines are believed to trace the migration
· Inflammatory linear verrucous epidermal nevi of embryonic cells. The stripes are a type of
· Verruca plana genetic mosaicism (post-zygotic mutation).
· Linear Porokeratosis · They do not correspond to nervous, muscular,
· Linear darier’s or lymphatic systems.
· Linear lichen nitidus · The lines can be observed in other animals such
as cats and dogs
· Adult blashkitis
Q. Is the lesion symptomatic?
Q. What is the age of onset?
A. Mostly asymptomatic, occasionally pruritic.
A. Young children (mean age 3 years)
Q. What are the nail changes?
Q. What is lichen striatus?
A. Only nails aligned with the skin lesions are affected.
A. Lichen striatus (also known as “Blaschko linear
There is :
acquired inflammatory skin eruption”(BLAISE) and
“Linear lichenoid dermatosis”) is an uncommon skin - thinning
condition that is seen primarily in children, most - ridging
frequently appearing ages 5-15.
- horizontal splitting and nail bed hyperkeratosis
Q. Is there any gender variation?
- dystrophy
A. Girls are affected more than boys.
- total nail loss
Q. What is the pattern of the lesion?
Q. How do lesions heal?
A. Linear pattern, along the line of Blaschko.
A. - Mostly. lesions heal spontaneously with
Q. What is Line of Blaschko? hypopigmentation
A. · German dermatologist Alfred Blaschko is - In less than 5%, hyperpigmentation is seen
522
524 DYP SURVIVAL GUIDE FOR POST GRADUATES

Q. What are the histological features ?


51. LINEAR AND WHORLED
A. · Band-like dermal infiltrate with necrotic
keratinocytes at dermo-epidermal junction NEVOID HYPERPIGMENTATION
· Granulomatous infiltrate may be seen
· Dense lymphoid infiltrate around eccrine sweat Q. Why is this linear nevoid hyperpigmentation?
glands and ducts A. Hyperpigmentation is characteristically symmetrical,
linear and has a dermatomal distribution following
Q. Is Koebner’s phenomenon seen?
lines of Blaschko producing a Zebra like appearance
A. Yes.
Q. Differential diagnosis?
Q. What is Grosshan’s Marot disease?
A. - Incontinentia Pigmenti
A. Syn: adult blaschkitis
- Linear epidermal Nevus
Q. How do adult and children blaschkitis differ? - Hypomelanosis of Ito
A. · In adult blaschkitis, papulovesicular lesions are - Goltz syndrome
seen
Q. What is it?
· Multiple areas are affected
A. A syndrome consisting of congenital anomalies and
· Resolves rapidly and relapses are common patterned melanocytic hyperpigmentation of the
· On histopathology, more of spongiosis trunk and extremities.
Q. What is the treatment of lichen striatus? Q. Synonyms?
A. · Counselling of the patient about the chronicity A. - Reticulate and Zosteriform Hyperpigmentation
of the disease and benign course - Reticulate Hyperpigmentation of Iijima and
· Topical steroids Naito
· Topical tacrolimus 0.1% or pimecrolimus 1% - Zebra-like hyperpigmentation.
· Antihistamines for symptomatic relief Q. What are the associated congenital abnormalities?
A. - Atrial septal defect
h - Dextrocardia
- Orbicular atresia
- Deafness
- Growth retardation
Q. Course of disease?
A. - Lesions increase after birth
525
526 DYP SURVIVAL GUIDE FOR POST GRADUATES

- Become stable, usually fades spontaneously.


Other abnormalities can be surgically corrected.
52. LIPOMA
Q. Any relation to Incontinentia Pigmenti?
Q. What do you find on examination?
A. Some authors consider it as a variation of
Incontinentia Pigmenti. A. A painless (rarely painful) swelling with normal
overlying skin with no increase in temperature, no
Q. Histopathology? tenderness, smooth lobulated surface, soft edges and
A. - Hyperpigmentation of basal keratinocytes compressible, slipping away from fingers, non-
- Prominent melanocytes fluctuating and freely mobile over deeper structures.
- Mild elongation of rete ridges Q. Differential diagnosis?
- Pigmentary incontinence with melanophages in A. 1) leiomyoma
papillary dermis 2) fibroma
Q. Treatment? 3) dermoid cyst
A. Q-switched lasers have been tried. 4) hibernoma
h 5) liposarcoma
6) lymphatic tumour
7) neurofibroma
8) rheumatic nodules
9) sebaceous cyst
Q. What is the cause of pain in lipoma?
A. - trauma or manipulation on part of the patient
- secondary infection
- angiolipoma
- haemorrhage in the lipoma
- malignancy (rare)
Q. D/D of tender tumors:
A. “BLEND AN EGG”:
B: Blue rubber bleb nevi
L: Leiomyoma

527
528 DYP SURVIVAL GUIDE FOR POST GRADUATES DYP SURVIVAL GUIDE FOR POST GRADUATES 529

E: Eccrine Spiroadenoma - Histopathology of hibernoma, on the other hand,


N: Neurilemmoma shows multiple cells with small vacuoles of fat
giving a granular appearance (brown fat)
D: Dermatofibroma
Q. What is the treatment of lipoma?
A: Angiolipoma
A. Excision of the lipoma cleanly at first operation to
N: Neuroma
prevent recurrence.
E: Endometrioma
Q. What is Bannayan Riley Ruvalcaba syndrome ?
G: Glomus cell tumour
A. Bannayan Riley Ruvalcaba syndrome is an autosomal
G: Granular cell tumour dominant disorder consisting of constellation of:
Q. What is it also called as? - multiple lipomas
A. - Universal tumour - macrocephaly
- Ubiquitious tumour
- lymphangiomas
Q. What is the etiology of lipoma?
- haemangiomas
A. Chromosome 12q 13-15
Q. What are the other disorders related to lipoma?
Q. What are the clinical features?
A. Other disorders include adiposis dolorosa,
A. - Seen in adults of age group 40-60 years. Madelung disease, Cowden syndrome and Gardner’s
- No gender predilection. syndrome, have an increased risk of multiple
- Lipomas can occur almost anywhere on the body, lipomas.
but are most commonly found on the trunk,
shoulders, neck, and armpits. h
- Lipomas can also form in muscles and internal
organs.
- Patient comes with a painless slowly enlarging
mass on any of the above sites.
Q. Histopathological findings?
A. - Epidermis normal
- Dermis normal
- Subcutaneous tissue: adipocytes are
univacuolated and mature with eccentrically
placed and compressed nuclei. Signet ring cells
seen.
DYP SURVIVAL GUIDE FOR POST GRADUATES 531

53. LIPODERMATOSCLEROSIS fat without primary inflammatory panniculitis


occurring multifocally and microscopically
Q. Why is this lipodermatosclerosis? throughout affected area.
A. Marked woody induration in a stocking distribution Q. What is designated as sclerosing panniculitis?
results in calves that resembles an inverted A. Areas of fat necrosis are larger and present as tender
champagne bottle. erythematous subcutaneous nodules or plaques.
Q. Differential diagnosis of lipodermatosclerosis? Q. What are ghost cells?
A. - Morphea A. Pale cell walls with no nuclei due to ischaemic
- DM scleromyxedema necrosis of fat globules.
- Panniculitis Q. Treatment of sclerosing panniculitis?
- Liposarcoma A. - Graded compression stockings
- Erythema nodosum - Elevation
- Diabetic dermopathy - Pressure dressings like Unna Boot (temporary but
Q. What is lipodermatosclerosis also called as? dramatic results)
A. Hypodermitis, sclerodermiformis, stasis panniculitis - Unna boot with coban and a foam buttress
Q. Age and sex of predilection? (bolster material to apply extra pressure to red
inflamed areas)
A. Women older than 40 years with an above average
body mass index - Pentoxyphylline 400-800 mg (three times a day),
Q. Distribution of lesions? Stanazol 2-5 mg, Oxandrolone 10 mg (twice a day)

A. Predilection for medial lower third of lower h


legs.Bilateral, if unilateral either left leg is only
affected or is more severe, if right leg is affected deep
venous thrombosis currently or in past or we suspect
venous injury to right leg.
Q. What is hypodermitis sclerodermiformis?
A. Marked woody induration in stocking distribution
results in calves that resemble inverted champagne
bottles induration is due to fibrosis of subcutaneous
530
DYP SURVIVAL GUIDE FOR POST GRADUATES 533

Q. What is amyloid ?
54. MACULAR AND LICHEN A. Ultrastructurally, amyloid is composed of non-
AMYLOIDOSIS branching fibrils , measuring 6-10 nm in diameter.
The amyloid protein has an anti-parallel, beta pleated
sheet configuration. It is composed of a non- fibrillary
Q. Why is it macular amyloidosis? protein known as the amyloid P component and a
A. Hyperpigmented macules showing rippled fibrillary component.
appearance on extensor aspect of arms and lateral Q. How is amyloid identified ?
aspect of lower legs. A. - In light microscopy it appears as amorphous,
Q. Why is it lichen amyloidosis? hyaline- like, eosinophilic deposits
- With congo red stain: green birefringence
A. Hyperpigmented plaques with acuminate papules
and excoriation marks. - With crystal violet: reddish metachromasia
Q. What are the differential diagnosis of macular - With thioflavin – T stain: yellow green
flourescence
amyloidosis?
Q. Clinical classification of amyloidosis?
A. 1. atopic dermatitis
A.
2. poikiloderma of civatte Primary systemic Amyloidosis: Cutaneous:
1. Plasma cell dyscrasias 1. Primary- macular, lichen, biphasic,
3. post inflammatory hyperpigmentation 2. Multiple myeloma associated dyschromic, nodular
4. phototoxic contact dermatitis Secondary Systemic Amyloidosis: 2. Secondary- Within skin tumors
1. Chronic inflammation (Rheumatoid Endocrine:medullary, carcinoma of thyroid,
5. Erythema dyschromiumperstans arthritis) insulinoma, type 2 diabetes mellitus
2. Chronic infection ( tuberculosis)
6. pityiriasis versicolor Heredofamilial amyloidosis: Cerebral: Alzheimer’s disease
1. Familial amyloidotic polyneuropathy
7. atrophic lichen planus 2. Familial Mediterranean fever

8. notalgia paraesthetica Q. What is primary cutaneous amyloidosis?


Q. What are the differential diagnosis of lichen A. It is associated with deposition of amyloid in skin
amyloidosis? only
A. 1. prurigo nodularis Q. What is the clinical presentation of localised
2. neurodermatitis cutaneous amyloidosis?
3. lichen simplex chronicus A. • Macular amyloidosis
4. scleroderma – May be pruritic and lesions are
hyperpigmented in a confluent or rippled
5. pretibial myxedema manner.
532
534 DYP SURVIVAL GUIDE FOR POST GRADUATES DYP SURVIVAL GUIDE FOR POST GRADUATES 535

- Sites of involvement are – upper back, Q. Describe the different treatment methods for
scapula, extensor surfaces of extremities. cutaneous amyloidosis?
- Presents in early adulthood affecting women A. - Treatment is aimed at breaking the itch – scratch
> men. cycle
• Lichen amyloidosis: - patients should be advised against frequent and
chronic rubbing with brushes, towels and loofahs
- Persistent pruritic papules on the shins or
during or after bathing.
other extensor surface of extremities. Initial
- Topical application of potent corticosteroids
lesions are discrete, firm, scaly, skin colored
under occlusion.
or hyperpigmented papules which later
coalesce into plaques that often have a - A mild keratolytic agent can also be used
rippled or ridged pattern. - Topical calcineurin inhibitors
• If both macular and lichen amyloidosis is present - intralesional corticosteroids
in a single patient then the term biphasic - phototherapy
amyloidosis can be used. - dermabrasion
• Biphasic amyloidosis – fine papular lesions are - CO2 laser
superimposed upon a background of - cyclosporine
hyperpigmentation. - systemic retinoids
Q. What is friction amyloidosis?
h
A. Friction from brushes, towels, loofahs and coconut
coir can produce macular amyloidosis .
Q. What is the histopathological findings in localized
amyloidosis?
A. - Amyloid deposits are restricted to upper dermis
in macular and lichen amyloidosis.
- In lichen amyloidosis , deposits expand the
papillae and displace the elongated rete ridges .
- Epidermis is acanthotic and hyperkeratotic.
- Pigment incontinence and perivascular
lymphohistiocytic infiltrate are noted .
DYP SURVIVAL GUIDE FOR POST GRADUATES 537

55. MILIARIA Q. What is miliaria?


A. It is the retention of the sweat as a result of occlusion
of eccrine sweat ducts which produces an eruption
Q. Why is this miliaria?
that is common in hot, humid climates, such as in
A. - Small superficial vesicles with no inflammatory tropics, hot summer months and in temperate
reaction (miliaria crystallina). climates.
- Pruritic, discrete, erythematous papulovesicles Q. Which organism can exacerbate miliaria?
accompanied by sensations of prickling, burning A. Staphylococcal epidermidis.
or tingling which later, become confluent on the
Q. How is miliaria caused?
bed of erythema(miliaria rubra).
A. The polysaccharide substance may obstruct the
- Pustules are distinct, superficial and delivery of the sweat to the skin surface. The occlusion
independent of hair follicles (miliaria pustulosa). prevents normal secretions from the sweat glands and
- Most common sites of predilection are eventually pressure causes rupture of the sweat
antecubital, popliteal fossa, trunk, glands or ducts at different levels. The escape of
inframammary areas, abdomen (especially substance in adjacent tissue produces miliaria.
waistline), inguinal regions. Q. What are the different types of miliaria?
- Miliaria profunda which occurs in the form of A. a) miliaria crystallina (sudamina)
asymptomatic nodules. b) miliaria rubra( prickly heat)
Q. What is the differential diagnosis of miliaria? c) miliaria pustulosa
A. • Depends on the type of miliaria. d) miliaria profunda
• In case of miliaria rubra: e) postmiliarial hypohydrosis
- Grover’s disease is an important DD. f) tropical anhidrotic asthenia
• In case of miliaria pustulosa: Q. What is morphology and clinical features of miliaria
crystallina?
- Acute generalized exanthematic pustulosis,
where there are non follicular pustules, but A. - It is characterized by small superficial vesicles
also systemic symptoms like fever and a with no inflammatory reaction.
history of exposure to drugs like antibiotics. - It appears in bedridden patients in whom fever
produces increased perspiration
• In case of miliaria profunda:
- Situations in which clothing prevents dissipation
- Furuncles and other pyodermas have to be
of heat and moisture, as in bundled children.
differentiated.
536
538 DYP SURVIVAL GUIDE FOR POST GRADUATES DYP SURVIVAL GUIDE FOR POST GRADUATES 539

- Lesions are generally asymptomatic and their Q. Which diseases are associated with miliaria
duration is short lived as they tend to rupture pustulosa?
due to slight trauma. A. Contact dermatitis, Lichen simplex chronicus and
- Lesions are self-limiting and resolve intertrigo which occur several weeks after the disease
spontaneously. is subsided.
Q. Name the drugs precipitating miliaria? Q. What is complication of miliaria pustulosa?
A. Isotretinoin, Bethanecol, Doxorubicin. A. Recurrent episodes may be the sign of type 1
pseudohypoaldosteronism, as salt losing crisis may
Q. What is the morphology and clinical features of precipitate miliaria pustulosa or rubra with
miliaria rubra? resolution after stabilization.
A. - They appear as extremely pruritic, discrete, Q. What is the morphology and clinical features of
erythematous papulovesicles accompanied by miliaria profunda?
sensations of prickling, burning or tingling. They A. - Non-pruritic, flesh coloured, deep seated whitish
later become confluent on the bed of erythema. papules characterize this form of miliaria
- Most common sites of predilection are - Usually asymptomatic
antecubital, popliteal fossa, trunk,
- Lasts for mostly 1 hr after the heating has
inframammary areas, abdomen (especially subsided
waistline), inguinal regions.
- Sites are trunk and extremities
- Due to impending evaporation of moisture, there
Q. What is the site of injury of the sweat gland in
is maceration of these areas.
miliaria profunda?
Q. At what level sweat gland is injured in miliaria A. The occlusion is in upper dermis.
rubra?
Q. What is post miliarial hypohidrosis?
A. Prickle cell layer, where spongiosis is produced.
A. It results from occlusion of sweat ducts and pores
Q. What is the morphology and clinical feature of and may be severe enough to impair individual’s
miliaria pustulosa? ability to perform sustained work in hot environment.
A. - Pustules are distinct, superficial and Q. What are the clinical features of postmiliarial
independent of hair follicles. hypohydrosis?
- Pruritic pustules occur most commonly on A. - decreasing efficiency
intertriginous areas, flexural surface of - irritability
extremities, scrotum and back of bedridden - anorexia
patients.
- drowsiness
540 DYP SURVIVAL GUIDE FOR POST GRADUATES

- vertigo
56. MILIUM
- headache
- the duration and severity of hypohydrosis is Q. Why is this milia?
related to degree of miliaria
A. Small, transluscent, yellowish white papules develop
Q. What is Tropical anhidrotic asthenia?
on areas exposed to sunlight, especially around the
A. - This is a rare form of miliaria with long lasting orbits, dorsa of the hands, back and sides of the neck
poral occlusion, which produces anhydrosis and and the ears.
heat retention
Q. Differential diagnosis of milium?
- It occurs in people who are accustomed to
temperate climates but then suddenly put in a A. - Trichoepithelioma
tropical environment in which they have to do a - Tuberous sclerosis
lot of physical work (e.g. soldiers)
- Hidrocystoma
Q. What is the treatment given for miliaria?
Q. What is milium?
A.- - Place the patient in cool environment
A. It is a degenerative change occurring in the skin.
- Circulating air fans to cool the skin
Q. What are the different types of milium?
- Anhydrous lanolin application
A. A. Juvenile type C. Primary
- Hydrophilic ointment
B. Adult type D. Secondary
- Soothing cooling bath containing colloidal
oatmeal or cornstarch is beneficial Q. What are primary and secondary milia?
- Dusting powder A. - Primary milia are derived from the lowest
portion of the infundibulum of the vellus hair at
h the level of the sebaceous duct. They represent a
keratinizing type of benign tumor and seen in
the epidermis.
- Secondary milia may arise from hair follicle,
sweat duct, sebaceous duct or epidermis . They
are seen in various blistering disorders like
bullous pemphigoid, dystrophic epidermolysis
bullosa, PCT, after dermabrasion or other trauma.
They represent retention cyst and are sub-
epidermal.

541
542 DYP SURVIVAL GUIDE FOR POST GRADUATES DYP SURVIVAL GUIDE FOR POST GRADUATES 543

Q. What are Bohn’s nodules and Epstein’s pearls? Q. What is the treatment of colloid milium?
A. - 40-50% infants show milia which resolve in the A. - Topical tretinoin
1st 4 weeks. - Mechanical expression
- Bohn’s nodules are milia on the hard palate - Electrodessication
- Epstein’s pearls are milia on the gum margins. - Dermabrasion
Q. What is milia en plaque? - Erbium: YAG laser
A. It is characterized by multiple milia within an
- Cryotherapy
erythematous edematous plaque, often seen in the
post-auricular area, may also be seen on anterior part - Chemical peels
of ear. - Resurfacing techniques
Q. In which condition are widespread milia seen? Q. How can milium be prevented?
A. - Marie-Unna hypotrichosis A. - Protection against sun, by use of sunscreens,
- Oral-facial-digital syndrome. covering the face before going into the sun.
Q. What is the etiology of milium? - Regular use of emollients or moisturizing creams
A. - Juvenile form is familial and begins before on areas of sun damage.
puberty - Lower concentrations of alfa hydroxyl acids helps
- Adult form is due to trauma, exposure to to improve skin texture.
sunlight, photodynamic effects of phenol in gas
fuel, prolonged application of hydroquinone
h
bleaching cream.
Q. Describe the histology of milium.
A. - Colloid globules can be seen on the tip of dermal
papillae.
- Fissured masses of colloid occur in the mid
dermis surrounded by collagen.
- In adult type, the colloid is separated from the
epidermis by a band of collagen and elastolysis
is present.
- In juvenile type, elastolysis is absent and colloid
is seen in the epidermis and immature civatte
bodies are also present.
DYP SURVIVAL GUIDE FOR POST GRADUATES 545

c) Genetic
57. MELASMA
d) Toxic (cosmetics)
Q. Why is this melasma ? e) Drugs (most commonly phenytoin)
A. Presence of symmetrically distributed light to dark Q. Histopathology?
brown macules and patches with defined borders A. - Melanocytes increased in number and activity.
over sun exposed areas. - Increased size and melanisation of melanosomes.
Q. Differential diagnosis? In dermal melasma, melanophages are increased
in the papillary dermis.
A. - Rash of LE
Q. Classification?
- Drug induced pigmentation e.g. due to
antimalarials, chlorpromazine, antidiabetics, A. As following:
chlofazamine, amiodarone, phenytoin, ingestion a) Clinical classification:
of silver (argyria) a. According to distribution
- Acanthosis nigricans in men (seborrhoiec i. Centro-facial
melanosis) ii. Malar
- pigmentory demarcation lines (Voight lines) iii. Mandibular
- LPP b. By Wood’s lamp examination
- actinic LP i. Epidermal
- riehl’s melanosis ii. Dermal
- exogenous ochronosis iii. Mixed
- erythema dyschromia perstans b) Histopathological classification:
- PIH a. Epidermal type (Melanin basal and
Q. Incidence? suprabasal layers)
A. - More in females, south Indians, Asians. b. Dermal type (Melanin – laden macrophages
in upper and mid dermis)
- More in darker individuals
Q. Clinical features?
Q. Etiology?
A. Well-defined, multiple hyperpigmented macules
A. Following factors: coalescing together to form larger patches, present
a) Sunlight over the forehead, nose, cheeks (mostly sun exposed
b) Hormones areas).
544
546 DYP SURVIVAL GUIDE FOR POST GRADUATES DYP SURVIVAL GUIDE FOR POST GRADUATES 547

Q. Dermascopy findings? Q. Prognosis?


A. Globules of pigmented areas arranged in a reticular A. - Epidermal type responds well to therapy.
or net like pattern. - Dermal type difficult to treat.
Q. Treatment? Q. What is exogenous ochronosis?
A. Following options considered: A. Due to prolonged use of topical high concentration
a) Sunscreens hydroquinone, globules of homogentisic acid, a
b) Modified Kligman’s formula derivarive of hydroquinone gets deposited in the
dermis (banana bodies)
c) Tretinoin
Q. Why is mono benzyl ether of hydroquinone (MBEH)
d) Azelaic acid
not used?
e) Glycolic acid
A. It causes irreversible depigmentation in a
f) Kojic acid characteristic speckled pattern.
g) Superficial chemical peels
h
h) Lasers
Q. What is Modified Kligman’s formula?
A. It contains:
a) Retinoic acid (0.1%)
b) Hydroquinone ( 2 or 4%)
c) Steroids (triamcinolone, mometasone,
fluticasone)
Q. What is original Kilgman’s formula?
A. - Hydroquinone 5%
- Retinoic acid 0.1%
- Dexamethasone 0.1%
Q. Which lasers are effective?
A. Following have been tried with success:
a) Q- switched ruby laser
b) Nd-YAG laser
c) Alexandrite
DYP SURVIVAL GUIDE FOR POST GRADUATES 549

58. MORPHEA Q. Why is the skin said to have a pig skin appearance?
A. As follicular openings are more prominent like in pig
Q. Types of morphea? skin.
A. Cutaneous scleroderma Q. Discuss morphology of Guttate morphea?
A. Multiple small chalk white flat or slightly depressed
Morphea Linear macules over chest, neck, shoulders and upper back.
- Localized - Scleroderma with Q. What are the manifestations of pansclerotic
or without morphea?
- Melorheostosis/ A. It involves dermis, panniculus, fascia, muscles, bones
- Generalized hemiatrophy and joints (disabling, limiting motion).
- Pan sclerotic Q. What does morphea profonda have an overlap with?
- Profunda A. a) Eosinophilic fasciitis
- Atrophoderma of pasini and pierini b) Eosinophilia myalgia
- Morphea-lichen sclerosus et atrophicus overlap c) Spanish toxic oil syndrome
Q. What is the ratio of men is to women in localized Q. What type of morphea will show widespread
morphea? indurated plaques with pigmentary changes and
A. 1:2 muscle atrophy?

Q. Age group and sites affected with localized morphea? A. Generalized morphea
Q. Morphea-lichen sclerosus et atrophicus overlap is
A. In childhood and in adults more on trunk than
common in which sex?
extremities.
A. Women
Q. Describe clinical course?
Q. Differentiate between Eosinophilic fasciitis and
A. Rose/violaceous macules appear first followed by
morphea profunda?
smooth hard somewhat depressed yellowish white
to ivory lesions. A.

Q. Why is it called hidebound skin? Eosinophilic Fascitis Morphea Profunda


Increased response to corticosteroids Less response
A. If skin is pinched or picked up between thumb and
Chronic debilitating course not present Chronic debilitating course
index finger it is rigid and difficult to pick.

548
550 DYP SURVIVAL GUIDE FOR POST GRADUATES DYP SURVIVAL GUIDE FOR POST GRADUATES 551

Q. What is linear atrophoderma of moulin? - Loss of adventitial fat resulting in trapped eccrine
A. A variant of atrophoderma in which induration and glands
pigmentary changes follow lines of Blaschko. - Sparse deep lymphoplasmacytic infiltrate
Q. Describe Atrophoderma of Pasini and Pierni? - Reduced number of CD34 positive cells in the
A. Brownish gray oval, round or irregular smooth dermis.
atrophic lesions depressed below level of skin with - Superficial dermal pallor may be present, but the
sharp sloping borders. vacuolar interface dermatitis and lymphoid band
Q. In Atrophoderma of Pasini and Pierni, why do we of lichen sclerosus are lacking
take a perilesional biopsy? Q. What is the treatment?
A. Because changes are subtle perilesional biopsy A. Local
allows us to compare to normal skin.
- Topical corticosteroids
Q. Describe the lesions of En coup de sabre?
- Intralesional corticosteroids
A. Induration and depression parasagitally along the
- Topical calcineurin inhibitors
parting extending way down the forehead.May be
- Vitamin A analogues
associated with cicatricial alopecia.
Q. What does "en coup de sabre" mean? - Vitamin D analogues

A. Cut of the sword Systemic

Q. Which morphea is difficult to differentiate from - Penicillin


lichen sclerosis et atrophicus? - Hydroxychloroquine or Chloroquine
A. Guttate morphea as lesions are less firm - Corticosteroids
Q. What is the histopathology? - Vitamin A analogues
A. The histopathology of morphea: - Vitamin D analogues
- Thick, closely packed, hyalinized collagen - Cyclosporine
bindles in the lower dermis. - Methotrexate
552 DYP SURVIVAL GUIDE FOR POST GRADUATES

Phototherapy 59. MYCETOMA


- Oral photochemotherapy
- Bath photochemotherapy Q. Define mycetoma?(Madura foot)
A. It is a chronic, slowly progressive, granulomatous
- Cream photochemotherapy
disease characterised by triad of painless
- UVA1 +++ 2 ++ 2 subcutaneous mass, multiple sinuses and sero-
- Photodynamic therapy + 3 ++ 3 purulent discharge containing grains. It may spread
to involve the skin and the deep structures resulting
- Extracorporeal photopheresis + 3 No experience
in destruction, deformity and loss of function.
- Others CO2 laser No experience ++ 3
Q. Who recognized mycetoma first?
- Surgery Selected patients Selected patients
A. Gill, who worked at a dispensary in the southern
- Physical therapy
province of Madura, first recognized mycetoma as a
disease entity in 1842
h Q. What is the mycetoma belt?
A. The mycetoma belt stretches in a band between the
latitudes of 150 South and 300 North. The belt
includes Sudan, Somalia, Senegal, India, Yemen,
Mexico among others.
Q. What is the incubation period?
A. Unknown but after animal inoculation,found to be
3 weeks.
Q. Why is occupation and residence important?
A. Mycetomas are mainly but not exclusively found in
the dry tropics where there is a low annual rainfall.
It is a disease of poverty, most commonly affecting
agricultural workers and people who are habitually
walk barefoot. In India, actinomycoticmycetoma is
more commonly encountered than eumycoticmycetoma.
However, eumycoticmycetoma is common in north
India.
553
554 DYP SURVIVAL GUIDE FOR POST GRADUATES DYP SURVIVAL GUIDE FOR POST GRADUATES 555

Q. What is Chalmers and Archibald classification? Q. What is the reason of the persistence of organism
A. Group 1 - Maduramycosis, caused by true fungi after an initial inoculation ?
exhibiting septate filaments usually with chlamydos- A. It appears to be related to its ability to evade host
pores defenses through a variety of adaptations such as cell
Group 2 - Actinomycosis, caused by delicate non- wall thickening and melanin production.
septate filaments of the Actinomyces which belong
Q. What are types of clinical material used
to higher bacteria.
fordiagnosis?
Q. Why is socioeconomic history important?
A. Serosanguinous fluid or seropurulent fluid, scrapings
A. Since it mostly affects young men, it has a
socioeconomic effect on the dependent family of sinuses, tissue biopsy or excised sinus should be
members examined for the presence of grains. Saline dressings
applied overnight over the swelling can also be
Q. Name common causative agents of actinomycotic-
mycetoma ? observed for the presence of grains.
A. Agent Grain colour Q. Why is mycetoma painless?
Nocardia asteroides White A. It has been suggested that mycetoma produces
Nocardia brasilienses White substances that have an anaesthetic action.
Nocardia otitidiscaviarum White Q. Why do you get pain in mycetoma?
Actinomadura madurae White A. • expansion of the bone with the mycetoma
Actinomadura pelletieri Red to pink granuloma and grains.
Streptomyces somaliensis White-to-yellow • secondary bacterial infection
Q. Common causative agents of eumycoticmycetoma:- Q. Why are areas of local hyperhidrosis seen over the
A. Agent Grain colour mycetoma lesion?
Madurellamycetomatis Black to brown A. They are either due to sympathetic over-stimulation
Madurell agrisea Black to brown or increased local temperature due to increased
Leptosphaeriase negalensis Black arterial blood flow caused by the chronic
inflammation.
Curvularialunata Black
Neotestidinarosatii Yellow Q. Why are neurological and trophic changes rare until
late in the disease process?
Acremonium spp. White to yellow
A. Because the tendons and the nerves are curiously
Fusarium spp. White to pale yellow spared,neurological and trophic changes are rare. The
Scedosporiumapio spermium White to pale yellow absence of the trophic changes may be explained by
Pseudoallescheria boydii White the adequate blood supply in the mycetoma.
556 DYP SURVIVAL GUIDE FOR POST GRADUATES DYP SURVIVAL GUIDE FOR POST GRADUATES 557

Q. Difference between eumycetoma and - Chromoblastomycosis


actinomycetoma? - Sporotrichosis
Eumycetoma Actinomycetoma - Atypical mycobacterial infection
Invasion of Gradual and delayed Earlier and extensive
Q. Complications
deeper
structures A. • secondary bacterial infection.
Multiple Larger in size, few in Usually smaller in size. • Cachexia
punched out number, well Numerous and have no
• Anaemia( due to malnutrition, sepsis and mental
cavities defined margins. definite margins.
depression)
throughout Osteolytic lesions in Both osteolytic and
the bone bones osterosclerotic lesions • disfigurement/ deformities/ankylosis
seen • Amputation(Chronic neglected infection)
USG Produce numerous Less distinct due to
• Invasive infection(Immunocompromised
sharp brightly their smaller size and
patients)
reflective echoes consistency. Individual
(grains) embedding of the • Lymphoedema(Lymphatic obstruction and
Multiple thick grains or the absence fibrosis)
walled cavities with of cement substances • toxicity due to prolonged antimicrobial or
absent acoustic in few of them. antifungal therapy.
enhancement
• Osteomyelitis (bacterial or tubercular)
Q. What are the causes of enlarged regional lymph Q. What are the causes of death in mycetoma?
node?
A. - cranial mycetoma
A. - secondary bacterial infection,
- sepsis
- genuine mycetoma lymphatic spread
Q. What are the common siites involved?
- local immune responses to mycetoma.
A. The foot (70%), specially the dorsal aspect of the fore-
Q. Whar are DDs of shotty lymphadenopathy? foot. The left foot is affected more. The hand(12%)
A. - Secondary syphilis knee, arm, leg, head and neck, thigh and the perineum
- Mycetoma Q. What is the mode of spread of mycetoma?
Q. What is the differential diagnosis? A. - along the fascial planes
A. - Osteomyelitis (bacterial or tubercular) - along the lymphatics to the regional lymph
- Actinomycosis nodes.
- Botryomycosis - blood borne spread
558 DYP SURVIVAL GUIDE FOR POST GRADUATES DYP SURVIVAL GUIDE FOR POST GRADUATES 559

Q. Why, in mycetoma, surgery under local anaesthesia and Codman triangle; an appearance that may be
is contra-indicated? indistinguishable from that of primary osteogenic
A. The apparent clinical features of mycetoma are not sarcoma.
always a reliable indicator of the extent and spread Q. What are the other Xray changes?
of the disease, as some small lesions with few sinuses
A. In the early stage, a soft tissue granuloma is seen as a
may have many deep connecting tracts, through which
dense shadow or as scattered multiple soft tissue
the disease has spread quite extensively.
shadows. Calcification and obliteration of the fascial
Q. What is the differential diagnosis?
planes may sometimes be seen. As the disease
A. - Kaposi’s sarcoma, progresses, the cortex may be compressed from
- Malignant melanoma outside by the granuloma leading to bone scalloping.
- Fibroma. Q. Advantages of USG?
- Thorn and foreign body granuloma A. • Differentiates between eumycetoma and
- Tuberculosis(The presence of bone destruction actinomycetoma as well as between mycetoma
in the absence of sinuses) and other conditions.
Q. Primary osseous mycetoma- Differential diagnosis • The size and extent of the lesion can be accurately
A. - Chronic osteomyelitis, determined ultrasonically and this is useful in
- Osteoclastoma, planning surgical incisions and procedures
- Bone cysts • Safe, simple, noninvasive
- Syphilitic osteitis Q. Advantages of MRI?
Q. Why are pathological fractures rare in mycetoma? A. Visualization of the extension of bone destruction,
A. They are rare because the cavities are usually filled periosteal reaction and soft tissue involvement.
with solid masses of grains and fibrous tissue, which Assessment of the extent of mycetoma in the soft
provides bone support. tissues.
Q. Where are the bony changes unique? Q. What is “dot-in-circle sign”?
A. In the skull. Purely sclerotic changes with dense bone A. MRI usually shows multiple 2-5 mm lesions of high
formation and loss of trabeculation. The cause of this signal intensity, which indicates the granuloma,
is unclear. interspersed within a low-intensity matrix which is
Q. What is Codman triangle? the fibrous tissue. The “dot-in-circle sign”, which
indicates the presence of grains, is characteristic of
A. Periosteal new bone spicules are laid down at right
mycetoma and it is highly specific.
angles to the cortex to create a sun-ray appearance
560 DYP SURVIVAL GUIDE FOR POST GRADUATES DYP SURVIVAL GUIDE FOR POST GRADUATES 561

Q. What is the differential diagnosis of mycetoma on material. This consists largely of pigmented cement
MRI? substance although hyphae are sometimes identified.
A. Chronic osteomyelitis,granulomas, soft tissue Type III reaction, is characterised by the formation
tumours, bone tuberculosis and cold abscesses. of a well-organized epithelioid granuloma with
Q. What is role of CT scan? Langhan’s type of giant cells. The centre of the
granuloma sometimes contains remnants of fungal
A. Not specific but are helpful to detect early bone
material
involvement.
Q. Role of Fine Needle Aspiration Cytology?
Q. How should you do a biopsy?
A. FNAC allows morphological identification of
A. It always needs surgical biopsy which should be a
mycetoma and its classification into eumycetoma and
deep biopsy done under general or regional
actinomycetoma. The technique is simple, rapid,
anaesthesia. The chance of local spread with the
sensitive and can be tolerated by patients.
surgical biopsy is high.
Q. What is the best way to obtain grains for culture?
Q. What are the different types of stainings or
procedures done on biopsy used? A. Deep surgical biopsy. The grains extracted through
the sinuses are usually contaminated and not viable
A. - Haematoxylin and eosin (H&E) stain and hence should be avoided.
- Immuno-fluorescent Q. Which are the media used to isolate organisms?
- antibody immuno-histochemistry techniques. A. Several media like Malt extract, Sabouraud’s and
There are three types of host tissue reaction Glucose nutrient.
against the organism:
Q. Disadvantages of culture?
Type I tissue reaction, the grains are usually
surrounded by a layer of polymorphonuclear A. The culture technique is cumbersome and time
leucocytes. The hyphae and cement substance consuming with chance of contamination happening
disappear and only remnants of brown pigmented which may give a false positive result. It also requires
cement are left behind. Outside the neutrophils zone experience to identify the causative organisms.
there is granulation tissuecontaining macrophages, Q. Any recent diagnostic techniques?
lymphocytes, plasma cells and few neutrophils. A. A specific PCR test amplifying a region of the internal
Theoutermost zone of the lesion consists of fibrous transcribed spacer in the ribosomal gene complex is
tissue. now available for molecular detection and
Type II tissue reaction, the neutrophils largely identification of the causative organism to understand
disappear and are replaced by macrophages and the disease aetiology, epidemiology and organisms
multinucleated giant cells. The latter engulfs the grain taxonomy, which ultimately improves patients’ care.
562 DYP SURVIVAL GUIDE FOR POST GRADUATES DYP SURVIVAL GUIDE FOR POST GRADUATES 563

Q. Explain Serodiagnosis in Mycetoma. Q. Why is ketoconazole not used these days?


A. In the absence of the classical triad of mycetoma, the A. Its because of side effects like hepatotoxicity,
demonstration of significant antibodies titres against gynaecomastia, lip dryness and ulceration, skin
the causative organism may be of diagnostic and darkness and decreased libido.
prognostic value. The common sero-diagnostic Q. Is Ketoconazole and Itraconazole curative?
techniques in use are counter-immuno-pheresis and A. In most eumycetoma patients they only help in
ELISA. These tests are tedious, need purified antigens localizing the disease by forming well localized,
and hence it is time consuming with cross reactivity thickly encapsulated lesions which are easily excised
between the different organisms commonly occurring surgically.
Q. How do you manage mycetoma? Q. What is the duration of treatment?
A. Combined medical and surgical treatment is the gold A. Medical treatment for both types of mycetoma must
standard in mycetoma. This regime facilitatessurgery, continue until the patient is clinically, radiologically,
accelerates healing and reduces the chance of relapse. ultrasonically and cytologically cured.
Combined drug therapy is always recommended to Q. What will suggest radiological cure?
avoid drug resistance and for disease eradication.
A. Reappearance of normal bone pattern and the
They are given in a form of cycles; each one consists disappearance of the soft tissue mass.
of Amikacin Sulfate in a dose of 15 mg/ kg twice
Q. What are the indications of Surgery for Mycetoma?
daily for three weeks and Co-tri-moxazole in a dose
of 1.5 mg/kg twice daily for five weeks; the cycles A. - small localized lesions
are repeated till cure.The number of cycles ranges - resistance to medical treatment
between five and ten cycles. - for better response to medical treatment in
Second line of treatment: Amoxicillin-Clavulanic patients with massive disease.
Acid, Rifampicin, Sulphonamides, Gentamicin, and Q. What are the surgical options?
Kanamycin. However, these drugs take a long time A. - wide local and debulking excisions
to achieve cure, the mean duration is around one year
- amputations.
and the recurrence rate was high.
Q. When is amputation indicated?
Q. Treatment of Eumycetoma
A. In advanced mycetoma not responding to medical
A. 400-800 mg/day Ketoconazole,
treatment with severe secondary bacterial infection.
200-400 mg/day Itraconazole or 300-400 mg/day As it can be a life saving procedure. The amputation
Voriconazole nine to twelve months. rate ranges from 10-25%.
564 DYP SURVIVAL GUIDE FOR POST GRADUATES

Q. Why is local anaesthesia contra-indicated in


mycetoma?
60. NEVUS COMEDONICUS
A. It is because the disease extension along tissue planes Q. What is the other name of Nevus Comedonicus?
is usually massive and un-predictable.
A. - It is also know as comedo nevus
Q. Why is it mandatory to use a tourniquet when
- It is a rare hamartoma of the pilosebaceous unit,
operating?
resulting in numerous dilated keratin filled
A. A bloodless operative field using a tourniquet is comedones
mandatory to identify the lesion margins to avoid
Q. Describe epidemiology of Nevus Comedonicus?
bursting, which will facilitate local disease spread.
The latter is an important cause of recurrence. A. - Most of the cases are evident at birth, with others
appearing during childhood, usually before ten
Q. What is the cause of recurrence in mycetoma?
years of age
A. The postoperative recurrence rate varies from 25 to
- Onset in adulthood is associated with trauma,
50% and this can be local or distant at the regional
lymph nodes. This could be due to the disease - There is no racial or sexual predilection
biology and behaviour, inadequate surgical excision Q. What is the pathogenesis behind development of
due to the use of local anaesthesia and lack of surgical Nevus Comedonicus?
experience and drug compliance due to financial A. - It is thought to represent growth dysregulation
reasons and/or lack of health education. affecting the mesodermal portion of the
Q. Why is post-operative medical treatment essential? pilosebaceous unit
A. Because if the treatment is stopped after apparent - The epithelial lined invaginations, are incapable
cure, recurrence may be immediate. Follow up of of forming mature terminal hairs and sebaceous
patients with mycetoma must be long enough to glands. This results in accumulation of a soft
detect early recurrence and to advise early treatment. cornified ostial product resulting in a comedo-
like plug
h - A FGFR2 mutation was also detected in the
nevus skin .
Q. What are the clinical features of nevus comedonicus?
A. - Single circumscribed area or linear streak
composed of clusters of dilated follicular ostia
containing firm darkly pigmented, cornified
material

565
566 DYP SURVIVAL GUIDE FOR POST GRADUATES DYP SURVIVAL GUIDE FOR POST GRADUATES 567

- Multiple linear plaques may be seen with Q. What are the other names of epidermal cyst?
midline demarcation A. Epidermoid cyst , infundibular cyst , epidermal
- Size ranges from few cm to extensive lesions inclusion cyst .
covering half of the body Q. Describe epidermoid cyst?
- Sites: face, trunk, neck and upper limb A. - Well demarcated dermal nodules which have a
- Hormonal influence on the nevus can worsen the visible central punctum representing the follicle
presentation from which cyst is derived, in size they are a
few, mms – cms in size
Q. What is the histopathology ?
- Tiny cysts are also known as milia
A. - Underdeveloped hair follicles
- They may be primary or secondary to disrupted
- Dilated invaginations filled with cornified debris follicular structures or traumatically implanted
devoid of hair shafts epithelium. Multiple epidermoid cyst can occur
- Epidermolytic hyperkeratosis may be seen in individuals with acne vulgaris
Q. What is the differential diagnosis? - Rupture of cyst can be painful
A. 1. Familial dyskeratotic comedones is a rare - Development of basal cell carcinoma is rare
autosomal dominant disorder in which Q. What is the histopathology of epidermoid cyst?
comedones arise during childhood and are A. - Cystic cavity is filled with laminated keratin lined
widely scattered on the trunk and are not linear by stratified squamous epithelium including a
2. Dilated pore nevus resembles nevus granular layer
comedonicus clinically but differs histologically - In individuals with Gardner Syndrome, some
by containing dilated follicular cysts cysts show columns of pilomatricoma like
Q. What is the treatment of nevus comedonicus? shadow cells projecting into the cyst cavitys
A. - Localized lesions can be surgically excised: it is Q. What is the treatment ?
difficult to excise larger lesions A. - Removal by excision is curative
- Manual comedo extraction - Incision and expression of the cyst contents is
- Dermabrasion done
- Keratolytic agents (salicylic acid, tretinoin and - If the entire cyst wall is not removed it may recur
ammonium lactate). - Inflamed lesions require intralesional triamcinolone
and also excision with antibiotic therapy
- Isotretinoin is not beneficial except for
preventing cyst formation h
DYP SURVIVAL GUIDE FOR POST GRADUATES 569

Q. Describe the histology of nevus depigmentosus?


61. NEVUS DEPIGMENTOSUS
A. A normal to reduced number of melanocytes may
Q. Why is this nevus depigmentosus? be observed, but DOPA reaction shows them to be
less reactive.
A. Asymptomatic depigmented macule of a variable
size with regular or serrated borders found on the Q. What is the electron microscopy of nevus
depigmentosus?
trunk, neck, face and proximal part of extremities and
it does not cross the midline. A. It shows stubby dendrites of melanocytes containing
autophagosomes with aggregates of melanosomes.
Q. Differential diagnosis?
Q. What is the treatment of nevus depigmentosus?
A. - Nevus anemicus (diascopy is positive),
A. - No treatment is necessary.
- Ash leaf macule
- For small lesions, plastic surgery may be
- Aplasia cutis (lesion is atrophic).
considered.
• In older children:
- segmental vitiligo and leprosy to be ruled h
out.
Q. What is nevus depigmentosus?
A. The disorder represents a congenital, stable and
leukodermic macular lesion that may occur in all
races in both sexes.
Q. What are the clinical features of nevus
depigmentosus?
A. It is an asymptomatic depigmented macule of a
variable size with regular or serrated borders present
on any region of the body, but is commonly found
on the trunk, neck, face and proximal part of
extrimities. It does not cross the midline. It may be
focal, segmental or generalized in distribution.
Q. What is the pathology of nevus depigmentosus?
A. It is condition due to aggravation of melanosomes in
melanocytes and a defect in transfer of melanosomes
to keratinocytes .
568
DYP SURVIVAL GUIDE FOR POST GRADUATES 571

Q. Why is there a characteristic blue color imparted to


62. NEVUS OF OTA AND the lesions?
NEVUS OF ITO A. The blue colour is due to melanin producing
melanocytes in the dermis, in which the Tyndall effect
Q. What is the classical presentation of a patient with occurs.
Nevus of Ota? Q. The lesions are located in the distribution of which
A. The presenting feature is a unilateral patch of nerve?
speckled brown, blue, black or purple discoloration A. Ophthalmic and Maxillary divisions of the
in the periorbital, temple, forehead, malar, earlobe Trigeminal nerve.
or nose regions. Q. Tanino’s classification
Q. What are Nevus of Ota and Nevus of Ito? • According to extent of cutaneous involvement:
A. These are dermal melanocytic nevi arising due to the I: IA- eyelids, periorbital, temporal region
failure of complete migration of melanocytes into the IB- infraorbital, nasolabial fold, zygomatic
epidermis. region
Q. Other names ? IC- Forehead
A. - Nevus fusco ceruleus-opthalmo-maxillaris: ID- Nasal region
Nevus of Ota II: Eyelids, periorbital, temporal, zygomatic
- Nevus fusco ceruleus-acromio-deltoidius: cheek, temporal region
Nevus of Ito III: Scalp, forehead, eyebrow, nose region
Q. What can be the differentials for these two IV: Bilateral
conditions? Q. What is the morphological classification?
A. 1) Mongolian spot – seen in mongoloid/black A. 1) Homogenous
infants over sacral region 2) Speckled
2) Blue Nevus – can be found in oral mucosa, 3) Mixed
uterine cervix, prostate Q. Describe the lesions in the eye.
3) Melasma – brown, sparing of sun-protected sites, A. The ipsilateral eye shows a diffuse bluish
no mucus memb. involved discoloration of the sclera and a patchy brownish
4) Hori’s Nevus color of the conjunctiva.

570
572 DYP SURVIVAL GUIDE FOR POST GRADUATES

Q. Which ophthalmological condition may be


associated with Nevus of Ota?
63. NEVUS SEBACEOUS
A. Glaucoma may be seen in 10% of patients with Nevus
of Ota. Q. Why is this nevus sebaceous?
A. Yellowish lobulated plaque with areas of alopecia.
Q. How does Nevus of Ito differ from Nevus of Ota? Q. Who described nevus sebaceous?
A. - Nevus of Ito differs from Nevus of Ota in terms A. - First described by Jadassohn and is also known
of area of involvement as organoid nevus
- Nevus of Ito involves the supraclavicular, - It is not a sebaceous malformation, but a
scapular, and deltoid regions, which are not hamartoma that exhibits follicular , sebaceous
involved in Nevus of Ota. and apocrine malformation .
Q. What is Hori’s Nevus? Q. What are the clinical features of nevus sebaceous?
A. Hori’s nevus is characterized by bilateral blue-gray
A. - It is a congenital malformation, the lesion is only
to gray-brown macules of the zygomatic region,
slightly raised.
forehead, upper outer eyelids, and nose.
Q. What are the histo-pathological findings in Nevus - It involves the scalp , neck and rarely trunk .
of Ota? - The lesions are distributed along the lines of
A. The non-infiltrated areas show pigmented, elongated, Blaschko.
stellate, dendritic melanocytes scattered among the - When present on scalp, the area involved is
collagen bundles. hairless.
Q. What are the treatment options for a patient with - Upon reaching adolescence, there is thickening
Nevus of Ota and Ito? and pebbling of the surface and lesions may also
A. 1) cosmetic camouflage become verrucous.
2) CO2 snow alone or with dermabrasion - Secondary neoplasms may also occur, 1 % of
3) Liquid Nitrogen therapy cases develop BCC.
4) Q-switched ruby, alexandrite, and Nd:Yag lasers - Other neoplasms are trichilemomma , sebaceous
Q. What is Panda’s sign? adenoma , apocrine adenoma and poroma .
A. Post laser treatment the periorbital pigmentation Q. What is the histopathology?
remains looking like a Panda’s eyes. Hence the name A. - Papillated epidermal hyperplasia, inter-
Panda’s sig.
anastamosis of rete and a coarsely fibrotic
h papillary dermis.

573
574 DYP SURVIVAL GUIDE FOR POST GRADUATES

- There are enlarged sebaceous lobules and buds


of follicular germinative cells.
64. NEVUS SPILUS
- The verrucous surface of nevus sebaceous is Q. What is nevus spilus?
histologically similar to verruca vulgaris.
A. It's a usually unilateral, segmental tan patch with
Q. What is the treatment ? superimposed speckles, that develop over time.
A. - A conservative complete excision can be done (spilus= speckled).
for the cosmetic purpose as verrucous lesions are
Q. What are the differential diagnoses?
highly unsightly.
A. Agminated nevi [junctional, compound, spitz, blue
- Removal by shave or laser ablation is usually not
nevi]
successful. It is one of the lesions where early
excision is rcommended though the chances of Q. What is the etiology of nevus spilus?
malignancy are rather low. A. There is controversy whether it is congenital or
acquired, however there is evidence that it is a
h congenital melanocytic nevus.
Q. What is the pathogenesis of nevus spilus?
A. - It may have segmental or zosteriform
distribution suggesting a localized 'field defect'.
- A nevus spilus has been linked to a garden of
melanocytes in which any type of nevus can
develop, simultaneously or sequentially.
Q. What are the clinical features of nevus spilus?
A. - These lesions most commonly affect the trunk and
extremities.
- The tan macular area commonly ranges from 1
to 4 cm in diameter.
- The darker speckles are approximately 1 to 6 mm
in greatest Diameter and may be macular or
papular.
- Larger varieties of nevus spilus may be
unilateral, segmental or follow Blaschko's lines.

575
576 DYP SURVIVAL GUIDE FOR POST GRADUATES

Q. What is the histopathological findings in nevus


spilus?
65. NEUROFIBROMATOSIS
A. - Lentiginous melanocytic hyperplasia
Q. Define Neurofibromatosis:
- Elongated epidermal rete ridges
A. Autosomal-Dominantly inherited Neurocutaneous
- The papular foci represent junctional,compound, disorder or Phakomatoses characterised by
blue, Spitz, and/or atypical nevi. cutaneous lesions as well as central and peripheral
Q. What is the treatment of nevus spilus? nervous system neoplasm.
A. - individuals be followed on a periodic basis Q. Define Phakomatoses:
- Areas of change or atypicality in a nevus spilus A. Inherited disorders of CNS that have congenital
should be evaluated by biopsy or excision retinal tumors and cutaneous involvement.
Includes:
h 1] Neurofibromatosis
2] Tuberous Sclerosis (Epiloia, Bourneville
Disease)
3] Ataxia Telangiectasia
4] Von Hipple Lindau Disease (Angiomatosis
Retinae)
5] Sturge Weber Syndrome
6] Nevoid Basal Cell Carcinoma Syndrome.
7] Nevus sebaceous syndrome
8] Epidermal nevus syndromes
Q. Classify Neurofibromatosis:
A. Riccardi Classification:
- NF-1- Von Recklinghausen Disease
- NF 2- Bilateral Acoustic Schwannoma
- NF 3- Mixed
- NF 4- Variant
- NF 5- Segmental Café-au-lait macules or
neurofibromas or both in one or more
dermatome. (Segmental NF-1)
577
578 DYP SURVIVAL GUIDE FOR POST GRADUATES DYP SURVIVAL GUIDE FOR POST GRADUATES 579

- NF 6- Café-au-lait macules only. - Sharply defined, light-brown macules


- NF 7- Late Onset that vary in size from 0.5 to 5 cm
- NF-NOS- Not otherwise specified. - The macules increase in size and number
during the first decade
Q. Describe Neurofibromatosis 1?
- Number is not indicative of clinical
A. - Also known as Von Recklinghausen’s severity, but it is important for diagnosis
neurofibromatosis as first described by Von
b) Axillary and/or Inguinal freckling(~80%)
Recklinghausen in 1882.
- Also known as Crowe’s Sign
- It accounts for 90% of neurofibromatosis
- Yesudian’s sign: Freckling over palms
- Incidence: approx. 1 in 3000
c) Neurofibromas (60-90%)
- It is the commonest phakomatoses
- CNFs are skin-colored to pink, tan or brown,
Q. Which gene is involved in NF-1?
polypoid or pedunculated nodules, soft /
A. NF-1 gene is located on chromosome 17q11.2 and slightly rubbery and range from a few mm
encodes for protein Neurofibromin. (p stands for to several cm in diameter.
the short-petit arm of the chromosome and q stands - May be cutaneous or subcutaneous.
for the long arm).
- ‘Button-Hole’ sign- Upon applying pressure
Q. Describe the pathogenesis of NF-1? to the neurofibroma, it completely goes
A. - Neurofibromin gene is a tumor suppressor inside the skin and again reappears on
gene. releasing the pressure.
- The neurofibromin protein shows significant - Usually asymptomatic but sometimes
regions of similarity to the GTPase-activating becomes pruritic also.
protein (GAP related protein) and is capable of - Appear as early as 4–5 years of age but
down-regulating Proto-oncogene Ras activity. increase in number during puberty with
- RAS proto-oncogene when activated and turned acceleration in pregnancy.
to oncogene causes tumorogenesis. d) Plexiform Neurofibroma (25%) [Plexiform- web-
Q. Describe the clinical features of NF-1? like]
A. 1] Cutaneous: - Common along branches of trigeminal and
cervical nerves, congenital in origin but
a) Cafe-au-lait macules (>90%)
become apparent at birth.
- Earliest skin lesion
- Large, diffuse elongated lesion with bag-of-
- Present at birth or appear in first year worms feel.
580 DYP SURVIVAL GUIDE FOR POST GRADUATES DYP SURVIVAL GUIDE FOR POST GRADUATES 581

- +/- overlying hyperpigmentation and b) SPINAL


hypertrichosis. - Scoliosis(5-10%)
- Elephantiasis neurofibromatosa-overgrowth
- Spina Bifida
of the subcutaneous tissue and of the skin
pendulous and wrinkled. c) LIMBS (congenital)
- Complication-May involve internal organs - Dysplasia of long bones; most commonly
especially spinal cord deformity (dumb bell Tibia (14%)
shaped NF).. Involvement of nerves leads to - Pseudoarthrosis (2%0
neurological deficit. Can undergo
sarcomatous change. - Thinning of long bone cortex
2] OCULAR: - Local bony overgrowth
a) LISCH NODULES (90%) - Absent Patellae
- Pigmented iris hamartomas appear as dome- 6] TUMORS:
shaped yellow-brown papules on surface of
- Optic Glioma (10-15%) +/- Precocious
iris on slit-lamp examination and rarely may
be visible to naked eye also. puberty
- Asymptomatic. - Malignant Peripheral nerve sheath tumor(3-
- Do not occur in segmental or NF-2 15%)
b) HYPERTELORISM (25%) - pheochromocytoma (1%)
c) GLAUCOMA - Juvenile Myelomonocytic leukaemia
3] ORAL LESIONS: (especially in patients with concomitant
juvenile xanthogranuloma)
- Papillomatous tumours of palate, buccal
mucous membrane, tongue and lips, - Astrocytoma, Ependymoma, Schwannoma
- Macroglossia, usually asymmetrical. - Rhabdomyosarcoma, especially genitourinary
4] SCALP: - Duodenal Carcinoid
- Cutis verticis gyrate (vertical longitudinal - Somatostatinoma
thickening of skin of scalp).
- Parathyroid adenoma
5] SKELETAL:
- Wilms Tumor
a) CRANIAL
- Macrocephaly (20-50%) - Malignant Melanoma
- Sphenoid wing dysplasia (<5%) - Retinoblastoma
582 DYP SURVIVAL GUIDE FOR POST GRADUATES DYP SURVIVAL GUIDE FOR POST GRADUATES 583

7] ENDOCRINE DISTURBANCES: with size 5 mm or more in children)also called


- Precocious puberty Crowe’s law)
— Two or more neurofibromas of any type or one
- Gynaecomastia
plexiform neurofibroma
- Acromegaly — ‘Freckling’ in the axillary or inguinal regions
- Addison’s disease — Optic gliomas
- Cushing syndrome — Two or more Lisch nodules (iris hamartomas)
- Hyperparathyroidism — Osseous lesion, such as sphenoid wing
- Pheochromocytoma dysplasia or thinning of long bone cortex, with
or without pseudarthrosis
8] NEUROLOGICAL:
— First-degree relative (parent, sibling or offspring)
- Unidentified bright object on MRI (50-75%) with NF1 by the above criteria
of unknown significance.
Q. Which investigation to be carried out in NF-1?
- Learning difficulties (30-50%)
A. - Diagnosis is mainly clinical
- Seizures (~5%)
- Thorough clinical examination of patient and
- Mental Retardation (severe, <5%) family member
- Aqueductal stenosis and Hydrocephalus - Slit Lamp examination of eye
(~2%)
- Skeletal survey
9] CARDIOVASCULAR:
- Neurological imaging- CT Scan and MRI
- Hypertension (~30%), mostly essential due
depending upon clinical features
to renal artery stenosis or
pheochromocytoma - Prenatal Screening
- Pulmonic stenosis Q. What are differential diagnosis of Cafe-au-lait
macule?
- Renal artery stenosis
A. - Normal Individuals (10-20%)
Q. What are the diagnostic criteria for NF-1?
- Noonan syndrome
A. • Suggested by National Institute of Health
Consensus Development Conference, 1988 - Ataxia telangiectasia
Two or more of the following must be present: - Piebaldism
— Six or more café-au-lait macules >5 mm in - LEOPARD Syndrome
prepubertal individuals and >15 mm in - McCune Albright Syndrome
postpubertal individuals and 5 or more CALM - Bloom’s Syndrome
584 DYP SURVIVAL GUIDE FOR POST GRADUATES DYP SURVIVAL GUIDE FOR POST GRADUATES 585

- Fanconi’s Anaemia Q. Which antihistiminics are used in pruritus of NF?


- Cowden’s Syndrome A. Mast cell stabilizers.
- Tuberous Sclerosis Q. What is the another name for NF-2?
- Watson Syndrome A. Bilateral acoustic schwannoma.
- MEN Syndrome Q. Which gene is involved in NF-2?
Q. What is the course and prognosis of NF-1? A. Gene on chromosome 22q11, encodes for protein
Neurofibromin 2 or Merlin or Schwanomin.
A. - Course- variable and unpredictable
Q. Describe NF-2:
- Psychological morbidity due to unsightly
appearence A. - Autosomal Dominant Inheritance
- Early onset, rapid progression before puberty - incidence 1 in 40000
and extensive involvement- Poor Prognosis. - Characterised by Bilateral Acoustic
- Enlargement and pain in individual lesion - Schwannoma
Haemorrhage or malignant transformation. - Associated with other meningeal and glial
- Sarcomatous change in neurofibromas occur in tumors
1.5-15% cases. - Vestibular lesions manifest in third decade of
life
- Highly aggressive malignant peripheral nerve
sheath tumor may occur. In women, screening - Juvenile Posterior Subcapsular Cataract
to be done for Breast carcinoma - CALM (fewer as in NF 1) are seen
Q. How will you manage NF-1? - Cutaneous neurofibroma are seen
A. - Treatment is mostly symptomatic. Café–au-lait - No Lisch nodules, MR, Learning disability
can be masked by cosmetics Q. What are the diagnostic criteria for NF-2?
- Large disfiguring or rapidly progressive A. Either of Two:
painful lesions- Excision or carbon dioxide laser 1] Bilateral eighth nerve masses as demonstrated
- Pruritis- Antihistaminics on CT or MRI.
- Orthopedic intervention 2] A First degree relative with NF2 and either
- Life long surveilliance usually an annual U/L eighth nerve mass or two of following-
assessment by neurologist, ophthalmologist, Neurofibroma, Meningioma, Glioma,
regular blood pressure monitoring Schwanoma, and juvenile posterior subcapsular
opacity.
- Counselling
586 DYP SURVIVAL GUIDE FOR POST GRADUATES

Q. Describe the clinical features of Segmental NF:


66. NIPPLE ECZEMA, HAND
A. Either cutaneous neurofibroma or cafe-au-lait
macule present or both present in segmental ECZEMA AND POMPHOLYX
distribution.
Q. If one parent is affected with Neurofibromatosis then Q. Define dyshidrotic eczema?
what is the chance of offspring being affected with A. This form of eczema is characterized by vesicle and
Neurofibromatosis? bullae on palms and soles. When it involves palms it
A. As Neurofibromatosis is transmitted as an is called cheiropompholyx, and when it involves
Autosomal Dominant disorder, so if one parent is soles it is called podopompholyx.
affected then there is 50% chance that the child will Q. What is pompholyx?
have neurofibromatosis. A. - Pompholyx or dyshidrotic eczema,account for
Q. If parents do not have NF, then what is the incidence 5%-10% of hand eczema. There is a family history
that the child will have NF? or a personal history of atopy. Direct allergens
A. - If both the parents are normal then incidence of implicated are primin, isopropyl-para-
child getting NF is equal to that in normal phenylenediamine, benzoiso-athiazolones and
population i.e., 1 in 3000 for NF-1 and 1 in 40000 dichromates, perfumes, fragrances, balsam
for NF-2. ingredients.
- It is due to sporadic mutation. - They cause palmar vesicles, appearing in crops
Q. Is it difficult for a women with NF to conceive? on palms and sides of fingers. Vesicles are very
itchy, deep seated and look like sago grains.
A. Yes. This is because NF patients frequently have They dry up and overlying skin peels off.
leiomyoma of uterus which makes it difficult for Lymphangitis and cellulitis can occur. Rubbing
women with NF to conceive. can produce secondary eczematous changes.
Nail changes can occur.
h
- Vesicles are seen over palms and soles. Nickel
sulphate was the commonest allergen (14%)
followed by potassium dichromate,
nitrofurazone. Patch testing is positive for one
or more allergens.
Q. What is the treatment of pompholyx?
A. - Severe cases require soaks and compresses of
potassium permanganate (1:8000) or burrows
solution 1%.A soothing lotion like calamine
587
588 DYP SURVIVAL GUIDE FOR POST GRADUATES DYP SURVIVAL GUIDE FOR POST GRADUATES 589

lotion is given. Blister fluid can be aspirated with be required and breastfeeding is suspended for
syringe. Topical steroids are useful in subacute the period. Pumping and the silicon nipple
and chronic cases. Topical tacrolimus shield are useful. Oral fluconazole 150 mg is
0.1%ointment with mometasone furaote can be effective in case of candidial infection. Topical
used. gentian violet 0.5%, mupirocin 2%,
- Systemic treatment- short course of steroids is betamethasone 0.1% ointment is effective.
used. Sedative antihistaminics are used. Low - Lactating consultant or nurse may be helpful in
dose methotrexate (5mg/week) and dapsone are managing these patients, since poor positioning
given. during breast-feeding is the co -factor.
- Other therapy are: local photochemotherapy, Q. What are the types of hand eczema?
intradermal botulinum toxin can be given.Tap A. a. Allergic contact dermatitis
water ionotophoresis can be used. b. Irritant contact dermatitis
Q. Describe nipple eczema? c. Atopic hand eczema
A. - Eczema of the breast affects the nipple, the d. Vesicular or vesiculobullous
areolae and may extend to surrounding skin. e. Hyperkeratotic endogenous hand eczema
- Rarely seen in men. Eczema of the nipple is the Q. Which is the strong risk for hand eczema?
moist type with oozing and crusting. Painful
A. Wet work (contact with liquid or gloves for more
fissuring is seen. Atopic dermatitis is a frequent
than 2 hrs per day or hand-washing more than 20
cause.
times per day) is a strong risk factor.
- It can be the only manifestation of atopy. Other
Q. What are the high risk occupations for hand eczema?
causes of nipple eczema are allergic contact
dermatitis, (jogger’s dermatitis), or from ill-fitting A. High risk occupations: bakers, hairdresser, dental
brassieres with seams in women with surgeon, kitchen worker/cooks, butchers, healthcare
asymmetrical large breast. workers, cleaners, doctor/dentist/veterinarians, and
laboratory technicians
Q. What are the signs of Pagets diease in breast?
Q. What are the allergens in hair dressing ?
A. - If eczema of nipple or areola has persisted for A. Glyceryl monothioglycolate, ammonium persulfate.
more than 3 months and unilateral, a biopsy is
Q. What is the allergen in cement?
to be done , to rule out Pagets diease. Retraction
of nipple is an ominous sign. A. Chromate and cobalt
Q What is the systemic treatment of hand eczema?
Q. Treatment of Nipple eczema?
A. Methotrexate , azathioprine and mycophenolate
A. - Oral antibiotics are preferred treatment for
mofetil (1-1.5g twice a day for adult), cyclosporine,
secondary bacterial infections. Analgesia may
oral retinoids, Alitretinoin 30 mg /day.
590 DYP SURVIVAL GUIDE FOR POST GRADUATES

Q. What are the barrier products used?


67. NUMMULAR ECZEMA
A. Foams containing dimethicone and glycerin.
Q. What is the topical treatment of hand eczema? Q. What are the other names for Nummular Eczema?
A. - Moisturizing is the critical component. A. Discoid Eczema, Nummular Dermatitis, microbial
Application of protective moisturizing cream or eczema
ointment after each hand wash. Petroleum jelly
Q. What is the meaning of the word Nummular?
is also used.
A. Coin-shaped
- Superpotent or potent topical steroids. Addition
of 2.5% zinc sulfate to clobetasol enhances the Q. Define Nummular Eczema?
efficacy of topical steroids. Soaks of tar bath oil A. It is a condition characterized by oozing
or application of 20% liquor carbonis detergent papulovesicular lesions or erythematous scaly
or 2% crude coal tar in an ointment base. eczematous lesions which are coin shaped with a clear
- Phototherapy: High dose of UVA-1, soaks or demarcated edge
cream PUVA and oral PUVA. Superficial Grenz Q. Which are the most common sites to be affected?
ray radiotherapy , in dose of 200cG are delivered
A. Usually extensors of limbs
at weekly interval for total 800-1000cG. Therapy
is repeated after months. Total lifetime dose Q. Age group most commonly affected?
should not exceed 5000cG A. More common in older age group ( above 50yrs )
h Q. Factors implicated in the etiology?
A. Xerotic Skin, Atopic Diathesis, Neurogenic
Contributors (like Substance P & Calcitonin gene
related peptide positive nerve fibres), alcohol,
allergic contact dermatitis ( to rubber, nickel,
fragrances, mercury), sensitization to certain
substances & drugs ( gold, dental amalgam, methyl
dopa)
Q. What are the clinical features?
A. Initially erythematous papules or vesicles which
coalesce to form large round lesions. Later lesion
starts oozing and crust is formed. In late stages,
lesions become dry, scaly, less erythematous that may
appear annular. Usually symmetrically distributed
591
592 DYP SURVIVAL GUIDE FOR POST GRADUATES DYP SURVIVAL GUIDE FOR POST GRADUATES 593

over extensors of limbs, occasionally trunk.Itching is - Antihistaminics


the main complaint.Hyperpigmentation or - Antibiotics
lichenification of skin can occur due to constant
- Topical emollients
scratching.
- Oral steroids( 40mg/day)
Q. What are the histopathological findings?
- Hand eczema
A. - New lesions: Spongiosis,Intraepidermal vesicles,
Papillary oedema, Perivascular infiltrate. Q. What are the morphological pattern of hand eczema?
- Old lesions: Psoriasiform epidermal changes, A. Pompholyx, hyperkeratotic palmar eczema, ring
Focal spongiosis, Mixed infiltrate with few eczema, fingertip eczema, housewife’s eczema, apron
eosinophils, Alternating ortho&parakeratosis eczema, discoid eczema, juvenile plantar dermatosis
Q. Classification of hand eczema?
Q. What is the prognosis?
A. Endogenous : atopic dermatitis
A. Chronic course, partial remission, relapses common,
worsens in winters Discoid eczema
Q. How to Diagnose? Pompholyx
A. Mainly clinical Palmoplantar pustulosis
Patch testing Hyperkeratotic eczema
Q. DD? Exogenous: irritant contact dermatitis
A. - Allergic Contact Dermatitis Allergic contact dermatitis
- Irritant Contact Dermatitis Dissemination from a focus
- Tinea Corporis Systemic allergen
Infective dermatitis
- Polymorphous Light Eruption
Q. Write a note on housewife’s eczema?
- Psoriasis
A. - It is a chronic, cumulative irritant dermatitis
- Atopic Dermatitis
caused by household contact with soap, soda,
- Mycosis Fungoides detergent, cleanser
- Autoeczematization - Friction,trauma,cold and heat play a part.
Q. Treatment? Atopics are more vulnerable. It develops usually
A. - All causative agents should be removed after marriage or childbirth
- Topical Steroids - It commonly occurs on palmar surface of fingers,
interdigital spaces, palms, and dorsal aspects of
- Coal tar ointments fingers mostly knuckles
594 DYP SURVIVAL GUIDE FOR POST GRADUATES

Q. What is ring eczema?


68. OTHER DEEP FUNGAL
A. This pattern of eczema affects young women. An itchy
patch of eczema appears beneath a ring finger and INFECTIONS
spreads to adjacent fingers and the palmar area. Very
rarely radioactive gold in the ring may cause Q. What is the causative agent of sporotrichosis?
radiation dermatitis called “wedding ring
A. Sporothrix schenkii
dermatitis”.
Q. What do you mean by dimorphic fungi?
A. Fungi which is seen in yeast and mycelial forms.
h
Q. What is dematiaceous fungi?
A. The pigment producing fungi.
Q. Is there any seasonal variation in the incidence of
sporotrichosis?
A. Highest in the first half of winter in which there is
high humidity and temperature is between 16-22
degree celsius because these conditions favour
saprophytic growth of S.schenkii.
Q. Why is history of occupation important?
A. After trauma, it is common in:
- Florists and gardeners(rose thorns)
- Forest tree workers
- Workers using straw as packing material
- Mine workers(decaying vegetable matter
eg.timber with soil as a source)
Q. Predisposing factors?
A. - Trauma- thorn/splinter/insect bite
- Adult males by view of occupation
- Walking barefoot(farmers)
- Alcoholics(systemic sporotrichosis)

595
596 DYP SURVIVAL GUIDE FOR POST GRADUATES DYP SURVIVAL GUIDE FOR POST GRADUATES 597

Q. What are the clinical features of systemic Q. Types of sporotrichosis?


sporotrichosis? A. - Cutaneous
A. Ulcerated nodules on body/mucous membrane - Fixed
lesions and visceral lesions.(bone/joints/lung/ - Lymphangitic(most common type is localised
meninges) lymphatic starting as pustule/nodule)
Q. Which animal can transmit sporotrichosis? - Atypical(Mycetoma like or cellulitic forms)
A. Cat has been reported to transmit sporotrichosis. - Systemic
Q. Portal of entry in systemic sporotrichosis? Q. What are the clinical types of fixed sporotrichosis?
A. Lung A. 1. Acneiform
Q. What is the incubation period? 2. Nodular
A. 8-30 days 3. Ulcerated
Q. Mode of spread? 4. Verrucous
A. 1. Localised to subcutaneous tissue 5. Infiltrated plaques(high immunity) or red scaly
2. Spreads via subcutaneous lymphatics patches.
Q. Why do some get lymphangitic and some fixed form
3. Disseminates in blood stream
of sporiotrichosis?
Q. What are Asteroid bodies? In which conditions are
A. Depends on hosts immune response and changes in
they seen?
temperature sensitivity of the organisms.
A. They are small cigar shaped or oval yeasts
Q. Which mycobacterial infection closely resembles
surrounded by thick radiate eosinophilic substance
lymphangitic sporotrichosis?
and are seen in
A. Mycobacterium marinum causing fish tank
1. Sporotrichosis granuloma
2. Sarcoidosis Q. Describe the colonies?
3. Tuberculosis A. Leathery moist, initially white later black due to
Q. What are the two reasons suggested to be the cause production of conidia and partly due to addition of
of asteroid bodies? thiamine to media.
A. 1. Trapped collagen bundles Microscopy- slender hyphae bear small oval/
2. Some components of cytoskeleton, pyriform conidia and conidiosphores arising at right
predominantly vimentin intermediate filaments. angles from the hyphae. Cigar shaped or oval yeasts
598 DYP SURVIVAL GUIDE FOR POST GRADUATES DYP SURVIVAL GUIDE FOR POST GRADUATES 599

on brain heart infusion agar incubated with sheeps Q. In which deep fungal infections can koebnerisation
blood at 37 degree Celsius. be seen?
Q. How do you treat? A. Chromoblastomycosis.
A. - Itraconazole 100mg-200mg daily Q. What are the complications?
- Terbinafine 250mg/day A. - Secondary infection- itching/pain
- Potassium Iodide 5 drops initially increasing to - Elephantiasis(secondary infection by
4-6 ml of SSKI three times a day for 3-4 weeks
after clinical cure. lymphaticstasis)
- i.v. Amphotericin B - Squamous cell carcinoma
- Local heat application (also in - Brain abscess(haematogenous spread)
chromoblastomycosis) Q. How do you differentiate it from blastomycosis?
Q. What are the agents causing chromoblastomycosis? A. By the absence of sharp border containing minute
A. Phialophora verrucosa, Fonsecaea pedrosoi, abscesses and also absence of pulmonary lesions.
Fonsecaea compacta, Cladophialophora carrionii.
Q. What are the DDs of verrucoushyperkeratotic
Q. Why is it common in rural communities? plaques?
A. Walking barefoot predisposes to trauma(splinter of
A. - Verruca vulgaris
wood) specially in adult male agricultural workers
(the organism is seen in wood and soil)and these - Tuberculosis verrucosa cutis
conditions are prevalent in rural areas. - Chromoblastomycosis
Q. Other names? - Leishmaniasis
A. Verrucous dermatitis/chromomycosis - Syphilis
Q. What are muriform or sclerotic cells or Medlarbodies
- Verrucous discoid lupus erythematosis
(Copper penny bodies)?
A. Within giant cells, group of fungal cells which are - Verrucous hemangioma
chestnut or golden brown in colour divided in several - Inlammatory verrucous epidermal nevus
planes of division by thick septa. (deeply pigmented Q. Culture characteristics?
and thick walled)
A. Dark grey/green to black and velvety or dawny with
Q. What are the clinical features?
a black reverse.
A. It starts as a warty papule leading to a verrucous
hypertrophic plaque, ulcereration, Psoriasiform Q. Treatment?
lesions A. - Itraconazole 100mg-200 mg daily
600 DYP SURVIVAL GUIDE FOR POST GRADUATES

- Terbinafine 250 mg daily 69. PARAPSORIASIS


- Flucytosine with amphotericin B
- Better with c.carionni Q. Define Parapsoriasis.
- Cryotherapy/local heat A. It is a Chronic, lymphoproliferative, asymptomatic,
Q. Whatis the indication of surgical excision? erythematous and scaly dermatosis, that includes ,
A. Very small lesions and in these it should be combined small (SPP) and large plaque (LPP), acute and chronic
with chemotherapy.(larger lesions,if excised may lead variants of pityriasis lichenoides and lymphomatoid
to development of satellite lesions). papulosis with significant overlap and progression
Q. What is welsh regime and Damle’s regime? to MF .
A. Welsh Regime: Q. What is D/D of parapsoriasis?
- It consists of Amikacin Sulphate 15mg/kg twice A. a. Mycosis fungoides
daily for 3 weeks and cotrimoxazole in a dose of b. Psoriasis
1.5mg/kg twice daily for 5 weeks, the cycles were
repeated till cure. The number of cycles ranged c. Pityriasis rosea
between 5-10 cycles. d. PR- like drug eruptions
Damle’s Regime: e. Dry nummular dermatitis
- It consists of adition of rifampicin to amikacin f. Secondary syphilis
sulphate and cotrimoxazole combination.
g. MF- like drug eruptions
h h. Autoimmune connective tissue diseases
Q. Who defined parapsoriasis?
A. Brocq in 1902
Q. Classify Parapsoriasis.
A. a. Small plaque Parapsoriasis
b. Large plaque parapsoriasis
c. Pityriasis lichenoides :
- Pityriasis Lichenoides Chronica (Juliusberg)
- Pityriasis Lichenoides et Varioliformis
Acuta (syn : PLEVA, Mucha-Habermann)
601
602 DYP SURVIVAL GUIDE FOR POST GRADUATES DYP SURVIVAL GUIDE FOR POST GRADUATES 603

Q. Clinical features of small plaque? Q. What are the various alternative names for
A. – It is chronic asymptomatic condition parapsoriasis ?
– Persistent small scaly plaques on trunk A. – Chronic superficial scaly dermatitis.
– Persistent superficial Dermatitis
– Peak age being 40-50 years
– Xanthoerythroderma
– It can affect all races with the male female ratio
being 3:1 Q. What are the other names of large plaque
parapsoriasis?
– It consists of round or oval erythematous patches
2-6 cm in diameter A. – Atrophic Parapsoriasis
– Poikilodermatous parapsoriasis
– Slight scaling with digitate appearance
– Poikiloderma vasculare atrophicans
– Poikiloderma
Q. What are the clinical features of large plaque
– The long axis of digitate lesion has has 10-20 cm
parapsoriasis?
forming fingernail appearence.
A. – It is a chronic condition characterised by
Q. What is digitate dermatosis? presence of fixed large atrophic erythematous
A. – Small plaque parapsoriasis with typical finger plaque mostly on trunk and some times limb.
print patches on the trunk which are more than 5 – Ill-defined irregular persistent yellow orange
cm and follows the lines of cleavage of the skin erythematous to brown macules or plaques.
and gives an image of a hug leaving a print. – Epidermal atrophy.
– When lesion has yellowish hue, it is called – Lesions are larger than 6 cms
xanthodermic perstans.
– Later they develop mottled hyperpigmentation
Q. What is the D/D of poikiloderma? and telangiectasias.
A. Large plaque parapsoriasis – Lesions are present on non-sun exposed parts
B. Dermatomyositis also.
C. LE Q. What is parapsoriasis variegata?
D. Chronic radiation dermatosis A. Brownish red scaly papules arranged in a net or zebra-
like pattern on trunk or extremities.
E. Bloom’s syndrome
Q. What are alternative names?
F. Rothmund- Thompson syndrome
A. – Parapsoriasis Lichenoids of Brocq
G. Dyskeratosis congenita
– Reuform parapsoriasis
H. Xeroderma pigmentosum
604 DYP SURVIVAL GUIDE FOR POST GRADUATES DYP SURVIVAL GUIDE FOR POST GRADUATES 605

Q. What is clonal dermatitis ? Q. What is Pityriasis Lichenoides ?


A. – It is a term which includes a variety of A. – Idiopathic acquired dermatosis of unknown
lymphocyte- rich dermatosis often characterised etiology
by clonal T-lymphocyte proliferation which – Acute form is known as PLEVA
includes presentation like primary erythroderma,
Q. Who discovered Pityriasis Lichenoids ?
non-specific chronic spongiotic dermatitis,
A. – PLEVA : Jadassohn and Nieser 1894
follicular mucinosis and large plaque
parapsoriasis and pagetoid reticulosis. It may – PLC : Juliusburg 1899
progress to MF. Q. Other name of PLEVA.
Q. Explain the histopathology of Parapsoriasis. A. Mucha Haberman disease
A. Small Plaque epidermis shows: Q. Epidemology of PLEVA.
– hyperkeratosis A. – young adults
– focal parakeratosis(focal scale crust) – male > female
– mild spongiosis – PLC > PLEVA
The dermis shows: Q. What is the etiology?
– perivascular lymphohistocytosis A. Infections:
– CD4 + T cells – Toxoplasma
Q. Explain the treatment of parapsoriasis. – Cytomegalovirus
A. – Topical emollients – Parvovirus
– Tacrolimus 0.1% – Adenovirus
– Coal Tar – EB virus
– Bexarotene – Streptococci
– Mechlorethamine – Staphylococci
– Steroids Q. PLC synonyms.
– Antihistaminics A. – Chronic guttate parapsoriasis
– UVB therapy – Dermatitis psoriasiform nodularis
– PUVA – Parapsoriasis Chronica
606 DYP SURVIVAL GUIDE FOR POST GRADUATES DYP SURVIVAL GUIDE FOR POST GRADUATES 607

Q. What are the clinical features of PLEVA ? Q. Explain the histopathology of PLEVA.
A. – Acute eruption associated with low grade fever, A. – Epidermal necrosis with hemorrhage and dense
headache, malasia, arthralgia perivascular infiltrate in superficial and deep
– Eruptions begin as crops of erythematous macule dermis.
-> edematous papules -> Central vesiculation – Lymphocytic vasculitis
– itching and burning sensation present – Vacuolar interface dermatitis.
– Lesions present over the trunk, flexures Q. D/ D of PLC :
– Spares the face, palms, soles, scalps, mucous A. – Guttate psoriasis
membrane – Pityriasis rosea
– Heals with varioliform scars, hypo and – Lichen planus
hyperpigmentation
– Secondary syphilis
– Polymorphous lesions are present
– Small plaque parapsoriasis
Q. D/D of varioliform scars.
– Drug eruption
A. – PLEVA
Q. What are the clinical features of PLC ?
– Varicella
A. – It is the more common form of PL.
– Hydroa vacciniforme
– Characteristically it shows a pleomorphic
– Papulonecrotic tuberculides
eruption.
– Disseminated pustular lesions
– Aymptomatic lesions
– Atrophic maculosa varioliformis cutis.
– Red-brown colored oval round lichenoid
Q. What are the D/D of PLEVA ? papules 3-10 mm with centrally adherent mica-
A. – Varicella like scales
– Pyoderma gangrenosum – Scales detached with shiny brown pigmented
– Leucocytoclastic vasculitis surface.
– Lymphomatoid papulosis – Site: trunk, proximal extremities; spares palms
– Gianotti-Crosti syndrome and soles.
– Arthropod bite, sting, infestations – Heals with hyperpigmentation or
– EM, DH, follicilitis hypopigmentation.
– Secondary syphilis – Scarring is unusual.
608 DYP SURVIVAL GUIDE FOR POST GRADUATES DYP SURVIVAL GUIDE FOR POST GRADUATES 609

Q. Explain the histopathology of PLC. – Oral cyclosporine


A. – Compact parakeratosis – Topical Tacrolimus 0.1%
– Exocytosis of lymphocytes causing pseudo- – PUVA
monro abcesses. – NBUVB
– Dermis shows lymphohistocytic perivascular – Severe cases can be treated with Methotrexate
infiltrate. and Etanercept or Dapsone
– Dermal edema. Q. Prognosis ?
– Vacuolar interface dermatitis. A. – Parapsoriasis : persists for years to decades
Q. What is Febrile ulceronecritic Mucha Haberman without any change; 20-30% proceed to MF.
disease (FUMHD) ? – PLEVA- recurrent crops with variable course-
A. – It is a febrile ulcerative acute ulceronecrotic form few months to years; Does not go into MF.
of PLEVA.
h
– Sudden onset of diffuse purpuric papules with
necrotic centres.
– Development of large necrotic ulcers asscociated
with high fever malaise arthralgia.
– GIT and CNS manifestations along with
pneumonitis
– Lymphadenopathy
– Myocarditis
– Oral mucosa may be involved
– Pruritus and pain is present
– Ulcer may get secondarily infected which heals
with atrophic scar
Q. Explain the treatment of PL.
A. – Oral erythromycin 2 gm/day
– Tetracycline 2 gm/day
– Oral antihistaminics
– Oral corticosteroids
DYP SURVIVAL GUIDE FOR POST GRADUATES 611

- Pachyonychia congenita
70. PALMOPLANTAR
- Carvajal syndrome
KERATODERMA
C. Punctate:

Q. Define palmoplantar keratoderma (tylosis). - Punctate keratoderma of palm and soles


A. Defined as heterogeneous group of disorders - Punctate keratoderma of palmar crease,
characterized by hyperkeratosis of palms and soles. marginal papular acrokeratoderma
Q. Classify hereditary palmoplantar keratoderma. Q. What is non -transgradiens keratoderma.
A. A. Diffuse: A. Types:
a) Non- transgradiens (confined to the palms 1. Vorner syndrome (Epidermolytic)
and soles only)
2. Unna-Thorst (non epidermolytic )
- Vorners
Clinical features:
- Unna-Thorst
- Thick yellowish hyperkeratosis involving the
b) Trans - gradiens (extending onto the dorsum
entire palms and soles with sharp eythematous
of palms & soles)
cut off borders
- Camisa syndrome - Vohwinkels syndrome
- Isolated keratotic lesions on dorsum of hands,
- Greithers syndrome - Huriez syndrome feet, knees and elbows.
- Mal de meleda - Olmsted syndrome - Dirty appearance in epidermolytic PPK
- Papillon-Lefevre syndrome - Waxy in non epidermolytic PPK
B. Focal:
Q. Treatment of non transgradiens PPK
- Focal PPK with oral leukokeratosis
A. - 6% salicyclic acid
- Striate PPK
- 50% propylene glycol under occlusion
- Howell-Evans syndrome
- lactic acid-urea combination
- Richner-Hanhart disease
- oral retinoid therapy
610
612 DYP SURVIVAL GUIDE FOR POST GRADUATES DYP SURVIVAL GUIDE FOR POST GRADUATES 613

Focal inherited PPKs : Q. What are the histopathological findings of Mal de


Type Inheritance Mutation Clincial Features Meleda ?
Focal PPK with oral AD K16 Oral cavity and nail involved
A. - hyperkeratosis
leukokeratosis - acanthosis
Striate Howell-Evans AD Dsg 1 Linear striate PPK, oral leukokeratosis,
Associated with malignancy - pseuodospongiosis
Richner- Hanhart AR Tat gene Painful PPK, Mental retardation
Pachyonychia AD K16, 17 Subungual Hyperkeratosis of nail - perivascular lymphohistiocytic infiltrate
congenita
Q. What is the treatment of Mal de Meleda
Q. What is Mal de Meleda ? A. Oral retinoids
A. Syn: keratoderma palmoplantar transgradiens Q. What is progressive keratoderma
- Autosomal Recessive A. Syn: 1. Grither disease
- Named after Croatian island Meleda from where 2. Acral keratoderma of Sybert
first patients were described. 3. Transgradiens et progrediens ppk
- Caused by mutation in ARS gene which encodes - AD
SLURP-1 (secreted by -6/PAR related protein)
- Mutations in gene encoding keratin 1 are
Q. Clinical features of Mal de Meleda. responsible
A. – Starts in early childhood as erythema of palms Q. What are the clinical features of progressive
and soles, followed by appearance of diffuse, keratoderma ?
waxy, ivory-yellow hyperkeratosis, extend on to
dorsum (transgradience) - Onset in late child hood, symmetrical, glove and
stocking distribution of PPK.
– Well-circumscribed psoriasis like plaques or
lichenoid patches may be present on elbow and - Diffuse PPK extends onto dorsal aspects of hands
knees. and feet, knees and elbows with characteristic
Q. How can you differentiate the above type from other involvement of tendo achilles.
PPK ? - Hyperhidrosis is common
A. – Hyperhydrosis - An early onset and a more severe form of pseudo-
– maceration ainhum formation and auto-amputation of digits
– malodor may occur.
– digital constriction Q. What is Vohwinkel’s syndrome ?
– nail changes A. It’s a cicatrizing keratoderma with hearing
impairment.
• All present in Mal de Meleda
614 DYP SURVIVAL GUIDE FOR POST GRADUATES

- AD trait
71. PARONYCHIA
- Honey comb-like palmoplantar keratoderma ,
pseudo-ainhum of digits and stellate keratoses
Q. Why is this paronychia?
of knuckles and other extensor surfaces.
- Mutations in gene encoding connexin 26 which A. There is a localized swelling on the lateral nail fold
is found in cochlea as well as palmoplantar with redness, edema, pain and occasionally purulent
epidermis. discharge with no change in the nail plate.
- Warty lesions on extensor surface and knuckles Q. Define Paronychia.
coalesce with spiky projections giving A. Paronychia is the term used for inflammation of nail
pathognomic starfish appearance on knuckles.
folds, and presents with painful periungual erythema
- Sensorineural hearing loss present. and sometimes with purulence.
Q. Histopathological findings in Vohwinkel’s Q. What are the types of Paronychia?
syndrome ?
A. Acute and Chronic
A. – Hyperkeratosis
– Acanthosis, thickened granular cell layer with Q. Which organisms cause paronychia?
retained nuclei in stratum corneum. A. 1. Acute Paronychia : Caused by bacteria such as
Q. Causes of acquired PPK ? Staphylococcus, Streptococcus, E.coli
A. – Menopause (keratoderma climactericum) 2. Chronic Paronychua : Due to Candida species,
– Hypothyroidism Scytalidium species, Syphilis, Tuberculosis,
– Acanthosis nigricans Leprosy
– Psoriasis Q. What are the clinical features of Acute Paronychia ?
– Pityriasis Rubra Pilaris A. – Usually bacterial in origin.
– Tinea Manum/pedis – May develop from a simple injury.
– Darier’s disease – Localized swelling on the lateral nail fold with
– Lichen planus redness, edema, pain and occasionally purulent
• Associated with esophageal carcinoma (Howel discharge.
Evans syndrome) – May subside on its own.
h – If it spreads it may cause nail deformity by
affecting the matrix

615
616 DYP SURVIVAL GUIDE FOR POST GRADUATES DYP SURVIVAL GUIDE FOR POST GRADUATES 617

Q. What are the predisposing factors for acute – Hypoparathyroidism


Paronychia ? – Celiac disease
A. Bacterial infection due thumb sucking in children – Acrodermatitis enteropathica
Q. What are the predisposing factors for Chronic – Reactive arthritis
Paronychia? – Acrokeratosis paraneoplastica
A. - Exposure to wet activities such as in housewives, – Retinoid therapy
nurses, exposure to irritants, excessive
– Pemphigus
manicuring, finger sucking. Trauma
– Myremecious wart
- Chemical damage
Q. What is chronic Paronychia ? h
A. – Infection maybe mixed (fungal and bacterial-
Gram Negative bacilli)
– Nail dystrophy is seen.
– Course is prolonged
– Eczematization and subungal abscess formation
may complicate the condition.
Q. How do you treat Acute Paronychia ?
A. – A quick and easy method to relieve pain and
hasten healing is to stab the purulent area with
no 11 scalpel blade and discharge the pus.
– Antibiotics and soaks should then be instituted.
Q. How do you treat Chronic Paronychia ?
A. – Avoidance of all causative and aggravating
factors
– Long term systemic antibiotics and topical
antibacterials or antifungal therapy
Q. Differential diagnosis of paronychia.
A. – Herpetic whitlow
– Hallopeau‘s acrodermatitis
72. PARRY ROMBERG 73. PEARLY PENILE
SYNDROME PAPULES - HIRSUTE PENIS
Q. What is seen in Parry Romberg syndrome (PRS)? Q. Why is it a case of Pearly penile papules ?
A. – They are pearly white dome-shaped papules
A. Progressive hemifacial atrophy, epilepsy,
present circumferentially on the coronal margin
exophthalmos, alopecia. and sulcus of glans penis.
Q. What do you see in Parry Romberg syndrome when – Seen more in uncircumsized males.
lower extremities are involved? Q. What else can it be ?
A. Associated spina bifida A. – Condyloma acuminatum
– Papillomas
Faulty limb development
– Hypertrophic sebaceous glands.
Hemiatrophy
Q. How will you manage a case of PPP ?
Flexion contractures A. – No treatment is necessary.
Q. Which is the only type of PRS showing spontaneous – If patient wants treatment, then laser abalation
resolution? or shave excision can be done.
A. Childhood type with involvement of extremities Q. What will you see on histology of Pearly penile
papules ?
Q. Why are roentgenograms done?
A. – There is dermal proliferation of fibroblasts in a
A. To detect melorheostosis as dense cortical collagenous stroma with increased dilated blood
hyperostosis vessels.
Q. What remains the mainstay of treatment? – Elastic fibres are decreased in number.
Q. What are the other cutaneous angiofibromas?
A. Physical therapy of involved limbs to prevent
A. – Adenoma sebaceum
contractures and frozen joints
– Fibrous papules of nose
h – MEN type 1
– Segmental angiofibroma of TS
– Cellular angiofibroma of vulva in older women
– Burt-Hogg-Dube syndrome
h
618 619
DYP SURVIVAL GUIDE FOR POST GRADUATES 621

74. PEDICULOSIS Q. What is the life cycle of lice?


A. The life cycle of the head louse has 3 stages: Egg,
Q. What are the clinical features of pediculosis? nymph and adult
A. - Itching Q. What is the mode of transmission of head louse?
- On examination: nits and live lice A. Close personal contact and sharing of headgear is the
chief mode of transmission among susceptible
- Lymph node swelling
contacts. They move from one infested hair to another
Q. What is the differential diagnosis of pediculosis
with the help of claws on their legs
capitis?
Q. What are the differentiating points between head
A. - Pityriasis Sicca (dandruff)
louse and body louse?
- Hair spray remnants
A. Head louse is relatively smaller (2.5-3 mm) in size
- Concretions compared to body louse (2.8-3.5 mm). It is darker or
- Piedra blackish in color whereas the body louse is grey or
- Monilethrix / Pseudomonilethrix brownish in color. Also, the segments of antennae in
- Scalp impetigo head louse are smaller as compared to body louse.
On an average, the female head louse lays 100 eggs
- Peripilar keratin casts
during its 30 day lifetime, whereas a female body
- Insect bite louse lays around 300 eggs
- Eczema Q. What are pseudonits?
- Psoriasis A. Retained internal root sheath on the hair shaft
- Pseudonits Q. What the diseases transmitted by head louse?
Q. What is pediculosis? What are the types? A. None.
A. Pediculosis is an infestation of lice, of any part of the
Q. What are pseudo knits?
body. It is caused by sucking lice of the order
phtiraptera and suborder anoplura. There are three A. Retained internal root sheath on the hair shaft.
types of lice which form the ectoparasite fauna of man: Q. What are the disease transmitted by body louse?
- Pediculosis humanus var capitis (head louse) A. - Epidemic typhus caused by R. Prowazekii
- Pediculosis humanus var corporis (body louse) - Relapsing fever caused by Borellia Recurrentis
- Phthirus pubis (pubic louse) - Trench fever caused by Bartonella Quintana
620
622 DYP SURVIVAL GUIDE FOR POST GRADUATES DYP SURVIVAL GUIDE FOR POST GRADUATES 623

Q. What the various treatment modalities of pediculosis (iii) 10% Crotamiton – safer, but less effective and
capitis? has to be left for >24 hrs
A. Treatment: (iv) Pyrethrins and carbaryl
1) GENERAL MEASURES (v) 1% Permethrin
- Fomite / environmental control 3) SYSTEMIC TREATMENT:
- Avoid contact with contaminated items such as (i) Ivermectin:
hats, hairbrushes, clothing, towel, combs, etc. - 200 mcg/kg
- Bedding, clothing should be washed and dried - two doses at 10 days interval reqd.
- Combing wet hair with the fine toothed bug - not in children <15 kgs
buster comb. It should be repeated every 4days
(Ivermectin is an anthelminthic drug,
for two weeks
structurally similar to the macrolide
- Robi comb: battery powered device with which antibiotics without antibacterial activity,
dry hair with lice can be combed which is derived from Streptomyces avermitilis.
- Isopropyl myristate & cyclomethicone Ivermectin binds to glutamate-gated
- Coconut & ylang-ylang spray: asphyxiate the lice chloride channels with high affinity and
specificity, which occur in invertebrate nerve
- Others: devices that deliver hot air to hair
and muscle cells, causing an increase in the
- Simple shaving permeability of the cell membrane to
- Plant derived essential hair oils: chloride ions of the nerve or muscle cell. This
lavender, tea-tree oil and neem results in hyperpolarization, leading to
- Head louse repellants containing piperonol flaccid paralysis culminating in the death of
the parasite.)
- Plant derived essential oils (citronella)
(ii) Cotrimoxazole:
2) TOPICAL APPLICATION:
- kills symbiotic bacteria Living in lice
(i) 0.5% Malathion –left on hair for 12 hrs and
washed off. It has a doubtful ovicidal action - controls pyoderma
but kills adult and larva. Therefore to be (When cotrimoxazole is administered to
applied second time, 10-14 days later infested individual, drug reaches the
(ii) Gamma-benzene hexachloride (lindane) 1% circulation of lice during blood meal, as a
- short contact period even for 5 mins is result, gut flora of Pediculus is killed, and
effective thereby depriving the lice of essential
624 DYP SURVIVAL GUIDE FOR POST GRADUATES DYP SURVIVAL GUIDE FOR POST GRADUATES 625

vitamins. Death results from vitamin B Q. What are the newer treatment modalities of
deficiency. Cotrimoxazole is not approved pediculosis capitis?
for treatment of head lice.) A. NEW TREATMENT MODALITIES:
(iii) Albendazole: (i) CETAPHIL CLEANSER (NUVO METHOD)
- 400 mg stat or repeated over 3 days - soak the scalp with cleanser
- repeat dose of 400 mg suggested at 7-10 days - remove excess by combing
interval - hair dryer to totally dry it
(Albendazole is a broad-spectrum - wash out > 8hrs later
antiparasitic, used worldwide for treatment
- repeat the process weekly x3wks
of various helminthic and protozoal
infections. It has manifold action against the (ii) 2010 – ULESFIA
parasite, chiefly: Inhibition of mitochondrial - Benzyl alcohol 5% plus mineral oil
function, uncoupling of oxidative - dose by length of hair
phosphorylation and inhibition of glucose - 2 doses (day 1 & day 7)
transport pathway, ultimately leading to
- 91%cure rate at day 8; Age> 6 months
ATP depletion and cell death)
(iii) 2011 – NATROBA
(iv) Levamisole:
- Benzyl alcohol 5% and 0.9% Spinosad
- 3.5 mg/kg for 10 days
- 10 mins application upto twice / wk
- for children weighing 10-19 kgs, 50 mg daily
- 84-87% cure rate at 40 days post therapy
- for children weighing 20-39 kgs, 100 mg
- Age> 4yrs
daily
(iv) 2012 – SKLICE
(Levamisole is an acetylcholine nicotinic
receptor agonist, which is rapidly and almost - Ivermectin 0.5% lotion
completely absorbed from the - single 10 mins application
gastrointestinal tract. Due to its agonistic - 71.4 - 76.4% cure rate
action on acetylcholine receptor, there is tonic
- NO nit combing required
paralysis of the ectoparasite and killing.
Levamisole also interferes with the - A/E: conjunctivitis, burning sensation
carbohydrate metabolism of the parasite by - Age> 6 months
inhibition of enzyme fumarate hydratase). - New, Expensive, But just one Application
626 DYP SURVIVAL GUIDE FOR POST GRADUATES DYP SURVIVAL GUIDE FOR POST GRADUATES 627

Q. What is vagabond disease? What is its treatment? - Two drops of Ivermectin 200 mcg/kg at an
A. Vagabond disease is the other name for pediculosis interval of 10 days is also effective.
corporis. It is usually seen in homeless people who - Topical pilocarpine & physostigmine
are wandering about or the nomads. It is characterised Q. What is called as permethrin/pediculosis resistance?
by parallel scratch marks and hyperpigmentation
A. - Resistance can be developed by 2 mechanisms:
especially of upper back.
Either by genetic mutation or enzymatically by
• Treatment: inhibition of oxidases in the lice.
- Laundering of clothes especially - To overcome the enzymatic resistance, co-
underclothes and bedding at high treatment with synergist piperonyl butoxide (3:1)
temperature. and longer duration of treatment is done.
- Use of hot iron with special attention to seams Q. Difference between live and dead lice?
of clothing destroys the lice and nits. Scrub
A. - Live knit: brown or yellow or white, oval in
bath is advised for the patient and change of
shape, attached at an acute angle, adherently to
clothing. Dusting of the garment with
the hair shaft. - Dead nit: clear or white, flat
gamma benzene hexachloride or malathion
surface, attached at obtuse angle
can also be done.
Q. What is Macula cerulae? h
A. Macula cerulae are blue-grey macules on the upper
thigh and lower abdomen which is probably
produced by altered blood pigment or as reaction to
louse saliva.
Q. What is the treatment for phthiriasis palpebrum?
A. Treatment consists of:
- Petrolatum applied thickly twice a day for two
weeks. This interferes with respiratory function
of the louse by blocking the spiracles.
- Freshly prepared 20% Flourescin eyedrops to the
lid margins.
DYP SURVIVAL GUIDE FOR POST GRADUATES 629

- Topical clindamycin and erythromycin


75. PERIORAL DERMATITIS
- Topical metronidazole 0.75%-1%
Q. What is perioral dermatitis? - Oral tetracycline for 6-8 weeks
A. it is a common acneiform eruption almost exclusively · Benzoyl peroxide
seen in women aged 20-30 years. It is characterized · Erythromycin/clindamycin
by erythema, scaling and follicular papules that occur · Tacrolimus
around the mouth, nose, and less frequently on the
· Tretinoin
eyelids.
- Oral medications
Q. what Is the etiology?
· Tetracycline
A. - fluorinated steroids
· Doxycycline
- sunlight
· Macrolides erythromycin,clindamycin
- allergic dermatitis
· Metronidazole
- demodex folliculorum
· Isotretinoin
- H.pylori
- Prevention-sunscreens, avoidance of cosmetics,
- Cosmetics
and triggers.
- OCP
- Hormones h
- Fluorinated toothpaste
- Rosacea
Q. what is the differential diagnosis ?
A. - Rosacea
- Acne
- Seborrheic dermatitis
- Contact dermatitis
- LMDF
Q. Treatment of perioral dermatitis?
A. - Avoidance of fluorinated compounds
- 1% hydrocortisone

628
DYP SURVIVAL GUIDE FOR POST GRADUATES 631

76. PHRYNODERMA – X1B : Bitot spots


– X2 : Corneal xerosis
Q. Meaning of the word phrynoderma? – X3 : Keratomalacia & corneal ulcerations
A. – Phryno = toad-like , Derma = skin – XS : Corneal scar
– Phrynoderma can now be considered as non- – XF : Xerophthalmic fundus
specific finding in multi-vitamin deficiencies.
Q. Other features of vitamin A deficiency?
Q. Etiology?
A. Other than skin and eye manifestations, patient is
A. – Mainly due to deficiency of vitamin A.
prone to develop recurrent respiratory tract infections
– Also due to deficiencies of vit.B-complex,
Q. Diagnosis?
vitamin E, vitamin C, essential fatty acids
– Protein-energy malnutrition and general A. – Clinical.
malnutrition state. – Serum Vit A levels < 20 mcg/dl
Q. Causes of vit A deficiency? Q. Histopathology?
A. – Decreased intake A. – Hyperkeratosis
– Malabsorption – Dilated follicular orifices with keratin plug
– Recent attack of measles – Sebaceous gland atrophy
Q. Classical lesions? Q. Daily allowance of Vit A?
A. Multiple, discrete, skin coloured to pigmented,
A. – Children : 3000 IU
keratotic, follicular papules with central
intrafollicular keratotic plug which projects as a – Adult female : 4000 IU
horny spine which leaves a pit when expressed. – Adult male : 5000 IU
Q. Sites affected? – Pregnant female : 6000 IU
A. – Extensor aspects of extremities, especially over Q. Treatment?
knees & elbows
A. Regimen-1) Skin+Eye involvement:
– Sometimes anterior aspect of thighs.
Inj. Vit. A on day 0,1,14
Q. Grade various eye symptoms due to vitamin A
deficiency? <6mths : 50000 IU/dose
A. – XN : Night blindness 6-12 mths : 100000 IU/dose
– X1A : Conjunctival xerosis >1yr : 200000 IU/dose

630
632 DYP SURVIVAL GUIDE FOR POST GRADUATES

Regimen-2) Skin moderate to severe involvement: 77. PIEBALDISM


Inj. Vit. A on day 0,3,7
100000IU/dose. Q. What is piebaldism?
Regimen-3) Skin mild involvement: A. It is an uncommon congenital autosomal dominant,
Vit. A capsule 50000IU/day till stable patterned leukoderma characterized by frontal,
symptoms subside median or paramedian depigmented patch with a
Q. Symptoms of hypervitaminosis A? white forelock. Also macules with scattered
hyperpigmented islets can be seen.
A. – Dry, scaly skin with desquamation,
Q. What is the cause of piebaldism?
– Peeling of palms and soles,
– Follicular hyperkeratosis, A. Mutation in KIT gene on chromosome 14 are the
cause.
– Cheilitis-fissuring of the lips and mouth,
– Raised ICT Q. What is the prevalence of piebaldism?
– Alopecia A. – It affects one in 20,000.
– Blurring of vision, pseudotumor cerebri, – It affects males and females equally.
– Headache, Arthralgia, myalgia, premature Q. Describe the clinical feature of piebaldism.
closure of epiphysis and spontaneous bone A. – The classical feature is a white forlock or a white
fracture, cortical hyperostosis
triangular elongated or diamond shaped macule
Q. Any teratogenic effect? on forehead since birth. The middle of eyebrows
A. Yes, cranio-facial syndrome and eyelashes may be white. White patches can
Q. Any precautions while giving Vit A? also be seen on abdominal wall and the anterior
chest, often extending dorsally on to the back,
A. Never given with oral retinoids because of risk of
mid thigh to calf and occasionally lower part of
pseudotumor cerebri
the neck and face, mid-upper am to wrist, while
h other areas remain normally pigmented.
– Another characteristic feature is the presence of
islets of hyperpigmented macules in
leukodermic patches and on normally coloured
skin.
Q. Describe the pathology of piebaldism.
A. – Absence of melanocytes and melanin in hair
bulbs and white patches of depigmented areas.
633
634 DYP SURVIVAL GUIDE FOR POST GRADUATES DYP SURVIVAL GUIDE FOR POST GRADUATES 635

– Negative DOPA reaction in depigmented skin.


78. PIGMENTED PURPURIC
– In the hyperpigmented macules, melanoytes are
normal in number but with abundant DERMATOSIS
melanosomes.
– The fault is mainly in differentiation, migration Q. Definition?
and survival of of melanoblasts. A. PPD are a group of benign asymptomatic disorders
Q. How will you diagnose a case of piebaldism? characterized by petechiae, pigmentation and
occasionally telangiectasia on the lower extremities.
A. – Congenital onset
Q. Etiology?
– Typical pattern of depigmented macules.
A. Following factors have been implicated:
– Presence of islets of hyperpigmented macules
within the depigmented and normal skin a) Contact with allergens (textile dyes, rubber,
cobalt, benzoyl peroxide and EMLA).
– Autosomal dominant inheritance pattern.
b) Drugs
Q. What is Wolffe’s syndrome?
c) Venous stasis/acroangiodermatitis
A. Piebaldism with deafness but without other features
d) Venous incompetence
of Waardenburg’s syndrome is known as Wolffe’s
syndrome. e) Step aerobics
Q. What is the treatment for piebaldism? f) RA/DM
A. – Photoprotective measures. g) Hodgkin’s disease
h) Hepatitis B,C
– Skin grafting from normal melanotic areas to
amelanotic sites. i) Purpuric MF
Q. Which drugs have been implicated in etiology?
h A. Following drugs:
a) NSAIDS
b) Diuretics
c) Glipizide
Q. Clinical features?
A. Multiple, well defined, erythematous to
hyperpigmented macules, few of them coalescing
together to form larger patches, are present most
commonly over lower limbs.

635
636 DYP SURVIVAL GUIDE FOR POST GRADUATES DYP SURVIVAL GUIDE FOR POST GRADUATES 637

Q. Clinical variants? Q. Dermascopy findings?


A. a) Schamberg’s disease A. Coppery background with more clear red dots and
b) Majocchi’s disease (purpura annularis globules.
telangiectoides) Q. Treatment?
c) Pigmented purpuric lichenoid dermatosis of A. a) Topical steroids
Gougerot and Blum b) Pentoxyfylline
d) Eczematoid-like purpura of Dukas and c) Ascorbic acid
Kapetanakis d) PUVA therapy
e) Lichen aureus e) Cyclosporine
f) Itching PPD (disseminated pruruginous
angiodermatitis)
h
g) Granulomatous PPD
h) Segmental PPD
i) PPD/MF overlap
Q. Which is the most common variant?
A. Schamberg’s disease (Cayenne pepper spots)
Q. Differential diagnosis?
A. – Contact dermatitis
– Cutaneous necrotising vaculitis
– Stasis dermatitis and purpura
– Angioma serpingnosa
– Kaposi sarcoma (Gougerot-Blum )
– Mycosis Fungoides
Q. Histopathology?
A. – Epidermis normal.
– Vasculitis in upper dermis.
– Extravasation of RBCs.
– Presence of hemosiderin.
638 DYP SURVIVAL GUIDE FOR POST GRADUATES DYP SURVIVAL GUIDE FOR POST GRADUATES 639

- These are asymptomatic ill-defined


79. PITYRIASIS ALBA hypopigmented slightly scaly macules present
primarily over face; though arms, neck and trunk
Q. Why is this Pityriasis Alba? may be rarely involved.
A. - Lesions are scaly hypopigmented macules on - In early stages, the lesion may be erythematous
face with no loss of sensation or hair. and show minimal scaling.
- There may be h/o waxing in winter and - It resolves spontaneously but may recur in
improvement in summer. subsequent years.
- Wood’s lamp examination shows no Q. What is progressive macular hypomelanosis?
accentuation or fluorescence.
A. It is characterized by ill-defined, nummular, non-
Q. What are the differential diagnosis? scaly hypopigmented spots on the trunk, often
A. 1. Early vitiligo confluent in and around the midline, and rarely
2. Pityriasis versicolor extending to the proximal extremities and neck/head
region.
3. Leprosy
Q. What is the treatment for Pityriasis alba?
4. Post inflammatory hypopigmentation
A. - An emollient may be advised.
5. Hypopigmented mycosis fungoides
- If mild signs of inflammation are present, a mild
6. Progressive macular hypomelanosis?
topical steroid may be used for short period.
• Associated with atopic dermatitis (minor
- Tacrolimus ointment 0.03% and pimecrolimus
criterion for its diagnosis).
cream 1% may hasten resolution.
Q. What is the histopathology?
A. - follicular plugging h
- follicular spongiosis
- atrophic sebaceous gland in early stages
- chronic dermatitis in later stage.
Q. What is the meaning of pityriasis alba ?
A. Pityriasis: bran-like
Alba: white
Q. What are the clinical features?
A. - It is the most common condition in children.

638
DYP SURVIVAL GUIDE FOR POST GRADUATES 641

80. PITYRIASIS ROSEA Q. Derivation of the word?


A. – Pityriasis = scale (latin)
Q. Define a classical primary lesion? – Rosea = red (greek)
A. – Primary lesion is known as a “herald patch” or – Also called as Gibert’s disease
“mother patch.” Q. Etiology?
– Erythematous, well defined, round to oval, 2-10 A. Two main factors:
cm in diameter, with collarette of scales a) Viral – HHV 7
– Sites : chest, back, neck, abdomen. b) Drugs
Q. Describe a classical secondary rash? Q. Reasons to support viral etiology?
A. – Appears 5-15 days following mother patch. A. Following facts support viral etiology:
– Comes in crops for a few week. a) Case clustering in families
– Discrete, oval, erythematous scaly plaques of 0.5- b) History of upper respiratory tract infection
1.5 cm diameter. c) Gap of few days between primary lesion and
widespread rash
– Fine, dry collarette of scales.
d) Recurrence is rare
– Lesions distributed in a Christmas tree pattern.
e) Cephalo-caudal distribution of rash
– Hanging curtain sign present.
f) HHV-7 in lesional skin
Q. Differential diagnosis?
g) Children (mainly affected)
A. – Drug rash, pityriasiform seborrhoeic dermatitis Q. Drugs causing PR like eruption?
– Secondary syphilis A. Barbiturates, propylthiouracil, captopril, clonidine,
– Guttate psoriasis arsenic, bismuth, gold, metronidazole, BCG vaccine,
– Pityriasis lichenoides chronica imatinib, mesylate, interferon, ketotifen,
methoxyprozamine, tripelennamine hydrochlride,
– Pityriasis alba ergotamine, lisinopril, acyclovir, lithium,
– Tinea circinata adalimumab.
Q. What is pityriasis rosea ? Q. Histopathology?
A. It is an acute, self-limiting, papulosquamous disorder A. Following features are observed:
with characteristic skin rash and clinical course, a) Tea-pot lid sign: focal angulated patchy
mainly involving children and young adults parakeratosis
640
642 DYP SURVIVAL GUIDE FOR POST GRADUATES DYP SURVIVAL GUIDE FOR POST GRADUATES 643

b) Coat-sleeve pattern inflammatory infiltrate Q. Treatment?


c) Spongiosis A. – Erythromycin (500 mg 4 times/day for 5 days)
d) Mild acanthosis – Acyclovir (800 mg 5 times/day for 7days)
e) Exocytosis – Emollients
Q. Comment on age group and seasonal variation? – Calamine
A. – Between 10-35 yrs, Peak in 20s. – Topical steroids
– More common in winters and spring – Sun exposure
Q. Distribution of lesions? – UVB phototherapy
A. – Christmas tree pattern on trunk. – PUVA therapy
– In adults, mainly on trunk. – Oral steroids
– In children, face may be involved. h
Q. Types of PR?
A. – Classical
– Inverse
– Atypical
– Papular
– Urticarial
– Purpuric
– Lichenoid
– Vesico-bullous
– Mucosal
– Giant PR.
– Large localized flexural lesions (Vidal PR)
Q. Prognosis?
A. – Self limiting.
– Mostly heals with hypopigmentation.
– Recurrences are 5%
DYP SURVIVAL GUIDE FOR POST GRADUATES 645

c) Sometimes erythroderma (islands of normal skin


81. PITYRIASIS RUBRA within the affected areas)
PILARIS (PRP) d) Yellowish hyperkeratosis of the palms & soles
(PRP sandal)
Q. Describe a classical skin lesion. e) Hyperkeratotic papules on knees and elbows
A. – Multiple, small, pale to pink, keratotic, scaly, f) Nails: thick, brittle, dull, rough, subungual
follicular papules coalescing together to form hyperkeratosis, splinter hemorrhage
large plaques. g) Rarely oral mucosa may be involved (diffuse
– Reddish-orange erythema is seen. ground glass appearance)
Q. Differential diagnosis ? Q. Name the classification system used for classifying
A. Following conditions should be kept in mind: PRP?
a) Psoriasis A. Griffith’s classification as follows:
b) Erythro-keratodermas a) Type-l : Classical adult type
c) Follicular ichthyosis b) Type-ll : Atypical adult type
d) Lichen scrofulosorum c) Type-lll : Classical juvenile type
e) Other follicular keratosis d) Type-IV : Circumscribed juvenile type
Q. Synonym ? e) Type-V : Atypical juvenile type
A. Devergie’s disease f) Type-Vl : HIV associated PRP (added later on)
Q. What are the etiological factors ? Q. Histopathology?
A. a) Disorders of Vit A metabolism A. a) Checker- board pattern of vertical and horizontal
b) Underlying immunological abnormality alternating ortho and parakeratosis
c) Preceeding febrile illness and infections b) Shouldering parakeratosis at the sides of hair
follicle
d) Bacterial superantigens
c) Hypergranulosis
e) Rarely hypogammaglobulinemias
d) Thick suprapapillary plates
Q. Describe other clinical features ?
e) Follicular plugging
A. a) Diffuse salmon-colored erythema and scaling
over scalp f) Broad rete ridges
b) Horny follicular papules on back of fingers g) Narrow dermal papillae
(nutmeg grater feel) h) Sparse inflammatory infiltrate
644
646 DYP SURVIVAL GUIDE FOR POST GRADUATES

Q. Treatment?
82. PRURIGO NODULARIS
A. Following drugs have been tried:
a) Keratolytics Q. Clinical features?
b) Emollients A. Multiple, well-defined, dry, scaly, warty, lichenified,
c) Topical steroids hyperpigmented, excoriated, firm nodules varying
d) Isotretinoin (0.5-2 mg/kg/day) in size from 1-3 cm. Mostly seen on extensor aspect
of limbs.
e) Acetretin (0.4mg/kg/day)
Q. Differential diagnosis?
f) Vit-A (50000IU thrice daily)
A. 1) Hypertrophic lichen planus
• One of the few causes of erythroderma for which
systemic steroids do not work. 2) Insect bite reactions
3) Benign epidermal tumors
h 4) Pemphigoid nodularis
5] Perforating disorders (e.g. acquired perforating
dermatosis)
4] Multiple keratoacanthomas
5] Granular cell tumors
6] Pruriginous epidermolysis bullosa
7] Nodular scabies (few lesions)
Q. What is prurigo nodularis also known as?
A. Hyde’s prurigo
Q. What is the etiology?
A. 1] Idiopathic
2] Emotional stress
3] Xerosis
4] Atopy
5] Systemic diseases like hyperthyroidism, hepatic
or renal dysfunction, lymphoma and iron
deficiency
6] Insect bite (20%)
647
648 DYP SURVIVAL GUIDE FOR POST GRADUATES DYP SURVIVAL GUIDE FOR POST GRADUATES 649

Q. Describe the clinical features? - Papillomatosis and irregular proliferation of the


A. – Pruritus with lesions that are hard, dome-shaped epidermis
papules or nodules with central scale-crust - The dermis may show a non-specific
– Variable colour ranging from erythematous to inflammatory infiltrate.
brown to skin-colored - Neural hyperplasia (variable degree).
– In longstanding disease, lesions may be Q. How will you investigate a case of prurigo nodularis?
verrucous, lichenified or fissured surface
A. 1] Complete blood cell count
– Multiple excoriations, post-inflammatory
2] liver function tests
hyperpigmentation and superficial scarring
– Patients may present with a few or hundreds of 3] Serum creatinine
lesions, usually in a symmetric distribution, with 4] Thyroid function tests.
a predominance on the extensor surfaces of the 5] Chest radiograph and abdominal CT scan.
limbs (suspicion of lymphoma)
Q. What is “butterfly sign” ? Q. How will you treat a case of prurigo nodularis?
A. Sparing of the mid upper back from the lesions is A. 1] General measures : To reduce excoriation,
referred to as the ‘butterfly’ sign.
- cutting the nails very short,
Q. Associated internal diseases?
- wearing gloves at night
A. It can be associated with:
- occlusion of the involved regions of skin
a) Metabolic disorders like uremia, hepatic
with bandages, dressings or steroid
dysfunction
impregnated tape.
b) Malignancies like Hodgkin’s disease
2] Topical therapy :
c) HIV infection
– Emollients,
d) Gluten-sensitive enteropathy
- Superpotent topical steroid under occlusion,
Q. Incidence?
- Topical Capsaicin (0.025–0.3%),
A. – Most common in 20-60 yrs.
- Tacrolimus, calcipotriol and calcitriol
– Rare in children.
- Topical antipruritic and moisturizing agents
– Equally seen in both sexes.
containing menthol and phenol or
Q. Describe the histopathology? anesthetics, such as pramoxine.
A. - Focal hyperkeratosis and acanthosis. 3] Intralesional steroid and Cryotherapy
650 DYP SURVIVAL GUIDE FOR POST GRADUATES

4] Systemic therapy
83. PSEUDO XANTHOMA
– Anti-histaminics
- Cyclosporine (3.0–4.5 mg/kg/day) ELASTICUM(PXE)
- Thalidomide (100–300 mg daily)
Q. Why is it PXE?
- Phototherapy
A. – Asymptomatic, small, skin-coloured to yellowish
- Azathioprine
papules arranged in a reticular fashion over soft
- Naltrexone lax wrinkled skin giving characteristic
- Anti-depressants cobblestone/chicken neck/Moroccan leather
- Anxiolytics appearance on the neck, axillae, groin.
– Occasionaly similar lesions are present over
h mucous membranes.
– Many individuals have oblique mental (chin)
creases.
Q. Differential diagnosis?
A. a) Buschke-Ollendorff syndrome
b) Juvenile elastoma
c) Other dermatochalasis like cutis laxa, ehlers-
danlos syndrome, granulomatous slack skin
Q. What is the other name?
A. Gronblad-Strandberg syndrome
Q. What is it?
A. It is an inherited autosomal recessive disorder
characterized by generalised fragmentation and
calcification of elastic fibers in various tissues and
clinically involving skin, eyes and cardiovascular
system.
Q. Genetic defect?
A. – Gene coding for ABCC6, on chromosome 16 is
defective.
651
652 DYP SURVIVAL GUIDE FOR POST GRADUATES DYP SURVIVAL GUIDE FOR POST GRADUATES 653

– Absence of normal ABCC6 protein results in • Angioid streaks are due to breaks in the Bruch’s
calcification of elastic fibers membrane of the retina
Q. Age of onset? Q. Cardiovascular manifestations?
A. – Age of onset is 13-15 yrs. A. a) Intermittent claudication
– More in women b) Diminished peripheral pulses
Q. Which all systems are affected? b) Angina pectoris
A. Following systems are affected: c) Myocardial infarction
a) Skin d) Mitral valve prolapse
b) Eyes e) Hypertension
c) Cardiovascular f) Brain haemorrhage
d) Others g) Seizures
Q. Ocular manifestations? Q. Other manifestations ?
A. In retina: A. a) Recurrent abortions
a) Angioid streaks b) Gastrointestinal haemorrhage
b) Leopard spotting c) Pulmonary involvement
c) Fibrovascular ingrowths in the retina lead to d) Urinary involvement
retinal haemorrhages, retinal detachment and Q. Any associations?
visual loss. A. a) Osteoectasia
In macula: b) Osteitis deformans (Paget’s disease)
a) String of pearls appearance (best visible on c) Sickle cell anemia
fluorescein angiography)
d) Beta thalassemia
Q. Other causes of angioid streaks ?
e) Tumoral calcinosis
A. P seudoxanthoma elasticum
Q. Cause of death in PXE ?
E hlers Danlos syndrome
A. – Cerebral haemorrhage
P aget’s disease of bone
– Coronary occlusion
S ickle cell anemia
– Massive haemorrhage in the gut
I diopathic and other conditions like
atherosclerosis, acromegaly, diabetes mellitus, Q. Histopathology?
hemochromatosis, lead poisoning, aging, NF, A. In mid and lower dermis elastic fibers are fragmented,
SWS clumped, degenerated, swollen and calcified
654 DYP SURVIVAL GUIDE FOR POST GRADUATES

– Appear dark blue on H&E staining.


84. PYODERMA FACIALE
– Special stains like van Gieson and von Kossa can
be used.
Q. What is pyoderma faciale?
Q. Diagnostic criteria?
A. It is a facial disorder that consists of an intense reddish
A. a) Major criteria:
or cyanotic erythema, combined with superficial and
- flexural yellow cobblestone lesions deep abscesses, cystic lesions and sometimes sinus
- characteristic histopathologic features in tracts .
lesional skin, using elastic and calcium stains
Q. How is it differentiated from acne?
- angioid streaks in retina
A. Absence of comedones, rapid onset, fulminating
b) Minor criteria: course.
- characteristic histologic changes in non- Q. How do you treat pyoderma faciale?
lesional skin
A. Prednisolone 40-60 mg daily tapered over 3-4 weeks.
- family history of PXE in first-degree
Isotretinoin should be used in conjuction with the
relatives.
prednisolone in a dose of 1mg/kg/day continued
Q. Treatment? for 4-6 mnths
A. Early diagnosis and appropriate treatment to prevent
complications
– Genetic advice h
– Avoid any activity that causes a sudden increase
in blood pressure or injury to eyes
– Mainstay is surgical correction of lax skin.
– Laser photocoagulation helpful in preventing
further retinal hemorrhage
– Other systemic manifestations should be treated
promptly.
– Restriction of dietary calcium may help.
– Recently anti-angiogeneic drugs like
Bevacizumab and Ranibizumab have been used

655
DYP SURVIVAL GUIDE FOR POST GRADUATES 657

capillary sized vessels in a loose and oedematous


85. PYOGENIC GRANULOMA stroma with an epidermal collarette formed at the
margins in form of rete ridges.
Q. What is pyogenic granuloma?
Q. What is the treatment?
A. Pyogenic granuloma is a a benign vascular tumor of
A. i) Simple curettage with electrocautery
skin or mucous membranes characterized by rapid
growth and friable surface. ii) Excision
Q. Differential diagnosis ? iii) LASER surgery with carbon dioxide and pulsed
dye
A. i) Bacillary angiomatosis
iv) Cryotherapy
ii) Epitheloid haemangioma
iii) Kaposi’s sarcoma
iv) Peripheral fibroma
v) Infantile haemangioma h
vi) Peripheral giant cell granuloma
Q. Other name for pyogenic granuloma?
A. Lobular capillary haemangioma
Q. Pyogenic granuloma is seen commonly in?
A. – Most commonly seen in infants and children.
– It can also be seen in adults particularly pregnant
women
Q. What are the Clinical features ?
A. It is a solitary, red, rapidly growing papule or nodule
with a subtle collarette of scale (other causes collarette
of scaling: PR, Staphhlococcal furuncle)
Q. What are the common sites?
A. Cheeks, forehead but virtually any body site.
Q. Histopathological findings?
A. Pyogenic granuloma is an exophytic often ulcerative
lesion characterized by lobulated proliferation of
656
86. RETICULATE 87. ROSACEA
ACROPIGMENTATION Q. Define rosacea?
OF KITAMURA A. Rosacea is characterized by a persistent erythema of
the convex surfaces of the face, nose most frequently
Q. What is it? affected followed by brow and chin.
A. Characterized by hyperpigmented, atrophic, Q. Types of rosacea
angulated macules which initially appear over A. - Erythemato-telengiectatic
dorsum of hands, later on involving the whole
extremities, rarely affecting the face. - Papulopustular
Q. Other clinical features? - Glandular
A. Palmar pits are characteristic. Q. Are the various stimuli for rosacea?
Q. Histopathology? A. - Emotional stress
A. Epidermal atrophy and increased basal melanocytes. - Hot drinks
Q. Relation with? - Alcohol
A. Many cases show histopathologic overlap, - Spicy foods
considered by some as varied presentations of same - Exercise
entity. - Cold or hot weather/baths/showers
Q. Differential diagnosis Q. What is the etiology?
A. - Dowling-Degos disease A. a) Abnormal vasomotor response to thermal and
- Acropigmentation of Dohi other stimuli
Q. Treatment b) Chronic solar damage
A. No treatment. c) Chronic vasodialation, edema and compromise
of lymphatic drainage and lead to telengiectasia
h
and fibrosis.
Q. Eye changes in rosacea
A. Blepharitis, recurrent chalazion, conjunctivitis,
keratitis, iritis, episcleritis.
Abnormal Schirmer test in 40% patients.
658 659
660 DYP SURVIVAL GUIDE FOR POST GRADUATES

Q. What is rhinophyma?
88. SCABIES
A. It is a subtype of glandular rosacea. Hypertrophic,
hyperemic, large nodular masses are centered over
Q. Define scabies?
the distal half of the nose.
A. Scabies is a disease caused by itch mite called
Q. Histopathology of rosacea?
Sarcoptes Scabiei var Hominis characterized by
A. Pilosebaceous hyperplasia with fibrosis, intense itching which is mostly worse at night.
inflammation and telangiectasia.
Q. What are the clinical features of scabies?
Q. What is Haber syndrome?
· Itching is predominant symptom, which is worse
A. Rosacea- like facial lesions and multiple verrucous at night
lesions on the non -sun exposed areas.
· In early stages, itching confined to areas of
Q. Differential diagnosis of rosacea? predisposition, but as disease progress due to
A. - Polycythemia vera delayed hypersensitivity response, patient may
- Carcinoid syndrome present with generalized itching
- Mastocytosis · Scabies involves web of fingers, flexor aspect of
wrists, elbows, anterior axillary folds, umbilicus
- Lupus erythematosus
and periumblical region, genitalia, upper thighs,
- Dermatomyositis knees, ankles. In females, nipple and areola are
- MCTD involved
Q. Treatment for rosacea · The lower part of buttocks and natal cleft are
A. - Control of inflammation common sites. In adults, scalp, face, palms and
soles are spared but this areas are involved in
- Topical Metronidazole
infants and children
- Sodium sulfacetamide-sulfur
· The burrow is the primary lesion. It is a
- Azelaic acid serpentine, threadlike, grayish or dark line. The
h open end is marked by papule or papulovesicle.
The mite may be seen as a whitish dot at closed
end. Lesions are often excoriated, eczematised
Q. What are differential diagnosis of scabies?
A. Infancy:
- Atopic dermatitis
- Prurigo mitis
661
662 DYP SURVIVAL GUIDE FOR POST GRADUATES DYP SURVIVAL GUIDE FOR POST GRADUATES 663

- Bacterial infection household is significant factor in transmission of


- Acropustulosis of infancy scabies
Adults: · Overcrowding with regard to sleeping space
- Contact dermatitis · Indirect spread by clothing, bedding or other
- Dermatitis herpetiformis fomites is uncommon
Insect bites Q. What is the morphology of Sarcoptes Scabiei?
Pruritus related to HIV disease A. · Sarcoptes Scabiei var Hominis is the causative
Elderly: agent of human scabies
- Asteatotic Eczema · Sarcoptes Scabiei are whitish hemispherical mites
- Pruritus related to systemic causes
· Female is 0.4 x0.3 mm, with four pair of legs
Q. What is epidemiology of scabies?
· In females, front two pairs end in suckers and
A. - Affects all races
hind two pairs in long trailing bristles
- Epidemics occur every 30 years according to
studies with smaller epidemics occurring every · Male is smaller about 0.2x0.15 mm and its fourth
7 years (7 years itch- the other 7 year itch is pair also end in suckers
marriage!) · Body of both male and females end in transverse
- Mostly a disease of children corrugation with spines and bristles on dorsal
- Highest in age group below 5 years surface
- Decreasing prevalence with age Q. What is life cycle of Sarcoptes Scabiei?
- No preponderance in either sex A. · Copulation –in a small burrow excavated by a
Q. What conditions are favourable for transmission of female
scabies? · After copulation, the fertile female enlarges the
A. · High humidity and low temperature favours burrow and begins laying eggs. Burrows are not
survival of scabies mite and high temperature confined to stratum corneum, but are inclined
and low humidity leads to death of the mite downward into epidermis
· Transfer of young fertilized female mites is · Gradual penetration appears to occur as a result
necessary for estabilshing infection of new host of production of a lytic secretion by mite
· Prolonged intimate, personal contact within · The burrow is lengthened by approximately
664 DYP SURVIVAL GUIDE FOR POST GRADUATES DYP SURVIVAL GUIDE FOR POST GRADUATES 665

2-3 mm daily. Eggs are deposited behind in the Q. What are different types of scabies?
burrow A. 1. Classical scabies:
· About 40-50 eggs are laid at each time during a - itching which worsens at night
lifespan of 4-6 weeks, during which time she does - burrows can be present as wavy thread-like,
not leave the burrow grayish lines, which open as papule or
vesicle
· Six-legged larvae emerge from the eggs after 3-4
- in severe cases, deep seated nodules are seen
days and escape the burrow
2. Scabies in clean:
· The larvae then dig short burrows (moulting - moderate itching with few scattered wheals
pockets) in which they transform into nymphs and papules
· After further moults, adult males and females - distribution limited to sites of
develop predisposition
- minute papules with gritty feeling in
Q. How is immune system related to scabies?
between web spaces may be present
A. - In scabies, cell mediated immune response is - genital and gluteal regions should be
more important than humoral response checked to see for the typical lesion
- The mite population is more in the early 3. Scabies in babies:
asymptomatic phase of infestation. Once - classical lesions of scabies are replaced by
symptomatic, their number reduces dramatically, excoriations, crusting, pyoderma
as the mites are removed mechanically by - vesicles are a prominent feature which may
scratching and the immune response causes an lead to blister formation
outpouring of fluid that makes stratum corneum - recurrent attack of vesicles and pustules of
inhospitable for mite hands and feet area a common feature
- the duration is usually less than 2 months
- In re-infestation, symptoms are earlier, mites are
and the mite population is low because of
fewer in number and disappear spontaneously.
severe inflammatory reaction as a result of
Rash and pruritis are limited to site of infestation.
scabies and pyoderma
- Immunosuppressed patients are more prone for
4. Nodular scabies:
severe type of scabies with unlimited production
of mites. Delayed hypersensitivity response help - persistant nodules occur mainly over the
to contain infestation . elbows, anterior axillary folds, genitalia and
gluteal region
666 DYP SURVIVAL GUIDE FOR POST GRADUATES DYP SURVIVAL GUIDE FOR POST GRADUATES 667

- firm, dull, red nodules may persist for - The distribution pattern is different
weeks, months or for a year - The lesions are most common on forearm,
5. Crusted scabies: lower chest, abdomen, thighs and areas
- hyperkeratotic, scaly, crusted lesions with exposed to mite by the person carrying the
large number of mites in them affected animal
- in addition to classical sites of involvement, - The interdigital webs and genitalia are
lesions are seen on face, scalp, palms, soles spared and burrow absent
and neck - Short incubation period
- the helix of the ear is commonly involved - The disease frequently goes undiagnosed
- the nails are dystrophic and discolored with - The dog may show patchy alopecia with
subungual hyperkeratosis scaling involving mainly the head, ears and
intertriginous folds, characteristic mousy
- poor personal hygiene, defective cell
odour (the disease is called as sarcoptic
mediated and humoral immunity may
mange because the dog actually gnaws at its
predispose to crusted scabies
own skin; mange = eating).
- the disease is frequently associated with
Q. What are the complications of scabies?
malnutrition, lepromatous leprosy,
tuberculosis, Down’s syndrome, diabetes A. · Secondary pyodermas presenting as impetigo,
mellitus, acute renal failure, pemphigus ecthyma, cellulitis, lymphangitis, furunculosis
vulgaris, etc. · Acute glomerulonephritis
6. Scabies incognito: · Pediatric acute glomerulonephritis is also
reported in some cases
- Scabies treated with topical or systemic
corticosteroids present with unusual clinical Q. How will you diagnose scabies?
manifestations, involving atypical sites A. The four cardinal features in scabies are
- While corticosteroids reduce the itching and · The presence of burrow specially on web-spaces
the inflammatory lesion, the mite population and penis
increases · The characteristic distribution pattern of lesion
- Mites are easily demonstrable from scraping · The presence of history of similar illness in other
of lesion members of household or other contacts
7) Animal transmitted scabies: · Intense pruritis, which tend to worsen at night
- it is acquired from animals like dogs, horses, · Demonstration of burrow, with the help of
etc. scalpel, slice of entire burrow, especially the
668 DYP SURVIVAL GUIDE FOR POST GRADUATES DYP SURVIVAL GUIDE FOR POST GRADUATES 669

blind end. This is placed on a slide. Addition of · Sulfur in petrolatum 6% applied as a cream or
xylol or 20% potassium hydroxide helps to clear ointment is one of the earliest known treatments
cellular debris. Examination under low for scabies. Sulfur is safe for use in pregnant
microscope reveals the mite, ova or fecal pellets women and infants
Q. What is the treatment of scabies? · Antihistamines can be useful in helping provide
relief from itching
A. Patient’s education:
- All persons in the household, whether itching or h
non itching should be treated.
- Even after effective treatment the itching may
persist for upto two weeks
- The clothes should be disinfected.
- Woolen blankets, etc. should be kept locked
inside the cupboard, as the mites die off if they
do not come in contact with humans for 3-5 days
Medical therapy:
- Apply 5% permethrin. These creams are applied
from the neck down, left on overnight, then
washed off. This application is usually repeated
in seven days. Permethrin is approved for use in
people 2 months of age and older
- An alternative treatment is 1 ounce of a 1% lotion
or 30 grams of cream of lindane, applied from
the neck down and washed off after
approximately eight hours
- Ivermectin, an oral medication, is an antiparasitic
medication that has also been shown to be an
effective scabicide, at a dosage of 200 micrograms
per kilogram body weight as a single dose,
followed by a repeat dose two weeks later
- Crotamiton lotion 10% and cream 10% is another
drug that has been approved for the treatment of
scabies in adults
DYP SURVIVAL GUIDE FOR POST GRADUATES 671

- The enlarged sebaceous lobules usually


89. SEBACEOUS HYPERPLASIA circumferentially surround a central
infundibulum .
Q. Why is this sebaceous hyperplasia?
Q. What are the treatment options ?
A. Multiple, yellowish, telangiectatic papules on the
central or upper face . A. - Lesions can be removed by shave excision, light
electrosurgical destruction, cryotherapy or laser
Q. Differential diagnosis? ablation .
A. - Rosacea - Long term topical retinoid may be helpful.
- Milia. - Oral isotretinoin can be used in individuals with
Q. Describe sebaceous hyperplasia? extensive disfiguring lesions.

A. It does not represent a true neoplasm, but it h


represents benign enlargement of the sebaceous
lobule around a follicular infundibulum .
Q. What are the clinical features of sebaceous
hyperplasia?
A. - It is relatively common and presents with one or
more multiple, yellowish, telangiectatic papules
usually on the central or upper face or sometimes
on the trunk.
- The lesions display a central dent that
corresponds to a central follicular infundibular
ostium
- Beaded lines represent a unique expression of
sebaceous hyperplasia in which a linear presence
of papules is seen.
Q. What is the histopathology of sebaceous
hyperplasia?
A. - Thin rim of seboblastic cells surrounding the
remainder of the gland, composed of mature
sebocytes .
670
DYP SURVIVAL GUIDE FOR POST GRADUATES 673

arranged parallel to surface. Thin, branched elastic


90. STRIAE fibres.
Q. Treatment?
Q. Synonyms?
A. Striae rubra can be treated with topical tretinoin and
A. Synonyms:
pulsed dye laser.
a) Striae atrophicans
Striae alba cannot be treated.
b) Striae distensae
Q. Definition? h
A. Striae are linear scars produced by stretching of skin
leading to dermal damage.
Q. Etiology?
A. Occurs secondary to stress rupture of connective
tissue network. This occurs in following conditions:
a) Growth spur in adolescence
b) Pregnancy
c) In weight lifters / bodybuilders
d) Cushing’s disease
Also observed in HIV patients on indinavir therapy.
Q. Clinical features?
A. Striae rubra: initial lesion, slightly raised, red to blue
in color. Fade after some years.
Striae alba: Atrophic linear scar like lesions.
Q. Sites affected?
A. Abdomen, thighs, shoulders, breasts.
Q. Striae in cushing’s syndrome?
A. Larger, more widely distributed, involve flexors
along with other regions.
Q. Histopathology?
A. Flattened epidermis, dermal oedema, perivascular
lymphocytic infiltrate, eosinophilic collagen bundles
672
DYP SURVIVAL GUIDE FOR POST GRADUATES 675

Q. Which dermatophyte causes the most fungal


91. SUPERFICIAL FUNGAL infections of the skin?
INFECTION A. T. rubrum
Tinea Barbae
Q. What is a dermatophyte?
Q. Why will you say this case is T.barbae?
A. Dermatophyte is a fungus that has developed
A. Since the lesion is nodular & boggy like kerion&
the ability to live on keratin( hair, nails or skin scale) associated with seropurulent discharge & crusting.
of animals. Dermatophytes are classified into 3 Seen mostly on the chin, neck, maxillary
genera- MICROSPORUM, TRICHOPHYTON & &submaxillary area.
EPIDERMOPHYTON.
Perifollicular pustules are present. Heals with
Q. How are superficial fungal infections diagnosed? scarring & alopecia.
A. Mostly by KOH mount ( skin scraping + ½ drops of Q. What the common types of T.barbae?
10-20% KOH) Hyphae of dermatophytes will be A. a) INFLAMMATORY TYPE (discussed above)
septate and demonstrate branching.
b) SUPERFICIAL/SYCOSIFORM TYPE
Q. Most commonly used culture medium?
Diffuse erythema with perifollicular papules &
A. a) SABOURAUD’S DEXTROSE AGAR pustules
Nonselective culture medium consisting of peptone, c) CIRCINATE/SPREADING TYPE
dextrose agar & distilled water.Allows growth of Vesiculopustular lesions at periphery with
bacteria & yeast. central scaling.
b) MYCOSEL/MYCOBIOTIC AGAR Q. What are the DD’s of T.barbae?
Selective growth medium. Consists of Sabouraud’s A. Sycosis barbae, Acne, Carbuncle, Furuncle, Herpes
+ Cycloheximide( inhibits saprophytic fungi) & Simplex
Chloramphenicol ( inhibits bacteria) Dermatophyte& Q. What are the atypical forms of T.barbae?
Candida grow on this.
A. - Granuloma annulare like/ Abscess like caused
c) DERMATOPHYTE TEST MEDIA by M.canis
Sabouraud’s + Cycloheximide + Gentamycin + - Verrucous granulomatous lesion caused by E.
Chlortetracycline Hydrochloride. Has a phenol red floccusum
indicator. If dermatophytes are present, the colour of Q. Most common method of transmission of T.barbae?
media changes from yellow to red.
A. Person to person by contaminated barbers razors
674
676 DYP SURVIVAL GUIDE FOR POST GRADUATES DYP SURVIVAL GUIDE FOR POST GRADUATES 677

Q. Treatment of T.barbae? Q. Treatment?


A. ORAL A. Mostly topical treatment such as Econazole,
Griseofulvin- 0.5-1 g/day x 2-4 weeks Miconazole and Ketoconazole cream. Applied BD for
4-6 weeks.
Itraconazole- 200 to 400mg OD x 1 week
Terbinafine- 250mg OD x 2 weeks Tinea cruris
Fluconazole- 150 to 300 mg x 4-6 weeks Q. Why will you say the case is of T.cruris?
TOPICAL A. Annular plaque with well -marginated raised border
composed of multiple, erythematous papulovesicles
- Econazole
and a clear centre. Common sites involved are
- Miconazole genitocrural area and on medial aspect of upper thigh
- Fluconazole and usually bilaterally present. They coalesce and
These creams applied BD for 4-6 weeks spread to buttock, lower back & abdomen. Itching
present.
Tinea faciei
Q. Why will you say the above case is T.faciei? Q. What are the complications?
A. The lesion begins as an erythematous scaly macule A. - Lichenification.
that extends peripherally & develops a raised border. - Secondary bacterial infection
Central area is hypopigmented/ brown. - Weeping, maceration and areas of pustualtion
It is associated with burning, itching & photo- present.
sensitivity. Q. Differential diagnosis of tinea cruris.
Some cases show simple papular lesions & flat A. Candidiasis, P.versicolor, Erythrasma, Intertrigo,
patches of erythema occur Psoriasis.
Q. What is T. faciei? Q. What is T.cruris?
A. Dermatophyte infection that occurs on non bearded
A. Dermatophytic infection of groins. Also called Dhobi’s
areas of face. Commonly seen in women &prepubertal
itch/ Jock itch.
boys.
Q. Why is T.cruris more common in males?
Q. Causative agents of T.faciei?
A. Because T.pedis (with which T.cruris is associated)
A. T.mentagrophytes & T.rubrum
is more prevalent in men, males perspire more,
Q. Differential diagnosis of T.faciei? anatomical difference and clothing difference.
A. Seborrhoeic Dermatitis, Atopic Dermatitis, Furuncle, Q. Why is T.cruris more prevalent in obese persons?
Carbuncle, Irritant Contact Dermatitis, P.alba,
Rosacea A. Constant mechanical irritation of apposed surface,
678 DYP SURVIVAL GUIDE FOR POST GRADUATES DYP SURVIVAL GUIDE FOR POST GRADUATES 679

decreased aeration, increased sweating, difficulty in - Manual labour


maintaining hygiene & inspecting area involved - Hyperhidrosis
Q. Causative agents? - Existing inflammatory dermatoses
A. T.rubrum, T.mentagrophytes&E.fluccosum Q. Types of T.manuum
Q. Mode of transmission of T.cruris? A. NON INFLAMMATORY SQUAMOUS FORM
A. Direct contact or fomites - Hyperkeratosis of palms and fingers affecting skin
Q. Treatment of T.cruris? - Accentuation of flexural creases
A. ORAL - Hyperhidrosis present
- Fluconazole- 150 to 300mg once a week x 4 weeks - Nails involved
- Itraconzaole- 200 to 400 OD x 1 week INFLAMMATORY VESICULAR/DYSHYDROTIC/
- Terbinafine- 250mg OD x 1 week ECZEMATOUS FORMà
- Griseofulvin- 500mg OD x 2-4 weeks Vesicles occur in clusters mainly on palms though
TOPICAL dorsal dorsal surface maybe affected. Lesions
arranged in annular/segmental pattern.
- Ketoconazole, Econazole, Terbinafine and
Ciclopirox cream. T.mentagrophytes is the usual causative organism in
this case.
Tinea manuum
Q. Other clinical variants?
Q. Define T.manuum
A. - Crescentric, circumscribed vesicular patches
A. Defined as ringworm of palmar skin and with
infections beginning under rings - Folliculopapular lesions
Q. Causative agents? - Dorsal erythematous type
A. T.rubrum&E.flucossum Q. DD of T.manuum?
Q. Pre-disposing factors for T.manuum. A. - Contact Dermatitis
A. - Pre-existing foot infection - Psoriasis
- Under rings - Eczema
- Watches - Tylosis
Tinea pedis
- Bangles
Q. Define T.pedis
- Poor circulation
A. Dermatophyte infection of feet. Also called Athlete’s
- Tylosis
foot.
680 DYP SURVIVAL GUIDE FOR POST GRADUATES DYP SURVIVAL GUIDE FOR POST GRADUATES 681

Q. Etiology of T.pedis anterior plantar surface. Scaling present. Infection


A. - Hands & feet are a favourable environment for flares up in summer
dermatophytebecause of their lack of sebaceous ACUTE ULCERATIVE VARIANT
glands that produce fungistatic lipids. Maceration, weeping and ulceration present. White
- Males> Females. hyperkeratosis and pungent odor present.
- Increased sweating washes off lipids that inhibit Complicated by secondary bacterial infection.
fungi Q. What are the DD’s of T. pedis?
- Lateral toes affected particularly between 4-5th A. Candidiasis, Erythrasma, Soft Corn, Pustular
toes Psoriasis, Pompholyx.
- Hot, humid climate. Q. Write a note on Athlete’s Foot.
- Occlusive shoes. A. - Caused by bacteria and fungi.
- Immune deficiency condition. - C/f are maceration, white hyperkeratosis and
Q. Types? erosions with pain and pruritus.
A. CHRONIC INTERTRIGINOUS TYPE Q. Why will you say the following case is of T.incognito
Commonest type. A. Suppression of inflammation with change in
Characterized by fissuring, scaling, maceration in morphology of lesion and decrease in associated
interdigital or subdigital areas. 3rd& 4th toe webs are symptoms. Raised margins are decreased, scaling lost
commonest sites of infection. Aggravating factor are and inflammation reduced.
warmth, humidity & hyperhidrosis Due to chronic use of topical steroids, local side effects
CHRONIC PAPULOSQUAMOUS TYPE become obvious ie concentric rings of erythema
within atrophy and telangiectasia.
Inflammation and a patchy/diffuse moccasin like
scaling over soles Tinea incognito
Affected areas are soles, heels and sides of feet Q. Define T.incognito
Multiple toenails involved A. Tinea modified by topical/systemic immunosuppressant
treatment due to misdiagnosis or given for some co-
One hand- 2 feet presentation existing dermatosis is known as T.incognito.
Seen in patients with Atopic Dermatitis Q. Who first described T.incognito?
VESICULAR/VESICULOBULLOUS TYPE A. Ive & Marks, 1968
T.mentagrophytes var interdigitale, E.fluccosum are Q. What is the recent cause of T.incognito?
main causative agents of this type. Small vesicles/
vesiculopustules seen near instep and on mid A. calcineurin inhibitors
682 DYP SURVIVAL GUIDE FOR POST GRADUATES DYP SURVIVAL GUIDE FOR POST GRADUATES 683

Q. Why will you say this is a case of Majocchi’s of fungus, immune status and extent of follicular
Granuloma? invasion. In inflammatory lesions- pustules/vesicles
A. Because the patient complains of non pruritic, present
solitary/multiple persistantpapulopustules/ In quieter lesions- scaling present
plaques on the legs. Associated with onychomycosis /
Central skin may show PIH
T.pedis
Q. DD?
Q. Describe the types of Majocchi’s Granuloma
A. P.rosea, Impetigo, Nummular Eczema, GA, Psoriasis,
A. FOLLICULAR TYPE
Seborrhoeic Dermatitis, Lichen Planus, Bacterial
Occurs secondary to trauma. Immunosuppression
infection
not associated. Common in young women who shave
their legs repeatedy. Q. Define T.corporis
SUBCUTANEOUS NODULAR TYPE A. Dermatophytosis of glabrous skin with exclusion of
Immunocompromised state which maybe primary/ palms, soles and groin
drug induced. Develops on hair bearing area mostly Q. PDF?
on the scalp, face, forearm, hands & legs. Lesion A. Poor hygiene, poor nutrition, DM, Leukemia
begins as single/multiple well defined oval patch/
scaly ones and evolve into peri-follicular papulo- Q. Causative agents?
pustules and nodules. A. T.rubrum, T.mentagrophytes, M.canis
Q. DD? Q. What is T.imbricata?
A. T.incognito, Folliculitis, Nodular Scabies, Kerion A. Variant of T.corporis caused by T.concentricum.
Q. How will you diagnose such a case? Limited to certain areas of Far east, South Pacific &
A. H & E – Hyphal invasion in cornified keratinocytes Central AmericA. Synonym is Indian TineA.
of hair follicle which produces suppurative Contracted in childhood & persist for lifetime. Lesions
folliculitis with rupture of hair follicle and spillage begin as squamous annular concentric plaques with
of contents into dermis. Healing is with fibrosis. erythemA. Later they became lamellar as they have
PAS – Fungal hyphae in tissue surrounded by foreign thick scales that adhere to 1 side giving appearance
body granulomatous reaction. Of tiles, fish scales, lace
Tinea Corporis Q. Types of hair invasion in T.capitis?
Q. Why will you say this is a case of T.corporis? A. ENDOTHRIX–Produced by fungi that invade the
A. Typical lesion is annular/polycyclic. Borders are inside of hair shaft & are composed of fungal
erythematous & vesicular/scaly but centre is clear. arthroconidia and hyphae. Causative agents- T.
Degree of inflammation varies depending on species tonsurans, T.violaceum, T.soudanese
684 DYP SURVIVAL GUIDE FOR POST GRADUATES DYP SURVIVAL GUIDE FOR POST GRADUATES 685

ECTOTHRIX–Produced by fungi that invade outside FAVUS–


of hair shaft Yellow cup shaped crust composed of dense mat of
FAVUS–Invasion of hair by hyphae mycelia & epithelial debris. Called scutulum due to
Wood’s lamp fluorescence is produced by: shield shape. As infection progresses scutulae enlarge
- Trichophyton schoenleinii & merge to form yellow crust. Odor present. Centre
of infected area is scarred & devoid of hair.
- Microsporum canis
Q. DD?
- Microsporum audouinii
A. - Alopecia Areata
- Trichophyton distortum
- Trichotillomania
- Trichophyton ferrugineum
- Secondary Syphilis
Non fluorescent tinea capitis caused by:
- Atopic Dermatitis
- Trichophyton tonsurans
- Seborrhoeic Dermatitis
- Trichophyton violaceum
- Psoriasis
- Trichophyton soudanense
Q. Complications?
Q. C/f of T.capitis?
A. Secondary bacterial infection, Cicatricial Alopecia
A. GRAY PATCH/ NON INFLAMMATORY
Q. Treatment?
Lesions begin as small, erythematous papule
surrounding hair shaft. Spreads centrifugally A. TOPICAL
involving all hair in its path. Patches of hair loss, - Ketoconazole
patches are circular in shape, show broken hair that ORAL
are dull, grey & lustreless. Caused by M. audouinii
- Griseofulvin 20-25 mg/kg/day for 8 weeks
KERION/INFLAMMATORY TYPE
- Ketoconazle 200mg OD for 4 weeks
Due to T.verrucosum/T.mentagrophytes.Kerion is an
inflamed boggy & tender swelling ie studded with - Itraconazole 5mg/kg/day for 4-6 weeks
broken/unbroken hair, vesicles and pustules. Thick - Terbinafine 3-6mg/kg/day for 4-8 weeks
crusting with matting of adjacent hair.Secondary - Fluconazole 3-6mg/kg/day for 6 weeks
bacterial infection present. Heals with scarring
Q. What are the types of types of Onychomycosis?
BLACK DOT
A. - Distal & Lateral SubungalOnychomycosis
Caused by endothrix organism. Hair shaft is brittle
& breaks at level of scalp. Remnant of hair left behind - White Superficial Onychomycosis
in follicle appears as black dot. When hair loss occurs - Proximal Subungal Onychomycosis
affected areas are multiple & polygonal. - Endonyx
686 DYP SURVIVAL GUIDE FOR POST GRADUATES DYP SURVIVAL GUIDE FOR POST GRADUATES 687

- Total dystrophic Q. DD?


- 20 nail dystrophy A. - Psoriasis
Q. Why will you say this is Distal & Lateral - Eczema
SubungalOnychomycosis? - Lichen Planus
A. Most common type - Darrier’s Disease
Mostly caused by T.rubrum TINEA NIGRA
Toenails > Fingernails Q. Why will you say this is a case of T.nigra?
Starts by invasion of stratum corneum of A. Lesions are asymptomatic , macular, sharply defined,
hyponychium of distal/lateral nail bed.Subungal non scaly patches. Most characteristic feature is the
hyperkeratosis results from hyperproliferative brown/ black colour which resembles AgNO3 stain
reaction of nail bed response to infection. Splinter & which is due to accumulation of melanin like
haemorrhages maybe seen due to mild inflammation substance in fungus.
compressing vessels. Q. Which are the most common sites affected?
Q. Why will you say it is White Superficial A. Palms are most commonly affected but other parts of
Onychomycosis? body such as soles, trunk, neck even penis maybe
A. Second most common type. There is invasion of involved
dorsal surface of nail plate. Causative agent is Q. DD?
T.mentagrophytes. Characterized by well A. - P.versicolor
circumscribed powdery white patches away from free
- PIH
edge of the nails. Ultimately whole surface of nail
plate may be affected. Fingernails are not affected. - Addison's Disease
Surface of nail is rough and friable. - Syphilis
Q. Why will you say it is Proximal Subungal - Pinta
Onychomycosis? - Junctional Naevi of palms
A. Least common variant of onychomycosis.Commonly - Melanoma
caused by T.rubrum. First clinical sign is whitish to Q. Treatment?
brownish area on proximal part of nail plate. The
A. Topical Antifungals
organisms invade the ventral surface of nail plate
from proximal nail fold. This type is an early indicator - Cotrimoxazole or Econazole ointment
of HIV and is associated with peripheral vascualar - Thiabendazole 10% suspension OD for 2 weeks
compromise. - Whitfield's Ointment
688 DYP SURVIVAL GUIDE FOR POST GRADUATES DYP SURVIVAL GUIDE FOR POST GRADUATES 689

- Topical Retinoic acid, Epidermal tape stripping Q. What is 'coup de ongle' sign?
& Ciclopirox Olamine A. When the lesion of P. Versicolor is scratched with
Oral Itraconazole & Ketoconazole fingernail, the branny scales become more obvious.
Q. What is the causative agent? Other names-Besnier's sign, scratch sign
A. Phaeoannellomyces werneckii Q. Treatment?
Q. Investigations? A. Effective topical agents include:
A. Scraped Material' shows Branched thick septate - selenium sulfide, sodium sulfacetamide,
hyphae. ciclopirox olamine, as well as azole and
Culture on Sabbouraud's Dextrose Agar & Oatmeal allylamine antifungals.
Agar. - Topical allylamines have been demonstrated to
Growing colonies that are white/grey at 1st & later be clinically and mycologically effective.
become more deeply pigmented & develop aerial Oral medications include:
hyphae. - ketoconazole 200-mg daily therapy for 5 -25 days
TINEA VERSICOLOR or 400mg stat dose
Q. Why will you say this is a case of P.versicolor? - Fluconazole as a single 400mg dose.
A. Usually the condition is asymptomatic. Chief lesion - Itraconazole is usually given at 200 mg/day for
is macule that maybe hypopigmented/ 7 days or 400mg stat.
hyperpigmented & covered with branny scales. - Pramiconazole and sertaconazole have also been
Upper trunk commonly involved but infection used in the management of tinea versicolor.
usually extends to upper arms, neck & abdomen.
Q. DD? h
A. - Vitiligo
- Pityriasis rosea
- Seborrhoeic Dermatitis
- Secondary Syphilis
- Hansens
- P. alba
Q. What does pityriasis versicolor mean?
A. Pityriasis means bran like and versicolor means
variegated color.
DYP SURVIVAL GUIDE FOR POST GRADUATES 691

Q. What are the complications of tattoos?


92. TATTOO
A. Infections(bacterial infections like syphilis, leprosy,
tuberculosis and atypical mycobacteria, viral
Q. What is the meaning of word "Tattoo"?
infections like HIV, Hepatitis B and deep fungal
A. It comes from the Hawaiian word "Tatau" meaning infection) and allergic reactions are the most common
rhythmic beat. side effects of tattoos.
Q. What are the types of tattoos? Reactions are mostly in form of erythematous nodules
A. There are two types: and plaques. Lichenoid and eczematous reaction
A. Decorative Tattoos may also be seen.
B. Accidental Tattoos: Pigmented particles may be Photo- allergic reactions occurs in red or yellow
introduced accidently into the skin as in road tattoos containing cadmium sulphide.
traffic accidents, blast injuries, penetrating Q. Explain the histological findings of tattoo.
wounds, therapeutic or occupational hazards. A. In inflamed tattoo: Different patterns of inflammatory
Examples: therapeutic : iron salts , gentian violet. response including sarcoidal, foreign body
Collier's stripes. granulomatous, lichenoid or pseudo-lymphomatous.
Q. What are Collier's stripes? In non-inflamed tattoos: Dermis shows granules
within macrophages and extracellulary, in the absence
A. Bluish grey angular or linear stripes at sites of
of an inflammatory infiltrate. Tattoo pigments appear
abrasion in coal miners.
black with H & E staining but sometimes red or
Q. What are the chemicals present in various inks and yellow colors are apparent histologically.
stains?
Q. What are the skin diseases associated with tattoos?
A. Mercury Red
A. Lichen planus, psoriasis and rarely lupus
Carbon Black erythematosus can be seen in association with tattoos.
Cinnabar and vegetable dyes Red Q. What are the modalities for tattoo removal?
Cadmium Yellow A. Lasers such as:
Cobalt Light blue - Q-switched ruby
Ochre and iron oxide Brown - Alexandrite
Chromium oxide Green - Nd-YAG
Manganese Purple The specific type of laser depends on the tattoo ink
690
692 DYP SURVIVAL GUIDE FOR POST GRADUATES

colors and the wavelengths to which they best 93. TRICHOTILLOMANIA


respond.
Q. How will you manage an inflamed tattoo? Q. Why trichotillomania?
A. Surgical excision is the treatment of choice.
A. Patchy loss of hair in an easily accessible area, with
Topical steroids are generally found ineffective; but hair broken a different length and no scaling or signs
intralesional steroids can be tried. of inflammation.
Laser treatment is contraindicated as there is a risk Q. What are the differential diagnosis of
of systemic reaction following release of pigment in
trichotillomania?
an already sensitized patient.
A. – Alopecia areata
Q. Relationship between syphilis and tattoos.
– Tinea capitis.
A. Syphilis can be inoculated through tattoos (Syphilis
d' emblee). Also in case of secondary syphilis, it is Q. Trichotillomania is also known as?
noted that the lesions spare a red tattoo since the red A. Habitual hair pulling.
pigment is mercury and mercury is known to be
Q. What does trichotillomania mean?
active against treponemes.
A. Hallopeau in 1889 and is derived from the Greek thrix
h hair, tillein pull out and mania madness. Literally it
means a morbid craving to pull out hair.
Q. In which age group is it more common?
A. Bimodal distribution.
1] Children- More common in boys.
2] Adolescent- More common in females.
Q. What are the diagnostic criterias for trichotillomania?
A. The revised DSM-IV diagnostic criteria for
trichotillomania are:
A) Recurrent pulling out of one’s own hair resulting
in hair loss
B) An increasing sense of tension immediately
before pulling out the hair or when attempting
to resist the behaviour.
693
694 DYP SURVIVAL GUIDE FOR POST GRADUATES DYP SURVIVAL GUIDE FOR POST GRADUATES 695

C) Pleasure, gratification or relief when pulling out Q. What is Trichobezoar?


the hair A. Bezoars are ball-like aggregations of vegetable or
D) The disturbance is not better accounted for by fibre-like materials in the stomach and small intestine
another mental disorder seen almost exclusively in girls and young women.
Swallowed hair is retained within the folds of the
E) The disturbance provokes clinically marked gastric mucosa. Action of peristalsis stimulate the
distress and/or impairment in occupational, hair to become enmeshed into a ball. The hair ball
social or other areas of functioning may eventually completely fill the stomach. Hair ball
Q. Which body parts hair are involved in trichotillomania? is decomposed to a degree by gastric acidity and
A. It involves scalp, eyebrows, eyelashes or pubic hair. fermentation of fats and food in the interstices of the
hair gives a putrid smell and significant halitosis.
Q. What are the clinical features of trichotillomania?
Q. What are the complications of trichobezoar?
A. – Hair pulling and plucking is commonest from A. 1] Halitosis
the scalp.
2] Gastric erosions
– Typically, the hairs are short, irregular, broken
3] Intestinal bleeding, pancreatitis or obstructive
and distorted. symptoms, particularly if the hair bolus extends
– The patterns of plucking activity are centrifugal down the small intestine.
from a single starting point or linear, in wave- Q. What is the Rapunzel syndrome?
like activity.
A. It is trichobezoar with a tail that extends at least to
– In extreme cases, the centrifugal pattern removes the jejunum producing gastrointestinal obstructive
all hair except the most difficult to access symptoms.
(occiput). This shows as a ‘tonsure pattern’ or ‘Friar Q. What are the other “tricho” seen in trichotillomania ?
Tuck’ distribution. A. – Trichomalacia : the process of injured follicles
– The eyelashes, eyebrows, facial and pubic hair forming soft twisted hair.
may also be primarily affected. – Trichotemnomania : the patient cuts his/her hair
– Plucking may be confined to a specific time and with scissors.
place, with ritualized manipulation of the pulled – Trichoteiromania : perpetual rubbing of hair
hairs prior to discarding. leading to alopecia because of hair shaft
– Some patients may practice trichophagy, the breakage
chewing and swallowing of the hair that has been Q. How will you investigate the case of trichotillomania?
pulled out, possibly leading to intestinal A. - Scalp biopsy.
obstruction from trichobezoars. - Barium contrast (trichobezoar).
696 DYP SURVIVAL GUIDE FOR POST GRADUATES

Q. What are the histological features?


94. TUBEROUS SCLEROSIS
A. - Normal hairs are seen amongst empty hair
follicles in a non-inflamed dermis. Many of the
Q. Why is this TSC?
empty hair follicles show evidence of changing
into the catagen phase. A. - Cutaneous angiofibroma (Adenoma sebaceum)
- Follicular plugging with keratin debris. - 10mm reddish pink, dome shaped papules,
distributed symmetrical on nasolabial folds,
- The hair shafts are often broken or reduced to cheeks, chin, eyelid, forhead, ears and scalp.
little dark bodies.
- Ash leaf spots- 1-3cm lanceloate hypopigmented
- Trichomalacia is seen as deep distortion and macules present on trunk or limb.
curling of the hair bulb. - Confetti like lesion- numerous 1-3mm sized
- In severe damage there is intraepithelial and white spots symmetrically distributed on
perifollicular damage. extremities.
Q. What is the treatment of trichotillomania? - Fibrotic Plaque/nodule. Skin colored, firm, large
plaque or nodule present on forehead or cheek
A. 1] Behavioural therapy or scalp.
2] Pharmacotherapy (tricyclic or SSRI antidepressants). - Shagreen’s patch- skin coloured, raised, soft
3] For many adolescents and children, identification plaque with irregular thickening and multiple
of stressful episodes with accompanying dimples at follicular opening stimulating orange
support and parent education. Teaching peel appearance, vary from 1cm to several inches
surrounded by satellite lesion. It commonly
awareness and relaxation training can be done. occurs in lumbosacral region,but other areas like
back of breast may be involved.
- Koenen Tumors- smooth,skin coloured,firm,
periungal or subungal fibroma arising from nail
folds.
Q. Define tuberous sclerosis complex?
A. Tuberous sclerosis complex is a multisystem
hamartomatous condition involving skin, eyes, CNS,
CVS, lung, kidney and bones. It is also called
Bourneville’s disease or epiloiA.
Q. What is etiopathognesis of tuberous sclerosis?
A. It is a genetic disorder following AD inheritance. It
occurs due to mutation of two genes. TSC1 and TSC2
697
698 DYP SURVIVAL GUIDE FOR POST GRADUATES DYP SURVIVAL GUIDE FOR POST GRADUATES 699

located on chromosome 9q34 and chromosome 16p13 - Confetti like lesion-they are numerous 1-3mm
respectively. Mutation may occur in TSC1 and TSC2. sized white spots symmetrical disturbuted on
TSC1 and TSC2 encode for two protein hamartin and extremities. More comman after age of 10years.
tuberin respectively. TSC1 is biologically similar to - Fibrotic Plaque/nodule. Its mostly present on
NF1 with tumor suppression activity. forhead or cheek or scalp. Lesion may be present
Loss of tuberin cause activation of Rap1 which is a at birth. It is skin colored, firm and large.
GTPase activating protein in patient of tsc.
- Shagreen’s patch-occurs in 14-20% patients,
Sporadic cases occur due to mutation in TSC2. concurrent with angiofibromA. The lesion is skin
Q. What is Ash leaf spot also known as? coloured, raisied, soft, irregular thickening and
A. Ash leaf spot is also known as Fitzpatrick macule or multiple dimples at follicular opening
lanceolate macule. stimulating orange peel appearance. They vary
Q. What are the different types of adenoma sebaceum? from 1cm to several inches. It is larger lesion
A. Pringle type, Balzer type, Butterworth and Wilson surrounded by satellite lesion. It commonly
type. occurs in lumbo-sacral region, but other areas
Q. What is the penetration of TSC? like back of breast may be involved.
A. High. - Koenen Tumors-periungal or subungal fibroma
found in 18% of patients. They are smooth, skin
Q. Why is adenoma sebaceum a misnomer?
coloured, firm projections arising from nail folds.
A. It is neither a adenoma nor does it arise from the
sebaceous gland; it is an angiofibroma. SYSTEMIC FINDINGS
Q. What are clinical features of TSC? 1) Ocular features-
A. - Cutaneous angiofibroma (Adenoma sebaceum) - Retinal hamartomatosis in 50% patients present
as yellow grey streaks along blood vessels or
- It appears at 3-4 years of age, but may manifest
from birth to third decade of life. Lesions are 1- elevated multinodular lesion resembling
10mm are reddish pink, dome shaped papules, mulberries.
distributed symmetrical on nasolabial folds, - White pendulous tumors of palpebral
cheeks, chin, eyelid, forhead, ears and scalp. It is conjunctiva
more extensive during puberty. - Glial hamartoma of optic disc
- Ash leaf spots-they are 1-3cm lanceloate - Retinal angioma
hypopigmented macules present on trunk or
limb. Visualisation of these in wood lamp seen - Colobomas
in 90% cases either present from birth or infancy. - White spots on iris
700 DYP SURVIVAL GUIDE FOR POST GRADUATES DYP SURVIVAL GUIDE FOR POST GRADUATES 701

2) CNS features- Q. What is diagnostic criteria of TSC?


- Sclerotic areas-subependymal nodules, tubers in A. major features
cerebral cortex in region of gyri and wall of · Facial angiofibroma or forehead plaque
ventricles.
· Non traumatic periungal fibroma
- Subependymal giant cell astrocytoma
· Hypomelanocytic macules>3
- Seizures, mental retardation, obstructive
hydrocephalus, behavioural disorders · Shagreen’s patch
3) Cardiac lesion · Retinal hamartomas
- Rhabdomyomas occur in infancy in 50% patients · Cortical tubers
- Asymptomatic · Subependymal nodules
- Second peak in puberty · Subependymal giant cell astrocytoma
- Cardiac arrhythmias · Cardiac rhabdomyoma
4) Renal lesions · Lymphangiomatosis
- Involves 15% patients. · Renal angiomyolipoma
- Most commonly angiomyolipoma, usually Minor features
asymptomatic · Dental enamel pits
- Bilateral multiple renal cyst · Hamartomatous rectal polyps
- Malignant angiomyolipoma · Bone cyst
- Renal cell carcinoma · Cerebral white matter migration lines
5) Pulmonary lesions · Gingival fibroma
- It is rare
· Non renal hamartomas
- Multifocal, multinodular pnemocyte hyperplasia
· Retinal achromic patch
giving rise to recurrent spontaneous pneumothorax.
· Confetti skin lesion
- Pulmonary lymphangio-leiomyoma gives rise to
chylothorax. · Multiple renal cyst
OTHER SYSTEMIC FINDINGS - Definite diagnosis
- Colonic polyposis 2 major features or 1 major +2 minor features
- Cyst in liver - Probable diagnosis
- Bony abnormalities 1 major +1 minor feature
702 DYP SURVIVAL GUIDE FOR POST GRADUATES DYP SURVIVAL GUIDE FOR POST GRADUATES 703

- Possible diagnosis DIFFERENTIAL DIAGNOSIS


- Either 1 major or 2 or more minor features. 1. Acne
Q. What are investigations? 2. Triepithelioma
A. Fully manifested syndrome present with few 3. Verruca
problems in diagnosis. 4. Millia
· Difficulty in diagnosis arises in childhood when 5. Xanthoma
disease evolution is incomplete. · Reddish hue of lesion is characteristic
· In infancy and childhood, presenting features are · Histopathology of the lesion reveals capillary
seizure disorder, developmental delay, mental dilation and dermal fibrosis and atrophy and
retardation, behavioural problem. downward displacement of sebaceous unit.
· Cutaneous lesion may be present at birth as ash- leaf · Ash leaf spots may be confused with vitiligo but
spot, forehead plaque. can be confirmed histopathologically by presence
of active melanocytes.
· In a child when other causes are ruled out, screening
for TSC done by wood lamp examination of whole Q. What is course and prognosis of TSC?
skin for ash leaf spots. A. - common causes of death are seizure and cranial
· Ophthalmic examination for retinal hamartomas disorder
- life expectancy of severely affected patient is
· CT /MRI scanning for brain
poor
· Neonatal 2 D echo for rhabdomyoma
- people without seizures may have normal
· Fetal ECG intelligence and life span
· In older patients, imaging of different organs. Q. What is treatment of TSC?
· Calcification of skull seen in 50% cases. A. Neurosurgical interventions are needed in presence
· Detection of calcified periventricular nodules in of obstructive hydrocephalus.
CT/MRIbrain Microsurgical removal of brain neoplasm causes
· Abdominal USG /CT helpful for detecting renal improvement in life expectancy and prognosis.
lesions. Angiofibroma can be removed by pulse dye laser.
· Other investigation-chest X-ray, ECG, X-ray of Recently, Rapamycin can induce regression of brain
hand and feet for cystic lesion of phalanges and astrocytes.
irregular thickening of cortex of meta tarsal
h
DYP SURVIVAL GUIDE FOR POST GRADUATES 705

Q. What treatment is available?


95. TWENTY NAIL DYSTROPHY
A. There is no specific treatment for twenty-nail
dystrophy. It is a very difficult condition to treat and
Q. Who 1st described 20 nail dystrophy?
often results are unsatisfactory.
A. Hazelrigg and colleagues in 1977
- Some treatments that have been tried include:
Q. What is 20 nail dystrophy?
- Systemic, topical and intralesional
A. Twenty-nail dystrophy is also known as corticosteroids
‘trachyonychia’. It could be said that twenty-nail
dystrophy is widespread trachyonychia involving all - Topical PUVA
20 nails. The condition is characterised by - Topical 5-fluorouracil
longitudinal ridging (alternating elevation and - Oral biotin
depression), pitting, loss of lustre, and roughening
(similar to sandpaper) of the nail surface. - Emollients
Q. What are the causes of 20 nail dystrophy? h
A. - Idiopathic
- Lichen Planus
- Rarely psoriasis
- IgA deficiency
- Primary Biliary Cirrhosis
- Pemphigus(rare)
Q. What are the types of nail changes that occur?
A. Type 1:Whole nail gives appearance of having been
sandpapered in longitudinal direction. Excessive
ridging and roughness that makes nails lustreless.
Type 2:Less severe, nail plate is shiny with myriad
miniscule superficial punctate depression.
Thinness, fragility, onychorrhexis, onychoschizia and
distal clipping of nail plate may occur. Yellowish
thickening of thumb and toe nails is seen.
Q. What is the h/p findings?
A. Spongiotic inflammation with fairly dense
mononuclear inflammatory infiltrate.
704
DYP SURVIVAL GUIDE FOR POST GRADUATES 707

Aquagenic urticaria: Water induced urticaria small


96. URTICARIA papules resembling micropapular hives of
cholinergic urticariausually within minutes of
Q. Causes of acute urticaria? exposure to water .
A. Common in children and young adults after allergy Autoimmune urticaria: 30 -40 % of patients with
to food, drugs , insect bites, or acute viral infection. idiopathic urticaria have clinically relevant functional
Q. Causes of chronic urticaria? autoantibodies, to high affinity ige receptor on
A. Can be due to physical stimulus: heat, pressure, basophils and mast cells. patients have chronic severe
vibration and cold or auto immune basis. urticaria with highly symptomatic , with systemic
complaints.
Q. Name few mediators of causing vasodilatation and
increasing vascular permeability? Hereditary angioedema: dominantly inherited
disorder in which a normal serum inhibitor of the
A. Histamine , prostaglandin d2, leukotriene c4 and d4,
activated firs componenent of complement of c1
platelet activating factor, bradykinin.
esrterase inhibitor is either deficient or non
Q. Causes of cholinergic urticaria? functional.
A. Exercise , warm bath , showers, emotions are triggers, Q. Describe lesions of urticaria ?
there is an increased efferent cholinergic output to
A. a. Urticarial wheals are raised, erythematous,
skin leading to mast cell degranulation.
edematous plaques with sharply defined,
Q. Classification of urticaria? serpiginous and polycyclic borders surrounded
A. If duration of symptoms less than 6 weeks then acute by erythematous halo.
urticaria if more than 6 weeks then chronic urticaria. b. Diameter ranging from few mm to cm’s,
Acute urticaria: Mediated by B-lymphocyte individual lesions last upto 12 hours, if lesion
produced ige , and seen in patients with anaphylaxis, lasting for more than 24 hours, then urticarial
serum sickness, atopy, reactions to food, drugs . vasculitis should be a differential diagnosis.
Food like, nuts, eggs, sstrawberries, tomatoes, c. Angioedema- edematous plaques involving the
chocolate, citrus. face, not associated with pruritus but with pain
Infections, - hepatitis B, H . pylori, gi parasites . or burning, the edema lasts upto 72 hrs. laryngeal
edema is noted too.
Chronic urticaria: causes – drugs , food additives ,
inhalants. d. Cold urticaria – may be familial or acquired.
Contact urticaria: direct contact of chemical with skin, e. Associated symptoms include fever, arthralgias,
may be immunologic ( drugs , cosmetics, natural latex leukocytosis. Patients develop localized or
rubber. Non- immunologic( stings, plants, chemicals.) generalized wheals or angioedema .

706
708 DYP SURVIVAL GUIDE FOR POST GRADUATES DYP SURVIVAL GUIDE FOR POST GRADUATES 709

f. Urticarial vasculitis- painful , burning urticarial of 20 mg/ day may cause ventricular
papules / plaques that last longer than 24 hours, arrhythmias.
postinflammatory hyperpigmentation or 3. Loratadine is intermediate between fast acting
purpura may be noted. Evaluate for complement terfenadine and slower astemizole but less
levels and systemic symptoms. ef-fective than cetirizine.The dose is 10 mg/day.
Q. History and investigations in urticaria?
4. Cetirizine is compared more favourably with
A. Medication history- aspirin, NSAID, COX -2 other non sedating anti-histamines. Dose of 10
AGENTS, ACE inhibitors, beta blockers . mg/day has a more rapid and long lasting effect
Investigations – CBC, ESR, urinaanalysis, lft . when compared to terfenadine or loratadine.
Leukocytosis indicates chronic infection, total 5. Fexofenadine has been used in the treatment of
eosinophilic count may indicate cause as drugs, food, chronic idiopathic urticaria at a dose of 180 mg
parasites, atopy. per day. Notably Q-T interval prolongation
If suspicion of necrotizing vasculitis or collagen occasion-ally seen with the parent compound,
vascular disease. terfenadine, does not occur with fexofenadine.
Autologous serum skin test (ASST) is a simple test Q. Can antihistamines be prescribed in pregnancy?
for diagnosing autoimmune urticaria. A. Antihistamines cross the placenta but are not
Q. How will we treat urticaria ? teratogenic. They are better avoided in pregnancy and
A. Traditional classic antihistamines exhibit se-dation, especially in the first trimester. Terfenadine is
anti cholnergic properties and paradoxical ex-citation relatively safer in pregnancy.
in children as side effects.An ideal antihis-tamine The ideal drug for chronic urticaria would need to
should have a quick onset of action and less side have a broad spectrum of activities antagonizing not
effects besides convenient dosage schedule. The new only histamine but also a range of other media-tors
generation antihistamines fulfill these criteria and are such as neuropeptides, and possibly PAF and IL-I as
the main stay of treatment especially in chronic well.
idiopathic urticaria.
Q. What are other lines of management?
1. Terfenadine: is as effective as chlorph-
eniramine and hydroxyzine but less sedative. A. In a limited number of patients, low-dose
Cardiac arrhythmias may result if given in excess cyclosporine - A, can be effective.
dosage i.e. more than 120 mg/day for adults. In resistant cases a brief course of systemic
2. Astemizole is very useful in chronic idiopathic corticosteroid therapy may be necessary but the
urticaria and also in angioedema; and extended use of systemic corticosteroids should be
dermog-raphism. The dose is 10 mg/day. A dose avoided because of significant adverse effects.
710 DYP SURVIVAL GUIDE FOR POST GRADUATES

Q. Can stress management be effective in treatment of


urticaria?
97. XANTHOMAS
A. As urticaria patients have signifi-cant psychological
Q. Definition?
problems it is suggested that ad-junctive treatment
of urticaria should focus primarily on stress A. Xanthoma is an uncommon skin manifestation of
management training aimed at relieving anxiety and generalised or localised disturbance of lipid
metabolism.
group therapy which focuses on an ex-ploration of
interpersonal issues. Q. Word derivation?
Doxepinhydro-chloride, a tricyclic antidepressant A. Xanthoma = yellow tumor (greek)
significantly sup-presses histamine and codeine Q. Associated / underlying conditions?
induced cutaneous weal response. A. a) Hyperlipoproteinemia
Q. Treatment of angioedema? b) Hypercholesterolemia
A. Adrenalin is used in the emergency treatment of c) Leprosy
nonheredi-tary angioedema involving larynx. It can d) DM
be injected intramuscularly, subcutaneously or e) Xanthoerythroderma
inhaled depend-ing upon the severity of the reaction.
f) Lipoid proteinosis
Q. Role of phototherapy in urticaria?
g) Histiocytosis X
A. UVB or PUVA therapy may improve dermographism.
h) Pancreatitis
Q. How do we treat cold and solar urticaria? i) Biliary cirrhosis
A. Cold urti-caria can be managed by repeated exposure j) Multiple myeloma
to cold and inducing tolerance apart from H1
Q. Types of xanthomas?
antagonists.
A. Following types:
Solar urticaria may be minimized by wearing
appro-priate clothing, avoidance of sunlight, or a) Eruptive xanthomas
applying broad spectrum sun screens. b) Plane xanthomas
c) Xanthelesma palpebrum
h
d) Tuberous xanthomas
e) Nodular xanthomas
f) Tendinous xanthomas
g) Generalised plane xanthomas
h) Xanthoma Disseminatum
711
712 DYP SURVIVAL GUIDE FOR POST GRADUATES DYP SURVIVAL GUIDE FOR POST GRADUATES 713

Q. Characteristics of each type? Tendinous Achilles tendon, dorsum of hand


A. xanthomas affected. Subcutaneous nodules
Type Characteristics attached to tendons. Associated
with hypercholesterolemia,
Eruptive xanthomas Buttocks, thighs, hyperlipoproteinemias.
flexors affected. Pin head sized,
Generalised Face, neck, chest affected. Yellow
reddish brown or yellow papules.
Sudden onset. Associated with plane macules or plaques. Associated
mixed hypertriglyceridemia, DM xanthomas with myelomas, lymphomas.
D/D: syringomas, sarcoidosis Xanthoma Flexures, face, oral mucosa,
Plane xanthomas Flexors affected. Yellow or orange Disseminatum conjunctiva affected. Discrete,
slightly elevated plaques. Yellow small, yellow - brown papules and
streaks in palmar crease (xanthoma nodules. Seen in normo lipemic
individuals.
striata Palmaris). Associated with
lymphoma / leukaemia. Q. Histopathology?
Xanthoma Most common type. Around the A. Xanthoma cells/ Foam cells present in dermis.
Touton giant cells seen. Special stains like Scarlet Red
palpabrum eyes, over upper eyelids. Soft or Sudan Black confirm presence of cholesterol laden
velvety, yellow – orange papules or cells in frozen sections.
plaques. Associated with familial Q. What are investigations advise?
hypercholesterolemia, hyperlipo- A. - CBC
proteinemias. Can be treated with
- blood sugars
lasers, chemical peals.
- Thyroid profile
Tuberous Face, buttocks, extensors of limbs - Lipid profile
xanthomas affected. Firm, yellow, indurated - Biopsy
nodules. Associated with - USG
hypercholesterolemia, hyper- - Serum amylase
thoroidism, biliary disease.
- Serum electrophoresis
Nodular On pressure sites, ear lobes, neck. Q. Treatment?
xanthomas Multiple, yellow, dome shaped A. Dietary control, treatment of underlying disorders,
nodular lesions. Associated with lipid lowering agents, cauterization, surgical
biliary cirrhosis, biliary atresia. excision, chemical peels with TCA and lasers
D/D: granuloma annulare, leprosy, (xanthelesma).
leishmaniasis h
98. XERODERMA 99. YELLOW NAIL SYNDROME
PIGMENTOSUM Q. Who 1st discovered the Yellow Nail Syndrome?
A. Yellow nail syndrome (YNS), which was first
Q. What is the mode of inheritance of Xeroderma
described by Samman and White in 1964 and by
pigmentosum?
Emerson in 1966.
A. Autosomal recessive disorder characterized by
Q. What is the Yellow Nail Syndrome?
defective DNA thymine dimer excision repair,
extreme sun sensitivity, freckling and skin cancer. A. Uncommon disorder that is characterized by a classic
triad of yellow dystrophic nails, lymphedema, and
Q. What is the De Sanctis – Cacchione syndrome? respiratory tract abnormalities.
A. Xeroderma pigmentosum with mental deficiency, Q. What is the criteria for diagnosis of this Syndrome?
dwarfism and gonadal hyperplasia.
A. Need to fulfil 2 out of the 3 in the triad.
Q. In which group is De Sanctis – Cacchione syndrome
Q. What is the pathogenesis of Yellow Nail Syndrome?
common?
A. The specific etiology of YELLOW NAIL SYNDROME
A. Complementation group D
remains obscure; however, the pathogenesis is
Q. Which group is spared of neurological symptoms? thought to reflect functional or anatomic defects in
A. Group C the lymphatic vasculature and flow. In a subset of
Q. At which age, are the skin cancers seen? YNS patients, lymphangiography demonstrated
hypoplastic, deficient, or sclerotic lymphatic vessels.
A. Below 10 years of age
Such abnormal lymphatic ducts are hypothesized to
Q. Treatment of xeroderma pigmentosum? result in reduced lymphatic drainage, which leads
A. - Retinoids prevent appearance of new cancers to peripheral edema, pleural effusions, and nail
- Photoprotection changes when affecting the nail bed.
- Topical application of recombinant liposomal Another subset of YNS patients had persistent
hypoalbuminemia due to increased enteric loss of
encapsulated
albumin, which suggested that increased
- T4 endonuclease microvascular permeability may contribute to
- V repairs pathogenesis of the syndrome.
- UV induced Q. What are the nail changes seen?
- Cyclobutane pyrimidine dimers A. Nail changes include slow growth (0.1-0.25 mm/
week, normal 0.5-2 mm/week), yellow and green
h
714 715
716 DYP SURVIVAL GUIDE FOR POST GRADUATES

discoloration, transverse and longitudinal over-


curvature that results in a hyperconvex nail plate,
100. MOLLUSCUM
swelling of periungal tissue, onycholysis, shedding, CONTAGIOSUM
and loss of lunulae and cuticles, nail detachment.
Affected nails are thickened and sclerotic. Erythema
Q. Why is it a case of molluscum?
and edema of the proximal nail fold or chronic
paronychia may be present. A. They are asymptomatic, discrete dome shaped,
umbilicated waxy papules. Pseudokoebner’s present.
Q. What are respiratory symptoms present?
In children it is present over face, trunk, thighs,. It is
A. Respiratory involvement may vary but most often
caused by direct contact. In adults genital lesions are
manifests as chronic cough, sinusitis, bronchiectasis,
seen due to sexual transmission.
and exudative pleural effusions.
Q. What are the respiratory abnormalities associated? Q. What is the differential diagnoses of molluscum
contagiosum?
A. Pleural Effusion, Chronic Sinusitis, Bronchiectasis,
Chronic Bronchitis, Pulmonary TB and COPD A. Pyogenic Granuloma
Q. What are the other systemic abnormalities associated Epithelioma
with YNS? Cutaneous cryptococosis
A. Immunodeficiency syndromes and autoimmune Keratoacanthoma
diseases, such as thyroiditis, systemic lupus
erythematosus, rheumatoid arthritis, and Guillain- Q. What is molluscum caused due to?
Barré syndrome. Lymphoma and carcinoma of the A. MCV 1-4. MCV 1 is the most common
breast, lung, larynx, endometrium, and gall bladder, Q. What will you see on histopathology?
Mycosis Fungoides, Xanthogranulomatous
A. Epithelial cells have large intracytoplasmic inclusion
pyelonephritis, Nephrotic Syndrome, D-penicillamine
therapy, Chronic Graft vs Host reaction. body (molluscum body) with the dilated follicle.
Q. Treatment? Q. How will you treat molluscumcontagiosum?
A. Topical and oral vitamin E, oral zinc supplementation, A. 1. Curretage
oral anti-fungals, and intralesional triamcinolone 2. Cryotherapy
injections.
3. Phenol
h 4. Cantaridin 0.9 %
5. 40% silver nitrate paste
6. 20 % salicylic acid
717
718 DYP SURVIVAL GUIDE FOR POST GRADUATES

7. 10% Potassium hydroxide 101. CHICKEN POX


8. 0.5 % podophyllin
9. 5% imiqumod cream on alternate nights Q. Why is it a case of chicken pox?
10. 1% cidofovir ointment useful in HIV patients A. They are pruritic vesicles, polymorphous rash in
Q. Is it necessary to treat molluscumcontagiosum? centripetal distribution which are present on an
erythematious base giving dew drop on a rose petal
A. No. since it resolves in 9 months
appearance, umbilicated pustule or crust. Lesions
Q. Why do you get umbilicated central area lesion? arise in crops over 2-5 days
A. The bond between the keratinocytes is weakened due Q. What is the differential diagnoses of chicken pox?
to the excessive intracytoplasmic molluscum bodies.
A. Disseminated herpes zoster, impetigo, inscet bite,
This central part collapses due to certain extent and
scabies, erythema multiformae, DH, drug eruptions,
causes umbilication.
secondary syphilis
Q. What are the complications of molluscum?
Q. What is chicken pox caused due to?
A. Eyelid involvement can cause toxic conjunctivitis.
A. VZV
Molluscum dermatitis patchy eczema develops
around the lesions Q. What will you see on tzanck smear?
Q. What are the lesions in HIV patients? A. Multinucleated giant cells, epithelial cells with
acidophilic intranuclear inclusion bodies
A. Lesions may be present over the face and neck. The
lesions are large, multiple. Atypical lesions Q. What will you see on histopathology?
resembling comedones, absecesses, furuncles, A. Ballooning degeneration of granular cells as well as
keratoacanthoma can be present cells of deeper layers like stratum spinosum
Q. What are the disesaes caused by pox virus? Q. How will you manage?
A. Molluscum contagiosum, monkey pox, small pox, A. Acyclovir 800mg QID for 7 days in adults. In children
cow pox 20mg/kg QID for 5 days
If visceral cancer or immunocompromised; IV
acyclovir or valacyclovir 1gm TDS
h
Q. What vaccines are available?
A. Live accentuated VZV vaccine (OKA stain) 0.5 ml
subcutaneous 2 doses; 3 months apart at 1 year of age
719
720 DYP SURVIVAL GUIDE FOR POST GRADUATES

Q. What are the complications of cp?


A. 1. Secondary bacterial infection
2. Neurological complications like acute cerebral
ataxia, meningoencephalitis, transverse myelitis
3. Rye’s Syndrome- Acute encephalopathy with
fatty degeneration of liver
4. Respiratory complications- Varicella Pneumonia
5. Others- Myocarditis, Glomerulonephritis,
Keratitis, iritis

S-ar putea să vă placă și